You are on page 1of 209

POLITICAL LAW 135385, Dec.

6, 2000, 347 SCRA 128, 271, En Banc


[Per Curiam])
1. Distinguish sovereignty from dominion.

Held: Sovereignty is the right to exercise the functions 3. Discuss the concept of "jura regalia" and how it
of a State to the exclusion of any other State. It is often evolved in the Philippines. Does it negate native title to
referred to as the power of imperium, which is defined as lands held in private ownership since time immemorial?
the government authority possessed by the State. On the
other hand, dominion, or dominium, is the capacity of the Held: Generally, under the concept of jura regalia,
State to own or acquire property such as lands and private title to land must be traced to some grant, express
natural resources. (Separate Opinion, Kapunan, J., in or implied, from the Spanish Crown or its successors, the
Isagani Cruz v. Secretary of DENR, G.R. No. American Colonial government, and thereafter, the
135385, Dec. 6, 2000, En Banc, See Footnote 86) Philippine Republic. The belief that the Spanish Crown is
the origin of all land titles in the Philippines has persisted
2. How did Spain acquire the Philippines? because title to land must emanate from some source for
it cannot issue forth from nowhere (Pena, Registration of
Held: 1. The Philippines passed to Spain by Land Titles and Deeds, 1994 rev. ed., p. 15).
virtue of “discovery” and conquest. Consequently, all
lands became the exclusive patrimony and dominion of In its broad sense, the term "jura regalia" refers to royal
the Spanish Crown. The Spanish Government took grants (1 Bouvier's Law Dictionary, 3rd revision, p. 1759),
charge of distributing the lands by issuing royal grants or those rights which the King has by virtue of his
and concessions to Spaniards, both military and civilian prerogatives (Black's Law Dictionary, 6th ed., p. 1282). In
(Antonio H. Noblejas, Land Titles and Deeds, p. 5 [1986]; Spanish law, it refers to a right which the sovereign has
These grants were better known as repartimientos and over anything in which a subject has a right of property
encomiendas. Repartimientos were handouts to the or propriedad (76 Corpus Juris Secundum, citing Hart v.
military as fitting reward for their services to the Spanish Burnett, 15 Cal. 530, 566). These were rights enjoyed
crown. The encomiendas were given to Spaniards to during feudal times by the king as the sovereign.
administer and develop with the right to receive and enjoy
for themselves the tributes of the natives assigned to The theory of the feudal system was that title to all lands
them. – Ponce, supra, p. 12, citing Benitez, History of the was originally held by the King, and while the use of lands
Philippines, pp. 125-126). Private land titles could only was granted out to others who were permitted to hold
be acquired from the government either by purchase or them under certain conditions, the King theoretically
by the various modes of land grant from the Crown retained the title (Washburn, p. 44; see also Williams,
(Narciso Pena, Registration of Land Titles and Deeds, p. Principles Of The Law On Real Property, 6th ed. [1886], p.
2 [1994]). (Separate Opinion, Puno, J., in Cruz v. 2; Bigelow, p. 2). By fiction of law, the King was regarded
Secretary of Environment and Natural Resources, as the original proprietor of all lands, and the true and
347 SCRA 128, 166, En Banc [Per Curiam]) only source of title, and from him all lands were held
(Warvelle, Abstracts and Examination of Title to Real
2. When Spain acquired sovereignty over the Philippines Property [1907], p. 18). The theory of jura regalia was
by virtue of its discovery and occupation thereof in the therefore nothing more than a natural fruit of conquest (1
16th century and the Treaty of Tordesillas of 1494 which Dictionary of English Law [Jowitt, ed.] p. 797).
it entered into with Portugal (Under the Treaty of
Tordesillas, the world was divided between Spain and The Regalian theory, however, does not negate native
Portugal, with the former having exclusive power to claim title to lands held in private ownership since time
all lands and territories west of the Atlantic Ocean immemorial. In the landmark case of Carino v. Insular
demarcation line [Lynch, The Legal Bases of Philippine Government (41 Phil. 935, 212 U.S. 449, 53 L. Ed. 594
Colonial Sovereignty, 62 Phil. L J 279, 283 [1987]) the [1909]), the United States Supreme Court, reversing the
continents of Asia, the Americas and Africa were decision of the pre-war Philippine Supreme Court, made
considered as terra nullius although already populated by the following pronouncement:
other peoples (See Akehurst, a Modern Introduction to
International Law, 5th ed., 142-143). The discovery and x x x Every presumption is and ought to be taken against
occupation by the European States, who were then the Government in a case like the present. It might,
considered as the only members of the international perhaps, be proper and sufficient to say that when, as far
community of civilized nations, of lands in the said back as testimony or memory goes, the land has been
continents were deemed sufficient to create title under held by individuals under a claim of private ownership, it
international law (See Cruz, International Law, 1996 ed., will be presumed to have been held in the same way from
pp. 106-107) (Separate Opinion, Kapunan, J., in before the Spanish conquest, and never to have been
Isagani Cruz v. Secretary of DENR, G.R. No. public land. x x x (Carino v. Insular Government, supra
note 75, at 941)
legislative or by the executive branch or entered into by
The above ruling institutionalized the recognition of the private persons for private purposes, is null and void and
existence of native title to land, or ownership of land by without any force and effect. Thus, since the Constitution
Filipinos by virtue of possession under a claim of is the fundamental, paramount and supreme law of the
ownership since time immemorial and independent of any nation, it is deemed written in every statute and contract .
grant from the Spanish Crown, as an exception to the (Manila Prince Hotel v. GSIS, 267 SCRA 408 [1997]
theory of jura regalia. [Bellosillo])

xxx 6. What are self-executing and non-self executing


provisions of the Constitution?
Carino was decided by the U.S. Supreme Court in 1909,
at a time when decisions of the U.S. Court were binding Held: A provision which lays down a general principle,
as precedent in our jurisdiction (Section 10, Philippine Bill such as those found in Article II of the 1987 Constitution,
of 1902). We applied the Carino doctrine in the 1946 case is usually not self-executing. But a provision which is
of Oh Cho v. Director of Lands (75 Phil. 890 [1946]), complete in itself and becomes operative without the aid
where we stated that “[a]ll lands that were not acquired of supplementary or enabling legislation, or that which
from the Government either by purchase or by grant, supplies sufficient rule by means of which the right it
belong to the public domain, but [a]n exception to the grants may be enjoyed or protected, is self-executing.
rule would be any land that should have been in the Thus a constitutional provision is self-executing if the
possession of an occupant and of his predecessors in nature and extent of the right conferred and the liability
interest since time immemorial, for such possession would imposed are fixed by the Constitution itself, so that they
justify the presumption that the land had never been part can be determined by an examination and construction of
of the public domain or that it had been private property its terms, and there is no language indicating that the
even before the Spanish conquest.” (Id., at 892). subject is referred to the legislature for action. (Manila
(Separate Opinion, Kapunan, J., in Isagani Cruz v. Prince Hotel v. GSIS, 267 SCRA 408 [1997]
Secretary of DENR, G.R. No. 135385, Dec. 6, 2000, [Bellosillo])
347 SCRA 128, 268-270, En Banc [Per Curiam]
7. Are provisions of the Constitution self-executing
4. What was the basis for the early Spanish decrees or non-self executing? Why?
embracing the theory of jura regalia? Is this also the basis
of the declaration in Section 2, Article XII of the 1987 Held: Unless it is expressly provided that a legislative act
Constitution that all lands of the public domain are owned is necessary to enforce a constitutional mandate, the
by the State? Consequently, did Spain acquire title over presumption now is that all provisions are self-executing.
all lands in the Philippines in the 16th century? If the constitutional provisions are treated as requiring
legislation instead of self-executing, the legislature would
Held: Dominium was the basis for the early Spanish have the power to ignore and practically nullify the
decrees embracing the theory of jura regalia. The mandate of the fundamental law. This can be
declaration in Section 2, Article XII of the 1987 cataclysmic. (Manila Prince Hotel v. GSIS, 267 SCRA
Constitution that all lands of the public domain are owned 408 [1997] [Bellosillo])
by the State is likewise founded on dominium. If
dominium, not imperium, is the basis of the theory of jura 8. Is the “Filipino First” Policy expressed in Section
regalia, then the lands which Spain acquired in the 16th 10, Article XII of the Constitution a self-executing
century were limited to non-private lands, because it provision?
could only acquire lands which were not yet privately-
owned or occupied by the Filipinos. Hence, Spain Held: Yes. It is a mandatory, positive command which
acquired title only over lands which were unoccupied and is complete in itself and which needs no further guidelines
unclaimed, i.e., public lands. (Separate Opinion, or implementing laws or rules for its enforcement. From
Kapunan, J., in Isagani Cruz v. Secretary of DENR, its very words the provision does not require any
G.R. No. 135385, Dec. 6, 2000, En Banc, See legislation to put it in operation. It is per se judicially
Footnote 86) enforceable. When our Constitution mandates that [i]n
the grant of rights, privileges, and concessions covering
the national economy and patrimony, the State shall give
preference to qualified Filipinos, it means just that –
5. What is the Doctrine of Constitutional qualified Filipinos must be preferred. (Manila Prince
Supremacy? Hotel v. GSIS, G.R. No. 118295, May 2, 1997, 267
SCRA 408 [Bellosillo])
Held: Under the doctrine of constitutional supremacy, if
a law or contract violates any norm of the Constitution, 9. Give examples of non-self executing provisions
that law or contract, whether promulgated by the of the Constitution.
to be accordingly restricted. (Department of
Held: By its very nature, Article II of the Constitution is Agriculture v. NLRC, 227 SCRA 693, Nov. 11, 1993
a “declaration of principles and state policies.” These [Vitug])
principles in Article II are not intended to be self-
executing principles ready for enforcement through the 12. Is the rule absolute, i.e., that the State may not
courts. They are used by the judiciary as aids or as guides be sued at all? How may consent of the State to be sued
in the exercise of its power of judicial review, and by the given?
legislature in its enactment of laws. As held in the leading
case of Kilosbayan, Incorporated v. Morato (246 SCRA Held: The rule, in any case, is not really absolute for it
540, 564, July 17, 1995), the principles and state policies does not say that the state may not be sued under any
enumerated in Article II and some sections of Article XII circumstances. On the contrary, as correctly phrased, the
are not “self-executing provisions, the disregard of which doctrine only conveys, “the state may not be sued without
can give rise to a cause of action in courts. They do not its consent;” its clear import then is that the State may at
embody judicially enforceable constitutional rights but times be sued. The State's consent may be given either
guidelines for legislation.” (Tanada v. Angara, 272 expressly or impliedly. Express consent may be made
SCRA 18 [1997], En Banc [Panganiban]) through a general law (i.e., Commonwealth Act No. 327,
as amended by Presidential Decree No. 1445 [Sections
10. When are acts of persons considered “State 49-50], which requires that all money claims against the
action” covered by the Constitution? government must first be filed with the Commission on
Audit which must act upon it within sixty days. Rejection
Held: In constitutional jurisprudence, the act of persons of the claim will authorize the claimant to elevate the
distinct from the government are considered “state matter to the Supreme Court on certiorari and, in effect,
action” covered by the Constitution (1) when the activity sue the State thereby) or a special law. In this
it engages in is a “public function”; (2) when the jurisdiction, the general law waiving the immunity of the
government is so significantly involved with the private state from suit is found in Act No. 3083, where the
actor as to make the government responsible for his Philippine government “consents and submits to be sued
action; and (3) when the government has approved or upon any money claim involving liability arising from
authorized the action. (Manila Prince Hotel v. GSIS, contract, express or implied, which could serve as a basis
267 SCRA 408 [1997] [Bellosillo]) of civil action between the private parties.” Implied
consent, on the other hand, is conceded when the State
itself commences litigation, thus opening itself to a
The Doctrine of State Immunity from Suit counterclaim or when it enters into a contract. In this
situation, the government is deemed to have descended
11. Discuss the basis of the doctrine of State to the level of the other contracting party and to have
immunity from suit. divested itself of its sovereign immunity. This rule x x x
is not, however, without qualification. Not all contracts
Held: The basic postulate enshrined in the Constitution entered into by the government operate as a waiver of its
that “[t]he State may not be sued without its consent,” non-suability; distinction must still be made between one
reflects nothing less than a recognition of the sovereign which is executed in the exercise of its sovereign function
character of the State and an express affirmation of the and another which is done in its proprietary capacity.
unwritten rule effectively insulating it from the jurisdiction
of courts. It is based on the very essence of sovereignty. In United States of America v. Ruiz (136 SCRA 487),
As has been aptly observed by Justice Holmes, a where the questioned transaction dealt with the
sovereign is exempt from suit, not because of any formal improvements on the wharves in the naval installation at
conception or obsolete theory, but on the logical and Subic Bay, we held:
practical ground that there can be no legal right as against
the authority that makes the law on which the right “The traditional rule of immunity exempts a State from
depends. True, the doctrine, not too infrequently, is being sued in the courts of another State without its
derisively called “the royal prerogative of dishonesty” consent or waiver. This rule is a necessary consequence
because it grants the state the prerogative to defeat any of the principle of independence and equality of States.
legitimate claim against it by simply invoking its non- However, the rules of International Law are not petrified;
suability. We have had occasion to explain in its defense, they are constantly developing and evolving. And
however, that a continued adherence to the doctrine of because the activities of states have multiplied, it has
non-suability cannot be deplored, for the loss of been necessary to distinguish them - between sovereign
governmental efficiency and the obstacle to the and governmental acts (jure imperii) and private,
performance of its multifarious functions would be far commercial and proprietary acts (jure gestionis). The
greater in severity than the inconvenience that may be result is that State immunity now extends only to acts jure
caused private parties, if such fundamental principle is to imperii. The restrictive application of State immunity is
be abandoned and the availability of judicial remedy is not
now the rule in the United States, the United Kingdom 14. State instances when a suit against the State is
and other states in Western Europe. proper.

Xxx Held: Some instances when a suit against the State is


proper are:
The restrictive application of State immunity is proper
only when the proceedings arise out of commercial 1) When the Republic is sued by name;
transactions of the foreign sovereign, its commercial 2) When the suit is against an unincorporated
activities or economic affairs. Stated differently, a State government agency;
may be said to have descended to the level of an 3) When the suit is on its face against a
individual and can thus be deemed to have tacitly given government officer but the case is such that ultimate
its consent to be sued only when it enters into business liability will belong not to the officer but to the
contracts. It does not apply where the contracts relate to government.
the exercise of its sovereign functions. In this case the Republic v. Sandoval, 220 SCRA 124, March 19,
projects are an integral part of the naval base which is 1993, En Banc [Campos, Jr.])
devoted to the defense of both the United States and the
Philippines, indisputably a function of the government of 15. Has the government waived its immunity from
the highest order; they are not utilized for nor dedicated suit in the Mendiola massacre, and, therefore, should
to commercial or business purposes.” indemnify the heirs and victims of the Mendiola incident?
(Department of Agriculture v. NLRC, 227 SCRA Consequently, is the suit filed against the Republic by
693, Nov. 11, 1993 [Vitug]) petitioners in said case really a suit against the State?

13. When is a suit against a public official deemed to Held: Petitioners x x x advance the argument that the
be a suit against the State? Discuss. State has impliedly waived its sovereign immunity from
suit. It is their considered view that by the
Held: 1. The doctrine of state immunity from suit applies recommendation made by the Commission for the
to complaints filed against public officials for acts done in government to indemnify the heirs and victims of the
the performance of their duties. The rule is that the suit Mendiola incident and by the public addresses made by
must be regarded as one against the State where the then President Aquino in the aftermath of the killings, the
satisfaction of the judgment against the public official State has consented to be sued.
concerned will require the State itself to perform a
positive act, such as appropriation of the amount Xxx
necessary to pay the damages awarded to the plaintiff.
This is not a suit against the State with its consent.
The rule does not apply where the public official is
charged in his official capacity for acts that are unlawful Firstly, the recommendation made by the Commission
and injurious to the rights of others. Public officials are regarding indemnification of the heirs of the deceased
not exempt, in their personal capacity, from liability and the victims of the incident by the government does
arising from acts committed in bad faith. not in any way mean that liability automatically attaches
to the State. It is important to note that A.O. 11 expressly
Neither does it apply where the public official is clearly states that the purpose of creating the Commission was
being sued not in his official capacity but in his personal to have a body that will conduct an “investigation of the
capacity, although the acts complained of may have been disorder, deaths and casualties that took place.” In the
committed while he occupied a public position. (Amado exercise of its functions, A.O. 11 provides guidelines, and
J. Lansang v. CA, G.R. No. 102667, Feb. 23, 2000, what is relevant to Our discussion reads:
2nd Div. [Quisumbing])
“1. Its conclusions regarding the existence of probable
2. As early as 1954, this Court has pronounced that an cause for the commission of any offense and of the
officer cannot shelter himself by the plea that he is a persons probably guilty of the same shall be sufficient
public agent acting under the color of his office when his compliance with the rules on preliminary investigation and
acts are wholly without authority. Until recently in 1991 the charges arising therefrom may be filed directly with
(Chavez v. Sandiganbayan, 193 SCRA 282 [1991]), this the proper court.”
doctrine still found application, this Court saying that
immunity from suit cannot institutionalize irresponsibility In effect, whatever may be the findings of the
and non-accountability nor grant a privileged status not Commission, the same shall only serve as the cause of
claimed by any other official of the Republic. (Republic action in the event that any party decides to litigate
v. Sandoval, 220 SCRA 124, March 19, 1993, En his/her claim. Therefore, the Commission is merely a
Banc [Campos, Jr.]) preliminary venue. The Commission is not the end in
itself. Whatever recommendation it makes cannot in any
way bind the State immediately, such recommendation subdivisions cannot be sued in the courts except when it
not having become final and executory. This is precisely has given its consent, it cannot be invoked by both the
the essence of it being a fact-finding body. military officers to release them from any liability, and by
the heirs and victims to demand indemnification from the
Secondly, whatever acts or utterances that then President government. The principle of state immunity from suit
Aquino may have done or said, the same are not does not apply, as in this case, when the relief demanded
tantamount to the State having waived its immunity from by the suit requires no affirmative official action on the
suit. The President’s act of joining the marchers, days part of the State nor the affirmative discharge of any
after the incident, does not mean that there was an obligation which belongs to the State in its political
admission by the State of any liability. In fact to borrow capacity, even though the officers or agents who are
the words of petitioner x x x, “it was an act of solidarity made defendants claim to hold or act only by virtue of a
by the government with the people.” Moreover, title of the state and as its agents and servants. This
petitioners rely on President Aquino’s speech promising Court has made it quite clear that even a “high position in
that the government would address the grievances of the the government does not confer a license to persecute or
rallyists. By this alone, it cannot be inferred that the State recklessly injure another.”
has admitted any liability, much less can it be inferred that
it has consented to the suit. The inescapable conclusion is that the State cannot be
held civilly liable for the deaths that followed the incident.
Although consent to be sued may be given impliedly, still Instead, the liability should fall on the named defendants
it cannot be maintained that such consent was given in the lower court. In line with the ruling of this Court in
considering the circumstances obtaining in the instant Shauf v. Court of Appeals (191 SCRA 713 [1990]), herein
case. public officials, having been found to have acted beyond
the scope of their authority, may be held liable for
Thirdly, the case does not qualify as a suit against the damages. (Republic v. Sandoval, 220 SCRA 124,
State. March 19, 1993, En Banc [Campos, Jr.])

Xxx
16. May the Government validly invoke the doctrine
While the Republic in this case is sued by name, the of State immunity from suit if its invocation will serve as
ultimate liability does not pertain to the government. an instrument for perpetrating an injustice on a citizen?
Although the military officers and personnel, then party
defendants, were discharging their official functions when Held: To our mind, it would be the apex of
the incident occurred, their functions ceased to be official injustice and highly inequitable for us to defeat
the moment they exceeded their authority. Based on the petitioners-contractors’ right to be duly compensated for
Commission findings, there was lack of justification by the actual work performed and services rendered, where
government forces in the use of firearms. Moreover, the both the government and the public have, for years,
members of the police and military crowd dispersal units received and accepted benefits from said housing
committed a prohibited act under B.P. Blg. 880 as there project and reaped the fruits of petitioners-contractors’
was unnecessary firing by them in dispersing the honest toil and labor.
marchers.
Incidentally, respondent likewise argues that the
As early as 1954, this Court has pronounced that an State may not be sued in the instant case, invoking the
officer cannot shelter himself by the plea that he is a constitutional doctrine of Non-suability of the State,
public agent acting under the color of his office when his otherwise known as the Royal Prerogative of Dishonesty.
acts are wholly without authority. Until recently in 1991
(Chavez v. Sandiganbayan, 193 SCRA 282 [1991]), this Respondent’s argument is misplaced inasmuch
doctrine still found application, this Court saying that as the principle of State immunity finds no application in
immunity from suit cannot institutionalize irresponsibility the case before us.
and non-accountability nor grant a privileged status not
claimed by any other official of the Republic. The military Under these circumstances, respondent may not
and police forces were deployed to ensure that the rally validly invoke the Royal Prerogative of Dishonesty and
would be peaceful and orderly as well as to guarantee the conveniently hide under the State’s cloak of invincibility
safety of the very people that they are duty-bound to against suit, considering that this principle yields to
protect. However, the facts as found by the trial court certain settled exceptions. True enough, the rule, in any
showed that they fired at the unruly crowd to disperse the case, is not absolute for it does not say that the state
latter. may not be sued under any circumstances. (Citations
omitted)
While it is true that nothing is better settled than the
general rule that a sovereign state and its political
Thus, in Amigable v. Cuenca, this Court, in
effect, shred the protective shroud which shields the Under both organic acts, all inhabitants of the
state from suit, reiterating our decree in the landmark Philippines who were Spanish subjects on April 11, 1899
case of Ministerio v. CFI of Cebu that “the doctrine of and resided therein including their children are deemed
governmental immunity from suit cannot serve as an to be Philippine citizens. Private respondent’s father,
instrument for perpetrating an injustice on a citizen.” It Telesforo Ybasco, was born on January 5, 1879 in Daet,
is just as important, if not more so, that there be fidelity Camarines Norte, a fact duly evidenced by a certified true
to legal norms on the part of officialdom if the rule of copy of an entry in the Registry of Births. Thus, under
law were to be maintained. (Citations omitted) the Philippine Bill of 1902 and the Jones Law, Telesforo
Ybasco was deemed to be a Philippine citizen. By virtue
Although the Amigable and Ministerio cases of the same laws, which were the laws in force at the time
generously tackled the issue of the State’s immunity of her birth, Telesforo’s daughter, herein private
from suit vis a vis the payment of just compensation for respondent Rosalind Ybasco Lopez, is likewise a citizen of
expropriated property, this Court nonetheless finds the the Philippines.
doctrine enunciated in the aforementioned cases
applicable to the instant controversy, considering that The signing into law of the 1935 Philippine Constitution
the ends of justice would be subverted if we were to has established the principle of jus sanguinis as basis for
uphold, in this particular instance, the State’s immunity the acquisition of Philippine citizenship x x x. So also, the
from suit. principle of jus sanguinis, which confers citizenship by
virtue of blood relationship, was subsequently retained
To be sure, this Court – as the staunch guardian under the 1973 and 1987 Constitutions. Thus, the herein
of the citizens’ rights and welfare – cannot sanction an private respondent, Rosalind Ybasco Lopez, is a Filipino
injustice so patent on its face, and allow itself to be an citizen, having been born to a Filipino father. The fact of
instrument in the perpetration thereof. Justice and equity her being born in Australia is not tantamount to her losing
sternly demand that the State’s cloak of invincibility her Philippine citizenship. If Australia follows the principle
against suit be shred in this particular instance, and that of jus soli, then at most, private respondent can also claim
petitioners-contractors be duly compensated – on the Australian citizenship resulting to her possession of dual
basis of quantum meruit – for construction done on the citizenship. (Valles v. COMELEC, 337 SCRA 543,
public works housing project. (EPG Construction Co. Aug. 9, 2000, En Banc [Purisima])
v. Vigilar, 354 SCRA 566, Mar.16, 2001, 2 nd
Div.
[Buena]) 18. What are the ways of acquiring citizenship?
Discuss.

Citizenship Held: There are two ways of acquiring citizenship: (1)


by birth, and (2) by naturalization. These ways of
17. To what citizenship principle does the Philippines acquiring citizenship correspond to the two kinds of
adhere to? Explain, and give illustrative case. citizens: the natural-born citizen, and the naturalized
citizen. A person who at the time of his birth is a citizen
Held: The Philippine law on citizenship adheres to the of a particular country, is a natural-born citizen thereof.
principle of jus sanguinis. Thereunder, a child follows the
nationality or citizenship of the parents regardless of the As defined in the x x x Constitution, natural-born citizens
place of his/her birth, as opposed to the doctrine of jus “are those citizens of the Philippines from birth without
soli which determines nationality or citizenship on the having to perform any act to acquire or perfect his
basis of place of birth. Philippine citizenship.”

Private respondent Rosalind Ybasco Lopez was On the other hand, naturalized citizens are those who
born on May 16, 1934 in Napier Terrace, Broome, have become Filipino citizens through naturalization,
Western Australia, to the spouses, Telesforo Ybasco, a generally under Commonwealth Act No. 473, otherwise
Filipino citizen and native of Daet, Camarines Norte, and known as the Revised Naturalization Law, which repealed
Theresa Marquez, an Australian. Historically, this was a the former Naturalization Law (Act No. 2927), and by
year before the 1935 Constitution took into effect and at Republic Act No. 530. (Antonio Bengson III v. HRET,
that time, what served as the Constitution of the G.R. No. 142840, May 7, 2001, En Banc [Kapunan])
Philippines were the principal organic acts by which the
United States governed the country. These were the 19. To be naturalized, what must an applicant prove?
Philippine Bill of July 1, 1902 and the Philippine Autonomy When and what are the conditions before the decision
Act of August 29, 1916, also known as the Jones Law. granting Philippine citizenship becomes executory?

Among others, these laws defined who were Held: To be naturalized, an applicant has to prove that
deemed to be citizens of the Philippine Islands. x x x he possesses all the qualifications and none of the
disqualifications provided by law to become a Filipino assassination for the success and predominance of their
citizen. The decision granting Philippine citizenship ideas;
becomes executory only after two (2) years from its (c) He must not be a polygamist or believer in
promulgation when the court is satisfied that during the the practice of polygamy;
intervening period, the applicant has (1) not left the (d) He must not have been convicted of any
Philippines; (2) has dedicated himself to a lawful calling crime involving moral turpitude;
or profession; (3) has not been convicted of any offense (e) He must not be suffering from mental
or violation of government promulgated rules; or (4) alienation or incurable contagious diseases;
committed any act prejudicial to the interest of the nation (f) He must have, during the period of his
or contrary to any government announced policies residence in the Philippines (or not less than six months
(Section 1, R.A. 530). (Antonio Bengson III v. HRET, before filing his application), mingled socially with the
G.R. No. 142840, May 7, 2001, En Banc [Kapunan]) Filipinos, or who have not evinced a sincere desire to learn
and embrace the customs, traditions and ideals of the
20. What qualifications must be possessed by an Filipinos;
applicant for naturalization? (g) He must not be a citizen or subject of a
nation with whom the Philippines is at war, during the
Held: Section 2, Act 473 provides the following period of such war;
qualifications: (h) He must not be a citizen or subject of a
foreign country whose laws do not grant Filipinos the right
(a) He must be not less than 21 years of age on to become naturalized citizens or subjects thereof.
the day of the hearing of the petition; (Antonio Bengson III v. HRET, G.R. No. 142840,
(b) He must have resided in the Philippines for a May 7, 2001, En Banc [Kapunan])
continuous period of not less than ten years;
(c) He must be of good moral character and 22. Can a legitimate child born under the 1935
believes in the principles underlying the Philippine Constitution of a Filipino mother and an alien father validly
Constitution, and must have conducted himself in a elect Philippine citizenship fourteen (14) years after he
proper and irreproachable manner during the entire has reached the age of majority?
period of his residence in the Philippines in his relation
with the constituted government as well as with the Held: Under Article IV, Section 1(3) of the 1935
community in which he is living; Constitution, the citizenship of a legitimate child born of
(d) He must own real estate in the Philippines a Filipino mother and an alien father followed the
worth not less than five thousand pesos, Philippine citizenship of the father, unless, upon reaching the age of
currency, or must have some known lucrative trade, majority, the child elected Philippine citizenship. C.A. No.
profession, or lawful occupation; 625 which was enacted pursuant to Section 1(3), Article
(e) He must be able to speak and write English IV of the 1935 Constitution, prescribes the procedure that
or Spanish and any of the principal languages; and should be followed in order to make a valid election of
(f) He must have enrolled his minor children of Philippine citizenship. However, the 1935 Constitution
school age, in any of the public schools or private schools and C.A. No. 625 did not prescribe a time period within
recognized by the Bureau of Private Schools of the which the election of Philippine citizenship should be
Philippines where Philippine history, government and civic made. The 1935 Charter only provides that the election
are taught or prescribed as part of the school curriculum, should be made “upon reaching the age of majority.” The
during the entire period of the residence in the Philippines age of majority then commenced upon reaching twenty-
required of him prior to the hearing of his petition for one (21) years. In the opinions of the Secretary of Justice
naturalization as Philippine citizen. on cases involving the validity of election of Philippine
(Antonio Bengson III v. HRET, G.R. No. 142840, citizenship, this dilemma was resolved by basing the time
May 7, 2001, En Banc [Kapunan]) period on the decisions of this Court prior to the effectivity
of the 1935 Constitution. In these decisions, the proper
21. What are the disqualifications under Section 4, period for electing Philippine citizenship was, in turn,
Act 473, in an application for naturalization? based on the pronouncements of the Department of State
of the United States Government to the effect that the
Held: Section 4, Act 473, provides the following election should be made within a “reasonable time” after
disqualifications: attaining the age of majority. The phrase “reasonable
time” has been interpreted to mean that the election
(a) He must not be opposed to organized should be made within three (3) years from reaching the
government or affiliated with any association or group of age of majority.
persons who uphold and teach doctrines opposing all
organized governments; The span of fourteen (14) years that lapsed from the time
(b) He must not be defending or teaching the that person reached the age of majority until he finally
necessity or propriety of violence, personal assault, or expressed his intention to elect Philippine citizenship is
clearly way beyond the contemplation of the requirement citizenship and do not militate against her claim of Filipino
of electing “upon reaching the age of majority.” citizenship. For renunciation to effectively result in the
loss of citizenship, the same must be express. As held by
Philippine citizenship can never be treated like a this Court in the aforecited case of Aznar, an application
commodity that can be claimed when needed and for an alien certificate of registration does not amount to
suppressed when convenient. One who is privileged to an express renunciation or repudiation of one’s
elect Philippine citizenship has only an inchoate right to citizenship. The application of the herein private
such citizenship. As such, he should avail of the right with respondent for an alien certificate of registration, and her
fervor, enthusiasm and promptitude. (Re: Application holding of an Australian passport, as in the case of
for Admission to the Philippine Bar, Vicente D. Mercado v. Manzano, were mere acts of assertion of her
Ching, Bar Matter No. 914, Oct. 1, 1999, En Banc Australian citizenship before she effectively renounced
[Kapunan]) the same. Thus, at the most, private respondent had dual
citizenship – she was an Australian and a Filipino, as well.
23. How may Philippine citizenship be renounced? Is
the application for an alien certificate of registration, and Moreover, under Commonwealth Act 63, the fact
the possession of foreign passport, tantamount to acts of that a child of Filipino parent/s was born in another
renunciation of Philippine citizenship? country has not been included as a ground for losing one’s
Philippine citizenship. Since private respondent did not
Held: Petitioner also contends that even on the lose or renounce her Philippine citizenship, petitioner’s
assumption that the private respondent is a Filipino claim that respondent must go through the process of
citizen, she has nonetheless renounced her Philippine repatriation does not hold water. (Valles v. COMELEC,
citizenship. To buttress this contention, petitioner cited 337 SCRA 543, Aug. 9, 2000, En Banc [Purisima])
private respondent’s application for an alien Certificate of
Registration (ACR) and Immigrant Certificate of 24. How may Filipino citizens who lost their
Residence (ICR), on September 19, 1988, and the citizenship reacquire the same?
issuance to her of an Australian passport on March 3,
1988. Answer: Filipino citizens who have lost their citizenship
may x x x reacquire the same in the manner provided by
Xxx law. Commonwealth Act No. 63 enumerates the three
modes by which Philippine citizenship may be reacquired
In order that citizenship may be lost by by a former citizen: (1) by naturalization, (2) by
renunciation, such renunciation must be express. repatriation, and (3) by direct act of Congress. (Frivaldo
Petitioner’s contention that the application of private v. COMELEC, 257 SCRA 727, June 28, 1996, En
respondent for an alien certificate of registration, and her Banc [Panganiban]; Antonio Bengson III v. HRET,
Australian passport, is bereft of merit. This issue was put G.R. No. 142840, May 7, 2001, En Banc [Kapunan])
to rest in the case of Aznar v. COMELEC (185 SCRA 703
[1990]) and in the more recent case of Mercado v. 25. Distinguish naturalization from repatriation.
Manzano and COMELEC (G.R. No. 135083, 307 SCRA 630,
May 26, 1999). Held: Naturalization is a mode for both acquisition and
reacquisition of Philippine citizenship. As a mode of
In the case of Aznar, the Court ruled that the initially acquiring Philippine citizenship, naturalization is
mere fact that he is an American did not mean that he is governed by Commonwealth Act No. 473, as amended.
no longer a Filipino, and that an application for an alien On the other hand, naturalization as a mode for
certificate of registration was not tantamount to reacquiring Philippine citizenship is governed by
renunciation of his Philippine citizenship. Commonwealth Act No. 63 (An Act Providing for the Ways
in Which Philippine Citizenship May Be Lost or Reacquired
And, in Mercado v. Manzano and COMELEC, it [1936]). Under this law, a former Filipino citizen who
was held that the fact that respondent Manzano was wishes to reacquire Philippine citizenship must possess
registered as an American citizen in the Bureau of certain qualifications and none of the disqualifications
Immigration and Deportation and was holding an mentioned in Section 4 of C.A. 473.
American passport on April 22, 1997, only a year before
he filed a certificate of candidacy for vice-mayor of Repatriation, on the other hand, may be had under
Makati, were just assertions of his American nationality various statutes by those who lost their citizenship due
before the termination of his American citizenship. to: (1) desertion of the armed forces (Section 4, C.A. No.
63); (2) service in the armed forces of the allied forces in
Thus, the mere fact that private respondent World War II (Section 1, Republic Act No. 965 [1953]);
Rosalind Ybasco Lopez was a holder of an Australian (3) service in the Armed Forces of the United States at
passport and had an alien certificate of registration are any other time (Sec. 1, Republic Act No. 2630 [1960]);
not acts constituting an effective renunciation of (4) marriage of a Filipino woman to an alien (Sec. 1,
Republic Act No. 8171 [1995]); and (5) political and 28. May a natural-born Filipino who became an
economic necessity (Ibid). American citizen still be considered a natural-born Filipino
upon his reacquisition of Philippine citizenship and,
As distinguished from the lengthy process of therefore, qualified to run for Congressman?
naturalization, repatriation simply consists of the taking of
an oath of allegiance to the Republic of the Philippines Held: Repatriation results in the recovery of the original
and registering said oath in the Local Civil Registry of the nationality. This means that a naturalized Filipino who
place where the person concerned resides or last resided. lost his citizenship will be restored to his prior status as a
naturalized Filipino citizen. On the other hand, if he was
In Angat v. Republic (314 SCRA 438 [1999]), we held: originally a natural-born citizen before he lost his
Philippine citizenship, he will be restored to his former
[P]arenthetically, under these statutes (referring to RA status as a natural-born Filipino.
Nos. 965 and 2630), the person desiring to reacquire
Philippine citizenship would not even be required to file a In respondent Cruz’s case, he lost his Filipino citizenship
petition in court, and all that he had to do was to take an when he rendered service in the Armed Forces of the
oath of allegiance to the Republic of the Philippines and United States. However, he subsequently reacquired
to register that fact with the civil registry in the place of Philippine citizenship under R.A. No. 2630, which
his residence or where he had last resided in the provides:
Philippines.
Section 1. Any person who had lost his Philippine
Moreover, repatriation results in the recovery of the citizenship by rendering service to, or accepting
original nationality. This means that a naturalized Filipino commission in, the Armed Forces of the United States, or
who lost his citizenship will be restored to his prior status after separation from the Armed Forces of the United
as a naturalized Filipino citizen. On the other hand, if he States, acquired United States citizenship, may reacquire
was originally a natural-born citizen before he lost his Philippine citizenship by taking an oath of allegiance to
Philippine citizenship, he will be restored to his former the Republic of the Philippines and registering the same
status as a natural-born Filipino. (Antonio Bengson III with Local Civil Registry in the place where he resides or
v. HRET, G.R. No. 142840, May 7, 2001, En Banc last resided in the Philippines. The said oath of allegiance
[Kapunan]) shall contain a renunciation of any other citizenship.

26. Who may validly avail of repatriation under R.A. Having thus taken the required oath of allegiance to the
No. 8171? Republic and having registered the same in the Civil
Registry of Mangatarem, Pangasinan in accordance with
Held: R.A. No. 8171, which has lapsed into law on the aforecited provision, respondent Cruz is deemed to
October 23, 1995, is an act providing for the repatriation have recovered his original status as a natural-born
(a) of Filipino women who have lost their Philippine citizen, a status which he acquired at birth as the son of
citizenship by marriage to aliens and (b) of natural-born a Filipino father. It bears stressing that the act of
Filipinos who have lost their Philippine citizenship on repatriation allows him to recover, or return to, his
account of political or economic necessity. (Gerardo original status before he lost his Philippine citizenship.
Angat v. Republic, G.R. No. 132244, Sept. 14, 1999
[Vitug]) Petitioner’s contention that respondent Cruz is no longer
a natural-born citizen since he had to perform an act to
27. Before what agency should application for regain his citizenship is untenable. [T]he term “natural-
repatriation under R.A 8171 be filed? born citizen” was first defined in Article III, Section 4 of
the 1973 Constitution as follows:
Held: Under Section 1 of P.D. No. 725, dated June 5,
1975, amending C.A. No. 63, an application for Section 4. A natural-born citizen is one who is a citizen
repatriation could be filed with the Special Committee on of the Philippines from birth without having to perform
Naturalization chaired by the Solicitor General with the any act to acquire or perfect his Philippine citizenship.
Undersecretary of Foreign Affairs and the Director of the
National Intelligence Coordinating Agency as the other Two requisites must concur for a person to be considered
members. Although the agency was deactivated by virtue as such: (1) a person must be a Filipino citizen from birth
of President Corazon C. Aquino’s Memorandum of March and (2) he does not have to perform any act to obtain or
27, 1987, it was not, however, abrogated. The perfect his Philippine citizenship.
Committee was reactivated on June 8, 1995. Hence, the
application should be filed with said Agency, not with the Under the 1973 Constitution definition, there were two
Regional Trial Court. (Gerardo Angat v. Republic, categories of Filipino citizens which were not considered
G.R. No. 132244, Sept. 14, 1999 [Vitug]) natural-born: (1) those who were naturalized and (2)
those born before January 17, 1973 (the date of
effectivity of the 1973 Constitution), of Filipino mothers individual’s volition. (Mercado v. Manzano, 307 SCRA
who, upon reaching the age of majority, elected Philippine 630, May 26, 1999, En Banc [Mendoza])
citizenship. Those “naturalized citizens” were not
considered natural-born obviously because they were not 30. What is the main concern of Section 5, Article IV,
Filipinos at birth and had to perform an act to acquire 1987 Constitution, on citizenship? Consequently, are
Philippine citizenship. Those born of Filipino mothers persons with mere dual citizenship disqualified to run for
before the effectivity of the 1973 Constitution were elective local positions under Section 40(d) of the Local
likewise not considered natural-born because they also Government Code?
had to perform an act to perfect their Philippine
citizenship. Held: In including Section 5 in Article IV on citizenship,
the concern of the Constitutional Commission was not
The present Constitution, however, now considers those with dual citizens per se but with naturalized citizens who
born of Filipino mothers before the effectivity of the 1973 maintain their allegiance to their countries of origin even
Constitution and who elected Philippine citizenship upon after their naturalization. Hence, the phrase “dual
reaching the majority age as natural-born. After defining citizenship” in R.A. No. 7160, Section 40(d) (Local
who are natural-born citizens, Section 2 of Article IV adds Government Code) must be understood as referring to
a sentence: “Those who elect Philippine citizenship in “dual allegiance.” Consequently, persons with mere dual
accordance with paragraph (3), Section 1 hereof shall be citizenship do not fall under this disqualification. Unlike
deemed natural-born citizens.” Consequently, only those with dual allegiance, who must, x x x, be subject to
naturalized Filipinos are considered not natural-born strict process with respect to the termination of their
citizens. It is apparent from the enumeration of who are status, for candidates with dual citizenship, it should
citizens under the present Constitution that there are only suffice if, upon the filing of their certificate of candidacy,
two classes of citizens: (1) those who are natural-born they elect Philippine citizenship to terminate their status
and (2) those who are naturalized in accordance with law. as persons with dual citizenship considering that their
A citizen who is not a naturalized Filipino, i.e., did not condition is the unavoidable consequence of conflicting
have to undergo the process of naturalization to obtain laws of different states.
Philippine citizenship, necessarily is a natural-born
Filipino. Noteworthy is the absence in the said By electing Philippine citizenship, such candidates at the
enumeration of a separate category for persons who, same time forswear allegiance to the other country of
after losing Philippine citizenship, subsequently reacquire which they are also citizens and thereby terminate their
it. The reason therefore is clear: as to such persons, they status as dual citizens. It may be that, from the point of
would either be natural-born or naturalized depending on view of the foreign state and of its laws, such an individual
the reasons for the loss of their citizenship and the mode has not effectively renounced his foreign citizenship. That
prescribed by the applicable law for the reacquisition is of no moment. (Mercado v. Manzano, G.R. No.
thereof. As respondent Cruz was not required by law to 135083, 307 SCRA 630, May 26, 1999 [Mendoza])
go through naturalization proceedings in order to
reacquire his citizenship, he is perforce a natural-born 31. Cite instances when a citizen of the Philippines
Filipino. As such, he possessed all the necessary may possess dual citizenship considering the citizenship
qualifications to be elected as member of the House of clause (Article IV) of the Constitution.
Representatives. (Antonio Bengson III v. HRET, G.R.
No. 142840, May 7, 2001, En Banc [Kapunan]) Held:

29. Distinguish dual citizenship from dual allegiance. 1) Those born of Filipino fathers and/or mothers
in foreign countries which follow the principle of jus soli;
Held: Dual citizenship arises when, as a result of the 2) Those born in the Philippines of Filipino
concurrent application of the different laws of two or more mothers and alien fathers if by the laws of their father’s
states, a person is simultaneously considered a national country such children are citizens of that country;
by the said states. For instance, such a situation may 3) Those who marry aliens if by the laws of the
arise when a person whose parents are citizens of a state latter’s country the former are considered citizens, unless
which adheres to the principle of jus sanguinis is born in by their act or omission they are deemed to have
a state which follows the doctrine of jus soli. Such a renounced Philippine citizenship.
person, ipso facto and without any voluntary act on his (Mercado v. Manzano, G.R. No. 135083, 307 SCRA
part, is concurrently considered a citizen of both states. 630, May 26, 1999 [Mendoza])

Dual allegiance, on the other hand, refers to a situation 32. Does res judicata apply in cases hinging on the issue
in which a person simultaneously owes, by some positive of citizenship?
act, loyalty to two or more states. While dual citizenship
is involuntary, dual allegiance is the result of an Held: Petitioner maintains further that when citizenship
is raised as an issue in judicial or administrative
proceedings, the resolution or decision thereon is manage the deployment of the Marines. It is, likewise,
generally not considered res judicata in any subsequent their duty to provide the necessary equipment to the
proceeding challenging the same; citing the case of Moy Marines and render logistical support to these soldiers. In
Ya Lim Yao v. Commissioner of Immigration (41 SCRA 292 view of the foregoing, it cannot be properly argued that
[1971]). He insists that the same issue of citizenship may military authority is supreme over civilian authority.
be threshed out anew.
Moreover, the deployment of the Marines to assist the
Petitioner is correct insofar as the general rule is PNP does not unmake the civilian character of the police
concerned, i.e., the principle of res judicata generally force. Neither does it amount to an “insidious incursion”
does not apply in cases hinging on the issue of citizenship. of the military in the task of law enforcement in violation
However, in the case of Burca v. Republic (51 SCRA 248 of Section 5[4], Article XVI of the Constitution.
[1973]), an exception to this general rule was recognized.
The Court ruled in that case that in order that the doctrine In this regard, it is not correct to say that General Angelo
of res judicata may be applied in cases of citizenship, the Reyes, Chief of Staff of the AFP, by his alleged
following must be present: involvement in civilian law enforcement, has been
virtually appointed to a civilian post in derogation of the
1) a person’s citizenship be raised as a material aforecited provision. The real authority in these
issue in a controversy where said person is a party; operations, as stated in the LOI, is lodged with the head
2) the Solicitor General or his authorized of a civilian institution, the PNP, and not with the military.
representative took active part in the resolution thereof, Such being the case, it does not matter whether the AFP
and Chief actually participates in the Task Force Tulungan
3) the finding on citizenship is affirmed by this since he does not exercise any authority or control over
Court. the same. Since none of the Marines was incorporated or
enlisted as members of the PNP, there can be no
Although the general rule was set forth in the case of Moy appointment to a civilian position to speak of. Hence, the
Ya Lim Yao, the case did not foreclose the weight of prior deployment of the Marines in the joint visibility patrols
rulings on citizenship. It elucidated that reliance may does not destroy the civilian character of the PNP.
somehow be placed on these antecedent official findings,
though not really binding, to make the effort easier or Considering the above circumstances, the Marines render
simpler. (Valles v. COMELEC, 337 SCRA 543, Aug. 9, nothing more than assistance required in conducting the
2000, En Banc [Purisima]) patrols. As such, there can be no “insidious incursion” of
the military in civilian affairs nor can there be a violation
of the civilian supremacy clause in the Constitution.
Civilian Supremacy Clause
It is worth mentioning that military assistance to civilian
33. The President issued Letter of Instruction (LOI) authorities in various forms persists in Philippine
ordering the deployment of members of the Philippine jurisdiction. The Philippine experience reveals that it is
Marines in the metropolis to conduct joint visibility patrols not averse to requesting the assistance of the military in
with members of the Philippine National Police in various the implementation and execution of certain traditionally
shopping malls. Will this not violate the civilian “civil” functions. x x x [S]ome of the multifarious activities
supremacy clause under Section 3, Article II of the wherein military aid has been rendered, exemplifying the
Constitution? Does this not amount to an "insidious activities that bring both the civilian and the military
incursion" of the military in the task of law enforcement together in a relationship of cooperation, are:
in violation of Section 5(4), Article XVI of the Constitution?
1. Elections;
Held: The deployment of the Marines does not constitute 2. Administration of the Philippine National
a breach of the civilian supremacy clause. The calling of Red Cross;
the marines in this case constitutes permissible use of 3. Relief and rescue operations during
military assets for civilian law enforcement. x x x The calamities and disasters;
limited participation of the Marines is evident in the 4. Amateur sports promotion and
provisions of the LOI itself, which sufficiently provides the development;
metes and bounds of the Marines' authority. It is 5. Development of the culture and the arts;
noteworthy that the local police forces are the ones in 6. Conservation of natural resources;
charge of the visibility patrols at all times, the real 7. Implementation of the agrarian reform
authority belonging to the PNP. In fact, the Metro Manila program;
Police Chief is the overall leader of the PNP-Philippine 8. Enforcement of customs laws;
Marines joint visibility patrols. Under the LOI, the police 9. Composite civilian-military law
forces are tasked to brief or orient the soldiers on police enforcement activities;
patrol procedures. It is their responsibility to direct and 10. Conduct of licensure examinations;
11. Conduct of nationwide tests for earth incapable of sustaining life. (Oposa v. Factoran,
elementary and high school students; Jr., 224 SCRA 792 [1993][Davide])
12. Anti-drug enforcement activities;
13. Sanitary inspections; 35. The Province of Palawan and the City of Puerto
14. Conduct of census work; Princesa enacted ordinances prohibiting the catching
15. Administration of the Civil Aeronautics and/or exportation of live tropical fishes, and imposing
Board; penalties for violations thereof, in order to stop the illegal
16. Assistance in installation of weather practice of cyanide fishing which destroys the corals and
forecasting devices; other marine resources. Several fishermen apprehended
17. Peace and order policy formulation in for violating the ordinances in question challenged their
local government units. constitutionality contending that the ordinances violated
their preferential right as subsistence and marginal
This unquestionably constitutes a gloss on executive fishermen to the use of our communal marine resources
power resulting from a systematic, unbroken, executive guaranteed by the Constitution, under Section 7, Article
practice, long pursued to the knowledge of Congress and, XIII. Will you sustain the challenge?
yet, never before questioned. What we have here is
mutual support and cooperation between the military and Held: The “preferential right” of subsistence or marginal
civilian authorities, not derogation of civilian supremacy. fishermen to the use of marine resources is not absolute.
In accordance with the Regalian Doctrine, marine
In the United States, where a long tradition of suspicion resources belong to the State, and, pursuant to the first
and hostility towards the use of military force for domestic paragraph of Section 2, Article XII of the Constitution,
purposes has persisted and whose Constitution, unlike their “exploration, development and utilization x x x shall
ours, does not expressly provide for the power to call, the be under the full control and supervision of the State.”
use of military personnel by civilian law enforcement Moreover, their mandated protection, development and
officers is allowed under circumstances similar to those conservation x x x imply certain restrictions on whatever
surrounding the present deployment of the Philippine right of enjoyment there may be in favor of anyone. What
Marines. (IBP v. Hon. Ronaldo B. Zamora, G.R. No. must be borne in mind is the State policy enshrined in the
141284, Aug. 15, 2000, En Banc [Kapunan]) Constitution regarding the duty of the State to protect and
advance the right of the people to a balanced and
healthful ecology in accord with the rhythm and harmony
The Right to a Balanced and Healthful Ecology of nature (Section 16, Article II). The ordinances in
question are meant precisely to protect and conserve our
34. Is the right to a balanced and healthful ecology any marine resources to the end that their enjoyment may be
less important than any of the civil and political rights guaranteed not only for the present generation, but also
enumerated in the Bill of Rights? Explain. for the generations to come. The right to a balanced and
healthful ecology carries with it a correlative duty to
Held: While the right to a balanced and healthful ecology refrain from impairing the environment. (Tano v. Gov.
is to be found under the Declaration of Principles and Salvador P. Socrates, G.R. No. 110249, Aug. 21,
State Policies and not under the Bill of Rights, it does not 1997)
follow that it is less important than any of the civil and
political rights enumerated in the latter. Such a right
belongs to a different category of rights altogether for it Academic Freedom
concerns nothing less than self-preservation and self-
perpetuation, the advancement of which may even be 36. How should the State’s power to regulate
said to predate all governments and constitutions. As a educational institutions be exercised?
matter of fact, these basic rights need not even be written
in the Constitution for they are assumed to exist from the Held: Section 4[1], Article XIV of the
inception of humankind. If they are now explicitly Constitution recognizes the State’s power to regulate
mentioned in the fundamental charter, it is because of the educational institutions:
well-founded fear of its framers that unless the rights to
a balanced and healthful ecology and to health are The State recognizes the complementary roles of public
mandated as state policies by the Constitution itself, and private institutions in the educational system and
thereby highlighting their continuing importance and shall exercise reasonable supervision and regulation of all
imposing upon the state a solemn obligation to preserve educational institutions.
the first and protect and advance the second, the day
would not be too far when all else would be lost not only As may be gleaned from the above provision,
for the present generation, but also for those to come – such power to regulate is subject to the requirement of
generations which stand to inherit nothing but parched reasonableness. Moreover, the Constitution allows
merely the regulation and supervision of educational
institutions, not the deprivation of their rights. (Miriam freedom” encompasses the freedom to determine for
College Foundation, Inc. v. Court of Appeals, 348 itself on academic grounds:
SCRA 265, 288, Dec. 15, 2000, 1st Div. [Kapunan])
(1) Who may teach,
(2) What may be taught,
37. Discuss the academic freedom of institutions of (3) How it shall be taught, and
higher learning. (4) Who may be admitted to study. (Isabelo, Jr.
v. Perpetual Help College of Rizal, Inc., 227 SCRA 591,
Held: 1. Equally mandated by Article XIV, 595 [1993]; Ateneo de Manila University v. Capulong, 222
Section 5[2] of the 1987 Constitution is that academic SCRA 643, 660 [1993]; Garcia v. The Faculty Admission
freedom shall be enjoyed in all institutions of higher Committee, Loyola School of Theology, 68 SCRA 277, 285
learning. Academic freedom of educational institutions [1975]. The above formulation was made by Justice Felix
has been defined as the right of the school or college to Frankfurter in his concurring opinion in Sweezy v. New
decide for itself, its aims and objectives, and how best to Hampshire, 354 U.S. 234, 263)
attain them - free from outside coercion or interference
save possibly when the overriding public welfare calls for The right of the school to discipline its students is at once
some restraint. It has a wide sphere of autonomy apparent in the third freedom, i.e., “how it shall be
certainly extending to the choice of students. Said taught.” A school certainly cannot function in an
constitutional provision is not to be construed in a atmosphere of anarchy.
niggardly manner or in a grudging fashion. That would
be to frustrate its purpose and nullify its intent (Garcia v. Thus, there can be no doubt that the establishment of an
The Faculty Admission Committee, et al., supra; educational institution requires rules and regulations
Tangonan v. Pano, et al., supra.) necessary for the maintenance of an orderly educational
program and the creation of an educational environment
While it is true that an institution of learning has conducive to learning. Such rules and regulations are
a contractual obligation to afford its students a fair equally necessary for the protection of the students,
opportunity to complete the course they seek to pursue faculty, and property (Angeles v. Sison, 112 SCRA 26, 37
(Licup, et al. v. University of San Carlos [USC], et al., [1982]).
supra.), since a contract creates reciprocal rights and
obligations, the obligation of the school to educate a Moreover, the school has an interest in teaching
student would imply a corresponding obligation on the the student discipline, a necessary, if not indispensable,
part of the student to study and obey the rules and value in any field of learning. By instilling discipline, the
regulations of the school (Capitol Medical Center, Inc., et school teaches discipline. Accordingly, the right to
al. v. Court of Appeals, et al., supra.). When a student discipline the student likewise finds basis in the freedom
commits a serious breach of discipline or failed to “what to teach.”
maintain the required academic standard, he forfeits his
contractual right. In this connection, this Court Incidentally, the school not only has the right but
recognizes the expertise of educational institutions in the the duty to develop discipline in its students. The
various fields of learning. Thus, they are afforded ample Constitution no less imposes such duty.
discretion to formulate reasonable rules and regulations
in the admission of students (Yap Chin Fah, et al. v. Court [All educational institutions] shall inculcate patriotism and
of Appeals, et al., G.R. No. 90063, December 12, 1989), nationalism, foster love of humanity, respect for human
including setting of academic standards. Within the rights, appreciation of the role of national heroes in the
parameters thereof, they are competent to determine historical development of the country, teach the rights
who are entitled to admission and re-admission. and duties of citizenship, strengthen ethical and spiritual
(University of San Agustin, Inc. v. Court of values, develop moral character and personal discipline,
Appeals, 230 SCRA 761, 774-775, March 7, 1994 encourage critical and creative thinking, broaden scientific
[Nocon]) and technological knowledge, and promote vocational
efficiency (Section 3[2], Article XIV, Constitution).
2. Section 5[2], Article XIV of the Constitution
guarantees all institutions of higher learning academic In Angeles v. Sison, we also said that discipline was a
freedom. This institutional academic freedom includes means for the school to carry out its responsibility to help
the right of the school or college to decide for itself, its its students “grow and develop into mature, responsible,
aims and objectives, and how best to attain them free effective and worthy citizens of the community.” (Supra,
from outside coercion or interference save possibly when at 37)
the overriding public welfare calls for some restraint
(Tangonan v. Pano, 137 SCRA 245, 256-257 [1985]). The Finally, nowhere in the above formulation is the
essential freedoms subsumed in the term “academic right to discipline more evident than in “who may be
admitted to study.” If a school has the freedom to
determine whom to admit, logic dictates that it also has freedom” encompassing not only “the freedom to
the right to determine whom to exclude or expel, as well determine x x x on academic grounds who may teach,
as upon whom to impose lesser sanctions such as what may be taught (and) how it shall be taught,” but
suspension and the withholding of graduation privileges. likewise “who may be admitted to study.” We have thus
sanctioned its invocation by a school in rejecting students
Thus, in Ateneo de Manila v. Capulong (222 SCRA who are academically delinquent (Tangonan v. Pano, 137
643 [1993]), the Court upheld the expulsion of students SCRA 245 [1985]), or a laywoman seeking admission to a
found guilty of hazing by petitioner therein, holding that: seminary (Garcia v. Loyola School of Theology, 68 SCRA
277 [1975]), or students violating “School Rules on
No one can be so myopic as to doubt that the immediate Discipline.” (Ateneo de Manila University v. Capulong,
reinstatement of respondent students who have been supra.) (Isabelo, Jr. v. Perpetual Help College of
investigated and found guilty by the Disciplinary Board to Rizal, Inc., 227 SCRA 595-597, Nov. 8, 1993, En
have violated petitioner university’s disciplinary rules and Banc [Vitug])
standards will certainly undermine the authority of the
administration of the school. This we would be most 40. Between the COA’s findings and conclusions and
loathe to do. that of private auditors, which should prevail?

More importantly, it will seriously impair petitioner Held: Moreover, as the constitutionally-mandated
university’s academic freedom which has been enshrined auditor of all government agencies, the COA’s findings
in the 1935, 1973 and the present 1987 Constitution (Id., and conclusions necessarily prevail over those of private
at 659-660). auditors, at least insofar as government agencies and
(Miriam College Foundation, Inc. v. Court of officials are concerned. The superiority or
Appeals, 348 SCRA 265, Dec. 15, 2000, 1st Div. preponderance of the COA audit over private audit can
[Kapunan]) be gleaned from the records of the Constitutional
Commission x x x. The findings and conclusions of the
38. May a university validly revoke a degree or honor private auditor may guide private investors or creditors
it has conferred to a student after the graduation of the who require such private audit. Government agencies
latter after finding that such degree or honor was and officials, however, remain bound by the findings and
obtained through fraud? conclusions of the COA, whether the matter falls under
the first or second paragraph of Section 2, unless of
Held: In Garcia v. Faculty Admission Committee, Loyola course such findings and conclusions are modified or
School of Theology (68 SCRA 277 [1975]), the SC pointed reversed by the courts.
out that academic freedom of institutions of higher
learning is a freedom granted to “institutions of higher The power of the COA to examine and audit government
learning” which is thus given a “wide sphere of authority agencies, while non-exclusive, cannot be taken away
certainly extending to the choice of students.” If such from the COA. Section 3, Article IX-C of the Constitution
institution of higher learning can decide who can and who mandates that:
cannot study in it, it certainly can also determine on whom
it can confer the honor and distinction of being its “Sec. 3. No law shall be passed exempting any entity of
graduates. the Government or its subsidiary in any guise
whatsoever, or any investment of public funds, from the
Where it is shown that the conferment of an honor or jurisdiction of the Commission on Audit.”
distinction was obtained through fraud, a university has
the right to revoke or withdraw the honor or distinction it The mere fact that private auditors may audit
has thus conferred. This freedom of a university does not government agencies does not divest the COA of its
terminate upon the “graduation” of a student, for it is power to examine and audit the same government
precisely the “graduation” of such a student that is in agencies. The COA is neither by-passed nor ignored
question. (UP Board of Regents v. Hon. Court of since even with a private audit the COA will still conduct
Appeals and Arokiaswamy William Margaret its usual examination and audit, and its findings and
Celine, G.R. No. 134625, Aug. 31, 1999, 2nd Div. conclusions will still bind government agencies and their
[Mendoza]) officials. A concurrent private audit poses no danger
whatsoever of public funds or assets escaping the usual
39. What are the essential freedoms subsumed in the scrutiny of a COA audit. (Development Bank of the
term “academic freedom”? Philippines v. Commission on Audit, 373 SCRA
356, January 16, 2002, En Banc [Carpio])
Held: In Ateneo de Manila University v. Capulong (G.R.
No. 99327, 27 May 1993), this Court cited with approval 41. Is the constitutional power of the COA to
the formulation made by Justice Felix Frankfurter of the examine and audit government banks and agencies
essential freedoms subsumed in the term “academic
exclusive? Does it preclude a concurrent audit by a provisions of Section 2 in the 1935 and 1973
private external auditor? Constitutions. There is no dispute that the COA’s
authority under the second paragraph of Section 2 is
Held: The resolution of the primordial issue of whether exclusive as the language of the Constitution admits of
or not the COA has the sole and exclusive power to no other meaning. Thus, the COA has the exclusive
examine and audit government banks involves an authority to decide on disallowances of unnecessary
interpretation of Section 2, Article IX-D of the 1987 government expenditures. Other government agencies
Constitution. This Section provides as follows: and their officials, as well as private auditors engaged by
them, cannot in any way intrude into this exclusive
“Sec. 2. (1) The Commission on Audit shall have the function of the COA.
power, authority, and duty to examine, audit, and settle
all accounts pertaining to the revenue and receipts of, The qualifying word “exclusive” in the second paragraph
and expenditures or uses of funds and property, owned of Section 2 cannot be applied to the first paragraph
and held in trust by, or pertaining to, the Government, which is another sub-section of Section 2. A qualifying
or any of its subdivisions, agencies, or instrumentalities, word is intended to refer only to the phrase to which it is
including government-owned or controlled corporations immediately associated, and not to a phrase distantly
with original charters, x x x. located in another paragraph or sub-section (Felipe v.
De la Cruz, 99 Phil. 940 [1956]; Tirona v. Cudiamat, 14
“(2) The Commission shall have the exclusive authority, SCRA 264 [1965]). Thus, the first paragraph of Section
subject to the limitations in this Article, to define the 2 must be read the way it appears, without the word
scope of its audit and examination, establish the “exclusive,” signifying that non-COA auditors can also
techniques and methods required therefore, and examine and audit government agencies. Besides, the
promulgate accounting and auditing rules and framers of the Constitution intentionally omitted the
regulations, including those for the prevention and word “exclusive” in the first paragraph of Section 2
disallowance of irregular, unnecessary, excessive, precisely to allow concurrent audit by private external
extravagant, or unconscionable expenditures, or uses of auditors.
government funds and properties.” (Emphasis supplied)
The clear and unmistakable conclusion from a reading of
The COA vigorously asserts that under the first the entire Section 2 is that the COA’s power to examine
paragraph of Section 2, the COA enjoys the sole and and audit is non-exclusive. On the other hand, the
exclusive power to examine and audit all government COA’s authority to define the scope of its audit,
agencies, including the DBP. The COA contends this is promulgate auditing rules and regulations, and disallow
similar to its sole and exclusive authority, under the unnecessary expenditures is exclusive.
same paragraph of the same section, to define the
scope of its audit, promulgate auditing rules and Xxx
regulations, including rules on the disallowance of
unnecessary expenditures of government agencies. The Manifestly, the express language of the
bare language of Section 2, however, shows that the Constitution, and the clear intent of its framers, point to
COA’s power under the first paragraph is not declared only one indubitable conclusion – the COA does not have
exclusive, while its authority under the second the exclusive power to examine and audit government
paragraph is expressly declared “exclusive.” There is a agencies. The framers of the Constitution were fully
significant reason for this marked difference in language. aware of the need to allow independent private audit of
certain government agencies in addition to the COA
During the deliberations of the Constitutional audit, as when there is a private investment in a
Commission, Commissioner Serafin Guingona proposed government-controlled corporation, or when a
the addition of the word “exclusive” in the first government corporation is privatized or publicly listed, or
paragraph of Section 2, thereby granting the COA the as in the case at bar when the government borrows
sole and exclusive power to examine and audit all money from abroad.
government agencies. However, the Constitutional
Commission rejected the addition of the word In these instances the government enters the
“exclusive” in the first paragraph of Section 2 and marketplace and competes with the rest of the world in
Guingona was forced to withdraw his proposal. X x x. attracting investments or loans. To succeed, the
government must abide with the reasonable business
Xxx practices of the marketplace. Otherwise no investor or
creditor will do business with the government,
In sharp contrast, the Constitutional Commission placed frustrating government efforts to attract investments or
the word “exclusive” to qualify the authority of the COA secure loans that may be critical to stimulate moribund
under the second paragraph of the same Section 2. This industries or resuscitate a badly shattered national
word “exclusive” did not appear in the counterpart economy as in the case at bar. By design the
Constitution is flexible enough to meet these exigencies.
Any attempt to nullify this flexibility in the instances Economic Policy
mentioned, or in similar instances, will be ultra vires, in
the absence of a statute limiting or removing such 42. Does the Constitutional policy of a “self-reliant
flexibility. and independent national economy” rule out foreign
competition?
The deliberations of the Constitutional Commission
reveal eloquently the intent of Section 2, Article IX-D of Held: The constitutional policy of a “self-reliant and
the Constitution. As this Court has ruled repeatedly, the independent national economy” does not necessarily rule
intent of the law is the controlling factor in the out the entry of foreign investments, goods and services.
interpretation of the law (People v. Purisima, 86 SCRA It contemplates neither “economic seclusion” nor
542 [1978]; others omitted). If a law needs “mendicancy in the international community.”
interpretation, the most dominant influence is the intent
of the law (De Jesus v. City of Manila, 29 Phil. 73 Aside from envisioning a trade policy based on “equality
[1914]). The intent of the law is that which is expressed and reciprocity,” the fundamental law encourages
in the words of the law, which should be discovered industries that are “competitive in both domestic and
within its four corners aided, if necessary, by its foreign markets,” thereby demonstrating a clear policy
legislative history (Manila Lodge No. 761 v. Court of against a sheltered domestic trade environment, but one
Appeals, 73 SCRA 162 [1976]). In the case of Section 2, in favor of the gradual development of robust industries
Article IX-D of the Constitution, the intent of the framers that can compete with the best in the foreign markets.
of the Constitution is evident from the bare language of (Tanada v. Angara, 272 SCRA 18 [1997])
Section 2 itself. The deliberations of the Constitutional
Commission confirm expressly and even elucidate
further this intent beyond any doubt whatsoever. 43. Is PHILSECO (Philippine Shipyard and
Engineering Corporation), as a shipyard, a public utility
There is another constitutional barrier to the and, hence, could be operated only by a corporation at
COA’s insistence of exclusive power to examine and least 60% of whose capital is owned by Filipino citizens in
audit all government agencies. The COA’s claim clashes accordance with Article XII, Section 10 of the
directly with the Central Bank’s constitutional power of Constitution?
“supervision” over banks under Section 20, Article XII of
the Constitution. X x x Held: Petitioner asserts that a shipyard is a public
utility pursuant to Section 13 (b) of Commonwealth Act
Historically, the Central Bank has been No. 146. Respondents, on the other hand, contend that
conducting periodic and special examination and audit of shipyards are no longer public utilities by express
banks to determine the soundness of their operations provision of Presidential Decree No. 666, which provided
and the safety of the deposits of the public. Undeniably, incentives to the shipbuilding and ship repair industry.
the Central Bank’s power of “supervision” includes the
power to examine and audit banks, as the banking laws Indeed, P.D. No. 666 dated March 5, 1975
have always recognized this power of the Central Bank. explicitly stated that a “shipyard” was not a “public utility.”
Hence, the COA’s power to examine and audit xxx
government banks must be reconciled with the Central
Bank’s power to supervise the same banks. The However, Section 1 of P.D. No. 666 was expressly
inevitable conclusion is that the COA and the Central repealed by Section 20 of Batas Pambansa Blg. 391, the
Bank have concurrent jurisdiction, under the Investment Incentive Policy Act of 1983. Subsequently,
Constitution, to examine and audit government banks. Executive Order No. 226, the Omnibus Investments Code
of 1987, was issued and Section 85 thereof expressly
However, despite the Central Bank’s concurrent repealed B.P. Blg. 391.
jurisdiction over government banks, the COA’s audit still
prevails over that of the Central Bank since the COA is The express repeal of B.P. Blg. 391 by E.O. No.
the constitutionally mandated auditor of government 226 did not revive Section 1 of P.D. No. 666, declassifying
banks. And in matters falling under the second the shipbuilding and ship repair industry as a public utility,
paragraph of Section 2, Article IX-D of the Constitution, as said executive order did not provide otherwise. When
the COA’s jurisdiction is exclusive. Thus, the Central a law which expressly repeals a prior law is itself repealed,
Bank is devoid of authority to allow or disallow the law first repealed shall not be thereby revived unless
expenditures of government banks since this function expressly so provided (Administrative Code of 1987, Book
belongs exclusively to the COA. (Development Bank I, Chapter 5, Section 21). Consequently, when the APT
of the Philippines v. Commission on Audit, 373 [Asset Privatization Trust] drafted the ASBR [Asset
SCRA 356, January 16, 2002, En Banc [Carpio]) Specific Bidding Rules] sometime in 1993, P.D. No. 666
no longer existed in our statute books. While it is true
that the repeal of a statute does not operate to impair The Rights of Indigenous Cultural
rights that have become vested or accrued while the Communities/Indigenous Peoples
statute was in force, there are no vested rights of the
parties that should be protected in the case at bar. The 44. Does R.A. 8371, otherwise known as “the
reason is simple: said decree was already inexistent when Indigenous People’s Rights Act” infringe upon the State’s
the ASBR was issued. ownership over the natural resources within the ancestral
domains?
A shipyard such as PHILSECO being a public
utility as provided by law, the following provision of the Held: Petitioners posit that IPRA deprives the State of its
Article XII of the Constitution applies: ownership over mineral lands of the public domain and
other natural resources, as well as the State’s full control
“Sec. 11. No franchise, certificate, or any other form of and supervision over the exploration, development and
authorization for the operation of a public utility shall be utilization of natural resources. Specifically, petitioners
granted except to citizens of the Philippines or to and the Solicitor General assail Sections 3[a], 5, and 7 of
corporations or associations organized under the laws of IPRA as violative of Section 2, Article XII of the
the Philippines at least sixty per centum of whose capital Constitution which states, in part, that “[a]ll lands of the
is owned by such citizens, nor shall such franchise, public domain, waters, minerals, coal, petroleum, and
certificate, or authorization be exclusive in character or other mineral oils, all forces of potential energy, fisheries,
for a longer period than fifty years. Neither shall any such forests or timber, wildlife, flora and fauna, and other
franchise or right be granted except under the condition natural resources are owned by the State.” (Section 2,
that it shall be subject to amendment, alteration, or repeal Article XII, Constitution) They would have the Court
by the Congress when the common good so requires. The declare as unconstitutional Section 3[a] of IPRA because
State shall encourage equity participation in public utilities the inclusion of natural resources in the definition of
by the general public. The participation of foreign ancestral domains purportedly results in the abdication of
investors in the governing body of any public utility State ownership over these resources.
enterprise shall be limited to their proportionate share in
its capital, and all the executive and managing officers of Xxx
such corporation or association shall be citizens of the
Philippines.” Section 3[a] merely defines the coverage of ancestral
domains, and describes the extent, limit and composition
The progenitor of this constitutional provision, of ancestral domains by setting forth the standards and
Article XIV, Section 5 of the 1973 Constitution, required guidelines in determining whether a particular area is to
the same proportion of 60%-40% capitalization. The JVA be considered as part of and within the ancestral
[Joint Venture Agreement] between NIDC [National domains. In other words, Section 3[a] serves only as a
Investment and Development Corporation] and Kawasaki yardstick which points out what properties are within the
[Kawasaki Heavy Industries, Ltd. of Kobe, Japan] entered ancestral domains. It does not confer or recognize any
into on January 27, 1977 manifests the intention of the right of ownership over the natural resources to the
parties to abide by the constitutional mandate on indigenous peoples. Its purpose is definitional and not
capitalization of public utilities. x x x declarative of a right or title.

A joint venture is an association of persons or companies The specification of what areas belong to the
jointly undertaking some commercial enterprise with all ancestral domains is, to our mind, important to ensure
of them generally contributing assets and sharing risks. that no unnecessary encroachment on private properties
x x x. Considered more of a partnership (Aurbach v. outside the ancestral domains will result during the
Sanitary Wares Manufacturing Corporation, G.R. No. delineation process. The mere fact that Section 3[a]
75875, 180 SCRA 130, 147 [1989]), a joint venture is defines ancestral domains to include the natural resources
governed by the laws on contracts and on partnership. found therein does not ipso facto convert the character of
The joint venture created between NIDC and Kawasaki such natural resources as private property of the
falls within the purview of an “association” pursuant to indigenous peoples. Similarly, Section 5 in relation to
Section 5 of Article XIV of the 1973 Constitution and Section 3[a] cannot be construed as a source of
Section 11 of Article XII of the 1987 Constitution. ownership rights of indigenous people over the natural
Consequently, a joint venture that would engage in the resources simply because it recognizes ancestral domains
business of operating a public utility, such as a shipyard, as their “private but community property.”
must observe the proportion of 60%-40% Filipino-
foreign capitalization. (JG Summit Holdings, Inc. v. The phrase “private but community property” is
Court of Appeals, 345 SCRA 143, Nov. 20, 2000, merely descriptive of the indigenous peoples’ concept of
1st Div. [Ynares-Santiago]) ownership as distinguished from that provided in the Civil
Code. x x x. In contrast, the indigenous peoples’ concept
of ownership emphasizes the importance of communal or
group ownership. By virtue of the communal character of immemorial possession in the concept of an owner was
ownership, the property held in common “cannot be sold, acknowledged and recognized as far back during the
disposed or destroyed” because it was meant to benefit Spanish colonization of the Philippines, there was no
the whole indigenous community and not merely the similar favorable treatment as regards natural resources.
individual member. The unique value of natural resources has been
acknowledged by the State and is the underlying reason
That IPRA is not intended to bestow ownership for its consistent assertion of ownership and control over
over natural resources to the indigenous peoples is also said natural resources from the Spanish regime up to the
clear from the deliberations of the bicameral conference present. Natural resources, especially minerals, were
committee on Section 7 which recites the rights of considered by Spain as an abundant source of revenue to
indigenous peoples over their ancestral domains x x x. finance its battle in wars against other nations. Hence,
Spain, by asserting its ownership over minerals wherever
Further, Section 7 makes no mention of any right these may be found, whether in public or private lands,
of ownership of the indigenous peoples over the natural recognized the separability of title over lands and that
resources. In fact, Section 7[a] merely recognizes the over minerals which may be found therein (Noblejas,
“right to claim ownership over lands, bodies of water Philippine Law on Natural Resources 1961 Revised Ed., p.
traditionally and actually occupied by indigenous peoples, 6).
sacred places, traditional hunting and fishing grounds,
and all improvements made by them at any time within On the other hand, the United States viewed
the domains.” Neither does Section 7[b], which natural resources as a source of wealth for its nationals.
enumerates certain rights of the indigenous peoples over As the owner of natural resources over the Philippines
the natural resources found within their ancestral after the latter’s cession from Spain, the United States
domains, contain any recognition of ownership vis-à-vis saw it fit to allow both Filipino and American citizens to
the natural resources. explore and exploit minerals in public lands, and to grant
patents to private mineral lands. x x x. Although the
What is evident is that the IPRA protects the United States made a distinction between minerals found
indigenous peoples’ rights and welfare in relation to the in public lands and those found in private lands, title in
natural resources found within their ancestral domains, these minerals was in all cases sourced from the State.
including the preservation of the ecological balance The framers of the 1935 Constitution found it necessary
therein and the need to ensure that the indigenous to maintain the State’s ownership over natural resources
peoples will not be unduly displaced when the State- to insure their conservation for future generations of
approved activities involving the natural resources located Filipinos, to prevent foreign control of the country through
therein are undertaken. (Separate Opinion, Kapunan, economic domination; and to avoid situations whereby
J., in Cruz v. Secretary of Environment and Natural the Philippines would become a source of international
Resources, 347 SCRA 128, 284-293, Dec. 6, 2000, conflicts, thereby posing danger to its internal security
En Banc [Per Curiam]) and independence.

45. Has the concept of native title to natural The declaration of State ownership and control
resources, like native title to land, been recognized in the over minerals and other natural resources in the 1935
Philippines? Constitution was reiterated in both the 1973 and 1987
Constitutions. (Separate Opinion, Kapunan, J., in
Held: The concept of native title to natural Cruz v. Secretary of Environment and Natural
resources, unlike native title to land, has not been Resources, 347 SCRA 128, 284-293, Dec. 6, 2000,
recognized in the Philippines. NCIP and Flavier, et al. En Banc [Per Curiam])
invoke the case of Reavies v. Fianza (40 Phil. 1017 [1909],
215 US 16, 54 L Ed 72) in support of their thesis that 46. Enumerate the Constitutional provisions
native title to natural resources has been upheld in this recognizing and protecting the rights and interests of the
jurisdiction. They insist that “it is possible for rights over indigenous peoples.
natural resources to vest on a private (as opposed to a
public) holder if these were held prior to the 1935 Held: The framers of the 1987 Constitution, looking back
Constitution.” However, a judicious examination of to the long destitution of our less fortunate brothers,
Reavies reveals that, contrary to the position of NCIP and fittingly saw the historic opportunity to actualize the ideals
Flavier, et al., the Court did not recognize native title to of people empowerment and social justice, and to reach
natural resources. Rather, it merely upheld the right of out particularly to the marginalized sectors of society,
the indigenous peoples to claim ownership of minerals including the indigenous peoples. They incorporated in
under the Philippine Bill of 1902. the fundamental law several provisions recognizing and
protecting the rights and interests of the indigenous
While x x x native title to land or private peoples, to wit:
ownership by Filipinos of land by virtue of time
Section 22. The State recognizes and promotes the rights Cultural Communities/Indigenous Peoples, Creating a
of indigenous peoples within the framework of national National Commission on Indigenous Peoples, Establishing
unity and development. (Article II of the Constitution, Implementing Mechanisms, Appropriating Funds
entitled State Principles and Policies) Therefor, and for Other Purposes." It is simply known as
"The Indigenous Peoples Rights Act of 1997" or the IPRA.
Section 5. The State, subject to the provisions of the
Constitution and national development policies and The IPRA recognizes the existence of the indigenous
programs, shall protect the rights of indigenous cultural cultural communities or indigenous peoples (ICCs/IPs) as
communities to their ancestral lands to ensure their a distinct sector in Philippine society. It grants these
economic, social, and cultural well-being. people the ownership and possession of their ancestral
domains and ancestral lands, and defines the extent of
The Congress may provide for the applicability of these lands and domains. The ownership given is the
customary laws governing property rights and relations in indigenous concept of ownership under customary law
determining the ownership and extent of ancestral which traces its origin to native title.
domains. (Article XII of the Constitution, entitled National
Economy and Patrimony) Xxx

Section 1. The Congress shall give the highest priority to Within their ancestral domains and ancestral lands, the
the enactment of measures that protect and enhance the ICCs/IPs are given the right to self-governance and
right of all the people to human dignity, reduce social, empowerment (Sections 13 to 20), social justice and
economic and political inequalities, and remove cultural human rights (Sections 21 to 28), the right to preserve
inequalities by equitably diffusing wealth and political and protect their culture, traditions, institutions and
power for the common good. community intellectual rights, and the right to develop
their own sciences and technologies (Sections 29 to 37).
To this end, the State shall regulate the acquisition, (Separate Opinion, Puno, J., in Isagani Cruz v.
ownership, use and disposition of property and its Secretary of DENR, et al., G.R. No. 135385, Dec. 6,
increments. (Article XIII of the Constitution, entitled 2000, En Banc)
Social Justice and Human Rights)
48. Define "indigenous peoples/indigenous cultural
Section 6. The State shall apply the principles of agrarian communities."
reform or stewardship, whenever applicable in
accordance with law, in the disposition and utilization of Held: 1. Drawing inspiration from both our fundamental
other natural resources, including lands of the public law and international law, IPRA now employs the
domain under lease or concession, subject to prior rights, politically-correct conjunctive term "indigenous
homestead rights of small settlers, and the rights of peoples/indigenous cultural communities" as follows:
indigenous communities to their ancestral lands. (Ibid.)
Section 3. Definition of Terms. - For purposes of this Act,
Section 17. The State shall recognize, respect, and protect the following terms shall mean:
the rights of cultural communities to preserve and develop
their cultures, traditions, and institutions. It shall consider (i) Indigenous peoples/Indigenous cultural
these rights in the formulation of national plans and communities. - refer to a group of people or homogenous
policies. (Article XIV of the Constitution, entitled societies identified by self-ascription and ascription by
Education, Science, Technology, Arts, Culture, and others, who have continuously lived as organized
Sports) community on communally bounded and defined
territory, and who have, under claims of ownership since
Section 12. The Congress may create a consultative body time immemorial, occupied, possessed and utilized such
to advise the President on policies affecting indigenous territories, sharing common bonds of language, customs,
cultural communities, the majority of the members of traditions, and other distinctive cultural traits, or who
which shall come from such communities. (Article XVI of have, through resistance to political, social and cultural
the Constitution, entitled General Provisions) inroads of colonization, non-indigenous religions and
(Separate Opinion, Kapunan, J., in Isagani Cruz v. cultures, became historically differentiated from the
Secretary of Environment and Natural Resources, majority of Filipinos. Indigenous peoples shall likewise
et al., G.R. No. 135385, Dec. 6, 2000, En Banc) include peoples who are regarded as indigenous on
account of their descent from the populations which
47. Discuss the Indigenous Peoples Rights Act (R.A. inhabited the country at the time of conquest or
No. 8371). colonization, or at the time of inroads of non-indigenous
religions and cultures, or the establishment of present
Held: Republic Act No. 8371 is entitled "An Act to State boundaries, who retain some or all of their own
Recognize, Protect and Promote the Rights of Indigenous social, economic, cultural and political institutions, but
who may have been displaced from their traditional communally or individually since time immemorial,
domains or who may have resettled outside their continuously until the present, except when interrupted
ancestral domains x x x. by war, force majeure or displacement by force, deceit,
(Separate Opinion, Kapunan, J., in Isagani Cruz stealth or as a consequence of government projects or
v. Secretary of Environment and Natural any other voluntary dealings with government and/or
Resources, et al., G.R. No. 135385, Dec. 6, 2000, private individuals or corporations. Ancestral domains
En Banc) comprise lands, inland waters, coastal areas, and natural
resources therein and includes ancestral lands, forests,
2. The IPRA is a law dealing with a specific group of pasture, residential, agricultural, and other lands
people, i.e., the Indigenous Cultural Communities (ICCs) individually owned whether alienable or not, hunting
or the Indigenous Peoples (IPs). The term "ICCs" is used grounds, burial grounds, worship areas, bodies of water,
in the 1987 Constitution while that of "IPs" is the mineral and other natural resources. They also include
contemporary international language in the International lands which may no longer be exclusively occupied by
Labor Organization (ILO) Convention 169 (Convention ICCs/IPs but from which they traditionally had access to
Concerning Indigenous and Tribal Peoples in Independent for their subsistence and traditional activities, particularly
Countries, June 27, 1989) and the United Nations (UN) the home ranges of ICCs/IPs who are still nomadic and/or
Draft Declaration on the Rights of Indigenous Peoples shifting cultivators (Section 3[a], IPRA).
(Guide to R.A. 8371, published by the Coalition for IPs
Rights and Ancestral Domains in cooperation with the ILO Ancestral lands are lands held by the ICCs/IPs under the
and Bilance-Asia Department, p. 4 [1999] - hereinafter same conditions as ancestral domains except that these
referred to as Guide to R.A. 8371). are limited to lands and that these lands are not merely
occupied and possessed but are also utilized by the
Indigenous Cultural Communities or Indigenous Peoples ICCs/IPs under claims of individual or traditional group
refer to a group of people or homogeneous societies who ownership. These lands include but are not limited to
have continuously lived as an organized community on residential lots, rice terraces or paddies, private forests,
communally bounded and defined territory. These groups swidden farms and tree lots (Section 3[b], IPRA).
of people have actually occupied, possessed and utilized (Separate Opinion, Puno, J., in Isagani Cruz v.
their territories under claim of ownership since time Secretary of DENR, et al., G.R. No. 135385, Dec. 6,
immemorial. They share common bonds of language, 2000, En Banc)
customs, traditions and other distinctive cultural traits, or,
they, by their resistance to political, social and cultural 50. How may ICCs/IPs acquire rights to their
inroads of colonization, non-indigenous religions and ancestral domains and ancestral lands?
cultures, became historically differentiated from the
Filipino majority. ICCs/IPs also include descendants of Held: The rights of the ICCs/IPs to their ancestral
ICCs/IPs who inhabited the country at the time of domains and ancestral lands may be acquired in two
conquest or colonization, who retain some or all of their modes: (1) by native title over both ancestral lands and
own social, economic, cultural and political institutions but domains; or (2) by torrens title under the Public Land Act
who may have been displaced from their traditional and the Land Registration Act with respect to ancestral
territories or who may have resettled outside their lands only. (Separate Opinion, Puno, J., in Isagani
ancestral domains. (Separate Opinion, Puno, J., in Cruz v. Secretary of DENR, et al., G.R. No. 135385,
Isagani Cruz v. Secretary of DENR, et al., G.R. No. Dec. 6, 2000, En Banc)
135385, Dec. 6, 2000, En Banc)
51. What is the concept of "native title"? What is a
49. Define "ancestral domains" and "ancestral lands." Certificate of Ancestral Domain Title (CADT)?
Do they constitute part of the land of the public domain?
Held: Native title refers to ICCs/IPs preconquest rights
Held: Ancestral domains and ancestral lands are the to lands and domains held under a claim of private
private property of indigenous peoples and do not ownership as far back as memory reaches. These lands
constitute part of the land of the public domain. are deemed never to have been public lands and are
indisputably presumed to have been held that way since
The IPRA grants to ICCs/IPs a distinct kind of ownership before the Spanish Conquest. The rights of ICCs/IPs to
over ancestral domains and ancestral lands. Ancestral their ancestral domains (which also include ancestral
lands are not the same as ancestral domains. These are lands) by virtue of native title shall be recognized and
defined in Section 3(a) and (b) of the Indigenous Peoples respected (Section 11, IPRA). Formal recognition, when
Rights Act x x x. solicited by ICCs/IPs concerned, shall be embodied in a
Certificate of Ancestral Domain Title (CADT), which shall
Ancestral domains are all areas belonging to ICCs/IPs held recognize the title of the concerned ICCs/IPs over the
under a claim of ownership, occupied or possessed by territories identified and delineated.
ICCs/IPs by themselves or through their ancestors,
Like a torrens title, a CADT is evidence of private from, relating or incident to, or involving ill-gotten wealth
ownership of land by native title. Native title, however, is contemplated in Section 15, Article XI of the Constitution
a right of private ownership peculiarly granted to ICCs/IPs may be barred by prescription. (Presidential Ad Hoc
over their ancestral lands and domains. The IPRA Fact-Finding Committee on Behest Loans, et al. v.
categorically declares ancestral lands and domains held Hon. Aniano A. Desierto, et al., G.R. No. 130140,
by native title as never to have been public land. Domains Oct. 25, 1999, En Banc [Davide, C.J.])
and lands held under native title are, therefore,
indisputably presumed to have never been public lands
and are private. STRUCTURE OF GOVERNMENT

The concept of native title in the IPRA was taken from the The Doctrine of Separation of Powers
1909 case of Carino v. Insular Government (41 Phil. 935
[1909], 212 U.S. 449, 53 L. Ed. 594). Carino firmly 55. May the Government, through the PCGG, validly
established a concept of private land title that existed bind itself to cause the dismissal of all cases against the
irrespective of any royal grant from the State. (Separate Marcos heirs pending before the Sandiganbayan and
Opinion, Puno, J., in Isagani Cruz v. Secretary of other courts in a Compromise Agreement entered into
DENR, et al., G.R. No. 135385, Dec. 6, 2000, En between the former and the latter?
Banc)
Held: This is a direct encroachment on judicial power,
particularly in regard to criminal jurisdiction. Well-settled
53. Distinguish ownership of land under native title is the doctrine that once a case has been filed before a
and ownership by acquisitive prescription against the court of competent jurisdiction, the matter of its dismissal
State. or pursuance lies within the full discretion and control of
the judge. In a criminal case, the manner in which the
Held: Ownership by virtue of native title presupposes prosecution is handled, including the matter of whom to
that the land has been held by its possessor and his present as witnesses, may lie within the sound discretion
predecessor-in-interest in the concept of an owner since of the government prosecutor; but the court decides,
time immemorial. The land is not acquired from the based on the evidence proffered, in what manner it will
State, that is, Spain or its successor-in-interest, the United dispose of the case. Jurisdiction, once acquired by the
States and the Philippine Government. There has been trial court, is not lost despite a resolution, even by the
no transfer of title from the State as the land has been justice secretary, to withdraw the information or to
regarded as private in character as far back as memory dismiss the complaint. The prosecution’s motion to
goes. In contrast, ownership of land by acquisitive withdraw or to dismiss is not the least binding upon the
prescription against the State involves a conversion of the court. On the contrary, decisional rules require the trial
character of the property from alienable public land to court to make its own evaluation of the merits of the case,
private land, which presupposes a transfer of title from because granting such motion is equivalent to effecting a
the State to a private person. (Separate Opinion, disposition of the case itself.
Kapunan, J., in Isagani Cruz v. Secretary of DENR,
G.R. No. 135385, Dec. 6, 2000, En Banc) Thus, the PCGG, as the government prosecutor of ill-
gotten wealth cases, cannot guarantee the dismissal of all
such criminal cases against the Marcoses pending in the
The Right of the State to Recover Properties Unlawfully courts, for said dismissal is not within its sole power and
Acquired by Public Officials or Employees discretion. (Chavez v. PCGG, 299 SCRA 744, Dec. 9,
1998 [Panganiban])
54. Does the right of the State to recover properties
unlawfully acquired by public officials or employees which
may not be barred by prescription, laches, or estoppel Delegation of Powers
under Section 15, Article XI of the Constitution apply to
criminal cases for the recovery of ill-gotten wealth? 56. What are the tests of a valid delegation of power?

Held: Section 15, Article XI, 1987 Constitution provides Held: Empowering the COMELEC, an administrative body
that “[T]he right of the State to recover properties exercising quasi-judicial functions, to promulgate rules
unlawfully acquired by public officials or employees, from and regulations is a form of delegation of legislative
them or from their nominees as transferees, shall not be authority x x x. However, in every case of permissible
barred by prescription, laches, or estoppel.” From the delegation, there must be a showing that the delegation
proceedings of the Constitutional Commission of 1986, itself is valid. It is valid only if the law (a) is complete in
however, it was clear that this provision applies only to itself, setting forth therein the policy to be executed,
civil actions for recovery of ill-gotten wealth, and not to carried out, or implemented by the delegate; and (b) fixes
criminal cases. Thus, the prosecution of offenses arising a standard – the limits of which are sufficiently
determinate and determinable – to which the delegate was no express provision in the 1935 Constitution giving
must conform in the performance of his functions . A such power to both houses of Congress, it was so
sufficient standard is one which defines legislative policy, incidental to the legislative function as to be implied.).
marks its limits, maps out its boundaries and specifies the Thus, Section 21, Article VI provides x x x.
public agency to apply it. It indicates the circumstances
under which the legislative command is to be effected . The power of both houses of Congress to conduct
(Santiago v. COMELEC, 270 SCRA 106, March 19, inquiries in aid of legislation is not, therefore, absolute or
1997) unlimited. Its exercise is circumscribed by the afore-
quoted provision of the Constitution. Thus, as provided
therein, the investigation must be “in aid of legislation in
The Legislative Department accordance with its duly published rules of procedure”
and that “the rights of persons appearing in or affected
57. May the Supreme Court properly inquire into the by such inquiries shall be respected.” It follows then that
motives of the lawmakers in conducting legislative the rights of persons under the Bill of Rights must be
investigations? Can it enjoin the Congress or any of its respected, including the right to due process and the right
regular and special committees from making inquiries in not to be compelled to testify against one’s self.
aid of legislation?
The power to conduct formal inquiries or
Held: In its comment, respondent Committee investigations is specifically provided for in Sec. 1 of the
claims that this Court cannot properly inquire into the Senate Rules of Procedure Governing Inquiries in Aid of
motives of the lawmakers in conducting legislative Legislation. Such inquiries may refer to the
investigations, much less can it enjoin the Congress or implementation or re-examination of any law or in
any of its regular and special committees x x x from connection with any proposed legislation or the
making inquiries in aid of legislation, under the doctrine formulation of future legislation. They may also extend
of separation of powers, which obtains in our present to any and all matters vested by the Constitution in
system of government. Congress and/or in the Senate alone.

The contention is untenable. X x x As held in Jean L. Arnault v. Leon Nazareno, et


al, (No. L-3820, July 18, 1950, 87 Phil. 29), the inquiry,
The “allocation of constitutional boundaries” is a to be within the jurisdiction of the legislative body making
task that this Court must perform under the Constitution. it, must be material or necessary to the exercise of a
Moreover, as held in a recent case (Neptali A. Gonzales, power in it vested by the Constitution, such as to legislate
et al. v. Hon. Catalino Macaraig, Jr., et al., G.R. No. or to expel a member.
87636, 19 November 1990, 191 SCRA 452, 463) , “[t]he
political question doctrine neither interposes an obstacle Under Sec. 4 of the aforementioned Rules, the
to judicial determination of the rival claims. The Senate may refer to any committee or committees any
jurisdiction to delimit constitutional boundaries has been speech or resolution filed by any Senator which in its
given to this Court. It cannot abdicate that obligation judgment requires an appropriate inquiry in aid of
mandated by the 1987 Constitution, although said legislation. In order therefore to ascertain the character
provision by no means does away with the applicability of or nature of an inquiry, resort must be had to the speech
the principle in appropriate cases.” (Section 1, Article VIII or resolution under which such an inquiry is proposed to
of the 1987 Constitution) be made. (Bengzon, Jr. v. Senate Blue Ribbon
Committee, 203 SCRA 767, Nov. 20, 1991, En Banc
The Court is thus of the considered view that it [Padilla])
has jurisdiction over the present controversy for the
purpose of determining the scope and extent of the power 59. On 13 September 1988, the Senate Minority
of the Senate Blue Ribbon Committee to conduct inquires Floor Leader, Hon. Juan Ponce Enrile delivered a speech
into private affairs in purported aid of legislation. “on a matter of personal privilege” before the Senate on
(Bengzon, Jr. v. Senate Blue Ribbon Committee, the alleged “take-over of SOLOIL Incorporated, the
203 SCRA 767, Nov. 20, 1991, En Banc [Padilla]) flagship on the First Manila Management of Companies
(FMMC) by Ricardo Lopa” and called upon “the Senate to
58. Is the power of both houses of Congress to look into the possible violation of the law in the case,
conduct inquiries in aid of legislation absolute or particularly with regard to Republic Act No. 3019, the
unlimited? Anti-Graft and Corrupt Practices Act.”

Held: The 1987 Constitution expressly On motion of Senator Orlando Mercado, the
recognizes the power of both houses of Congress to matter was referred by the Senate to the Committee on
conduct inquiries in aid of legislation (In Arnault v. Accountability of Public Officers (Blue Ribbon Committee).
Nazareno, 87 Phil. 29, this Court held that although there Thereafter, the Senate Blue Ribbon Committee started its
investigation on the matter. Petitioners and Ricardo Lopa investigation is to find out whether or not the relatives of
were subpoenaed by the Committee to appear before it the President or Mr. Ricardo Lopa had violated Section 5
and testify on “what they know” regarding the “sale of the of RA No. 3019, the “Anti-Graft and Corrupt Practices
thirty-six (36) corporations belonging to Benjamin ‘Kokoy’ Act”, a matter that appears more within the province of
Romualdez.” the courts rather than of the legislature. Besides, the
Court may take judicial notice that Mr. Ricardo Lopa died
At the hearing held on 23 May 1989, Ricardo Lopa during the pendency of this case. In John T. Watkins v.
declined to testify on the ground that his testimony may United States (354 U.S. 178, 1 L. ed. 2D 1273 [1957]), it
“unduly prejudice” the defendants in Civil Case No. 0035 was held:
before the Sandiganbayan. Petitioner Jose F.S. Bengzon,
Jr. likewise refused to testify invoking his constitutional “x x x. The power of Congress to conduct inquiries in aid
right to due process, and averring that the publicity of legislation is inherent in the legislative process. That
generated by respondent Committee’s inquiry could power is broad. It encompasses inquiries concerning the
adversely affect his rights as well as those of the other administration of existing laws as well as proposed or
petitioners who are his co-defendants in Civil Case No. possibly needed statutes. It includes surveys of defects
0035 before the Sandiganbayan. in our social, economic, or political system for the purpose
of enabling Congress to remedy them. It comprehends
The Senate Blue Ribbon Committee, thereupon, probes into departments of the Federal Government to
suspended its inquiry and directed the petitioners to file expose corruption, inefficiency or waste. But broad as is
their memorandum on the constitutional issues raised, this power of inquiry, it is not unlimited. There is no
after which, it issued a resolution dated 5 June 1989 general authority to expose the private affairs of
rejecting the petitioners’ plea to be excused from individuals without justification in terms of the functions
testifying, and the Committee voted to pursue and of Congress. This was freely conceded by the Solicitor
continue its investigation of the matter. X x x General in his arguments in this case. Nor is the Congress
a law enforcement or trial agency. These are functions
Claiming that the Senate Blue Ribbon Committee of the executive and judicial departments of government.
is poised to subpoena and require their attendance and No inquiry is an end in itself; it must be related to and in
testimony in proceedings before the Committee, in excess furtherance of a legislative task of Congress.
of its jurisdiction and legislative rights, and that there is Investigations conducted solely for the personal
no appeal nor any other plain, speedy and adequate aggrandizement of the investigators or to ‘punish’ those
remedy in the ordinary course of law, the petitioners filed investigated are indefensible.” (italics supplied)
the present petition for prohibition with a prayer for
temporary restraining order and/or injunctive relief. It cannot be overlooked that when respondent
Committee decided to conduct its investigation of the
Held: A perusal of the speech of Senator Enrile petitioners, the complaint in Civil Case No. 0035 had
reveals that he (Senator Enrile) made a statement which already been filed with the Sandiganbayan. A perusal of
was published in various newspapers on 2 September that complaint shows that one of its principal causes of
1988 accusing Mr. Ricardo “Baby” Lopa of “having taken action against herein petitioners, as defendants therein,
over the FMMC Group of Companies.” X x x is the alleged sale of the 36 (or 39) corporations belonging
to Benjamin “Kokoy” Romualdez. Since the issues in said
Verily, the speech of Senator Enrile contained no complaint had long been joined by the filing of petitioners’
suggestion of contemplated legislation; he merely called respective answers thereto, the issue sought to be
upon the Senate to look into a possible violation of Sec. 5 investigated by the respondent Committee is one over
of RA No. 3019, otherwise known as “The Anti-Graft and which jurisdiction had been acquired by the
Corrupt Practices Act.” In other words, the purpose of Sandiganbayan. In short, the issue has been pre-empted
the inquiry to be conducted by respondent Blue Ribbon by that court. To allow the respondent Committee to
Committee was to find out whether or not the relatives of conduct its own investigation of an issue already before
President Aquino, particularly Mr. Ricardo Lopa, had the Sandiganbayan would not only pose the possibility of
violated the law in connection with the alleged sale of the conflicting judgments between a legislative committee
36 or 39 corporations belonging to Benjamin “Kokoy” and a judicial tribunal, but if the Committee’s judgment
Romualdez to the Lopa Group. There appears to be, were to be reached before that of the Sandiganbayan, the
therefore, no intended legislation involved. possibility of its influence being made to bear on the
ultimate judgment of the Sandiganbayan can not be
Xxx discounted.

It appears, therefore, that the contemplated In fine, for the respondent Committee to probe and
inquiry by respondent Committee is not really “in aid of inquire into the same justiciable controversy already
legislation” because it is not related to a purpose within before the Sandiganbayan, would be an encroachment
the jurisdiction of Congress, since the aim of the into the exclusive domain of judicial jurisdiction that had
(Bengzon, Jr. v. Senate Blue
much earlier set in. X x x Given, then, the power of the Senate to
Ribbon Committee, 203 SCRA 767, Nov. 20, 1991, propose amendments, the Senate can propose its own
En Banc [Padilla]) version even with respect to bills which are required by
the Constitution to originate in the House.
60. Petitioners’ contention is that Republic Act No.
7716 (The Expanded-VAT Law) did not “originate It is insisted, however, that S. No. 1630 was
exclusively” in the House of Representatives as required passed not in substitution of H. No. 11197 but of another
by Art. VI, Sec. 24 of the Constitution, because it is in fact Senate bill (S. No. 1129) earlier filed and that what the
the result of the consolidation of two distinct bills, H. No. Senate did was merely to “take (H. No. 11197) into
11197 and S. No. 1630. In this connection, petitioners consideration” in enacting S. No. 1630. There is really no
point out that although Art. VI, Sec. 24 was adopted from difference between the Senate preserving H. No. 11197
the American Federal Constitution, it is notable in two up to the enacting clause and then writing its own version
respects: the verb “shall originate” is qualified in the following the enacting clause (which, it would seem,
Philippine Constitution by the word “exclusively” and the petitioners admit is an amendment by substitution), and,
phrase “as on other bills” in the American version is on the other hand, separately presenting a bill of its own
omitted. This means, according to them, that to be on the same subject matter. In either case the result are
considered as having originated in the House, Republic two bills on the same subject.
Act No. 7716 must retain the essence of H. No. 11197.
Indeed, what the Constitution simply means is
Held: This argument will not bear analysis. To that the initiative for filing revenue, tariff, or tax bills, bills
begin with, it is not the law - but the revenue bill - which authorizing an increase of the public debt, private bills and
is required by the Constitution to “originate exclusively” in bills of local application must come from the House of
the House of Representatives. It is important to Representatives on the theory that, elected as they are
emphasize this, because a bill originating in the House from the districts, the members of the House can be
may undergo such extensive changes in the Senate that expected to be more sensitive to the local needs and
the result may be a rewriting of the whole. The possibility problems. On the other hand, the senators, who are
of a third version by the conference committee will be elected at large, are expected to approach the same
discussed later. At this point, what is important to note problems from the national perspective. Both views are
is that, as a result of the Senate action, a distinct bill may thereby made to bear on the enactment of such laws.
be produced. To insist that a revenue statute - and not
only the bill which initiated the legislative process Nor does the Constitution prohibit the filing in
culminating in the enactment of the law - must the Senate of a substitute bill in anticipation of its
substantially be the same as the House bill would be to receipt of the bill from the House, so long as action by
deny the Senate's power not only to “concur with the Senate as a body is withheld pending receipt of the
amendments” but also to “propose amendments.” It House bill. The Court cannot, therefore, understand the
would be to violate the coequality of legislative power of alarm expressed over the fact that on March 1, 1993,
the two houses of Congress and in fact make the House eight months before the House passed H. No. 11197, S.
superior to the Senate. No. 1129 had been filed in the Senate. After all it does
not appear that the Senate ever considered it. It was
The contention that the constitutional design is to only after the Senate had received H. No. 11197 on
limit the Senate's power in respect of revenue bills in November 23, 1993 that the process of legislation in
order to compensate for the grant to the Senate of the respect of it began with the referral to the Senate
treaty-ratifying power (Art. VII, Sec. 21) and thereby Committee on Ways and Means of H. No. 11197 and the
equalize its powers and those of the House overlooks the submission by the Committee on February 7, 1994 of S.
fact that the powers being compared are different. We No. 1630. For that matter, if the question were simply
are dealing here with the legislative power which under the priority in the time of filing of bills, the fact is that it
the Constitution is vested not only in any particular was in the House that a bill (H. No. 253) to amend the
chamber but in the Congress of the Philippines, consisting VAT law was first filed on July 22, 1992. Several other
of “a Senate and a House of Representatives.” (Art. VI, bills had been filed in the House before S. No. 1129 was
Sec. 1) The exercise of the treaty-ratifying power is not filed in the Senate, and H. No. 11197 was only a
the exercise of legislative power. It is the exercise of a substitute of those earlier bills. (Tolentino v.
check on the executive power. There is, therefore, no Secretary of Finance, 235 SCRA 630, 661-663,
justification for comparing the legislative powers of the Aug. 25, 1994, En Banc [Mendoza])
House and of the Senate on the basis of the possession
of a similar non-legislative power by the Senate. The
possession of a similar power by the U.S. Senate has 61. Discuss the nature of the Party-List system. Is it,
never been thought of as giving it more legislative powers without any qualification, open to all?
than the House of Representatives.
Held: 1. The party-list system is a social justice tool party, a sectoral party, a national party, UNIDO,
designed not only to give more law to the great masses Magsasaka, or a regional party in Mindanao.” x x x.
of our people who have less in life, but also to enable
them to become veritable lawmakers themselves, Indeed, Commissioner Monsod stated that the purpose of
empowered to participate directly in the enactment of the party-list provision was to open up the system, in
laws designed to benefit them. It intends to make the order to give a chance to parties that consistently place
marginalized and the underrepresented not merely third or fourth in congressional district elections to win a
passive recipients of the State’s benevolence, but active seat in Congress. He explained: “The purpose of this is
participants in the mainstream of representative to open the system. In the past elections, we found out
democracy. Thus, allowing all individuals and groups, that there were certain groups or parties that, if we count
including those which now dominate district elections, to their votes nationwide, have about 1,000,000 or
have the same opportunity to participate in party-list 1,500,000 votes. But they were always third or fourth
elections would desecrate this lofty objective and place in each of the districts. So, they have no voice in
mongrelize the social justice mechanism into an atrocious the Assembly. But this way, they would have five or six
veneer for traditional politics. (Ang Bagong Bayani – representatives in the assembly even if they would not
OFW Labor Party v. COMELEC, G.R. No. 147589, win individually in legislative districts. So, that is
June 26, 2001, En Banc [Panganiban]) essentially the mechanics, the purpose and objective of
the party-list system.”
2. Crucial to the resolution of this case is the fundamental
social justice principle that those who have less in life For its part, Section 2 of RA 7941 also provides for “a
should have more in law. The party-list system is one party-list system of registered national, regional and
such tool intended to benefit those who have less in life. sectoral parties or organizations or coalitions thereof, x x
It gives the great masses of our people genuine hope and x.” Section 3 expressly states that a “party” is “either a
genuine power. It is a message to the destitute and the political party or a sectoral party or a coalition of parties.”
prejudiced, and even to those in the underground, that More to the point, the law defines “political party” as “an
change is possible. It is an invitation for them to come organized group of citizens advocating an ideology or
out of their limbo and seize the opportunity. platform, principles and policies for the general conduct
of government and which, as the most immediate means
Clearly, therefore, the Court cannot accept the of securing their adoption, regularly nominates and
submissions x x x that the party-list system is, without supports certain of its leaders and members as candidates
any qualification, open to all. Such position does not only for public office.”
weaken the electoral chances of the marginalized and
underrepresented; it also prejudices them. It would gut Furthermore, Section 11 of RA 7941 leaves no doubt as
the substance of the party-list system. Instead of to the participation of political parties in the party-list
generating hope, it would create a mirage. Instead of system. X x x
enabling the marginalized, it would further weaken them
and aggravate their marginalization. (Ang Bagong Indubitably, therefore, political parties – even the major
Bayani – OFW Labor Party v. COMELEC, G.R. No. ones – may participate in the party-list elections.
147589, June 26, 2001, En Banc [Panganiban])
That political parties may participate in the party-list
62. Are political parties – even the major ones – elections does not mean, however, that any political party
prohibited from participating in the party-list elections? – or any organization or group for that matter – may do
so. The requisite character of these parties or
Held: Under the Constitution and RA 7941, private organizations must be consistent with the purpose of the
respondents cannot be disqualified from the party-list party-list system, as laid down in the Constitution and RA
elections, merely on the ground that they are political 7941. X x x (Ang Bagong Bayani – OFW Labor Party
parties. Section 5, Article VI of the Constitution, provides v. COMELEC, G.R. No. 147589, June 26, 2001, En
that members of the House of Representatives may “be Banc [Panganiban])
elected through a party-list system of registered national,
regional, and sectoral parties or organizations. 63. Who are the marginalized and underrepresented
sectors to be represented under the party-list system?
Furthermore, under Sections 7 and 8, Article IX [C] of the
Constitution, political parties may be registered under the Held: The marginalized and underrepresented sectors to
party-list system. X x x be represented under the party-list system are
enumerated in Section 5 of RA 7941 x x x.
During the deliberations in the Constitutional Commission,
Comm. Christian S. Monsod pointed out that the While the enumeration of marginalized and
participants in the party-list system may “be a regional underrepresented sectors is not exclusive, it
demonstrates the clear intent of the law that not all
sectors can be represented under the party-list system. X fundamental difference between the congressional district
xx elections and the party-list elections.

[W]e stress that the party-list system seeks to enable As earlier noted, the purpose of the party-list provision
certain Filipino citizens – specifically those belonging to was to open up the system, in order to enhance the
marginalized and underrepresented sectors, chance of sectoral groups and organizations to gain
organizations and parties – to be elected to the House of representation in the House of Representatives through
Representatives. The assertion x x x that the party-list the simplest scheme possible. Logic shows that the
system is not exclusive to the marginalized and system has been opened to those who have never gotten
underrepresented disregards the clear statutory policy. a foothold within it – those who cannot otherwise win in
Its claim that even the super-rich and overrepresented regular elections and who therefore need the “simplest
can participate desecrates the spirit of the party-list scheme possible” to do so. Conversely, it would be
system. illogical to open the system to those who have long been
within it – those privileged sectors that have long
Indeed, the law crafted to address the peculiar dominated the congressional district elections.
disadvantage of Payatas hovel dwellers cannot be
appropriated by the mansion owners of Forbes Park. The Xxx
interests of these two sectors are manifestly disparate;
hence, the x x x position to treat them similarly defies Verily, allowing the non-marginalized and
reason and common sense. X x x overrepresented to vie for the remaining seats under the
party-list system would not only dilute, but also prejudice
While the business moguls and the mega-rich are, the chance of the marginalized and underrepresented,
numerically speaking, a tiny minority, they are neither contrary to the intention of the law to enhance it. The
marginalized nor underrepresented, for the stark reality is party-list system is a tool for the benefit of the
that their economic clout engenders political power more underprivileged; the law could not have given the same
awesome than their numerical limitation. Traditionally, tool to others, to the prejudice of the intended
political power does not necessarily emanate from the size beneficiaries. (Ang Bagong Bayani – OFW Labor
of one’s constituency; indeed, it is likely to arise more Party v. COMELEC, G.R. No. 147589, June 26, 2001,
directly from the number and amount of one’s bank En Banc [Panganiban])
accounts.
64. Section 5(2), Article VI of the Constitution
It is ironic, therefore, that the marginalized and provides that "[t]he party-list representatives shall
underrepresented in our midst are the majority who constitute twenty per centum of the total number of
wallow in poverty, destitution and infirmity. It was for representatives including those under the party-list."
them that the party-list system was enacted – to give Does the Constitution require all such allocated seats to
them not only genuine hope, but genuine power; to give be filled up all the time and under all circumstances?
them opportunity to be elected and to represent the
specific concerns of their constituencies; and simply to Held: The Constitution simply states that "[t]he party-
give them a direct vote in Congress and in the larger list representatives shall constitute twenty per centum of
affairs of the State. In its noblest sense, the party-list the total number of representatives including those under
system truly empowers the masses and ushers a new the party-list."
hope for genuine change. Verily, it invites those
marginalized and underrepresented in the past – the Xxx
farm hands, the fisher folk, the urban poor, even those in
the underground movement – to come out and We rule that a simple reading of Section 5, Article VI of
participate, as indeed many of them came out and the Constitution, easily conveys the equally simple
participated during the last elections. The State cannot message that Congress was vested with the broad power
now disappoint and frustrate them by disabling the to define and prescribe the mechanics of the party-list
desecrating this social justice vehicle. system of representation. The Constitution explicitly sets
down only the percentage of the total membership in the
Because the marginalized and underrepresented had not House of Representatives reserved for party-list
been able to win in the congressional district elections representatives.
normally dominated by traditional politicians and vested
groups, 20 percent of the seats in the House of In the exercise of its constitutional prerogative, Congress
Representatives were set aside for the party-list system. enacted RA 7941. As said earlier, Congress declared
In arguing that even those sectors who normally therein a policy to promote "proportional representation"
controlled 80 percent of the seats in the House could in the election of party-list representatives in order to
participate in the party-list elections for the remaining 20 enable Filipinos belonging to the marginalized and
percent, the OSG and the Comelec disregard the underrepresented sectors to contribute legislation that
would benefit them. It however deemed it necessary to incorporation, bylaws, history, platform of government
require parties, organizations and coalitions participating and track record – that it represents and seeks to uplift
in the system to obtain at least two percent of the total marginalized and underrepresented sectors. Verily,
votes cast for the party-list system in order to be entitled majority of its membership should belong to the
to a party-list seat. Those garnering more than this marginalized and underrepresented. And it must
percentage could have "additional seats in proportion to demonstrate that in a conflict of interest, it has chosen or
their total number of votes." Furthermore, no winning is likely to choose the interest of such sectors.
party, organization or coalition can have more than three
seats in the House of Representatives. X x x Second, while even major political parties are expressly
allowed by RA 7941 and the Constitution to participate in
Considering the foregoing statutory requirements, it will the party-list system, they must comply with the declared
be shown x x x that Section 5(2), Article VI of the statutory policy of enabling “Filipino citizens belonging to
Constitution is not mandatory. It merely provides a marginalized and underrepresented sectors x x x to be
ceiling for party-list seats in Congress. (Veterans elected to the House of Representatives.” In other words,
Federation Party v. COMELEC, G.R. No. 136781, while they are not disqualified merely on the ground that
Oct. 6, 2000, En Banc [Panganiban]) they are political parties, they must show, however, that
they represent the interests of the marginalized and
65. What are the inviolable parameters to determine underrepresented. X x x.
the winners in a Philippine-style party-list election?
Third, in view of the objections directed against the
Held: To determine the winners in a Philippine-style registration of Ang Buhay Hayaang Yumabong, which is
party-list election, the Constitution and Republic Act No. allegedly a religious group, the Court notes the express
7941 mandate at least four inviolable parameters. These constitutional provision that the religious sector may not
are: be represented in the party-list system. x x x

First, the twenty percent allocation - the combined Furthermore, the Constitution provides that “religious
number of all party-list congressmen shall not exceed denominations and sects shall not be registered.” (Sec. 2
twenty percent of the total membership of the House of [5], Article IX [C]) The prohibition was explained by a
Representatives, including those elected under the party member of the Constitutional Commission in this wise:
list. “[T]he prohibition is on any religious organization
registering as a political party. I do not see any
Second, the two percent threshold - only those prohibition here against a priest running as a candidate.
garnering a minimum of two percent of the total valid That is not prohibited here; it is the registration of a
votes cast for the party-list system are "qualified" to have religious sect as a political party.”
a seat in the House of Representatives.
Fourth, a party or an organization must not be disqualified
Third, the three seat limit - each qualified party, under Section 6 of RA 7941, which enumerates the
regardless of the number of votes it actually obtained, is grounds for disqualification as follows:
entitled to a maximum of three seats; that is, one
"qualifying" and two additional seats. 1) It is a religious sect or denomination,
organization or association organized for religious
Fourth, proportional representation - the additional purposes;
seats which a qualified party is entitled to shall be 2) It advocates violence or unlawful means to
computed "in proportion to their total number of votes." seek its goal;
(Veterans Federation Party v. COMELEC, G.R. No. 3) It is a foreign party or organization;
136781 and Companion Cases, Oct. 6, 2000, En 4) It is receiving support from any foreign
Banc [Panganiban]) government, foreign political party, foundation,
organization, whether directly or through any of its
66. State the guidelines for screening Party-List officers or members or indirectly through third parties for
Participants. partisan election purposes;
5) It violates or fails to comply with laws, rules
Held: In this light, the Court finds it appropriate to lay or regulations relating to elections;
down the following guidelines, culled from the law and 6) It declares untruthful statements in its
the Constitution, to assist the Comelec in its work. petition;
7) It has ceased to exist for at least one (1)
First, the political party, sector, organization or coalition year; or
must represent the marginalized and underrepresented 8) It fails to participate in the last two (2)
groups identified in Section 5 of RA 7941. In other words, preceding elections or fails to obtain at least two per
it must show – through its constitution, articles of centum (2%) of the votes cast under the party-list system
in the two (2) preceding elections for the constituency in any right or interest – not even the police power of the
which it had registered.” State. Resolve.

Note should be taken of paragraph 5, which disqualifies a Held: The immunity from arrest or detention of Senators
party or group for violation of or failure to comply with and members of the House of Representatives x x x arises
election laws and regulations. These laws include Section from a provision of the Constitution. The history of the
2 of RA 7941, which states that the party-list system seeks provision shows that the privilege has always been
to “enable Filipino citizens belonging to marginalized and granted in a restrictive sense. The provision granting an
underrepresented sectors, organizations and parties x x x exemption as a special privilege cannot be extended
to become members of the House of Representatives.” A beyond the ordinary meaning of its terms. It may not be
party or organization, therefore, that does not comply extended by intendment, implication or equitable
with this policy must be disqualified. considerations.

Fifth, the party or organization must not be an adjunct of, The 1935 Constitution provided in its Article VI on the
or a project organized or an entity funded or assisted by, Legislative Department:
the government. By the very nature of the party-list
system, the party or organization must be a group of Sec. 15. The Senators and Members of the House of
citizens, organized by citizens and operated by citizens. Representatives shall in all cases except treason, felony,
It must be independent of the government. The and breach of the peace, be privileged from arrest during
participation of the government or its officials in the their attendance at the sessions of Congress, and in going
affairs of a party-list candidate is not only illegal and to and returning from the same; x x x.
unfair to other parties, but also deleterious to the
objective of the law: to enable citizens belonging to Because of the broad coverage of felony and breach of
marginalized and underrepresented sectors and the peace, the exemption applied only to civil arrests. A
organization to be elected to the House of congressman like the accused-appellant, convicted under
Representatives. Title Eleven of the Revised Penal Code could not claim
parliamentary immunity from arrest. He was subject to
Sixth, the party must not only comply with the the same general laws governing all persons still to be
requirements of the law; its nominees must likewise do tried or whose convictions were pending appeal.
so. x x x
The 1973 Constitution broadened the privilege of
Seventh, not only the candidate party or organization immunity as follows:
must represent marginalized and underrepresented
sectors; so also must its nominees. To repeat, under Article VIII, Sec. 9. A Member of the Batasang Pambansa
Section 2 of RA 7941, the nominees must be Filipino shall, in all offenses punishable by not more than six years
citizens “who belong to marginalized and imprisonment, be privileged from arrest during his
underrepresented sectors, organizations and parties.” attendance at its sessions and in going to and returning
Surely, the interests of the youth cannot be fully from the same.
represented by a retiree; neither can those of the urban
poor or the working class, by an industrialist. To allow For offenses punishable by more than six years
otherwise is to betray the State policy to give genuine imprisonment, there was no immunity from arrest. The
representation to the marginalized and underrepresented. restrictive interpretation of immunity and the intent to
confine it within carefully defined parameters is illustrated
Eighth, x x x while lacking a well-defined political by the concluding portion of the provision, to wit:
constituency, the nominee must likewise be able to
contribute to the formulation and enactment of X x x but the Batasang Pambansa shall surrender the
appropriate legislation that will benefit the nation as a member involved to the custody of the law within twenty
whole. x x x (Ang Bagong Bayani – OFW Labor four hours after its adjournment for a recess or for its next
Party v. COMELEC, G.R. No. 147589, June 26, 2001, session, otherwise such privilege shall cease upon its
En Banc [Panganiban]) failure to do so.

67. Accused-appellant Congressman Romeo G. The present Constitution adheres to the same restrictive
Jalosjos filed a motion before the Court asking that he be rule minus the obligation of Congress to surrender the
allowed to fully discharge the duties of a Congressman, subject Congressman to the custody of the law. The
including attendance at legislative sessions and requirement that he should be attending sessions or
committee meetings despite his having been convicted in committee meetings has also been removed. For
the first instance of a non-bailable offense. He contended relatively minor offenses, it is enough that Congress is in
that his reelection being an expression of popular will session.
cannot be rendered inutile by any ruling, giving priority to
The accused-appellant argues that a member of The accused-appellant states that the plea of the
Congress’ function to attend sessions is underscored by electorate which voted him into office cannot be
Section 16(2), Article VI of the Constitution which states supplanted by unfounded fears that he might escape
that – eventual punishment if permitted to perform
congressional duties outside his regular place of
(2) A majority of each House shall constitute a quorum to confinement.
do business, but a smaller number may adjourn from day
to day and may compel the attendance of absent It will be recalled that when a warrant for accused-
Members in such manner, and under such penalties, as appellant’s arrest was issued, he fled and evaded capture
such House may provide. despite a call from his colleagues in the House of
Representatives for him to attend the sessions ands to
However, the accused-appellant has not given any reason surrender voluntarily to the authorities. Ironically, it is
why he should be exempted from the operation of Section now the same body whose call he initially spurned which
11, Article VI of the Constitution. The members of accused-appellant is invoking to justify his present
Congress cannot compel absent members to attend motion. This can not be countenanced because, x x x
sessions if the reason for the absence is a legitimate one. aside from its being contrary to well-defined
The confinement of a Congressman charged with a crime Constitutional restrains, it would be a mockery of the aims
punishable by imprisonment of more than six years is not of the State’s penal system.
merely authorized by law, it has constitutional
foundations. Accused-appellant argues that on several occasions, the
Regional Trial Court of Makati granted several motions to
Accused-appellant’s reliance on the ruling in Aguinaldo v. temporarily leave his cell at the Makati City Jail, for official
Santos (212 SCRA 768, at 773 [1992]), which states, inter or medical reasons x x x.
alia, that –
He also calls attention to various instances, after his
The Court should never remove a public officer for acts transfer at the New Bilibid Prison in Muntinlupa City, when
done prior to his present term of office. To do otherwise he was likewise allowed/permitted to leave the prison
would be to deprive the people of their right to elect their premises x x x.
officers. When the people have elected a man to office,
it must be assumed that they did this with the knowledge There is no showing that the above privileges are peculiar
of his life and character, and that they disregarded or to him or to a member of Congress. Emergency or
forgave his fault or misconduct, if he had been guilty of compelling temporary leaves from imprisonment are
any. It is not for the Court, by reason of such fault or allowed to all prisoners, at the discretion of the authorities
misconduct, to practically overrule the will of the people. or upon court orders.

will not extricate him from his predicament. It can be What the accused-appellant seeks is not of an emergency
readily seen x x x that the Aguinaldo case involves the nature. Allowing accused-appellant to attend
administrative removal of a public officer for acts done congressional sessions and committee meetings for five
prior to his present term of office. It does not apply to (5) days or more in a week will virtually make him a free
imprisonment arising from the enforcement of criminal man with all the privileges appurtenant to his position.
law. Moreover, in the same way that preventive Such an aberrant situation not only elevates accused-
suspension is not removal, confinement pending appeal is appellant’s status to that of a special class, it also would
not removal. He remains a Congressman unless expelled be a mockery of the purposes of the correction system.
by Congress or, otherwise, disqualified. Xxx

One rationale behind confinement, whether pending The accused-appellant avers that his constituents in the
appeal or after final conviction, is public self-defense. First District of Zamboanga del Norte want their voices to
Society must protect itself. It also serves as an example be heard and that since he is treated as bona fide member
and warning to others. of the House of Representatives, the latter urges a co-
equal branch of government to respect his mandate. He
A person charged with crime is taken into custody for also claims that the concept of temporary detention does
purposes of the administration of justice. As stated in not necessarily curtail his duty to discharge his mandate
United States v. Gustilo (19 Phil. 208, 212), it is the injury and that he has always complied with the
to the public which State action in criminal law seeks to conditions/restrictions when he is allowed to leave jail.
redress. It is not the injury to the complainant. After
conviction in the Regional Trial Court, the accused may We remain unpersuaded.
be denied bail and thus subjected to incarceration if there
is risk of his absconding. Xxx
When the voters of his district elected the accused- VOTERS, ADOPTING A SYSTEM OF CONTINUING
appellant to Congress, they did so with full awareness of REGISTRATION, PRESCRIBING THE PROCEDURES
the limitations on his freedom of action. They did so with THEREOF AND AUTHORIZING THE APPROPRIATION OF
the knowledge that he could achieve only such legislative FUNDS THEREFOR." Section 44, which provides for the
results which he could accomplish within the confines of reassignment of election officers, is relevant to the
prison. To give a more drastic illustration, if voters elect subject matter of registration as it seeks to ensure the
a person with full knowledge that he is suffering from a integrity of the registration process by providing guideline
terminal illness, they do so knowing that at any time, he for the COMELEC to follow in the reassignment of election
may no longer serve his full term in office. (People v. officers. It is not an alien provision but one which is
Jalosjos, 324 SCRA 689, Feb. 3, 2000, En Banc related to the conduct and procedure of continuing
[Ynares-Santiago]) registration of voters. In this regard, it bears stressing
that the Constitution does not require Congress to employ
68. Discuss the objectives of Section 26(1), Article VI in the title of an enactment, language of such precision
of the 1987 Constitution, that "[e]very bill passed by the as to mirror, fully index or catalogue, all the contents and
Congress shall embrace only one subject which shall be the minute details therein. (Agripino A. De Guzman,
expressed in the title thereof." Jr., et al. v. COMELEC, G.R. No. 129118, July 19,
2000, En Banc [Purisima])
Held: The objectives of Section 26(1), Article VI of the
1987 Constitution are: 70. Do courts have the power to inquire into
allegations that, in enacting a law, a House of Congress
1) To prevent hodge-podge or log-rolling failed to comply with its own rules?
legislation;
2) To prevent surprise or fraud upon the Held: The cases, both here and abroad, in varying forms
legislature by means of provisions in bills of which the of expression, all deny to the courts the power to inquire
titles gave no information, and which might therefore be into allegations that, in enacting a law, a House of
overlooked and carelessly and unintentionally adopted; Congress failed to comply with its own rules, in the
and absence of showing that there was a violation of a
3) To fairly apprise the people, through such constitutional provision or the right of private individuals.
publication of legislative proceedings as is usually made, In Osmena v. Pendatun (109 Phil. At 870-871), it was
of the subjects of legislation that are being considered, in held: “At any rate, courts have declared that ‘the rules
order that they may have opportunity of being heard adopted by deliberative bodies are subject to revocation,
thereon by petition or otherwise if they shall so desire. modification or waiver at the pleasure of the body
adopting them.’ And it has been said that ‘Parliamentary
Section 26(1) of Article VI of the 1987 Constitution is rules are merely procedural, and with their observance,
sufficiently complied with where x x x the title is the courts have no concern. They may be waived or
comprehensive enough to embrace the general objective disregarded by the legislative body.’ Consequently, ‘mere
it seeks to achieve, and if all the parts of the statute are failure to conform to parliamentary usage will not
related and germane to the subject matter embodied in invalidate that action (taken by a deliberative body) when
the title or so long as the same are not inconsistent with the requisite number of members have agreed to a
or foreign to the general subject and title. (Agripino A. particular measure.’”
De Guzman, Jr., et al. v. COMELEC, G.R. No.
129118, July 19, 2000, en Banc [Purisima]) It must be realized that each of the three departments of
our government has its separate sphere which the others
69. Section 44 of R.A. No. 8189 (The Voter's may not invade without upsetting the delicate balance on
Registration Act of 1996) which provides for automatic which our constitutional order rests. Due regard for the
transfer to a new station of any Election Officer who has working of our system of government, more than mere
already served for more than four years in a particular city comity, compels reluctance on the part of the courts to
or municipality was assailed for being violative of Section enter upon an inquiry into an alleged violation of the rules
26(1) of Article VI of the Constitution allegedly because it of the House. Courts must accordingly decline the
has an isolated and different subject from that of RA 8189 invitation to exercise their power. (Arroyo v. De
and that the same is not expressed in the title of the law. Venecia, 277 SCRA 268, Aug. 14, 1997 [Mendoza])
Should the challenge be sustained?
71. What is the Bicameral Conference Committee?
Held: Section 44 of RA 8189 is not isolated considering Discuss the nature of its function and its jurisdiction.
that it is related and germane to the subject matter stated
in the title of the law. The title of RA 8189 is "The Voter's Held: While it is true that a conference committee is the
Registration Act of 1996" with a subject matter mechanism for compromising differences between the
enunciated in the explanatory note as "AN ACT Senate and the House, it is not limited in its jurisdiction
PROVIDING FOR A GENERAL REGISTRATION OF to this question. Its broader function is described thus:
and abroad. The enrolled bill rule rests on the following
A conference committee may deal generally with the considerations:
subject matter or it may be limited to resolving the precise
differences between the two houses. Even where the X x x. As the President has no authority to approve a bill
conference committee is not by rule limited in its not passed by Congress, an enrolled Act in the custody of
jurisdiction, legislative custom severely limits the freedom the Secretary of State, and having the official attestations
with which new subject matter can be inserted into the of the Speaker of the House of Representatives, of the
conference bill. But occasionally a conference committee President of the Senate, and of the President of the United
produces unexpected results, results beyond its mandate. States, carries, on its face, a solemn assurance by the
These excursions occur even where the rules impose strict legislative and executive departments of the government,
limitations on conference committee jurisdiction. This is charged, respectively, with the duty of enacting and
symptomatic of the authoritarian power of conference executing the laws, that it was passed by Congress. The
committee. (Philippine Judges Association v. Prado, respect due to coequal and independent departments
227 SCRA 703, Nov. 11, 1993, En Banc [Cruz]) requires the judicial department to act upon that
assurance, and to accept, as having passed Congress, all
72. Discuss the Enrolled Bill Doctrine. bills authenticated in the manner stated; leaving the court
to determine, when the question properly arises, whether
Held: Under the enrolled bill doctrine, the signing of H. the Act, so authenticated, is in conformity with the
Bill No. 7189 by the Speaker of the House and the Constitution. (Marshall Field & Co. v. Clark, 143 U.S. 649,
President of the Senate and the certification by the 672, 36 L. Ed. 294, 303 [1891])
secretaries of both Houses of Congress that it was passed
on November 21, 1996 are conclusive of its due To overrule the doctrine now, x x x is to repudiate the
enactment. x x x To be sure, there is no claim either massive teaching of our cases and overthrow an
here or in the decision in the EVAT cases (Tolentino v. established rule of evidence. (Arroyo v. De Venecia,
Secretary of Finance) that the enrolled bill embodies a 277 SCRA 268, Aug. 14, 1997 [Mendoza])
conclusive presumption. In one case (Astorga v. Villegas,
56 SCRA 714 [1974]) we “went behind” an enrolled bill 73. When should the Legislative Journal be regarded
and consulted the Journal to determine whether certain as conclusive upon the courts, and why?
provisions of a statute had been approved by the Senate.
Held: The Journal is regarded as conclusive with respect
But, where as here there is no evidence to the contrary, to matters that are required by the Constitution to be
this Court will respect the certification of the presiding recorded therein. With respect to other matters, in the
officers of both Houses that a bill has been duly passed. absence of evidence to the contrary, the Journals have
Under this rule, this Court has refused to determine claims also been accorded conclusive effects. Thus, in United
that the three-fourths vote needed to pass a proposed States v. Pons (34 Phil. 729, 735 [1916]], quoting ex rel.
amendment to the Constitution had not been obtained, Herron v. Smith, 44 Ohio 348 [1886]), this Court spoke of
because “a duly authenticated bill or resolution imports the imperatives of public policy for regarding the Journals
absolute verity and is binding on the courts.” x x x as “public memorials of the most permanent character,”
thus: “They should be public, because all are required to
This Court has refused to even look into allegations that conform to them; they should be permanent, that rights
the enrolled bill sent to the President contained provisions acquired today upon the faith of what has been declared
which had been “surreptitiously” inserted in the to be law shall not be destroyed tomorrow, or at some
conference committee x x x. (Tolentino v. Secretary of remote period of time, by facts resting only in the memory
Finance) of individuals.” X x x. (Arroyo v. De Venecia, 277
SCRA 268, 298-299, Aug. 14, 1997 [Mendoza])
It has refused to look into charges that an amendment
was made upon the last reading of a bill in violation of 74. What matters are required to be entered on the
Art. VI, Sec. 26(2) of the Constitution that “upon the last Journal?
reading of a bill, no amendment shall be allowed.”
(Philippine Judges Ass’n v. Prado, 227 SCRA 703, 710 Held:
[1993])
1) The yeas and nays on the third and final
In other cases, this Court has denied claims that the tenor reading of a bill (Art. VI, Sec. 26[2]);
of a bill was otherwise than as certified by the presiding 2) The yeas and nays on any question, at the
officers of both Houses of Congress. request of one-fifth of the members present (Id., Sec.
16[4]);
The enrolled bill doctrine, as a rule of evidence, is well- 3) The yeas and nays upon repassing a bill over
established. It is cited with approval by text writers here the President’s veto (Id., Sec. 27[1]); and
4) The President’s objection to a bill he had authority to pass upon the question of qualification” finds
vetoed (Id.). no basis in law, because even after the elections the
(Arroyo v. De Venecia, 277 SCRA 268, 298, Aug. 14, COMELEC is empowered by Section 6 (in relation to
1997 [Mendoza]) Section 7) of R.A. 6646 to continue to hear and decide
questions relating to qualifications of candidates. X x x.
75. A disqualification case was filed against a
candidate for Congressman before the election with the Under the above-quoted provision, not only is a
COMELEC. The latter failed to resolve that disqualification disqualification case against a candidate allowed to
case before the election and that candidate won, although continue after the election (and does not oust the
he was not yet proclaimed because of that pending COMELEC of its jurisdiction), but his obtaining the highest
disqualification case. Is the COMELEC now ousted of number of votes will not result in the suspension or
jurisdiction to resolve the pending disqualification case termination of the proceedings against him when the
and, therefore, should dismiss the case, considering that evidence of guilt is strong. While the phrase “when the
jurisdiction is now vested with the House of evidence of guilt is strong” seems to suggest that the
Representatives Electoral Tribunal (HRET)? provisions of Section 6 ought to be applicable only to
disqualification cases under Section 68 of the Omnibus
Held: 1. In his first assignments of error, petitioner Election Code, Section 7 of R.A. 6646 allows the
vigorously contends that after the May 8, 1995 elections, application of the provisions of Section 6 to cases
the COMELEC lost its jurisdiction over the question of involving disqualification based on ineligibility under
petitioner’s qualifications to run for member of the House Section 78 of BP. Blg. 881. X x x. (Aquino v. COMELEC,
of Representatives. He claims that jurisdiction over the 248 SCRA 400, 417-419, Sept. 18, 1995, En Banc
petition for disqualification is exclusively lodged with the [Kapunan, J.])
House of Representatives Electoral Tribunal (HRET).
Given the yet-unresolved question of jurisdiction, 2. As to the House of Representatives Electoral Tribunal’s
petitioner avers that the COMELEC committed serious supposed assumption of jurisdiction over the issue of
error and grave abuse of discretion in directing the petitioner’s qualifications after the May 8, 1995 elections,
suspension of his proclamation as the winning candidate suffice it to say that HRET’s jurisdiction as the sole judge
in the Second Congressional District of Makati City. We of all contests relating to the elections, returns and
disagree. qualifications of members of Congress begins only after a
candidate has become a member of the House of
Petitioner conveniently confuses the distinction between Representatives (Art. VI, Sec. 17, 1987 Constitution).
an unproclaimed candidate to the House of Petitioner not being a member of the House of
Representatives and a member of the same. Obtaining Representatives, it is obvious that the HRET at this point
the highest number of votes in an election does not has no jurisdiction over the question. (Romualdez-
automatically vest the position in the winning candidate. Marcos v. COMELEC, 248 SCRA 300, 340-341, Sept.
Section 17 of Article VI of the 1987 Constitution reads: 18, 1995, En Banc [Kapunan, J.])

The Senate and the House of Representatives shall have 76. Will the rule be the same if that candidate wins
an Electoral Tribunal which shall be the sole judge of all and was proclaimed winner and already assumed office
contests relating to the election, returns and qualifications as Congressman?
of their respective Members.
Held: While the COMELEC is vested with the power to
Under the above-stated provision, the electoral tribunal declare valid or invalid a certificate of candidacy, its
clearly assumes jurisdiction over all contests relative to refusal to exercise that power following the proclamation
the election, returns and qualifications of candidates for and assumption of the position by Farinas is a recognition
either the Senate or the House only when the latter of the jurisdictional boundaries separating the COMELEC
become members of either the Senate or the House of and the Electoral Tribunal of the House of Representatives
Representatives. A candidate who has not been (HRET). Under Article VI, Section 17 of the Constitution,
proclaimed and who has not taken his oath of office the HRET has sole and exclusive jurisdiction over all
cannot be said to be a member of the House of contests relative to the election, returns, and
Representatives subject to Section 17 of Article VI of the qualifications of members of the House of
Constitution. While the proclamation of a winning Representatives. Thus, once a winning candidate has
candidate in an election is ministerial, B.P. Blg. 881 in been proclaimed, taken his oath, and assumed office as a
conjunction with Sec. 6 of R.A. 6646 allows suspension of member of the House of Representatives, COMELEC’s
proclamation under circumstances mentioned therein. jurisdiction over election contests relating to his election,
Thus, petitioner’s contention that “after the conduct of returns, and qualifications ends, and the HRET’s own
the election and (petitioner) has been established the jurisdiction begins. Thus, the COMELEC’s decision to
winner of the electoral exercise from the moment of discontinue exercising jurisdiction over the case is
election, the COMELEC is automatically divested of
justifiable, in deference to the HRET’s own jurisdiction manifestly constituting such grave abuse of discretion
and functions. that there has to be a remedy for such abuse.”

Xxx The Court does not x x x venture into the perilous area of
correcting perceived errors of independent branches of
Petitioner further argues that the HRET assumes the Government; it comes in only when it has to vindicate
jurisdiction only if there is a valid proclamation of the a denial of due process or correct an abuse of discretion
winning candidate. He contends that if a candidate fails so grave or glaring that no less than the Constitution itself
to satisfy the statutory requirements to qualify him as a calls for remedial action. (Libanan v. HRET, 283 SCRA
candidate, his subsequent proclamation is void ab initio. 520, Dec. 22, 1997 [Vitug])
Where the proclamation is null and void, there is no
proclamation at all and the mere assumption of office by
the proclaimed candidate does not deprive the COMELEC The Executive Department
at all of its power to declare such nullity, according to
petitioner. But x x x, in an electoral contest where the 78. What are the limitations on the veto power of
validity of the proclamation of a winning candidate who the President?
has taken his oath of office and assumed his post as
congressman is raised, that issue is best addressed to the Held: The act of the Executive in vetoing the
HRET. The reason for this ruling is self-evident, for it particular provisions is an exercise of a constitutionally
avoids duplicity of proceedings and a clash of jurisdiction vested power. But even as the Constitution grants the
between constitutional bodies, with due regard to the power, it also provides limitations to its exercise. The
people’s mandate. (Guerrero v. COMELEC, 336 SCRA veto power is not absolute.
458, July 26, 2000, En Banc [Quisumbing])
Xxx
77. Is there an appeal from a decision of the Senate
or House of Representatives Electoral Tribunal? What The OSG is correct when it states that the
then is the remedy, if any? Executive must veto a bill in its entirety or not at all. He
or she cannot act like an editor crossing out specific lines,
Held: The Constitution mandates that the House of provisions, or paragraphs in a bill that he or she dislikes.
Representatives Electoral Tribunal and the Senate In the exercise of the veto power, it is generally all or
Electoral Tribunal shall each, respectively, be the sole nothing. However, when it comes to appropriation,
judge of all contests relating to the election, returns and revenue or tariff bills, the Administration needs the money
qualifications of their respective members. to run the machinery of government and it can not veto
the entire bill even if it may contain objectionable
The Court has stressed that “x x x so long as the features. The President is, therefore, compelled to
Constitution grants the HRET the power to be the sole approve into law the entire bill, including its undesirable
judge of all contests relating to the election, returns and parts. It is for this reason that the Constitution has wisely
qualifications of members of the House of provided the “item veto power” to avoid inexpedient
Representatives, any final action taken by the HRET on a riders being attached to an indispensable appropriation or
matter within its jurisdiction shall, as a rule, not be revenue measure.
reviewed by this Court. The power granted to the
Electoral Tribunal x x x excludes the exercise of any The Constitution provides that only a particular
authority on the part of this Court that would in any wise item or items may be vetoed. The power to disapprove
restrict it or curtail it or even affect the same.” any item or items in an appropriate bill does not grant the
authority to veto a part of an item and to approve the
The Court did recognize, of course, its power of judicial remaining portion of the same item. (Gonzales v.
review in exceptional cases. In Robles v. HRET (181 SCRA Macaraig, Jr., 191 SCRA 452, 464 [1990]) (Bengzon v.
780), the Court has explained that while the judgments Drilon, 208 SCRA 133, 143-145, April 15, 1992, En
of the Tribunal are beyond judicial interference, the Court Banc [Gutierrez])
may do so, however, but only “in the exercise of this
Court’s so-called extraordinary jurisdiction x x x upon a 79. Distinguish an “item” from a “provision” in
determination that the Tribunal’s decision or resolution relation to the veto power of the President.
was rendered without or in excess of its jurisdiction, or
with grave abuse of discretion or paraphrasing Morrero Held: The terms item and provision in budgetary
(Morrero v. Bocar [66 Phil. 429]), upon a clear showing legislation and practice are concededly different. An item
of such arbitrary and improvident use by the Tribunal of in a bill refers to the particulars, the details, the distinct
its power as constitutes a denial of due process of law, or and severable parts x x x of the bill (Bengzon, supra, at
upon a demonstration of a very clear unmitigated error, 916). It is an indivisible sum of money dedicated to a
stated purpose (Commonwealth v. Dodson, 11 S.E., 2d
120, 124, 125, etc., 176 Va. 281). The United States of peace and in order to begin the healing process of our
Supreme Court, in the case of Bengzon v. Secretary of nation. He did not say he was leaving the Palace due to
Justice (299 U.S. 410, 414, 57 Ct 252, 81 L. Ed., 312) any kind of inability and that he was going to re-assume
declared “that an item” of an appropriation bill obviously the presidency as soon as the disability disappears; (3) he
means an item which in itself is a specific appropriation of expressed his gratitude to the people for the opportunity
money, not some general provision of law, which happens to serve them. Without doubt, he was referring to the
to be put into an appropriation bill. (Bengzon v. Drilon, past opportunity given him to serve the people as
208 SCRA 133, 143-145, April 15, 1992, En Banc President; (4) he assured that he will not shirk from any
[Gutierrez]) future challenge that may come ahead on the same
service of our country. Petitioner’s reference is to a future
80. May the President veto a law? May she veto a challenge after occupying the office of the president
decision of the SC which has long become final and which he has given up; and (5) he called on his supporters
executory? to join him in the promotion of a constructive national
spirit of reconciliation and solidarity. Certainly, the
Held: We need no lengthy justifications or national spirit of reconciliation and solidarity could not be
citations of authorities to declare that no President may attained if he did not give up the presidency. The press
veto the provisions of a law enacted thirty-five (35) years release was petitioner’s valedictory, his final act of
before his or her term of office. Neither may the President farewell. His presidency is now in the past tense.
set aside or reverse a final and executory judgment of this (Estrada v. Desierto, G.R. Nos. 146710-15, March
Court through the exercise of the veto power. (Bengzon 2, 2001, en Banc [Puno])
v. Drilon, 208 SCRA 133, 143-145, April 15, 1992,
En Banc [Gutierrez]) 82. Discuss our legal history on executive immunity.

81. Did former President Estrada resign as President Held: The doctrine of executive immunity in this
or should be considered resigned as of January 20, 2001 jurisdiction emerged as a case law. In the 1910 case of
when President Gloria Macapagal Arroyo took her oath as Forbes, etc. v. Chuoco Tiaco and Crossfield (16 Phil. 534
the 14th President of the Republic? [1910]), the respondent Tiaco, a Chinese citizen, sued
petitioner W. Cameron Forbes, Governor-General of the
Held: Resignation x x x is a factual question and its Philippine Islands, J.E. Harding and C.R. Trowbridge,
elements are beyond quibble: there must be an intent to Chief of Police and Chief of the Secret Service of the City
resign and the intent must be coupled by acts of of Manila, respectively, for damages for allegedly
relinquishment. The validity of a resignation is not conspiring to deport him to China. In granting a writ of
governed by any formal requirement as to form. It can prohibition, this Court, speaking thru Mr. Justice Johnson,
be oral. It can be written. It can be express. It can be held:
implied. As long as the resignation is clear, it must be
given legal effect. “The principle of nonliability x x x does not mean that the
judiciary has no authority to touch the acts of the
In the cases at bar, the facts show that petitioner did not Governor-General; that he may, under cover of his office,
write any formal letter of resignation before he evacuated do what he will, unimpeded and unrestrained. Such a
Malacanang Palace in the afternoon of January 20, 2001 construction would mean that tyranny, under the guise of
after the oath-taking of respondent Arroyo. the execution of the law, could walk defiantly abroad,
Consequently, whether or not petitioner resigned has to destroying rights of person and of property, wholly free
be determined from his acts and omissions before, during from interference of courts or legislatures. This does not
and after January 20, 2001 or by the totality of prior, mean, either, that a person injured by the executive
contemporaneous and posterior facts and circumstantial authority by an act unjustifiable under the law has no
evidence bearing a material relevance on the issue. remedy, but must submit in silence. On the contrary, it
means, simply, that the Governor-General, like the judges
Using this totality test, we hold that petitioner resigned as of the courts and the members of the Legislature, may
President. not be personally mulcted in civil damages for the
consequences of an act executed in the performance of
Xxx his official duties. The judiciary has full power to, and
will, when the matter is properly presented to it and the
In sum, we hold that the resignation of the petitioner occasion justly warrants it, declare an act of the
cannot be doubted. It was confirmed by his leaving Governor-General illegal and void and place as nearly as
Malacanang. In the press release containing his final possible in status quo any person who has been deprived
statement, (1) he acknowledged the oath-taking of the his liberty or his property by such act. This remedy is
respondent as President of the Republic albeit with assured to every person, however humble or of whatever
reservation about its legality; (2) he emphasized he was country, when his personal or property rights have been
leaving the Palace, the seat of the presidency, for the sake invaded, even by the highest authority of the state. The
thing which the judiciary can not do is mulct the
Governor-General personally in damages which result In his second Vicente G. Sinco Professorial Chair Lecture
from the performance of his official duty, any more than entitled, “Presidential Immunity And All The King’s Men:
it can a member of the Philippine Commission or the The Law Of Privilege As A Defense To Actions For
Philippine Assembly. Public policy forbids it. Damages,” (62 Phil. L.J. 113 [1987]) petitioner’s learned
counsel, former Dean of the UP College of Law, Atty.
Neither does this principle of nonliability mean that the Pacifico Agabin, brightened the modifications effected by
chief executive may not be personally sued at all in this constitutional amendment on the existing law on
relation to acts which he claims to perform as such official. executive privilege. To quote his disquisition:
On the contrary, it clearly appears from the discussion
heretofore had, particularly that portion which touched “In the Philippines though, we sought to do the
the liability of judges and drew an analogy between such American one better by enlarging and fortifying the
liability and that of the Governor-General, that the latter absolute immunity concept. First, we extended it to shield
is liable when he acts in a case so plainly outside of his the President not only from civil claims but also from
power and authority that he can not be said to have criminal cases and other claims. Second, we enlarged its
exercised discretion in determining whether or not he had scope so that it would cover even acts of the President
the right to act. What is held here is that he will be outside the scope of official duties. And third, we
protected from personal liability for damages not only broadened its coverage so as to include not only the
when he acts within his authority, but also when he is President but also other persons, be they government
without authority, provided he actually used discretion officials or private individuals, who acted upon orders of
and judgment, that is, the judicial faculty, in determining the President. It can be said that at that point most of us
whether he had authority to act or not. In other words, were suffering from AIDS (or absolute immunity defense
he is entitled to protection in determining the question of syndrome).”
his authority. If he decide wrongly, he is still protected
provided the question of his authority was one over which The Opposition in the then Batasang Pambansa sought
two men, reasonably qualified for that position, might the repeal of this Marcosian concept of executive
honestly differ; but he is not protected if the lack of immunity in the 1973 Constitution. The move was led by
authority to act is so plain that two such men could not then Member of Parliament, now Secretary of Finance,
honestly differ over its determination. In such case, he Alberto Romulo, who argued that the after incumbency
acts, not as Governor-General but as a private individual, immunity granted to President Marcos violated the
and, as such, must answer for the consequences of his principle that a public office is a public trust. He
act.” denounced the immunity as a return to the anachronism
“the king can do no wrong.” The effort failed.
Mr. Justice Johnson underscored the consequences if the
Chief Executive was not granted immunity from suit, viz: The 1973 Constitution ceased to exist when President
“x x x. Action upon important matters of state delayed; Marcos was ousted from office by the People Power
the time and substance of the chief executive spent in revolution in 1986. When the 1987 Constitution was
wrangling litigation; disrespect engendered for the person crafted, its framers did not reenact the executive
of one of the highest officials of the State and for the immunity provision of the 1973 Constitution. X x x
office he occupies; a tendency to unrest and disorder; (Estrada v. Desierto, G.R. Nos. 146710-15, March
resulting in a way, in a distrust as to the integrity of 2, 2001, en Banc [Puno])
government itself.”
83. Can former President Estrada still be prosecuted
Our 1935 Constitution took effect but it did not contain criminally considering that he was not convicted in the
any specific provision on executive immunity. Then came impeachment proceedings against him?
the tumult of the martial law years under the late
President Ferdinand E. Marcos and the 1973 Constitution Held: We reject his argument that he cannot be
was born. In 1981, it was amended and one of the prosecuted for the reason that he must first be convicted
amendments involved executive immunity. Section 17, in the impeachment proceedings. The impeachment trial
Article VII stated: of petitioner Estrada was aborted by the walkout of the
prosecutors and by the events that led to his loss of the
“The President shall be immune from suit during his presidency. Indeed, on February 7, 2001, the Senate
tenure. Thereafter, no suit whatsoever shall lie for official passed Senate Resolution No. 83 “Recognizing that the
acts done by him or by others pursuant to his specific Impeachment Court is Functus Officio.” Since the
orders during his tenure. Impeachment Court is now functus officio, it is untenable
for petitioner to demand that he should first be
The immunities herein provided shall apply to the impeached and then convicted before he can be
incumbent President referred to in Article XVII of this prosecuted. The plea if granted, would put a perpetual
Constitution.” bar against his prosecution. Such a submission has
nothing to commend itself for it will place him in a better 85. Enumerate the groups of officers who are to be
situation than a non-sitting President who has not been appointed by the President under Section 16, Article VII
subjected to impeachment proceedings and yet can be of the 1987 Constitution, and identify those officers
the object of a criminal prosecution. To be sure, the whose appointments shall require confirmation by the
debates in the Constitutional Commission make it clear Commission on Appointments.
that when impeachment proceedings have become moot
due to the resignation of the President, the proper Held: Conformably, as consistently interpreted and ruled
criminal and civil cases may already be filed against him in the leading case of Sarmiento III v. Mison (Ibid.), and
x x x. in the subsequent cases of Bautista v. Salonga (172 SCRA
160), Quintos-Deles v. Constitutional Commission (177
This is in accord with our ruling in In Re: Saturnino SCRA 259), and Calderon v. Carale (208 SCRA 254), under
Bermudez (145 SCRA 160 [1986]) that “incumbent Section 16, Article VII, of the Constitution, there are four
Presidents are immune from suit or from being brought groups of officers of the government to be appointed by
to court during the period of their incumbency and the President:
tenure” but not beyond. Considering the peculiar
circumstance that the impeachment process against the First, the heads of the executive departments,
petitioner has been aborted and thereafter he lost the ambassadors, other public ministers and consuls, officers
presidency, petitioner Estrada cannot demand as a of the armed forces from the rank of colonel or naval
condition sine qua non to his criminal prosecution before captain, and other officers whose appointments are
the Ombudsman that he be convicted in the impeachment vested in him in this Constitution;
proceedings. (Estrada v. Desierto, G.R. Nos. 146710-
15, Mar. 2, 2001, en Banc [Puno]) Second, all other officers of the Government whose
appointments are not otherwise provided for by law;
84. State the reason why not all appointments made
by the President under the 1987 Constitution will no Third, those whom the President may be authorized by
longer require confirmation by the Commission on law to appoint;
Appointments.
Fourth, officers lower in rank whose appointments the
Held: The aforecited provision (Section 16, Article VII) Congress may by law vest in the President alone.
of the Constitution has been the subject of several cases
on the issue of the restrictive function of the Commission It is well-settled that only presidential appointees
on Appointments with respect to the appointing power of belonging to the first group require the confirmation by
the President. This Court touched upon the historical the Commission on Appointments. (Manalo v. Sistoza,
antecedent of the said provision in the case of Sarmiento 312 SCRA 239, Aug. 11, 1999, En Banc [Purisima])
III v. Mison (156 SCRA 549) in which it was ratiocinated
upon that Section 16 of Article VII of the 1987 86. Under Republic Act 6975 (the DILG Act of 1990),
Constitution requiring confirmation by the Commission on the Director General, Deputy Director General, and other
Appointments of certain appointments issued by the top officials of the Philippine National Police (PNP) shall
President contemplates a system of checks and balances be appointed by the President and their appointments
between the executive and legislative branches of shall require confirmation by the Commission on
government. Experience showed that when almost all Appointments. Respondent Sistoza was appointed
presidential appointments required the consent of the Director General of the PNP but he refused to submit his
Commission on Appointments, as was the case under the appointment papers to the Commission on Appointments
1935 Constitution, the commission became a venue of for confirmation contending that his appointment shall no
"horse trading" and similar malpractices. On the other longer require confirmation despite the express provision
hand, placing absolute power to make appointments in of the law requiring such confirmation. Should his
the President with hardly any check by the legislature, as contention be upheld?
what happened under the 1973 Constitution, leads to
abuse of such power. Thus was perceived the need to Held: It is well-settled that only presidential appointees
establish a "middle ground" between the 1935 and 1973 belonging to the first group (enumerated under the first
Constitutions. The framers of the 1987 Constitution sentence of Section 16, Article VII of the 1987
deemed it imperative to subject certain high positions in Constitution) require the confirmation by the Commission
the government to the power of confirmation of the on Appointments. The appointments of respondent
Commission on Appointments and to allow other positions officers who are not within the first category, need not be
within the exclusive appointing power of the President. confirmed by the Commission on Appointments. As held
(Manalo v. Sistoza, 312 SCRA 239, Aug. 11, 1999, in the case of Tarrosa v. Singson (232 SCRA 553),
En Banc [Purisima]) Congress cannot by law expand the power of confirmation
of the Commission on Appointments and require
confirmation of appointments of other government
officials not mentioned in the first sentence of Section 16 to those appointments made within two months
of Article VII of the 1987 Constitution. preceding the Presidential election and are similar to
those which are declared election offenses in the Omnibus
Consequently, unconstitutional are Sections 26 and 31 of Election Code; while the second consists of the so-called
Republic Act 6975 which empower the Commission on “midnight” appointments. The SC in In Re: Hon. Mateo
Appointments to confirm the appointments of public A. Valenzuela and Hon. Placido B. Vallarta, (298
officials whose appointments are not required by the SCRA 408, Nov. 9, 1998, En Banc [Narvasa C.J.])
Constitution to be confirmed. x x x. (Manalo v. Sistoza, clarified this when it held:
312 SCRA 239, Aug. 11, 1999, En Banc [Purisima])
“Section 15, Article VII has a broader scope than the
87. Will it be correct to argue that since the Philippine Aytona ruling. It may not unreasonably be deemed to
National Police is akin to the Armed Forces of the contemplate not only “midnight” appointments – those
Philippines, therefore, the appointments of police officers made obviously for partisan reasons as shown by their
whose rank is equal to that of colonel or naval captain will number and the time of their making – but also
require confirmation by the Commission on appointments presumed made for the purpose of
Appointments? influencing the outcome of the Presidential election.”

Held: This contention is x x x untenable. The Philippine 89. Discuss the nature of an ad-interim
National Police is separate and distinct from the Armed appointment. Is it temporary and, therefore, can be can
Forces of the Philippines. The Constitution, no less, sets be withdrawn or revoked by the President at her
forth the distinction. Under Section 4 of Article XVI of the pleasure?
1987 Constitution,
Held: An ad interim appointment is a
"The Armed Forces of the Philippines shall be composed permanent appointment because it takes effect
of a citizen armed force which shall undergo military immediately and can no longer be withdrawn by the
training and service, as may be provided by law. It shall President once the appointee has qualified into office.
keep a regular force necessary for the security of the The fact that it is subject to confirmation by the
State." Commission on Appointments does not alter its
permanent character. The Constitution itself makes an
On the other hand, Section 6 of the same Article of the ad interim appointment permanent in character by
Constitution ordains that: making it effective until disapproved by the Commission
on Appointments or until the next adjournment of
"The State shall establish and maintain one police force, congress. The second paragraph of Section 16, Article
which shall be national in scope and civilian in character VII of the Constitution provides as follows:
to be administered and controlled by a national police
commission. The authority of local executives over the “The President shall have the power to make
police units in their jurisdiction shall be provided by law." appointments during the recess of the Congress,
whether voluntary or compulsory, but such
To so distinguish the police force from the armed forces, appointments shall be effective only until disapproval by
Congress enacted Republic Act 6975 x x x. the Commission on Appointments or until the next
adjournment of the Congress.”
Thereunder, the police force is different from and
independent of the armed forces and the ranks in the Thus, the ad interim appointment remains effective until
military are not similar to those in the Philippine National such disapproval or next adjournment, signifying that it
Police. Thus, directors and chief superintendents of the can no longer be withdrawn or revoked by the President.
PNP x x x do not fall under the first category of The fear that the President can withdraw or revoke at
presidential appointees requiring confirmation by the any time and for any reason an ad interim appointment
Commission on Appointments. (Manalo v. Sistoza, 312 is utterly without basis.
SCRA 239, Aug. 11, 1999, En Banc [Purisima])
More than half a century ago, this Court had
88. To what types of appointments is Section 15, already ruled that an ad interim appointment is
Article VII of the 1987 Constitution (prohibiting the permanent in character. In Summers v. Ozaeta (81 Phil.
President from making appointments two months before 754 [1948]), decided on October 25, 1948, we held that:
the next presidential elections and up to the end of his
term) directed against? “x x x an ad interim appointment is one made in
pursuance of paragraph (4), Section 10, Article VII of
Held: Section 15, Article VII is directed against two types the Constitution, which provides that the ‘President shall
of appointments: (1) those made for buying votes and (2) have the power to make appointments during the recess
those made for partisan considerations. The first refers of the Congress, but such appointments shall be
effective only until disapproval by the Commission on intended in the context of Philippine law. In referring to
Appointments or until the next adjournment of the Dr. Esteban’s appointments, the term is not descriptive
Congress.’ It is an appointment permanent in nature, of the nature of the appointments given to him. Rather,
and the circumstance that it is subject to confirmation it is used to denote the manner in which said
by the Commission on Appointments does not alter its appointments were made, that is, done by the President
permanent character. An ad interim appointment is of the Pamantasan in the meantime, while the Board of
disapproved certainly for a reason other than that its Regents, which is originally vested by the University
provisional period has expired. Said appointment is of Charter with the power of appointment, is unable to act.
course distinguishable from an ‘acting’ appointment X x x.”
which is merely temporary, good until another
permanent appointment is issued.” Thus, the term “ad interim appointment”, as
used in letters of appointment signed by the President,
The Constitution imposes no condition on the means a permanent appointment made by the President
effectivity of an ad interim appointment, and thus an ad in the meantime that Congress is in recess. It does not
interim appointment takes effect immediately. The mean a temporary appointment that can be withdrawn
appointee can at once assume office and exercise, as a or revoked at any time. The term, although not found in
de jure officer, all the powers pertaining to the office. the text of the Constitution, has acquired a definite legal
In Pacete v. Secretary of the Commission on meaning under Philippine jurisprudence. The Court had
Appointments (40 SCRA 58 [1971]), this Court again occasion to explain the nature of an ad interim
elaborated on the nature of an ad interim appointment appointment in the more recent case of Marohombsar v.
as follows: Court of Appeals (326 SCRA 62 [2000]), where the Court
stated:
“A distinction is thus made between the exercise of such
presidential prerogative requiring confirmation by the “We have already mentioned that an ad interim
Commission on Appointments when Congress is in appointment is not descriptive of the nature of the
session and when it is in recess. In the former, the appointment, that is, it is not indicative of whether the
President nominates, and only upon the consent of the appointment is temporary or in an acting capacity,
Commission on Appointments may the person thus rather it denotes the manner in which the appointment
named assume office. It is not so with reference to ad was made. In the instant case, the appointment
interim appointments. It takes effect at once. The extended to private respondent by then MSU President
individual chosen may thus qualify and perform his Alonto, Jr. was issued without condition nor limitation as
function without loss of time. His title to such office is to tenure. The permanent status of private respondent’s
complete. In the language of the Constitution, the appointment as Executive Assistant II was recognized
appointment is effective ‘until disapproval by the and attested to by the Civil Service Commission Regional
Commission on Appointments or until the next Office No. 12. Petitioner’s submission that private
adjournment of the Congress.’” respondent’s ad interim appointment is synonymous
with a temporary appointment which could be validly
Petitioner cites Black’s Law Dictionary which terminated at any time is clearly untenable. Ad interim
defines the term “ad interim” to mean “in the meantime” appointments are permanent appointment but their
or “for the time being.” Hence, petitioner argues that an terms are only until the Board disapproves them.”
ad interim appointment is undoubtedly temporary in
character. This argument is not new and was answered An ad interim appointee who has qualified and
by this Court in Pamantasan ng Lungsod ng Maynila v. assumed office becomes at that moment a government
Intermediate Appellate Court (140 SCRA 22 [1985]), employee and therefore part of the civil service. He
where we explained that: enjoys the constitutional protection that “[n]o officer or
employee in the civil service shall be removed or
“x x x From the arguments, it is easy to see why the suspended except for cause provided by law.” (Section
petitioner should experience difficulty in understanding 2[3], Article IX-B of the Constitution) Thus, an ad
the situation. Private respondent had been extended interim appointment becomes complete and irrevocable
several ‘ad interim’ appointments which petitioner once the appointee has qualified into office. The
mistakenly understands as appointments temporary in withdrawal or revocation of an ad interim appointment is
nature. Perhaps, it is the literal translation of the word possible only if it is communicated to the appointee
‘ad interim’ which creates such belief. The term is before the moment he qualifies, and any withdrawal or
defined by Black to mean ‘in the meantime’ or ‘for the revocation thereafter is tantamount to removal from
time being’. Thus, an officer ad interim is one appointed office (See concurring opinion of Justice Cesar Bengzon
to fill a vacancy, or to discharge the duties of the office in Erana v. Vergel de Dios, 85 Phil. 17 [1949]). Once an
during the absence or temporary incapacity of its regular appointee has qualified, he acquires a legal right to the
incumbent (Black’s Law Dictionary, Revised Fourth office which is protected not only by statute but also by
Edition, 1978). But such is not the meaning nor the use the Constitution. He can only be removed for cause,
after notice and hearing, consistent with the an acting Commissioner would undermine the
requirements of due process. independence of the COMELEC and hence violate the
Constitution. We declared then: “It would be more in
An ad interim appointment can be terminated for two keeping with the intent, purpose and aim of the framers
causes specified in the Constitution. The first cause is of the Constitution to appoint a permanent
the disapproval of his ad interim appointment by the Commissioner than to designate one to act temporarily.”
Commission on Appointments. The second cause is the
adjournment of Congress without the Commission on In the instant case, the President did in fact
Appointments acting on his appointment. These two appoint permanent Commissioners to fill the vacancies in
causes are resolutory conditions expressly imposed by the COMELEC, subject only to confirmation by the
the Constitution on all ad interim appointments. These Commission on Appointments. Benipayo, Borra and
resolutory conditions constitute, in effect, a Sword of Tuason were extended permanent appointments during
Damocles over the heads of ad interim appointees. No the recess of Congress. They were not appointed or
one, however, can complain because it is the designated in a temporary or acting capacity, unlike
Constitution itself that places the Sword of Damocles Commissioner Haydee Yorac in Brillantes v. Yorac and
over the heads of the ad interim appointees. Solicitor General Felix Bautista in Nacionalista Party v.
Bautista. The ad interim appointments of Benipayo,
While an ad interim appointment is permanent and Borra and Tuason are expressly allowed by the
irrevocable except as provided by law, an appointment Constitution which authorizes the President, during the
or designation in a temporary or acting capacity can be recess of Congress, to make appointments that take
withdrawn or revoked at the pleasure of the appointing effect immediately.
power (Binamira v. Garrucho, 188 SCRA 154 [1990];
Santiago v. Commission on Audit, 199 SCRA 125 [1991]; While the Constitution mandates that the COMELEC
Sevilla v. Court of Appeals, 209 SCRA 637 [1992]). A “shall be independent,” this provision should be
temporary or acting appointee does not enjoy any harmonized with the President’s power to extend ad
security of tenure, no matter how briefly. This is the interim appointments. To hold that the independence of
kind of appointment that the Constitution prohibits the the COMELEC requires the Commission on Appointments
President from making to the three independent to first confirm ad interim appointees before the
constitutional commissions, including the COMELEC. appointees can assume office will negate the President’s
Thus, in Brillantes v. Yorac (192 SCRA 358 [1990]), this power to make ad interim appointments. This is
Court struck down as unconstitutional the designation by contrary to the rule on statutory construction to give
then President Corazon Aquino of Associate meaning and effect to every provision of the law. It will
Commissioner Haydee Yorac as Acting Chairperson of also run counter to the clear intent of the framers of the
the COMELEC. This Court ruled that: Constitution.

“A designation as Acting Chairman is by its very terms The original draft of Section 16, Article VII of the
essentially temporary and therefore revocable at will. Constitution – on the nomination of officers subject to
No cause need be established to justify its revocation. confirmation by the Commission on Appointments – did
Assuming its validity, the designation of the respondent not provide for ad interim appointments. The original
as Acting Chairman of the Commission on Elections may intention of the framers of the Constitution was to do
be withdrawn by the President of the Philippines at any away with ad interim appointments because the plan
time and for whatever reason she sees fit. It is doubtful was for Congress to remain in session throughout the
if the respondent, having accepted such designation, will year except for a brief 30-day compulsory recess.
not be estopped from challenging its withdrawal. However, because of the need to avoid disruptions in
essential government services, the framers of the
The Constitution provides for many safeguards to the Constitution thought it wise to reinstate the provisions of
independence of the Commission on Elections, foremost the 1935 Constitution on ad interim appointments. X x
among which is the security of tenure of its members. x.
That guarantee is not available to the respondent as
Acting Chairman of the Commission on Elections by Xxx
designation of the President of the Philippines.”
Clearly, the reinstatement in the present Constitution of
Earlier, in Nacionalista Party v. Bautista (85 Phil. the ad interim appointing power of the President was for
101 [1949]), a case decided under the 1935 the purpose of avoiding interruptions in vital government
Constitution, which did not have a provision prohibiting services that otherwise would result from prolonged
temporary or acting appointments to the COMELEC, this vacancies in government offices, including the three
Court nevertheless declared unconstitutional the constitutional commissions. In his concurring opinion in
designation of the Solicitor General as acting member of Guevarra v. Inocentes (16 SCRA 379 [1966]), decided
the COMELEC. This Court ruled that the designation of under the 1935 Constitution, Justice Roberto
Concepcion, Jr. explained the rationale behind ad interim successful conduct of the May 2001 national elections,
appointments in this manner: right after the tumultuous EDSA II and EDSA III events,
was certainly essential in safeguarding and
“Now, why is the lifetime of ad interim appointments so strengthening our democracy.
limited? Because, if they expired before the session of
Congress, the evil sought to be avoided – interruption in Evidently, the exercise by the President in the instant
the discharge of essential functions – may take place. case of her constitutional power to make ad interim
Because the same evil would result if the appointments appointments prevented the occurrence of the very evil
ceased to be effective during the session of Congress sought to be avoided by the second paragraph of
and before its adjournment. Upon the other hand, once Section 16, Article VII of the Constitution. This power to
Congress has adjourned, the evil aforementioned may make ad interim appointments is lodged in the President
easily be conjured by the issuance of other ad interim to be exercised by her in her sound judgment. Under
appointments or reappointments.” the second paragraph of Section 16, Article VII of the
Constitution, the President can choose either of two
Indeed, the timely application of the last sentence of modes in appointing officials who are subject to
Section 16, Article VII of the Constitution barely avoided confirmation by the Commission on Appointments. First,
the interruption of essential government services in the while Congress is in session, the President may
May 2001 national elections. Following the decision of nominate the prospective appointee, and pending
this Court in Gaminde v. Commission on Appointments consent of the Commission on Appointments, the
(347 SCRA 655 [2000]), promulgated on December 13, nominee cannot qualify and assume office. Second,
2000, the terms of office of constitutional officers first during the recess of Congress, the President may extend
appointed under the Constitution would have to be an ad interim appointment which allows the appointee to
counted starting February 2, 1987, the date of immediately qualify and assume office.
ratification of the Constitution, regardless of the date of
their actual appointment. By this reckoning, the terms Whether the President chooses to nominate the
of office of three Commissioners of the COMELEC, prospective appointee or extend an ad interim
including the Chairman, would end on February 2, 2001 appointment is a matter within the prerogative of the
(See Section 1[2], Article IX-C of the Constitution). President because the Constitution grants her that
power. This Court cannot inquire into the propriety of
Xxx the choice made by the President in the exercise of her
constitutional power, absent grave abuse of discretion
During an election year, Congress normally goes on amounting to lack or excess of jurisdiction on her part,
voluntary recess between February and June considering which has not been shown in the instant case.
that many of the members of the House of
Representatives and the Senate run for re-election. In The issuance by Presidents of ad interim appointments
2001, the Eleventh Congress adjourned from January 9, to the COMELEC is a long-standing practice. X x x
2001 to June 3, 2001. Concededly, there was no more
time for Benipayo, Borra and Tuason, who were The President’s power to extend ad interim
originally extended ad interim appointments only on appointments may indeed briefly put the appointee at
March 22, 2001, to be confirmed by the Commission on the mercy of both the appointing and confirming
Appointments before the May 14, 2001 elections. powers. This situation, however, in only for a short
period – from the time of issuance of the ad interim
If Benipayo, Borra and Tuason were not extended ad appointment until the Commission on Appointments
interim appointments to fill up the three vacancies in the gives or withholds its consent. The Constitution itself
COMELEC, there would only have been one division sanctions this situation, as a trade-off against the evil of
functioning in the COMELEC instead of two during the disruptions in vital government services. This is also
May 2001 elections. Considering that the Constitution part of the check-and-balance under the separation of
requires that “all x x x election cases shall be heard and powers, as a trade-off against the evil of granting the
decided in division,” the remaining one division would President absolute and sole power to appoint. The
have been swamped with election cases. Moreover, Constitution has wisely subjected the President’s
since under the Constitution motions for reconsideration appointing power to the checking power of the
“shall be decided by the Commission en banc”, the mere legislature.
absence of one of the four remaining members would
have prevented a quorum, a less than ideal situation This situation, however, does not compromise the
considering that the Commissioners are expected to independence of the COMELEC as a constitutional body.
travel around the country before, during and after the The vacancies in the COMELEC are precisely staggered
elections. There was a great probability that disruptions to insure that the majority of its members hold
in the conduct of the May 2001 elections could occur confirmed appointments, and no one President will
because of the three vacancies in the COMELEC. The appoint all the COMELEC members. X x x. The special
constitutional safeguards that insure the independence decision by the Commission on Appointments to
of the COMELEC remain in place (See Sections, 3, 4, 5 withhold its consent to the appointment.
and 6, Article IX-A of the Constitution).
An ad interim appointment that is by-passed
In fine, we rule that the ad interim appointments because of lack of time or failure of the Commission on
extended by the President to Benipayo, Borra and Appointments to organize is another matter. A by-
Tuason, as COMELEC Chairman and Commissioners, passed appointment is one that has not been finally
respectively, do not constitute temporary or acting acted upon on the merits by the Commission on
appointments prohibited by Section 1 (2), Article IX-C of Appointments at the close of the session of Congress.
the Constitution. (Matibag v. Benipayo, 380 SCRA There is no final decision by the Commission on
49, April 2, 2002, En Banc [Carpio]) Appointments to give or withhold its consent to the
appointment as required by the Constitution. Absent
90. Does the renewal of ad interim appointments such decision, the President is free to renew the ad
violate the prohibition on reappointment under Section interim appointment of a by-passed appointee. This is
1(2), Article IX-C of the 1987 Constitution? recognized in Section 17 of the Rules of the Commission
on Appointments x x x. Hence, under the Rules of the
Held: Petitioner also argues that assuming the Commission on Appointments, a by-passed appointment
first ad interim appointment and the first assumption of can be considered again if the President renew the
office of Benipayo, Borra and Tuason are constitutional, appointment.
the renewal of their ad interim appointments and their
subsequent assumption of office to the same positions It is well-settled in this jurisdiction that the
violate the prohibition on reappointment under Section 1 President can renew the ad interim appointments of by-
(2), Article IX-C of the Constitution, which provides as passed appointees. Justice Roberto Concepcion, Jr.
follows: lucidly explained in his concurring opinion in Guevarra v.
Inocentes (Supra, note 34) why by-passed ad interim
“The Chairman and the Commissioners shall be appointees could be extended new appointments, thus:
appointed by the President with the consent of the
Commission on Appointments for a term of seven years “In short, an ad interim appointment ceases to be
without reappointment. Of those first appointed, three effective upon disapproval by the Commission, because
Members shall hold office for seven years, two Members the incumbent can not continue holding office over the
for five years, and the last Members for three years, positive objection of the Commission. It ceases, also,
without reappointment.” upon “the next adjournment of the Congress”, simply
because the President may then issue new appointments
Petitioner theorizes that once an ad interim appointee is – not because of implied disapproval of the Commission
by-passed by the Commission on Appointments, his ad deduced from its intention during the session of
interim appointment can no longer be renewed because Congress, for, under the Constitution, the Commission
this will violate Section 1 (2), Article IX-C of the may affect adversely the interim appointments only by
Constitution which prohibits reappointments. Petitioner action, never by omission. If the adjournment of
asserts that this is particularly true to permanent Congress were an implied disapproval of ad interim
appointees who have assumed office, which is the appointments made prior thereto, then the President
situation of Benipayo, Borra and Tuason if their ad could no longer appoint those so by-passed by the
interim appointments are deemed permanent in Commission. But, the fact is that the President may
character. reappoint them, thus clearly indicating that the reason
for said termination of the ad interim appointments is
There is no dispute that an ad interim appointee not the disapproval thereof allegedly inferred from said
disapproved by the Commission on Appointments can no omission of the Commission, but the circumstance that
longer be extended a new appointment. The upon said adjournment of the Congress, the President is
disapproval is a final decision of the Commission on free to make ad interim appointments or
Appointments in the exercise of its checking power on reappointments.”
the appointing authority of the President. The
disapproval is a decision on the merits, being a refusal Guevarra was decided under the 1935 Constitution from
by the Commission on Appointments to give its consent where the second paragraph of Section 16, Article VII of
after deliberating on the qualifications of the appointee. the present Constitution on ad interim appointments was
Since the Constitution does not provide for any appeal lifted verbatim. The jurisprudence under the 1935
from such decision, the disapproval is final and binding Constitution governing ad interim appointments by the
on the appointee as well as on the appointing power. In President is doubtless applicable to the present
this instance, the President can no longer renew the Constitution. The established practice under the present
appointment not because of the constitutional Constitution is that the President can renew the
prohibition on appointment, but because of a final appointments of by-passed ad interim appointees. This
is a continuation of the well-recognized practice under
the 1935 Constitution, interrupted only by the 1973 The framers of the Constitution made it quite
Constitution which did not provide for a Commission on clear that any person who has served any term of office
Appointments but vested sole appointing power in the as COMELEC member – whether for a full term of seven
President. years, a truncated term of five or three years, or even
an unexpired term for any length of time – can no
The prohibition on reappointment in Section 1 longer be reappointed to the COMELEC. X x x
(2), Article IX-C of the Constitution applies neither to
disapproval nor by-passed ad interim appointments. A Xxx
disapproved ad interim appointment cannot be revived
by another ad interim appointment because the In Visarra v. Miraflor (8 SCRA 1 [1963]), Justice Angelo
disapproval is final under Section 16, Article VII of the Bautista, in his concurring opinion, quoted Nacionalista
Constitution, and not because a reappointment is v. De Vera (85 Phil. 126 [1949]) that a [r]eappointment
prohibited under Section 1 (2), Article IX-C of the is not prohibited when a Commissioner has held, office
Constitution. A by-passed ad interim appointment only for, say, three or six years, provided his term will
cannot be revived by a new ad interim appointment not exceed nine years in all.” This was the
because there is no final disapproval under Section 16, interpretation despite the express provision in the 1935
Article VII of the Constitution, and such new Constitution that a COMELEC member “shall hold office
appointment will not result in the appointee serving for a term of nine years and may not be reappointed.”
beyond the fixed term of seven years.
To foreclose this interpretation, the phrase
Section 1 (2), Article IX-C of the Constitution “without reappointment” appears twice in Section 1 (2),
provides that “[t]he Chairman and the Commissioners Article IX-C of the present Constitution. The first phrase
shall be appointed x x x for a term of seven years prohibits reappointment of any person previously
without reappointment.” There are four situations appointed for a term of seven years. The second phrase
where this provision will apply. The first situation is prohibits reappointment of any person previously
where an ad interim appointee to the COMELEC, after appointed for a term of five or three years pursuant to
confirmation by the Commission on Appointments, the first set of appointees under the Constitution. In
serves his full seven-year term. Such person cannot be either case, it does not matter if the person previously
reappointed to the COMELEC, whether as a member or appointed completes his term of office for the intention
as a chairman, because he will then be actually serving is to prohibit any reappointment of any kind.
more than seven years. The second situation is where
the appointee, after confirmation, serves a part of his However, an ad interim appointment that has
term and then resigns before his seven-year term of lapsed by inaction of the Commission on Appointments
office ends. Such person cannot be reappointed, does not constitute a term of office. The period from
whether as a member or as a chair, to a vacancy arising the time the ad interim appointment is made to the time
from retirement because a reappointment will result in it lapses is neither a fixed term nor an unexpired term.
the appointee also serving more than seven years. The To hold otherwise would mean that the President by his
third situation is where the appointee is confirmed to unilateral action could start and complete the running of
serve the unexpired term of someone who died or a term of office in the COMELEC without the consent of
resigned, and the appointee completes the unexpired the Commission on Appointments. This interpretation
term. Such person cannot be reappointed, whether as a renders inutile the confirming power of the Commission
member or chair, to a vacancy arising from retirement on Appointments.
because a reappointment will result in the appointee
also serving more than seven years. The phrase “without reappointment” applies
only to one who has been appointed by the President
The fourth situation is where the appointee has and confirmed by the Commission on Appointments,
previously served a term of less than seven years, and a whether or not such person completes his term of office.
vacancy arises from death or resignation. Even if it will There must be a confirmation by the Commission on
not result in his serving more than seven years, a Appointments of the previous appointment before the
reappointment of such person to serve an unexpired prohibition on reappointment can apply. To hold
term is also prohibited because his situation will be otherwise will lead to absurdities and negate the
similar to those appointed under the second sentence of President’s power to make ad interim appointments.
Section 1 (2), Article IX-C of the Constitution. This
provision refers to the first appointees under the In the great majority of cases, the Commission
Constitution whose terms of office are less than seven on Appointments usually fails to act, for lack of time, on
years, but are barred from ever being reappointed under the ad interim appointments first issued to appointees.
any situation. Not one of these four situations applies to If such ad interim appointments can no longer be
the case of Benipayo, Borra and Tuason. renewed, the President will certainly hesitate to make ad
interim appointments because most of her appointees reappointments in Section 1 (2), Article IX-C of the
will effectively be disapproved by mere inaction of the Constitution. (Matibag v. Benipayo, 380 SCRA 49,
Commission on Appointments. This will nullify the April 2, 2002, En Banc [Carpio])
constitutional power of the President to make ad interim
appointments, a power intended to avoid disruptions in
vital government services. This Court cannot subscribe 91. Ma. Evelyn S. Abeja was a municipal mayor. She
to a proposition that will wreak havoc on vital ran for reelection but lost. Before she vacated her office,
government services. though, she extended permanent appointments to
fourteen new employees of the municipal government.
The prohibition on reappointment is common to The incoming mayor, upon assuming office, recalled said
the three constitutional commissions. The framers of appointments contending that these were “midnight
the present Constitution prohibited reappointments for appointments” and, therefore, prohibited under Sec. 15,
two reasons. The first is to prevent a second Art. VII of the 1987 Constitution. Should the act of the
appointment for those who have been previously new mayor of recalling said appointments on the
appointed and confirmed even if they served for less aforestated ground be sustained?
than seven years. The second is to insure that the
members of the three constitutional commissions do not Held: The records reveal that when the petitioner
serve beyond the fixed term of seven years. X x x. brought the matter of recalling the appointments of the
fourteen (14) private respondents before the CSC, the
Xxx only reason he cited to justify his action was that these
were “midnight appointments” that are forbidden under
Plainly, the prohibition on reappointment is intended to Article VII, Section 15 of the Constitution. However, the
insure that there will be no reappointment of any kind. CSC ruled, and correctly so, that the said prohibition
On the other hand, the prohibition on temporary or applies only to presidential appointments. In truth and in
acting appointments is intended to prevent any fact, there is no law that prohibits local elective officials
circumvention of the prohibition on reappointment that from making appointments during the last days of his or
may result in an appointee’s total term of office her tenure. (De Rama v. Court of Appeals (353
exceeding seven years. The evils sought to be avoided SCRA 94, Feb. 28, 2001, En Banc [Ynares-
by the twin prohibitions are very specific – Santiago])
reappointment of any kind and exceeding one’s term in
office beyond the maximum period of seven years.

Not contented with these ironclad twin prohibitions, the 92. Distinguish the President’s power to call out the
framers of the Constitution tightened even further the armed forces as their Commander-in-Chief in order to
screws on those who might wish to extend their terms of prevent or suppress lawless violence, invasion or
office. Thus, the word “designated” was inserted to plug rebellion, from his power to proclaim martial and suspend
any loophole that might be exploited by violators of the the privilege of the writ of habeas corpus. Explain why
Constitution x x x. the former is not subject to judicial review while the latter
two are.
The ad interim appointments and subsequent
renewals of appointments of Benipayo, Borra and Held: There is a clear textual commitment under the
Tuason do not violate the prohibition on reappointments Constitution to bestow on the President full discretionary
because there were no previous appointments that were power to call out the armed forces and to determine the
confirmed by the Commission on Appointments. A necessity for the exercise of such power. Section 18,
reappointment presupposes a previous confirmed Article VII of the Constitution, which embodies the powers
appointment. The same ad interim appointments and of the President as Commander-in-Chief, provides in part:
renewal of appointments will also not breach the seven-
year term limit because all the appointments and The President shall be the Commander-in-Chief of all
renewals of appointments of Benipayo, Borra and armed forces of the Philippines and whenever it becomes
Tuason are for a fixed term expiring on February 2, necessary, he may call out such armed forces to prevent
2008. Any delay in their confirmation will not extend the or suppress lawless violence, invasion or rebellion. In
expiry date of their terms of office. Consequently, there case of invasion or rebellion, when the public safety
is no danger whatsoever that the renewal of the ad requires it, he may, for a period not exceeding sixty days,
interim appointments of these three respondents will suspend the privilege of the writ of habeas corpus, or
result in any of the evils intended to be exorcised by the place the Philippines or any part thereof under martial
twin prohibitions in the Constitution. The continuing law.
renewal of the ad interim appointment of these three
respondents, for so long as their terms of office expire The full discretionary power of the President to determine
on February 2, 2008, does not violate the prohibition on the factual basis for the exercise of the calling out power
is also implied and further reinforced in the rest of Section information might be difficult to verify, or wholly
18, Article VII x x x. unavailable to the courts. In many instances, the
evidence upon which the President might decide that
Under the foregoing provisions, Congress may revoke there is a need to call out the armed forces may be of a
such proclamations (of martial law) or suspension (of the nature not constituting technical proof.
privilege of the writ of habeas corpus) and the Court may
review the sufficiency of the factual basis thereof. On the other hand, the President as Commander-in-Chief
However, there is no such equivalent provision dealing has a vast intelligence network to gather information,
with the revocation or review of the President's action to some of which may be classified as highly confidential or
call out the armed forces. The distinction places the affecting the security of the state. In the exercise of the
calling out power in a different category from the power power to call, on-the-spot decisions may be imperatively
to declare martial law and the power to suspend the necessary in emergency situations to avert great loss of
privilege of the writ of habeas corpus, otherwise, the human lives and mass destruction of property. Indeed,
framers of the Constitution would have simply lumped the decision to call out the military to prevent or suppress
together the three powers and provided for their lawless violence must be done swiftly and decisively if it
revocation and review without any qualification. were to have any effect at all. Such a scenario is not
Expressio unios est exclusio alterius. X x x. That the farfetched when we consider the present situation in
intent of the Constitution is exactly what its letter says, Mindanao, where the insurgency problem could spill over
i.e., that the power to call is fully discretionary to the the other parts of the country. The determination of the
President, is extant in the deliberation of the necessity for the calling out power if subjected to
Constitutional Commission x x x. unfettered judicial scrutiny could be a veritable
prescription for disaster, as such power may be unduly
The reason for the difference in the treatment of the straitjacketed by an injunction or a temporary restraining
aforementioned powers highlights the intent to grant the order every time it is exercised.
President the widest leeway and broadest discretion in
using the power to call out because it is considered as the Thus, it is the unclouded intent of the Constitution to vest
lesser and more benign power compared to the power to upon the President, as Commander-in-Chief of the Armed
suspend the privilege of the writ of habeas corpus and the Forces, full discretion to call forth the military when in his
power to impose martial law, both of which involve the judgment it is necessary to do so in order to prevent or
curtailment and suppression of certain basic civil rights suppress lawless violence, invasion or rebellion. Unless
and individual freedoms, and thus necessitating the petitioner can show that the exercise of such
safeguards by Congress and review by this Court. discretion was gravely abused, the President's exercise of
judgment deserves to be accorded respect from this
Moreover, under Section 18, Article VII of the Court. (Integrated Bar of the Philippines v. Hon.
Constitution, in the exercise of the power to suspend the Ronaldo B. Zamora, G.R. No. 141284, Aug. 15,
privilege of the writ of habeas corpus or to impose martial 2000, En Banc [Kapunan])
law, two conditions must concur: (1) there must be an
actual invasion or rebellion and, (2) public safety must 93. By issuing a TRO on the date convicted rapist Leo
require it. These conditions are not required in the case Echegaray is to be executed by lethal injection, the
of the power to call out the armed forces. The only Supreme Court was criticized on the ground, among
criterion is that "whenever it becomes necessary," the others, that it encroached on the power of the President
President may call the armed forces "to prevent or to grant reprieve under Section 19, Article VII, 1987
suppress lawless violence, invasion or rebellion." The Constitution. Justify the SC's act.
implication is that the President is given full discretion and
wide latitude in the exercise of the power to call as Held: Section 19, Article VII of the 1987 Constitution is
compared to the two other powers. simply the source of power of the President to grant
reprieves, commutations, and pardons and remit fines
If the petitioner fails, by way of proof, to support the and forfeitures after conviction by final judgment. This
assertion that the President acted without factual basis, provision, however, cannot be interpreted as denying the
then this Court cannot undertake an independent power of courts to control the enforcement of their
investigation beyond the pleadings. The factual necessity decisions after the finality. In truth, an accused who has
of calling out the armed forces is not easily quantifiable been convicted by final judgment still possesses collateral
and cannot be objectively established since matters rights and these rights can be claimed in the appropriate
considered for satisfying the same is a combination of courts. For instance, a death convict who becomes insane
several factors which are not always accessible to the after his final conviction cannot be executed while in a
courts. Besides the absence of textual standards that the state of insanity (See Article 79 of the Revised Penal
court may use to judge necessity, information necessary Code). The suspension of such a death sentence is
to arrive at such judgment might also prove undisputably an exercise of judicial power. It is not
unmanageable for the courts. Certain pertinent usurpation of the presidential power of reprieve though
its effect is the same – the temporary suspension of the filed against him, warrants the same. Courts have no
execution of the death convict. In the same vein, it authority to interfere with the grant by the President of a
cannot be denied that Congress can at any time amend pardon to a convicted criminal. It has been our fortified
R.A. No. 7659 by reducing the penalty of death to life ruling that a final judicial pronouncement as to the guilt
imprisonment. The effect of such an amendment is like of a pardonee is not a requirement for the President to
that of commutation of sentence. But by no stretch of determine whether or not there has been a breach of the
the imagination can the exercise by Congress of its terms of a conditional pardon. There is likewise nil a basis
plenary power to amend laws be considered as a violation for the courts to effectuate the reinstatement of a
of the President’s power to commute final sentences of conditional pardon revoked by the President in the
conviction. The powers of the Executive, the Legislative exercise of powers undisputably solely and absolutely in
and the Judiciary to save the life of a death convict do not his office. (In Re: Wilfredo Sumulong Torres, 251
exclude each other for the simple reason that there is no SCRA 709, Dec. 29, 1995 [Hermosisima])
higher right than the right to life. (Echegaray v.
Secretary of Justice, 301 SCRA 96, Jan. 19, 1999, 95. Who has the power to ratify a treaty?
En Banc [Puno])
Held: In our jurisdiction, the power to ratify is vested in
94. Discuss the nature of a conditional pardon. Is its the President and not, as commonly believed, in the
grant or revocation by the President subject to judicial legislature. The role of the Senate is limited only to giving
review? or withholding its consent, or concurrence, to the
ratification. (BAYAN [Bagong Alyansang
Held: A conditional pardon is in the nature of a contract Makabayan] v. Executive Secretary Ronaldo
between the sovereign power or the Chief Executive and Zamora, G.R. No. 138570, Oct. 10, 2000, En Banc
the convicted criminal to the effect that the former will [Buena])
release the latter subject to the condition that if he does
not comply with the terms of the pardon, he will be 96. What is the power of impoundment of the
recommitted to prison to serve the unexpired portion of President? What are its principal sources?
the sentence or an additional one (Alvarez v. Director of
Prisons, 80 Phil. 50). By the pardonee’s consent to the Held: Impoundment refers to the refusal of the
terms stipulated in this contract, the pardonee has President, for whatever reason, to spend funds made
thereby placed himself under the supervision of the Chief available by Congress. It is the failure to spend or
Executive or his delegate who is duty-bound to see to it obligate budget authority of any type.
that the pardonee complies with the terms and conditions
of the pardon. Under Section 64(i) of the Revised Proponents of impoundment have invoked at least three
Administrative Code, the Chief Executive is authorized to principal sources of the authority of the President.
order “the arrest and re-incarceration of any such person Foremost is the authority to impound given to him either
who, in his judgment, shall fail to comply with the expressly or impliedly by Congress. Second is the
condition, or conditions of his pardon, parole, or executive power drawn from the President’s role as
suspension of sentence.” It is now a well-entrenched rule Commander-in-Chief. Third is the Faithful Execution
in this jurisdiction that this exercise of presidential Clause.
judgment is beyond judicial scrutiny. The determination
of the violation of the conditional pardon rests exclusively The proponents insist that a faithful execution of the laws
in the sound judgment of the Chief Executive, and the requires that the President desist from implementing the
pardonee, having consented to place his liberty on law if doing so would prejudice public interest. An
conditional pardon upon the judgment of the power that example given is when through efficient and prudent
has granted it, cannot invoke the aid of the courts, management of a project, substantial savings are made.
however erroneous the findings may be upon which his In such a case, it is sheer folly to expect the President to
recommitment was ordered. spend the entire amount budgeted in the law.
(PHILCONSA v. Enriquez, 235 SCRA 506, Aug. 9,
It matters not that the pardonee has allegedly been 1994 [Quiason])
acquitted in two of the three criminal cases filed against
him subsequent to his conditional pardon, and that the 97. Distinguish the President’s power of general
third remains pending for thirteen (13) years in apparent supervision over local governments from his control
violation of his right to a speedy trial. power.

Ultimately, solely vested in the Chief Executive, who in Held: On many occasions in the past, this
the first place was the exclusive author of the conditional Court has had the opportunity to distinguish the power
pardon and of its revocation, is the corollary prerogative of supervision from the power of control. In Taule v.
to reinstate the pardon if in his own judgment, the Santos (200 SCRA 512 [1991]), we held that the Chief
acquittal of the pardonee from the subsequent charges Executive wielded no more authority than that of
checking whether a local government or the officers
thereof perform their duties as provided by statutory It is the considered view of the Court, given the
enactments. He cannot interfere with local governments above disquisition, that the phrase “upon
provided that the same or its officers act within the recommendation of the Secretary,” found in Section 9,
scope of their authority. Supervisory power, when Chapter II, Title III, Book IV, of the Revised
contrasted with control, is the power of mere oversight Administrative Code, should be interpreted, as it is
over an inferior body; it does not include any restraining normally so understood, to be a mere advise, exhortation
authority over such body (Ibid.). Officers in control lay or indorsement, which is essentially persuasive in
down the rules in the doing of an act. If they are not character and not binding or obligatory upon the party to
followed, it is discretionary on his part to order the act whom it is made. The recommendation is here nothing
undone or redone by his subordinate or he may even really more than advisory in nature. The President, being
decide to do it himself. Supervision does not cover such the head of the Executive Department, could very well
authority. Supervising officers merely see to it that the disregard or do away with the action of the departments,
rules are followed, but he himself does not lay down bureaus or offices even in the exercise of discretionary
such rules, nor does he have the discretion to modify or authority, and in so opting, he cannot be said as having
replace them. If the rules are not observed, he may acted beyond the scope of his authority.
order the work done or re-done to conform to the
prescribed rules. He cannot prescribe his own manner The doctrine in San Juan, relied upon by
for the doing of the act (Drilon v. Lim, supra, 142). petitioners, is tangential. While the tenor of the legal
(Bito-Onon v. Fernandez, 350 SCRA 732, Jan. 31, provision in Executive Order No. 112 has some similarity
2001, 3rd Div. [Gonzaga-Reyes]) with the provision in the 1987 Administrative Code in
question, it is to be pointed out, however, that San Juan
98. Is the absence of a recommendation of the (196 SCRA 69), in construing the law, has distinctively
Secretary of Justice to the President fatal to the given stress to the constitutional mandate on local
appointment of respondent as prosecutor? autonomy; x x x. The Court there has explained that the
President merely exercises general supervision over local
Held: This question would x x x pivot on the proper government units and local officials (Section 4, Article X,
understanding of the provision of the Revised Constitution); hence, in the appointment of a Provincial
Administrative Code of 1987 (Book IV, Title III, Chapter Budget Officer, the executive department, through the
II, Section 9) to the effect that – Secretary of Budget and Management, indeed had to
share the questioned power with the local government.
“All provincial and city prosecutors and their assistants
shall be appointed by the President upon the In the instant case, the recommendation of the
recommendation of the Secretary.” Secretary of Justice and the appointment of the President
are acts of the Executive Department itself, and there is
Petitioners contend that an appointment of a provincial no sharing of power to speak of, the latter being deemed
prosecutor mandatorily requires a prior recommendation for all intents and purposes as being merely an extension
of the Secretary of Justice endorsing the intended of the personality of the President. (Bermudez v.
appointment citing, by analogy, the case of San Juan v. Executive Secretary Ruben Torres, G.R. No.
CSC (196 SCRA 69) x x x. 131429, Aug. 4, 1999, 3rd Div. [Vitug])

When the Constitution or the law clothes the 99. Discuss the three distinct powers of the
President with the power to appoint a subordinate officer, President under Section 18, Art. VII of the 1987
such conferment must be understood as necessarily Constitution. Are they subject to judicial review, or are
carrying with it an ample discretion of whom to appoint. they political questions?
It should be here pertinent to state that the President is
the head of government whose authority includes the Ans.: There are three distinct powers of the
power of control over all “executive departments, bureaus President under Sec. 18, Art. VII of the Constitution, to
and offices.” Control means the authority of an wit: 1) her calling out power, as Commander-in-Chief of
empowered officer to alter or modify, or even nullify or the Armed Forces; 2) her martial law power; and 3) her
set aside, what a subordinate officer has done in the power to suspend the privilege of the writ of habeas
performance of his duties, as well as to substitute the corpus.
judgment of the latter, as and when the former deems it
to be appropriate. Expressed in another way, the Her martial law power and her power to suspend
President has the power to assume directly the functions the privilege of the writ of habeas corpus are subject to
of an executive department, bureau and office. It can judicial review as expressly provided under Sec. 18, Art.
accordingly be inferred therefrom that the President can VII of the 1987 Constitution because these two are the
interfere in the exercise of discretion of officials under him greater powers, compared with her calling out power, as
or altogether ignore their recommendations. they involve the curtailment and suppression of certain
basic civil rights and individual freedoms (IBP v. Zamora, Held: An "actual case or controversy" means an existing
G.R. No. 141284, Aug. 15, 2000, En Banc [Kapunan]). case or controversy which is both ripe for resolution and
susceptible of judicial determination, and that which is not
Her calling out power is a political question and conjectural or anticipatory, or that which seeks to resolve
not subject to judicial power as this is the lesser and more hypothetical or feigned constitutional problems. A
benign of the three powers under Sec. 18, Art. VII of the petition raising a constitutional question does not present
1987 Constitution (IBP v. Zamora, G.R. No. 141284, Aug. an "actual controversy," unless it alleges a legal right or
15, 2000, En Banc [Kapunan]). It is a question in regard power. Moreover, it must show that a conflict of rights
to which full discretionary authority has been delegated exists, for inherent in the term "controversy" is the
by the Constitution to the President, as their Commander- presence of opposing views or contentions. Otherwise,
in-Chief, to call out the armed forces whenever she deems the Court will be forced to resolve issues which remain
it necessary in order to prevent or suppress lawless unfocused because they lack such concreteness provided
violence, invasion, or rebellion. To subject such calling when a question emerges precisely framed from a clash
out power to unfettered judicial scrutiny could be a of adversary arguments exploring every aspect of a multi-
veritable prescription for disaster as such power may be faceted situation embracing conflicting and demanding
unduly straitjacketed by an injunction or a TRO every time interests. The controversy must also be justiciable; that
it is exercised. is, it must be susceptible of judicial determination.
(Integrated Bar of the Philippines v. Hon. Ronaldo
Unless it can be shown that the exercise of such B. Zamora, G.R. No. 141284, Aug. 15, 2000, En
discretion to call out the armed forces was gravely Banc [Kapunan])
abused, the President’s exercise of judgment deserves to
be accorded respect from the Court. And the burden to 102. Petitioners Isagani Cruz and Cesar Europa
show that the President gravely abused her discretion in brought a suit for prohibition and mandamus as citizens
calling out the armed forces to prevent or suppress and taxpayers, assailing the constitutionality of certain
lawless violence, invasion, or rebellion, lies provisions of Republic Act No. 8371, otherwise known as
the Indigenous Peoples Rights Act of 1997 (IPRA), and its
Implementing Rules and Regulations. A preliminary issue
resolved by the SC was whether the petition presents an
The Judicial Department actual controversy.

100. What are the requisites before the Court can Held: Courts can only decide actual controversies, not
exercise the power of judicial review? hypothetical questions or cases. The threshold issue,
therefore, is whether an "appropriate case" exists for the
Held: 1. The time-tested standards for the exercise of exercise of judicial review in the present case.
judicial review are: (1) the existence of an appropriate
case; (2) an interest personal and substantial by the party Xxx
raising the constitutional question; (3) the plea that the
function be exercised at the earliest opportunity; and (4) In the case at bar, there exists a live controversy involving
the necessity that the constitutional question be passed a clash of legal rights. A law has been enacted, and the
upon in order to decide the case (Separate Opinion, Implementing Rules and Regulations approved. Money
Kapunan, J., in Isagani Cruz v. Secretary of has been appropriated and the government agencies
Environment and Natural Resources, et al., G.R. concerned have been directed to implement the statute.
No. 135385, Dec. 6, 2000, En Banc). It cannot be successfully maintained that we should await
the adverse consequences of the law in order to consider
2. When questions of constitutional significance are the controversy actual and ripe for judicial resolution. It
raised, the Court can exercise its power of judicial review is precisely the contention of the petitioners that the law,
only if the following requisites are complied with, namely: on its face, constitutes an unconstitutional abdication of
(1) the existence of an actual and appropriate case; (2) a State ownership over lands of the public domain and
personal and substantial interest of the party raising the other natural resources. Moreover, when the State
constitutional question; (3) the exercise of judicial review machinery is set into motion to implement an alleged
is pleaded at the earliest opportunity; and (4) the unconstitutional statute, this Court possesses sufficient
constitutional question is the lis mota of the case. authority to resolve and prevent imminent injury and
(Integrated Bar of the Philippines v. Hon. Ronaldo violation of the constitutional process. (Separate
B. Zamora, G.R. No. 141284, Aug. 15, 2000, En Opinion, Kapunan, J., in Isagani Cruz v. Secretary
Banc [Kapunan]) of Environment and Natural Resources, et al., G.R.
No. 135385, Dec. 6, 2000, En Banc)
101. What is an "actual case or controversy"?
103. What is the meaning of "legal standing" or locus
standi?
affected by the deployment of the Marines. x x x
Held: "Legal standing" or locus standi has been defined Moreover, the IBP x x x has not shown any specific injury
as a personal and substantial interest in the case such which it has suffered or may suffer by virtue of the
that the party has sustained or will sustain direct injury as questioned governmental act. Indeed, none of its
a result of the governmental act that is being challenged . members, whom the IBP purportedly represents, has
The term "interest" means a material interest, an interest sustained any form of injury as a result of the operation
in issue affected by the decree, as distinguished from of the joint visibility patrols. Neither is it alleged that any
mere interest in the question involved, or a mere of its members has been arrested or that their civil
incidental interest. The gist of the question of standing is liberties have been violated by the deployment of the
whether a party alleges "such personal stake in the Marines. What the IBP projects as injurious is the
outcome of the controversy as to assure that concrete supposed "militarization" of law enforcement which might
adverseness which sharpens the presentation of issues threaten Philippine democratic institutions and may cause
upon which the court depends for illumination of difficult more harm than good in the long run. Not only is the
constitutional questions." (Integrated Bar of the presumed "injury" not personal in character, it is likewise
Philippines v. Hon. Ronaldo B. Zamora, G.R. No. too vague, highly speculative and uncertain to satisfy the
141284, Aug. 15, 2000) requirement of standing. Since petitioner has not
successfully established a direct and personal injury as a
In addition to the existence of an actual case or consequence of the questioned act, it does not possess
controversy, a person who assails the validity of a statute the personality to assail the validity of the deployment of
must have a personal and substantial interest in the case, the Marines. This Court, however, does not categorically
such that, he has sustained, or will sustain, a direct injury rule that the IBP has absolutely no standing to raise
as a result of its enforcement. Evidently, the rights constitutional issues now or in the future. The IBP must,
asserted by petitioners as citizens and taxpayers are held by way of allegations and proof, satisfy this Court that it
in common by all the citizens, the violation of which may has sufficient stake to obtain judicial resolution of the
result only in a "generalized grievance". Yet, in a sense, controversy. (Integrated Bar of the Philippines v.
all citizen's and taxpayer's suits are efforts to air Hon. Ronaldo B. Zamora, G.R. No. 141284, Aug. 15,
generalized grievances about the conduct of government 2000, En Banc [Kapunan])
and the allocation of power. (Separate Opinion,
Kapunan, J., in Isagani Cruz v. Secretary of 105. Considering the lack of requisite standing of the
Environment and Natural Resources, et al., G.R. IBP to file the petition questioning the validity of the order
No. 135385, Dec. 6, 2000, En Banc) of the President to deploy and utilize the Philippine
Marines to assist the PNP in law enforcement, may the
104. Asserting itself as the official organization of Court still properly take cognizance of the case?
Filipino lawyers tasked with the bounden duty to uphold
the rule of law and the Constitution, the Integrated Bar of Held: Having stated the foregoing, it must be
the Philippines (IBP) filed a petition before the SC emphasized that this Court has the discretion to take
questioning the validity of the order of the President cognizance of a suit which does not satisfy the
commanding the deployment and utilization of the requirement of legal standing when paramount interest is
Philippine Marines to assist the Philippine National Police involved. In not a few cases, the Court has adopted a
(PNP) in law enforcement by joining the latter in visibility liberal attitude on the locus standi of a petitioner where
patrols around the metropolis. The Solicitor General the petitioner is able to craft an issue of transcendental
questioned the legal standing of the IBP to file the significance to the people . Thus, when the issues raised
petition? Resolve. are of paramount importance to the public, the Court may
brush aside technicalities of procedure. In this case, a
Held: In the case at bar, the IBP primarily anchors its reading of the petition shows that the IBP has advanced
standing on its alleged responsibility to uphold the rule of constitutional issues which deserve the attention of this
law and the Constitution. Apart from this declaration, Court in view of their seriousness, novelty and weight as
however, the IBP asserts no other basis in support of its precedents. Moreover, because peace and order are
locus standi. The mere invocation by the IBP of its duty under constant threat and lawless violence occurs in
to preserve the rule of law and nothing more, while increasing tempo, undoubtedly aggravated by the
undoubtedly true, is not sufficient to clothe it with Mindanao insurgency problem, the legal controversy
standing in this case. This is too general an interest which raised in the petition almost certainly will not go away. It
is shared by other groups and the whole citizenry. Based will stare us in the face again. It, therefore, behooves the
on the standards above-stated, the IBP has failed to Court to relax the rules on standing and to resolve the
present a specific and substantial interest in the resolution issue now, rather than later. (Integrated Bar of the
of the case. Its fundamental purpose which, under Philippines v. Hon. Ronaldo B. Zamora, G.R. No.
Section 2, Rule 139-A of the Rules of Court, is to elevate 141284, Aug. 15, 2000)
the standards of the law profession and to improve the
administration of justice is alien to, and cannot be
106. What are the requisites for the proper exercise On the other hand, if Benipayo is the lawful
of the power of judicial review? Illustrative case. COMELEC Chairman because he assumed office in
accordance with the Constitution, then petitioner’s
Held: Respondents assert that the petition fails reassignment is legal and she has no cause to complain
to satisfy all the four requisites before this Court may provided the reassignment is in accordance with the Civil
exercise its power of judicial review in constitutional Service Law. Clearly, petitioner has a personal and
cases. Out of respect for the acts of the Executive material stake in the resolution of the constitutionality of
department, which is co-equal with this Court, Benipayo’s assumption of office. Petitioner’s personal
respondents urge this Court to refrain from reviewing and substantial injury, if Benipayo is not the lawful
the constitutionality of the ad interim appointments COMELEC Chairman, clothes her with the requisite locus
issued by the President to Benipayo, Borra and Tuason standi to raise the constitutional issue in this petition.
unless all the four requisites are present. X x x
Respondents harp on petitioner’s belated act of
Respondents argue that the second, third and questioning the constitutionality of the ad interim
fourth requisites are absent in this case. Respondents appointments of Benipayo, Borra and Tuason. Petitioner
maintain that petitioner does not have a personal and filed the instant petition only on August 3, 2001, when
substantial interest in the case because she has not the first ad interim appointments were issued as early as
sustained a direct injury as a result of the ad interim March 22, 2001. However, it is not the date of filing of
appointments of Benipayo, Borra and Tuason and their the petition that determines whether the constitutional
assumption of office. Respondents point out that issue was raised at the earliest opportunity. The earliest
petitioner does not claim to be lawfully entitled to any of opportunity to raise a constitutional issue is to raise it in
the positions assumed by Benipayo, Borra or Tuason. the pleadings before a competent court that can resolve
Neither does petitioner claim to be directly injured by the same, such that, “if it is not raised in the pleadings,
the appointments of these three respondents. it cannot be considered on appeal.” (Joaquin G. Bernas,
The 1987 Constitution of the Republic of the Philippines:
Respondents also contend that petitioner failed A Commentary, p. 858 [1996], citing People v. Vera, 65
to question the constitutionality of the ad interim Phil. 56 [1937]). Petitioner questioned the
appointments at the earliest opportunity. Petitioner filed constitutionality of the ad interim appointments of
the petition only on August 3, 2001 despite the fact that Benipayo, Borra and Tuason when she filed her petition
the ad interim appointments of Benipayo, Borra and before this Court, which is the earliest opportunity for
Tuason were issued as early as March 22, 2001. pleading the constitutional issue before a competent
Moreover, the petition was filed after the third time that body. Furthermore, this Court may determine, in the
these three respondents were issued ad interim exercise of sound discretion, the time when a
appointments. constitutional issue may be passed upon (Ibid., citing
Sotto v. Commission on Elections, 76 Phil. 516 [1946]).
Respondents insist that the real issue in this There is no doubt petitioner raised the constitutional
case is the legality of petitioner’s reassignment from the issue on time.
EID to the Law Department. Consequently, the
constitutionality of the ad interim appointments is not Moreover, the legality of petitioner’s
the lis mota of this case. reassignment hinges on the constitutionality of
Benipayo’s ad interim appointment and assumption of
We are not persuaded. office. Unless the constitutionality of Benipayo’s ad
interim appointment and assumption of office is
Benipayo reassigned petitioner from the EID, resolved, the legality of petitioner’s reassignment from
where she was Acting Director, to the Law Department, the EID to the Law Department cannot be determined.
where she was placed on detail. Respondents claim that Clearly, the lis mota of this case is the very
the reassignment was “pursuant to x x x Benipayo’s constitutional issue raised by petitioner.
authority as Chairman of the Commission on Elections,
and as the Commission’s Chief Executive Officer.” In any event, the issue raised by petitioner is of
Evidently, respondents’ anchor the legality of petitioner’s paramount importance to the public. The legality of the
reassignment on Benipayo’s authority as Chairman of directives and decisions made by the COMELEC in the
the COMELEC. The real issue then turns on whether or conduct of the May 14, 2001 national elections may be
not Benipayo is the lawful Chairman of the COMELEC. put in doubt if the constitutional issue raised by
Even if petitioner is only an Acting director of the EID, petitioner is left unresolved. In keeping with this Court’s
her reassignment is without legal basis if Benipayo is not duty to determine whether other agencies of
the lawful COMELEC Chairman, an office created by the government have remained within the limits of the
Constitution. Constitution and have not abused the discretion given
them, this Court may even brush aside technicalities of
procedure and resolve any constitutional issue raised
(Ople v. Torres, 293 SCRA 1412 [1998]; others omitted). [1955]) the petition contesting the validity of a
Here the petitioner has complied with all the requisite regulation issued by the Secretary of Education requiring
technicalities. Moreover, public interest requires the private schools to secure a permit to operate was
resolution of the constitutional issue raised by petitioner. dismissed on the ground that all the petitioners have
(Matibag v. Benipayo, 380 SCRA 49, April 2, 2002, permits and are actually operating under the same. The
En Banc [Carpio]) petitioners questioned the regulation because of the
possibility that the permit might be denied them in the
107. What is the meaning of “justiciable controversy” future. This Court held that there was no justiciable
as requisite for the proper exercise of the power of controversy because the petitioners suffered no wrong
judicial review? Illustrative case. by the implementation of the questioned regulation and
therefore, they are not entitled to relief. A mere
Held: From a reading of the records it appears apprehension that the Secretary of Education will
to us that the petition was prematurely filed. Under the withdraw the permit does not amount to justiciable
undisputed facts there is as yet no justiciable controversy. The questioned regulation in the PACU
controversy for the court to resolve and the petition case may be questioned by a private school whose
should have been dismissed by the appellate court on permit to operate has been revoked or one whose
this ground. application therefore has been denied (Bernas, supra.).

We gather from the allegations of the petition This Court cannot rule on the basis of
and that of the petitioner’s memorandum that the petitioners’ speculation that the DENR will approve the
alleged application for certificate of ancestral land claim application of the heirs of Carantes. There must be an
(CALC) filed by the heirs of Carantes under the assailed actual governmental act which directly causes or will
DENR special orders has not been granted nor the CALC imminently cause injury to the alleged legal right of the
applied for, issued. The DENR is still processing the petitioner to possess the land before the jurisdiction of
application of the heirs of Carantes for a certificate of this Court may be invoked. There is no showing that the
ancestral land claim, which the DENR may or may not petitioners were being evicted from the land by the heirs
grant. It is evident that the adverse legal interests of Carantes under orders from the DENR. The
involved in this case are the competing claims of the petitioners’ allegation that certain documents from the
petitioners and that of the heirs of Carantes to possess a DENR were shown to them by the heirs of Carantes to
common portion of a piece of land. As the undisputed justify eviction is vague, and it would appear that the
facts stand there is no justiciable controversy between petitioners did not verify if indeed the respondent DENR
the petitioners and the respondents as there is no actual or its officers authorized the attempted eviction. Suffice
or imminent violation of the petitioners’ asserted right to it to say that by the petitioners’ own admission that the
possess the land by reason of the implementation of the respondents are still processing and have not approved
questioned administrative issuance. the application of the heirs of Carantes, the petitioners
alleged right to possess the land is not violated nor is in
A justiciable controversy has been defined as, “a definite imminent danger of being violated, as the DENR may or
and concrete dispute touching on the legal relations of may not approve Carantes’ application. Until such time,
parties having adverse legal interests” (Sinco, Philippine the petitioners are simply speculating that they might be
Political Law, 1962 ed., quoting from the U.S. evicted from the premises at some future time.
Declaratory Judgment Act of 1934, p. 360) which may Borrowing from the pronouncements of this Court in the
be resolved by a court of law through the application of PACU case, “They (the petitioners) have suffered no
a law (Macasiano v. National Housing Authority, 224 wrong under the terms of the law – and, naturally need
SCRA 238 [1993]; Bernas, The Constitution of the no relief in the form they now seek to obtain.” (PACU,
Republic of the Philippines: A Commentary, Vol. II, 1988 supra, at p. 810) If indeed the heirs of Carantes are
ed., pp. 274-275). Courts have no judicial power to trying to enter the land and disturbing the petitioners’
review cases involving political questions and as a rule, possession thereof even without prior approval by the
will desist from taking cognizance of speculative or DENR of the claim of the heirs of Carantes, the case is
hypothetical cases, advisory opinions and in cases that simply one of forcible entry. (Cutaran v. DENR, 350
has become moot (Cruz, Philippine Political Law, 1998 SCRA 697, Jan. 31, 2001, 3rd Div. [Gonzaga-
ed., p. 257-259). Subject to certain well-defined Reyes])
exceptions (Solicitor-General v. MMA, December 11,
1991, 204 SCRA 837; Dumlao v. Comelec, 95 SCRA 392 108. Should the Court still resolve the case despite that
[1980]) courts will not touch an issue involving the the issue has already become moot and academic?
validity of a law unless there has been a governmental Exception.
act accomplished or performed that has a direct adverse
effect on the legal right of the person contesting its Held: Neither do we agree that merely because a
validity (Tan v. Macapagal, 43 SCRA 678 [1972]). In the plebiscite had already been held in the case of the
case of PACU v. Secretary of Education (97 Phil. 806 proposed Barangay Napico, the petition of the
Municipality of Cainta has already been rendered moot Arroyo and the warrantless arrests allegedly effected by
and academic. The issue raised by the Municipality of virtue thereof, as having no basis both in fact and in
Cainta in its petition before the COMELEC against the law. Significantly, on May 6, 2001, President
holding of the plebiscite for the creation of Barangay Macapagal-Arroyo ordered the lifting of the declaration
Napico are still pending determination before the Antipolo of a “state of rebellion” in Metro Manila. Accordingly,
Regional Trial Court. the instant petitions have been rendered moot and
academic. As to petitioners’ claim that the proclamation
In Tan v. Commission on Elections (G.R. No. 73155, 142 of a “state of rebellion” is being used by the authorities
SCRA 727, 741-742 [1986]), we struck down the moot to justify warrantless arrests, the Secretary of Justice
and academic argument as follows – denies that it has issued a particular order to arrest
specific persons in connection with the “rebellion.” He
“Considering that the legality of the plebiscite itself is states that what is extant are general instructions to law
challenged for non-compliance with constitutional enforcement officers and military agencies to implement
requisites, the fact that such plebiscite had been held and Proclamation No. 38. x x x With this declaration,
a new province proclaimed and its officials appointed, the petitioners’ apprehensions as to warrantless arrests
case before Us cannot truly be viewed as already moot should be laid to rest. (Lacson v. Perez, 357 SCRA
and academic. Continuation of the existence of this newly 756, May 10, 2001, En Banc [Melo])
proclaimed province which petitioners strongly profess to
have been illegally born, deserves to be inquired into by 110. When is an action considered “moot”? May the
this Tribunal so that, if indeed, illegality attaches to its court still resolve the case once it has become moot and
creation, the commission of that error should not provide academic?
the very excuse for perpetration of such wrong. For this
Court to yield to the respondents’ urging that, as there Held: 1. It is alleged by respondent that, with respect
has been fait accompli, then this Court should passively to the PCCR [Preparatory Commission on Constitutional
accept and accede to the prevailing situation is an Reform], this case has become moot and academic. We
unacceptable suggestion. Dismissal of the instant agree.
petition, as respondents so propose is a proposition
fraught with mischief. Respondents’ submission will An action is considered “moot” when it no longer
create a dangerous precedent. Should this Court decline presents a justiciable controversy because the issues
now to perform its duty of interpreting and indicating involved have become academic or dead. Under E.O. No.
what the law is and should be, this might tempt again 43, the PCCR was instructed to complete its task on or
those who strut about in the corridors of power to before June 30, 1999. However, on February 19, 1999,
recklessly and with ulterior motives, create, merge, divide the President issued Executive Order No. 70 (E.O. No.
and/or alter the boundaries of political subdivisions, either 70), which extended the time frame for the completion of
brazenly or stealthily, confident that this Court will abstain the commission’s work x x x. The PCCR submitted its
from entertaining future challenges to their acts if they recommendations to the President on December 20, 1999
manage to bring about a fait accompli.” and was dissolved by the President on the same day. It
(City of Pasig v. COMELEC, 314 SCRA 179, Sept. had likewise spent the funds allocated to it. Thus, the
10, 1999, En Banc [Ynares-Santiago]) PCCR has ceased to exist, having lost its raison d’être.
Subsequent events have overtaken the petition and the
109. On May 1, 2001, President Macapagal-Arroyo, Court has nothing left to resolve.
faced by an “angry and violent mob armed with
explosives, firearms, bladed weapons, clubs, stones and The staleness of the issue before us is made more
other deadly weapons” assaulting and attempting to manifest by the impossibility of granting the relief prayed
break into Malacanang, issued Proclamation No. 38 for by petitioner. Basically, petitioner asks this Court to
declaring that there was a state of rebellion in the enjoin the PCCR from acting as such. Clearly, prohibition
National Capital Region. She likewise issued General is an inappropriate remedy since the body sought to be
Order No. 1 directing the Armed Forces of the enjoined no longer exists. It is well-established that
Philippines and the Philippine National Police to suppress prohibition is a preventive remedy and does not lie to
the rebellion in the National Capital Region. Warrantless restrain an act that is already fait accompli. At this point,
arrests of several alleged leaders and promoters of the any ruling regarding the PCCR would simply be in the
“rebellion” were thereafter effected. Hence, several nature of an advisory opinion, which is definitely beyond
petitions were filed before the SC assailing the the permissible scope of judicial power. (Gonzales v.
declaration of State of Rebellion by President Gloria Narvasa, 337 SCRA 733, Aug. 14, 2000, En Banc
Macapagal-Arroyo and the warrantless arrests allegedly [Gonzaga-Reyes])
effected by virtue thereof.
2. The petition which was filed by private
Held: All the foregoing petitions assail the declaration respondents before the trial court sought the issuance of
of state of rebellion by President Gloria Macapagal- a writ of mandamus, to command petitioners to admit
them for enrolment. Taking into account the admission subdivision or instrumentality and in doing so, a law is
of private respondents that they have finished their violated or some irregularity is committed, and that the
Nursing course at the Lanting College of Nursing even petitioner is directly affected by the alleged ultra vires act .
before the promulgation of the questioned decision, this The same pronouncement was made in Kilosbayan, Inc.
case has clearly been overtaken by events and should v. Guingona, Jr., (232 SCRA 110 [1994], where the Court
therefore be dismissed. However, the case of Eastern also reiterated its liberal stance in entertaining so-called
Broadcasting Corporation (DYRE) v. Dans, etc., et al., taxpayer's suits, especially when important issues are
G.R. No. 59329, July 19, 1985, 137 SCRA 628 is the involved. A closer examination of the facts of this case
authority for the view that "even if a case were moot and would readily demonstrate that petitioner's standing
academic, a statement of the governing principle is should not even be made an issue here, "since standing
appropriate in the resolution of dismissal for the guidance is a concept in constitutional law and here no
not only of the parties but of others similarly situated .” constitutional question is actually involved."
We shall adhere to this view and proceed to dwell on the
merits of this petition. (University of San Agustin, In the case at bar, disbursement of public funds
Inc. v. Court of Appeals, 230 SCRA 761, 770, March was only made in 1975 when the Province bought the
7, 1994 [Nocon]) lands from Ortigas at P110.00 per square meter in line
with the objectives of P.D. 674. Petitioner never referred
111. In connection with the May 11, 1998 elections, to such purchase as an illegal disbursement of public
the COMELEC issued a resolution prohibiting the conduct funds but focused on the alleged fraudulent reconveyance
of exit polls on the ground, among others, that it might of said property to Ortigas because the price paid was
cause disorder and confusion considering the randomness lower than the prevailing market value of neighboring
of selecting interviewees, which further makes the exit lots. The first requirement, therefore, which would make
polls unreliable. The constitutionality of this resolution this petition a taxpayer's suit is absent. The only
was challenged by ABS-CBN Broadcasting Corporation as remaining justification for petitioner to be allowed to
violative of freedom of expression. The Solicitor General pursue this action is whether it is, or would be, directly
contends that the petition has been rendered moot and affected by the act complained of. As we stated in
academic because the May 11, 1998 election has already Kilosbayan, Inc. v. Morato (supra.),
been held and done with and, therefore, there is no longer
any actual controversy to be resolved. Resolve. "Standing is a special concern in constitutional law
because in some cases suits are brought not by parties
Held: While the assailed Resolution referred specifically who have been personally injured by the operation of a
to the May 11, 1998 election, its implications on the law or by official action taken, but by concerned citizens,
people’s fundamental freedom of expression transcend taxpayers or voters who actually sue in the public interest.
the past election. The holding of periodic elections is a Hence the question in standing is whether such parties
basic feature of our democratic government. By its very have 'alleged such a personal stake in the outcome of the
nature, exit polling is tied up with elections. To set aside controversy as to assure that concrete adverseness which
the resolution of the issue now will only postpone a task sharpens the presentation of issues upon which the court
that could well crop up again in future elections. so largely depends for illumination of difficult
constitutional questions.' (Citing Baker v. Carr, 369 U.S.
In any event, in Salonga v. Cruz Pano (134 SCRA 438, 186, 7l. Ed. 2d 633 [1962])"
463, Feb. 18, 1985), the Court had occasion to reiterate
that it “also has the duty to formulate guiding and Undeniably, as a taxpayer, petitioner would somehow be
controlling constitutional principles, precepts, doctrines, adversely affected by an illegal use of public money.
or rules. It has the symbolic function of educating bench When, however, no such unlawful spending has been
and bar on the extent of protection given by constitutional shown, as in the case at bar, petitioner, even as a
guarantees.” Since the fundamental freedoms of speech taxpayer, cannot question the transaction validly
and of the press are being invoked here, we have resolved executed by and between the Province and Ortigas for the
to settle, for the guidance of posterity, whether they simple reason that it is not privy to said contract. In other
likewise protect the holding of exit polls and the words, petitioner has absolutely no cause of action, and
dissemination of data derived therefrom. (ABS-CBN consequently no locus standi, in the instant case. (The
Broadcasting Corporation v. COMELEC, G.R. No. Anti-Graft League of the Philippines, Inc. v. San
133486, Jan. 28, 2000, En Banc [Panganiban]) Juan, 260 SCRA 250, 253-255, Aug. 1, 1996, En
Banc [Romero])
112. Discuss the nature of a taxpayer’s suit. When
may it be allowed? 2. A taxpayer is deemed to have the standing to raise a
constitutional issue when it is established that public
Held: 1. Petitioner and respondents agree that to funds have been disbursed in alleged contravention of the
constitute a taxpayer's suit, two requisites must be met, law or the Constitution. Thus, a taxpayer’s action is
namely, that public funds are disbursed by a political properly brought only when there is an exercise by
Congress of its taxing or spending power (Flast v. Cohen, Meanwhile, in Bugnay Construction and
392 US 83, 20 L Ed 2d 947, 88 S Ct 1942). This was our Development Corporation v. Laron (176 SCRA 251
ruling in a recent case wherein petitioners [1989]), the Court declared that the trial court was wrong
Telecommunications and Broadcast Attorneys of the in allowing respondent Ravanzo to bring an action for
Philippines (TELEBAP) and GMA Network, Inc. questioned injunction in his capacity as a taxpayer in order to
the validity of Section 92 of B.P. Blg. 881 (otherwise question the legality of the contract of lease covering the
known as the “Omnibus Election Code”) requiring radio public market entered into between the City of Dagupan
and television stations to give free air time to the and petitioner. The Court declared that Ravanzo did not
Commission on Elections during the campaign period possess the requisite standing to bring such taxpayer’s
(Telecommunications and Broadcast Attorneys of the suit since “[o]n its face, and there is no evidence to the
Philippines, Inc. v. Commission on Elections, 289 SCRA contrary, the lease contract entered into between
337 [1998]). The Court held that petitioner TELEBAP did petitioner and the City shows that no public funds have
not have any interest as a taxpayer since the assailed law been or will be used in the construction of the market
did not involve the taxing or spending power of Congress. building.”

Many other rulings have premised the grant or denial of Coming now to the instant case, it is readily apparent that
standing to taxpayers upon whether or not the case there is no exercise by Congress of its taxing or spending
involved a disbursement of public funds by the legislature. power. The PCCR was created by the President by virtue
In Sanidad v. Commission on Elections (73 SCRA 333 of E.O. No. 43, as amended by E.O. No. 70. Under
[1976]), the petitioners therein were allowed to bring a Section 7 of E.O. No. 43, the amount of P3 million is
taxpayer’s suit to question several presidential decrees "appropriated" for its operational expenses "to be sourced
promulgated by then President Marcos in his legislative from the funds of the Office of the President.” x x x. The
capacity calling for a national referendum, with the Court appropriations for the PCCR were authorized by the
explaining that – President, not by Congress. In fact, there was no
appropriation at all. “In a strict sense, appropriation has
X x x [i]t is now an ancient rule that the valid source of a been defied ‘as nothing more than the legislative
statute – Presidential Decrees are of such nature – may authorization prescribed by the Constitution that money
be contested by one who will sustain a direct injury as a may be paid out of the Treasury,’ while appropriation
result of its enforcement. At the instance of taxpayers, made by law refers to ‘the act of the legislature setting
laws providing for the disbursement of public funds may apart or assigning to a particular use a certain sum to be
be enjoined, upon the theory that the expenditure of used in the payment of debt or dues from the State to its
public funds by an officer of the State for the purpose of creditors.’” The funds used for the PCCR were taken from
executing an unconstitutional act constitutes a funds intended for the Office of the President, in the
misapplication of such funds. The breadth of Presidential exercise of the Chief Executive’s power to transfer funds
Decree No. 991 carries an appropriation of Five Million pursuant to Section 25 (5) of Article VI of the Constitution.
Pesos for the effective implementation of its purposes.
Presidential Decree No. 1031 appropriates the sum of In the final analysis, it must be stressed that the
Eight Million Pesos to carry out its provisions. The interest Court retains the power to decide whether or not it will
of the aforenamed petitioners as taxpayers in the lawful entertain a taxpayer’s suit. In the case at bar, there being
expenditure of these amounts of public money sufficiently no exercise by Congress of its taxing or spending power,
clothes them with that personality to litigate the validity petitioner cannot be allowed to question the creation of
of the Decrees appropriating said funds x x x. the PCCR in his capacity as a taxpayer, but rather, he
must establish that he has a “personal and substantial
In still another case, the Court held that petitioners – the interest in the case and that he has sustained or will
Philippine Constitution Association, Inc., a non-profit civic sustain direct injury as a result of its enforcement.” In
organization – had standing as taxpayers to question the other words, petitioner must show that he is a real party
constitutionality of Republic Act No. 3836 insofar as it in interest – that he will stand to be benefited or injured
provides for retirement gratuity and commutation of by the judgment or that he will be entitled to the avails of
vacation and sick leaves to Senators and Representatives the suit. Nowhere in his pleadings does petitioner
and to the elective officials of both houses of Congress presume to make such a representation. (Gonzales v.
(Philippine Constitution Association, Inc. v. Gimenez, 15 Narvasa, 337 SCRA 733, Aug. 14, 2000, En Banc
SCRA 479 [1965]). And in Pascual v. Secretary of Public [Gonzaga-Reyes])
Works (110 Phil. 331 [1960]), the Court allowed petitioner
to maintain a taxpayer’s suit assailing the constitutional 113. What is a justiciable controversy? What are
soundness of Republic Act No. 920 appropriating P85,000 political questions?
for the construction, repair and improvement of feeder
roads within private property. All these cases involved Held: As a general proposition, a controversy is
the disbursement of public funds by means of a law. justiciable if it refers to a matter which is appropriate for
court review. It pertains to issues which are inherently
susceptible of being decided on grounds recognized by one of legality or validity, not its wisdom. Moreover, the
law. Nevertheless, the Court does not automatically jurisdiction to delimit constitutional boundaries has been
assume jurisdiction over actual constitutional cases given to this Court. When political questions are involved,
brought before it even in instances that are ripe for the Constitution limits the determination as to whether or
resolution. One class of cases wherein the Court hesitates not there has been a grave abuse of discretion amounting
to rule on are "political questions." The reason is that to lack or excess of jurisdiction on the part of the official
political questions are concerned with issues dependent whose action is being questioned.
upon the wisdom, not the legality, of a particular act or
measure being assailed. Moreover, the political question By grave abuse of discretion is meant simply capricious or
being a function of the separation of powers, the courts whimsical exercise of judgment that is patent and gross
will not normally interfere with the workings of another as to amount to an evasion of positive duty or a virtual
co-equal branch unless the case shows a clear need for refusal to perform a duty enjoined by law, or to act at all
the courts to step in to uphold the law and the in contemplation of law, as where the power is exercised
Constitution. in an arbitrary and despotic manner by reason of passion
or hostility. Under this definition, a court is without power
As Tanada v. Angara (103 Phil. 1051 [1957]) puts it, to directly decide matters over which full discretionary
political questions refer "to those questions which, under authority has been delegated. But while this Court has
the Constitution, are to be decided by the people in their no power to substitute its judgment for that of Congress
sovereign capacity, or in regard to which full discretionary or of the President, it may look into the question of
authority has been delegated to the legislative or whether such exercise has been made in grave abuse of
executive branch of government." Thus, if an issue is discretion. A showing that plenary power is granted
clearly identified by the text of the Constitution as matters either department of government may not be an obstacle
for discretionary action by a particular branch of to judicial inquiry, for the improvident exercise or abuse
government or to the people themselves then it is held to thereof may give rise to justiciable controversy.
be a political question. In the classic formulation of (Integrated Bar of the Philippines v. Hon. Ronaldo
Justice Brennan in Baker v. Carr (369 U.S. 186, 82 S Ct. B. Zamora, G.R. No. 141284, Aug. 15, 2000, En
691, 7 L. Ed. 663, 678 [1962]), "[p]rominent on the Banc [Kapunan])
surface of any case held to involve a political question is
found a textually demonstrable constitutional 114. Is the legitimacy of the assumption to the
commitment of the issue to a coordinate political Presidency of President Gloria Macapagal Arroyo a
department; or a lack of judicially discoverable and political question and, therefore, not subject to judicial
manageable standards for resolving it; or the impossibility review? Distinguish EDSA People Power I from EDSA
of deciding without an initial policy determination of a kind People Power II.
clearly for nonjudicial discretion; or the impossibility of a
court's undertaking independent resolution without Held: Respondents rely on the case of Lawyers League
expressing lack of the respect due coordinate branches of for a Better Philippines and/or Oliver A. Lozano v.
government; or an unusual need for unquestioning President Corazon C. Aquino, et al. and related cases to
adherence to a political decision already made; or the support their thesis that since the cases at bar involve the
potentiality of embarrassment from multifarious legitimacy of the government of respondent Arroyo, ergo,
pronouncements by various departments on the one they present a political question. A more cerebral reading
question." of the cited cases will show that they are inapplicable. In
the cited cases, we held that the government of former
The 1987 Constitution expands the concept of judicial President Aquino was the result of a successful revolution
review by providing that "(T)he Judicial power shall be by the sovereign people, albeit a peaceful one. No less
vested in one Supreme Court and in such lower courts as than the Freedom Constitution declared that the Aquino
may be established by law. Judicial power includes the government was installed through a direct exercise of the
duty of the courts of justice to settle actual controversies power of the Filipino people “in defiance of the provisions
involving rights which are legally demandable and of the 1973 Constitution, as amended.” It is familiar
enforceable, and to determine whether or not there has learning that the legitimacy of a government sired by a
been a grave abuse of discretion amounting to lack or successful revolution by people power is beyond judicial
excess of jurisdiction on the part of any branch or scrutiny for that government automatically orbits out of
instrumentality of the Government." (Article VIII, Sec. 1 the constitutional loop. In checkered contrast, the
of the 1987 Constitution) Under this definition, the Court government of respondent Arroyo is not revolutionary in
cannot agree x x x that the issue involved is a political character. The oath that she took at the EDSA Shrine is
question beyond the jurisdiction of this Court to review. the oath under the 1987 Constitution. In her oath, she
When the grant of power is qualified, conditional or categorically swore to preserve and defend the 1987
subject to limitations, the issue of whether the prescribed Constitution. Indeed, she has stressed that she is
qualifications or conditions have been met or the discharging the powers of the presidency under the
limitations respected, is justiciable - the problem being authority of the 1987 Constitution.
that grave abuse was committed because the power to
In fine, the legal distinction between EDSA People Power call was exercised in such a manner as to violate the
I and EDSA People Power II is clear. EDSA I involves the constitutional provision on civilian supremacy over the
exercise of the people power of revolution which military. In the performance of this Court's duty of
overthrows the whole government. EDSA II is an exercise "purposeful hesitation" before declaring an act of another
of people power of freedom of speech and freedom of branch as unconstitutional, only where such grave abuse
assembly to petition the government for redress of of discretion is clearly shown shall the Court interfere with
grievances which only affected the office of the President. the President's judgment. To doubt is to sustain.
EDSA I is extra constitutional and the legitimacy of the (Integrated Bar of the Philippines v. Hon. Ronaldo
new government that resulted from it cannot be the B. Zamora, G.R. No. 141284, Aug. 15, 2000, En
subject of judicial review, but EDSA II is intra Banc [Kapunan])
constitutional and the resignation of the sitting President
that it caused and the succession of the Vice President as 116. Do lower courts have jurisdiction to consider the
President are subject to judicial review. EDSA I presented constitutionality of a law? If so, how should they act in
a political question; EDSA II involves legal questions. X x the exercise of this jurisdiction?
x
Held: We stress at the outset that the lower court had
Needless to state, the cases at bar pose legal and not jurisdiction to consider the constitutionality of Section
political questions. The principal issues for resolution 187, this authority being embraced in the general
require the proper interpretation of certain provisions in definition of the judicial power to determine what are the
the 1987 Constitution, notably Section 1 of Article II, and valid and binding laws by the criterion of their conformity
Section 8 of Article VII, and the allocation of to the fundamental law. Specifically, BP 129 vests in the
governmental powers under Section 11 of Article VII. The regional trial courts jurisdiction over all civil cases in which
issues likewise call for a ruling on the scope of presidential the subject of the litigation is incapable of pecuniary
immunity from suit. They also involve the correct estimation (Sec. 19[1]), even as the accused in a criminal
calibration of the right of petitioner against prejudicial action has the right to question in his defense the
publicity. As early as the 1803 case of Marbury v. Madison constitutionality of a law he is charged with violating and
(1 Cranch [5 US] 137, L Ed 60 [1803]), the doctrine has of the proceedings taken against him, particularly as they
been laid down that “it is emphatically the province and contravene the Bill of Rights. Moreover, Article VIII,
duty of the judicial department to say what the law is x x Section 5(2), of the Constitution vests in the Supreme
x.” Thus, respondent’s invocation of the doctrine of Court appellate jurisdiction over final judgments and
political question is but a foray in the dark. (Joseph E. orders of lower courts in all cases in which the
Estrada v. Aniano Desierto, G.R. Nos. 146710-15, constitutionality or validity of any treaty, international or
March 2, 2001, En Banc [Puno]) executive agreement, law, presidential decree,
proclamation, order, instruction, ordinance, or regulation
115. Is the President’s power to call out the armed is in question.
forces as their Commander-in-Chief in order to prevent or
suppress lawless violence, invasion or rebellion subject to In the exercise of this jurisdiction, lower courts are
judicial review, or is it a political question? Clarify. advised to act with the utmost circumspection, bearing in
mind the consequences of a declaration of
Held: When the President calls the armed forces to unconstitutionality upon the stability of laws, no less than
prevent or suppress lawless violence, invasion or on the doctrine of separation of powers. As the
rebellion, he necessarily exercises a discretionary power questioned act is usually the handiwork of the legislative
solely vested in his wisdom. This is clear from the intent or the executive departments, or both, it will be prudent
of the framers and from the text of the Constitution itself. for such courts, if only out of a becoming modesty, to
The Court, thus, cannot be called upon to overrule the defer to the higher judgment of this Court in the
President's wisdom or substitute its own. However, this consideration of its validity, which is better determined
does not prevent an examination of whether such power after a thorough deliberation by a collegiate body and
was exercised within permissible constitutional limits or with the concurrence of the majority of those who
whether it was exercised in a manner constituting grave participated in its discussion (Art. VIII, Sec. 4[2],
abuse of discretion. In view of the constitutional intent Constitution) (Drilon v. Lim, 235 SCRA 135, 139-
to give the President full discretionary power to determine 140, Aug. 4, 1994, En Banc [Cruz])
the necessity of calling out the armed forces, it is
incumbent upon the petitioner to show that the 117. What cases are to be heard by the Supreme Court
President's decision is totally bereft of factual basis. The en banc?
present petition fails to discharge such heavy burden as
there is no evidence to support the assertion that there Held: Under Supreme Court Circular No. 2-89, dated
exists no justification for calling out the armed forces. February 7, 1989, as amended by the Resolution of
There is, likewise, no evidence to support the proposition November 18, 1993:
to fiscal autonomy and violative not only of the express
X x x, the following are considered en banc cases: mandate of the Constitution but especially as regards the
Supreme Court, of the independence and separation of
1) Cases in which the constitutionality or validity powers upon which the entire fabric of our constitutional
of any treaty, international or executive agreement, law, system is based. (Bengzon v. Drilon, 208 SCRA 133,
executive order, or presidential decree, proclamation, April 15, 1992, En Banc [Gutierrez])
order, instruction, ordinance, or regulation is in question;
2) Criminal cases in which the appealed decision 119. May the Ombudsman validly entertain criminal
imposes the death penalty; charges against a judge of the regional trial court in
3) Cases raising novel questions of law; connection with his handling of cases before the court.
4) Cases affecting ambassadors, other public
ministers and consuls; Held: Petitioner criticizes the jurisprudence
5) Cases involving decisions, resolutions or (Maceda v. Vasquez, 221 SCRA 464 [1993] and Dolalas v.
orders of the Civil Service Commission, Commission on Office of the Ombudsman-Mindanao, 265 SCRA 818
Elections, and Commission on Audit; [1996]) cited by the Office of the Ombudsman as
6) Cases where the penalty to be imposed is the erroneous and not applicable to his complaint. He insists
dismissal of a judge, officer or employee of the judiciary, that since his complaint involved a criminal charge against
disbarment of a lawyer, or either the suspension of any a judge, it was within the authority of the Ombudsman
of them for a period of more than one (1) year or a fine not the Supreme Court to resolve whether a crime was
exceeding P10,000.00 or both; committed and the judge prosecuted therefor.
7) Cases where a doctrine or principle laid down
by the court en banc or in division may be modified or The petition can not succeed.
reversed;
8) Cases assigned to a division which in the Xxx
opinion of at least three (3) members thereof merit the
attention of the court en banc and are acceptable to a We agree with the Solicitor General that the Ombudsman
majority of the actual membership of the court en banc; committed no grave abuse of discretion warranting the
and writs prayed for. The issues have been settled in the case
9) All other cases as the court en banc by a of In Re: Joaquin Borromeo (241 SCRA 408, 460 [1995]).
majority of its actual membership may deem of sufficient There, we laid down the rule that before a civil or criminal
importance to merit its attention. action against a judge for a violation of Arts. 204 and 205
(Firestone Ceramics, Inc. v. Court of Appeals, 334 (knowingly rendering an unjust judgment or order) can
SCRA 465, 471-472, June 28, 2000, En Banc be entertained, there must first be “a final and
[Purisima]) authoritative judicial declaration” that the decision or
order in question is indeed “unjust.” The pronouncement
118. What is fiscal autonomy? The fiscal autonomy may result from either:
clause?
(a) an action of certiorari or prohibition in a
Held: As envisioned in the Constitution, the fiscal higher court impugning the validity of the judgment; or
autonomy enjoyed by the Judiciary, the Civil Service (b) an administrative proceeding in the Supreme
Commission, the Commission on Audit, the Commission Court against the judge precisely for promulgating an
on Elections, and the Office of the Ombudsman unjust judgment or order.
contemplates a guarantee of full flexibility to allocate and
utilize their resources with the wisdom and dispatch that Likewise, the determination of whether a judge has
their needs require. It recognizes the power and maliciously delayed the disposition of the case is also an
authority to levy, assess and collect fees, fix rates of exclusive judicial function (In Re: Borromeo, supra, at
compensation not exceeding the highest rates authorized 461).
by law for compensation and pay plans of the government
and allocate and disburse such sums as may be provided “To repeat, no other entity or official of the government,
by law or prescribed by them in the course of the not the prosecution or investigation service of any other
discharge of their functions. branch, not any functionary thereof, has competence to
review a judicial order or decision – whether final and
Fiscal autonomy means freedom from outside control. executory or not – and pronounce it erroneous so as to
The Judiciary, the Constitutional Commissions, and the lay the basis for a criminal or administrative complaint for
Ombudsman must have the independence and flexibility rendering an unjust judgment or order. That prerogative
needed in the discharge of their constitutional duties. The belongs to the courts alone.
imposition of restrictions and constraints on the manner
the independent constitutional offices allocate and utilize This having been said, we find that the
the funds appropriated for their operations is anathema Ombudsman acted in accordance with law and
jurisprudence when he referred the cases against Judge court. The idea is to avoid having to repeat in the body
Pelayo to the Supreme Court for appropriate action. (De of the latter decision the findings or conclusions of the
Vera v. Pelayo, 335 SCRA 281, July 6, 2000, 1st Div. lower court since they are being approved or adopted
[Pardo]) anyway.), a specie of succinctly written decisions by
appellate courts in accordance with the provisions of
120. Discuss the validity of “Memorandum Decisions.” Section 40, B.P. Blg. 129 on the grounds of expediency,
practicality, convenience and docket status of our courts.
Held: 1. The constitutional mandate that no decision We have also declared that memorandum decisions
shall be rendered by any court without expressing therein comply with the constitutional mandate.
clearly and distinctly the facts and the law on which it is
based does not preclude the validity of "memorandum In Francisco v. Permskul, however, we laid the
decisions" which adopt by reference the findings of fact conditions for the validity of memorandum decisions,
and conclusions of law contained in the decisions of thus:
inferior tribunals. X x x
The memorandum decision, to be valid, cannot
Hence, even in this jurisdiction, incorporation by incorporate the findings of fact and the conclusions of law
reference is allowed if only to avoid the cumbersome of the lower court only by remote reference, which is to
reproduction of the decision of the lower courts, or say that the challenged decision is not easily and
portions thereof, in the decisions of the higher court immediately available to the person reading the
(Francisco v. Permskul, 173SCRA 324, 333). This is memorandum decision. For the incorporation by
particularly true when the decision sought to be reference to be allowed, it must provide for direct access
incorporated is a lengthy and thorough discussion of the to the facts and the law being adopted, which must be
facts and conclusions arrived at x x x. (Oil and Natural contained in a statement attached to the said decision. In
Gas Commission v. Court of Appeals, 293 SCRA 26, other words, the memorandum decision authorized under
July 23, 1998 [Martinez]) Section 40 of B.P. Blg. 129 should actually embody the
findings of fact and conclusions of law of the lower court
2. We have sustained decisions of lower courts in an annex attached to and made an indispensable part
as having substantially or sufficiently complied with the of the decision.
constitutional injunction notwithstanding the laconic and
terse manner in which they were written and even if It is expected that this requirement will allay the suspicion
“there [was left] much to be desired in terms of [their] that no study was made of the decision of the lower court
clarity, coherence and comprehensibility” provided that and that its decision was merely affirmed without a prior
they eventually set out the facts and the law on which examination of the facts and the law on which it is based.
they were based, as when they stated the legal The proximity at least of the annexed statement should
qualifications of the offense constituted by the facts suggest that such examination has been undertaken. It
proved, the modifying circumstances, the participation of is, of course, also understood that the decision being
the accused, the penalty imposed and the civil liability; or adopted should, to begin with, comply with Article VIII,
discussed the facts comprising the elements of the Section 14 as no amount of incorporation or adoption will
offense that was charged in the information, and rectify its violation.
accordingly rendered a verdict and imposed the
corresponding penalty; or quoted the facts narrated in the The Court finds necessary to emphasize that the
prosecution’s memorandum but made their own findings memorandum decision should be sparingly used lest it
and assessment of evidence, before finally agreeing with become an additive excuse for judicial sloth. It is an
the prosecution’s evaluation of the case. additional condition for the validity of this kind of decision
may be resorted to only in cases where the facts are in
We have also sanctioned the use of memorandum the main accepted by both parties and easily
decisions (In Francisco v. Permskul, 173 SCRA 324, 333 determinable by the judge and there are no doctrinal
[1989], the Court described “[t]he distinctive features of complications involved that will require an extended
a memorandum decision are, first, it is rendered by an discussion of the laws involved. The memorandum
appellate court, second, it incorporates by reference the decision may be employed in simple litigations only, such
findings of fact or the conclusions of law contained in the as ordinary collection cases, where the appeal is obviously
decision, order, or ruling under review. Most likely, the groundless and deserves no more than the time needed
purpose is to affirm the decision, although it is not to dismiss it.
impossible that the approval of the findings of facts by the
lower court may lead to a different conclusion of law by Xxx
the higher court. At any rate, the reason for allowing the
incorporation by reference is evidently to avoid the Henceforth, all memorandum decisions shall comply with
cumbersome reproduction of the decision of the lower the requirements herein set forth as to the form
court, or portions thereof, in the decision of the higher prescribed and the occasions when they may be
rendered. Any deviation will summon the strict higher court, if permitted, should he believe that the
enforcement of Article VIII, Section 14 of the Constitution decision should be reversed. A decision that does not
and strike down the flawed judgment as a lawless clearly and distinctly state the facts and the law on which
disobedience. it is based leaves the parties in the dark as to how it was
reached and is precisely prejudicial to the losing party,
Tested against these standards, we find that the who is unable to pinpoint the possible errors of the court
RTC decision at bar miserably failed to meet them and, for review by a higher tribunal. More than that, the
therefore, fell short of the constitutional injunction. The requirement is an assurance to the parties that, in
RTC decision is brief indeed, but it is starkly hallow, reaching judgment, the judge did so through the
otiosely written, vacuous in its content and trite in its processes of legal reasoning. It is, thus, a safeguard
form. It achieved nothing and attempted at nothing, not against the impetuosity of the judge, preventing him from
even at a simple summation of facts which could easily be deciding ipse dixit. Vouchsafed neither the sword nor the
done. Its inadequacy speaks for itself. purse by the Constitution but nonetheless vested with the
sovereign prerogative of passing judgment on the life,
We cannot even consider or affirm said RTC liberty or property of his fellowmen, the judge must
decision as a memorandum decision because it failed to ultimately depend on the power of reason for sustained
comply with the measures of validity laid down in public confidence in the justness of his decision.
Francisco v. Permskul. It merely affirmed in toto the
MeTC decision without saying more. A decision or Thus the Court has struck down as void, decisions of
resolution, especially one resolving an appeal, should lower courts and even of the Court of Appeals whose
directly meet the issues for resolution; otherwise, the careless disregard of the constitutional behest exposed
appeal would be pointless (See ABD Overseas Manpower their sometimes cavalier attitude not only to their
Corporation v. NLRC, 286 SCRA 454, 464 [1998]). magisterial responsibilities but likewise to their avowed
fealty to the Constitution.
We therefore reiterate our admonition in Nicos
Industrial Corporation v. Court of Appeals (206 SCRA 127, Thus, we nullified or deemed to have failed to comply with
134 [1992]), in that while we conceded that brevity in the Section 14, Article VIII of the Constitution, a decision,
writing of decisions is an admirable trait, it should not and resolution or order which: contained no analysis of the
cannot be substituted for substance; and again in evidence of the parties nor reference to any legal basis in
Francisco v. Permskul, where we cautioned that reaching its conclusions; contained nothing more than a
expediency alone, no matter how compelling, cannot summary of the testimonies of the witnesses of both
excuse non-compliance with the constitutional parties; convicted the accused of libel but failed to cite
requirements. any legal authority or principle to support conclusions that
the letter in question was libelous; consisted merely of
This is not to discourage the lower courts to write one (1) paragraph with mostly sweeping generalizations
abbreviated and concise decisions, but never at the and failed to support its conclusion of parricide; consisted
expense of scholarly analysis, and more significantly, of of five (5) pages, three (3) pages of which were
justice and fair play, lest the fears expressed by Justice quotations from the labor arbiter’s decision including the
Feria as the ponente in Romero v. Court of Appeals come dispositive portion and barely a page (two [2] short
true, i.e., if an appellate court failed to provide the appeal paragraphs of two [2] sentences each) of its own
the attention it rightfully deserved, said court deprived the discussion or reasonings; was merely based on the
appellant of due process since he was accorded a fair findings of another court sans transcript of stenographic
opportunity to be heard by a fair and responsible notes, or failed to explain the factual and legal bases for
magistrate. This situation becomes more ominous in the award of moral damages.
criminal cases, as in this case, where not only property
rights are at stake but also the liberty if not the life of a In the same vein do we strike down as a nullity the RTC
human being. decision in question. (Yao v. Court of Appeals, 344
SCRA 202, Oct. 24, 2000, 1st Div. [Davide])
Faithful adherence to the requirements of Section 14,
Article VIII of the Constitution is indisputably a paramount 121. Does the period for decision making under
component of due process and fair play. It is likewise Section 15, Article VIII, 1987 Constitution, apply to the
demanded by the due process clause of the Constitution. Sandiganbayan? Explain.
The parties to a litigation should be informed of how it
was decided, with an explanation of the factual and legal Held: The above provision does not apply to the
reasons that led to the conclusions of the court. The court Sandiganbayan. The provision refers to regular courts of
cannot simply say that judgment is rendered in favor of X lower collegiate level that in the present hierarchy applies
and against Y and just leave it at that without any only to the Court of Appeals.
justification whatsoever for its action. The losing party is
entitled to know why he lost, so he may appeal to the
The Sandiganbayan is a special court of the same presently sitting in five [5] divisions, functions as a trial
level as the Court of Appeals and possessing all the court. The term “trial” is used in its broad sense,
inherent powers of a court of justice, with functions of a meaning, it allows introduction of evidence by the parties
trial court. in the cases before it. The Sandiganbayan, in original
cases within its jurisdiction, conducts trials, has the
Thus, the Sandiganbayan is not a regular court discretion to weigh the evidence of the parties, admit the
but a special one. The Sandiganbayan was originally evidence it regards as credible and reject that which they
empowered to promulgate its own rules of procedure . consider perjurious or fabricated. (Re: Problem of
However, on March 30, 1995, Congress repealed the Delays in Cases Before the Sandiganbayan, A.M.
Sandiganbayan’s power to promulgate its own rules of No. 00-8-05-SC, Nov. 28, 2001, En Banc [Pardo])
procedure and instead prescribed that the Rules of Court
promulgated by the Supreme Court shall apply to all cases
and proceedings filed with the Sandiganbayan.
CONSTITUTIONAL LAW
“Special courts are judicial tribunals exercising limited
jurisdiction over particular or specialized categories of
actions. They are the Court of Tax Appeals, the 122. What is the effect of declaration of
Sandiganbayan, and the Shari’a Courts.” (Supra, Note 23, unconstitutionality of a law? Illustrative case.
at p. 8)
Held: Respondents are seeking a
Under Article VIII, Section 5[5] of the reconsideration of the Court’s 25 January 2000 decision,
Constitution “Rules of procedure of special courts and wherein we declared section 8 of Republic Act No. 8551
quasi-judicial bodies shall remain effective unless (RA 8551) to be violative of petitioners’ constitutionally
disapproved by the Supreme Court.” mandated right to security of tenure. As a consequence
of our ruling, we held that petitioners’ removal as
In his report, the Court Administrator would commissioners of the National Police Commission
distinguish between cases which the Sandiganbayan has (NAPOLCOM) and the appointment of new Commissioners
cognizance of in its original jurisdiction, and cases which in their stead were nullities and ordered the reinstatement
fall within the appellate jurisdiction of the Sandiganbayan. of petitioners and the payment of full backwages to be
The Court Administrator posits that since in the first class computed from the date they were removed from office.
of cases, the Sandiganbayan acts more as a trial court,
then for that classification of cases, the three [3] month Xxx
reglementary period applies. For the second class of
cases, the Sandiganbayan has the twelve-month An unconstitutional act is not a law; it confers no rights,
reglementary period for collegiate courts. We do not imposes no duties, and affords no protection (Fernandez
agree. v. Cuerva, 21 SCRA 1095 [1967]). Therefore, the
unavoidable consequence of the Court’s declaration that
The law creating the Sandiganbayan, P.D. No. section 8 of RA 8551 violates the fundamental law is
1606 is clear on this issue. It provides: that all acts done pursuant to such provision shall be null
and void, including the removal of petitioners and
“Sec. 6. Maximum period for termination of cases – As far Adiong from their positions in the NAPOLCOM and the
as practicable, the trial of cases before the appointment of new commissioners in their stead.
Sandiganbayan once commenced shall be continuous When a regular government employee is illegally
until terminated and the judgment shall be rendered dismissed, his position does not become vacant and the
within three [3] months from the date the case was new appointment made in order to replace him is null
submitted for decision.” and void ab initio (Aquino v. Civil Service Commission,
208 SCRA 240 [1992]). Rudimentary is the precept that
On September 18, 1984, the Sandiganbayan there can be no valid appointment to a non-vacant
promulgated its own rules, thus: position (Garces v. Court of Appeals, 259 SCRA 99
[1996]). Accordingly, Adiong’s appointment on 11
“Sec. 3. Maximum Period to Decide Cases – The judgment March 1998 for a term of two years, pursuant to section
or final order of a division of the Sandiganbayan shall be 8 of RA 8551, is null and void. However, he should now
rendered within three [3] months from the date the case be permitted to enjoy the remainder of his term under
was submitted for decision.” RA 6975. Therefore, based on our foregoing
disquisition, there should no longer be any doubt as to
Given the clarity of the rule that does not distinguish, we the proper execution of our 25 January 2000 decision –
hold that the three [3] month period, not the twelve [12] all the Commissioners appointed under RA 8551 should
month period, to decide cases applies to the be removed from office, in order to give way to the
Sandiganbayan. Furthermore, the Sandiganbayan reinstatement of petitioners and respondent Adiong.
(Canonizado v. Aguirre, 351 SCRA 659, Feb. 15, x x (People v. Dela Piedra, 350 SCRA 163, Jan. 24,
2001, En Banc [Gonzaga-Reyes]) 2001, 1st Div. [Kapunan])

2. The doctrine has been formulated in various


123. Discuss the “Void for Vagueness” Doctrine, and ways, but is commonly stated to the effect that a statute
why is it repugnant to the Constitution. Distinguish a establishing a criminal offense must define the offense
“perfectly vague act” from “legislation couched in with sufficient definiteness that persons of ordinary
imprecise language.” intelligence can understand what conduct is prohibited
by the statute. It can only be invoked against that
Held: 1. Due process requires that the terms of a specie of legislation that is utterly vague on its face, i.e.,
penal statute must be sufficiently explicit to inform those that which cannot be clarified either by a saving clause
who are subject to it what conduct on their part will or by construction.
render them liable to its penalties (Connally v. General
Construction Co., 269 US 385, 70 L Ed 322 46 S Ct 126 A statute or act may be said to be vague when it
[1926]). A criminal statute that “fails to give a person of lacks comprehensible standards that men of common
ordinary intelligence fair notice that his contemplated intelligence must necessarily guess at its meaning and
conduct is forbidden by the statute,” or is so indefinite differ in its application. In such instance, the statute is
that “it encourages arbitrary and erratic arrests and repugnant to the Constitution in two (2) respects – it
convictions,” is void for vagueness (Colautti v. Franklin, violated due process for failure to accord persons,
439 US 379, 58 L Ed 2d 596, 99 S Ct 675 [1979]). The especially the parties targeted by it, fair notice of what
constitutional vice in a vague or indefinite statute is the conduct to avoid; and, it leaves law enforcers unbridled
injustice to the accused in placing him on trial for an discretion in carrying out its provisions and becomes an
offense, the nature of which he is given no fair warning arbitrary flexing of the Government muscle (See People
(American Communications Asso. v. Douds, 339 US 382, v. Nazario, No. L-44143, 31 August 1988, 165 SCRA 186,
94 L Ed 925, 70 S Ct 674 [1950]) 195-196). But the doctrine does not apply as against
legislations that are merely couched in imprecise
We reiterated these principles in People v. language but which nonetheless specify a standard
Nazario (165 SCRA 186 [1988]): though defectively phrased; or to those that are
apparently ambiguous yet fairly applicable to certain
As a rule, a statute or act may be said to be vague when types of activities. The first may be “saved” by proper
it lacks comprehensible standards that men “of common construction, while no challenge may be mounted as
intelligence must necessarily guess at its meaning and against the second whenever directed against such
differ as to its application.” It is repugnant to the activities (Ibid.) With more reason, the doctrine cannot
Constitution in two respects: (1) it violates due process be invoked where the assailed statute is clear and free
for failure to accord persons, especially the parties from ambiguity, as in this case.
targeted by it, fair notice of the conduct to avoid; and
(2) it leaves law enforcers unbridled discretion in The test in determining whether a criminal
carrying out its provisions and become an arbitrary statute is void for uncertainty is whether the language
flexing of the Government muscle. conveys a sufficiently definite warning as to the
proscribed conduct when measured by common
We added, however, that: understanding and practice (State v. Hill, 189 Kan 403,
369 P2d 365, 91 ALR 2d 750). It must be stressed,
X x x the act must be utterly vague on its face, that is to however, that the “vagueness” doctrine merely requires
say, it cannot be clarified by either a saving clause or by a reasonable degree of certainty for the statute to be
construction. Thus, in Coates v. City of Cincinnati, the upheld – not absolute precision or mathematical
U.S. Supreme Court struck down an ordinance that had exactitude, as petitioner seems to suggest. Flexibility,
made it illegal for “three or more persons to assemble rather than meticulous specificity, is permissible as long
on any sidewalk and there conduct themselves in a as the metes and bounds of the statute are clearly
manner annoying to persons passing by.” Clearly, the delineated. An act will not be held invalid merely
ordinance imposed no standard at all “because one may because it might have been more explicit in its wordings
never know in advance what annoys some people but or detailed in its provisions, especially where, because of
does not annoy others.” the nature of the act, it would be impossible to provide
all the details in advance as in all other statutes.
Coates highlights what has been referred to as a (Joseph Ejercito Estrada v. Sandiganbayan [Third
“perfectly vague” act whose obscurity is evident on its Division], G.R. No. 148560, Nov. 19, 2001, En
face. It is to be distinguished, however, from legislation Banc [Bellosillo])
coached in imprecise language – but which nonetheless
specifies a standard though defectively phrased – in
which case, it may be “saved” by proper construction. X
124. Does Article 13 (b) of the Labor Code defining for overbreadth provisions prohibiting the posting of
“recruitment and placement” violate the due process election propaganda in any place – including private
clause? vehicles – other than in the common poster areas
sanctioned by the COMELEC. We held that the
Held: In support of her submission that Article 13 (b) is challenged provisions not only deprived the owner of the
void for vagueness, appellant invokes People v. Panis vehicle the use of his property but also deprived the
(142 SCRA 664 [1986]), where this Court x x x citizen of his right to free speech and information. The
“criticized” the definition of “recruitment and placement” prohibition in Adiong, therefore, was so broad that it
x x x. covered even constitutionally guaranteed rights and,
hence, void for overbreadth. In the present case,
Appellant further argues that the acts that constitute however, appellant did not even specify what
“recruitment and placement” suffer from overbreadth constitutionally protected freedoms are embraced by the
since by merely “referring” a person for employment, a definition of “recruitment and placement” that would
person may be convicted of illegal recruitment. render the same constitutionally overbroad. (People v.
Dela Piedra, 350 SCRA 163, Jan. 24, 2001, 1st Div.
These contentions cannot be sustained. [Kapunan])

Appellant’s reliance on People v. Panis is misplaced. The 125. Is the Plunder Law unconstitutional for being
issue in Panis was whether, under the proviso of Article vague?
13(b), the crime of illegal recruitment could be
committed only “whenever two or more persons are in Held: As it is written, the Plunder Law contains
any manner promised or offered any employment for a ascertainable standards and well-defined parameters
fee.” The Court held in the negative x x x. which would enable the accused to determine the nature
of his violation. Section 2 is sufficiently explicit in its
X x x The Court, in Panis, merely bemoaned the lack of description of the acts, conduct and conditions required
records that would help shed light on the meaning of the or forbidden, and prescribes the elements of the crime
proviso. The absence of such records notwithstanding, with reasonable certainty and particularity. X x x
the Court was able to arrive at a reasonable
interpretation of the proviso by applying principles in As long as the law affords some comprehensible
criminal law and drawing from the language and intent guide or rule that would inform those who are subject to
of the law itself. Section 13(b), therefore, is not a it what conduct would render them liable to its penalties,
“perfectly vague act” whose obscurity is evident on its its validity would be sustained. It must sufficiently guide
face. If at all, the proviso therein is merely couched in the judge in its application; the counsel, in defending
imprecise language that was salvaged by proper one charged with its violation; and more importantly,
construction. It is not void for vagueness. the accused, in identifying the realm of the proscribed
conduct. Indeed, it can be understood with little
Xxx difficulty that what the assailed statute punishes is the
act of a public officer in amassing or accumulating ill-
That Section 13(b) encompasses what appellant gotten wealth of at least P50,000,000.00 through a
apparently considers as customary and harmless acts series or combination of acts enumerated in Sec. 1, par.
such as “labor or employment referral” (“referring” an (d), of the Plunder Law.
applicant, for employment to a prospective employer)
does not render the law overbroad. Evidently, appellant In fact, the amended Information itself closely
misapprehends concept of overbreadth. tracks the language of the law, indicating with
reasonable certainty the various elements of the offense
A statute may be said to be overbroad where it operates which petitioner is alleged to have committed x x x.
to inhibit the exercise of individual freedoms
affirmatively guaranteed by the Constitution, such as the We discern nothing in the foregoing that is
freedom of speech or religion. A generally worded vague or ambiguous – as there is obviously none – that
statute, when construed to punish conduct which cannot will confuse petitioner in his defense. Although subject
be constitutionally punished is unconstitutionally vague to proof, these factual assertions clearly show that the
to the extent that it fails to give adequate warning of the elements of the crime are easily understood and provide
boundary between the constitutionally permissible and adequate contrast between the innocent and the
the constitutionally impermissible applications of the prohibited acts. Upon such unequivocal assertions,
statute (Wright v. Georgia, 373 US 284, 10 L Ed 2d 349, petitioner is completely informed of the accusations
83 S Ct 1240 [1963]). against him as to enable him to prepare for an
intelligent defense.
In Blo Umpar Adiong v. Commission on Elections (207
SCRA 712 [1992]), for instance, we struck down as void
Petitioner, however, bewails the failure of the
law to provide for the statutory definition of the terms Xxx
“combination” and “series” in the key phrase “a
combination or series of overt or criminal acts” found in Thus when the Plunder Law speaks of
Sec. 1, par. (d), and Sec. 2, and the word “pattern” in “combination,” it is referring to at least two (2) acts
Sec. 4. These omissions, according to petitioner, render falling under different categories or enumeration
the Plunder Law unconstitutional for being impermissibly provided in Sec. 1, par. (d), e.g., raids on the public
vague and overbroad and deny him the right to be treasury in Sec. 1, par. (d), subpar. (1), and fraudulent
informed of the nature and cause of the accusation conveyance of assets belonging to the National
against him, hence, violative of his fundamental right to Government under Sec. 1, par. (d), subpar. (3).
due process.
On the other hand, to constitute a “series” there
The rationalization seems to us to be pure must be two (2) or more overt or criminal acts falling
sophistry. A statute is not rendered uncertain and void under the same category of enumeration found in Sec.
merely because general terms are used therein, or 1, par. (d), say, misappropriation, malversation and
because of the employment of terms without defining raids on the public treasury, all of which fall under Sec.
them (82 C.J.S. 68, P. 113; People v. Ring, 70 P.2d 281, 1, par. (d), subpar. (1). Verily, had the legislature
26 Cal. App. 2d Supp. 768); much less do we have to intended a technical or distinctive meaning for
define every word we use. Besides, there is no positive “combination” and “series,” it would have taken greater
constitutional or statutory command requiring the pains in specifically providing for it in the law.
legislature to define each and every word in an
enactment. Congress is not restricted in the form of As for “pattern,” we agree with the observations
expression of its will, and its inability to so define the of the Sandiganbayan that this term is sufficiently
words employed in a statute will not necessarily result in defined in Sec. 4, in relation to Sec. 1, par. (d), and Sec.
the vagueness or ambiguity of the law so long as the 2–
legislative will is clear, or at least, can be gathered from
the whole act, which is distinctly expressed in the x x x under Sec. 1 (d) of the law, a ‘pattern’ consists of
Plunder Law. at least a combination or series of overt or
criminal acts enumerated in subsections (1) to
Moreover, it is a well-settled principle of legal (6) of Sec. 1 (d). Secondly, pursuant to Sec. 2 of the
hermeneutics that words of a statute will be interpreted law, the pattern of overt or criminal acts is directed
in their natural, plain and ordinary acceptation and towards a common purpose or goal which is to
signification (Mustang Lumber, Inc. v. Court of Appeals, enable the public officer to amass, accumulate or
G.R. No. 104988, 18 June 1965, 257 SCRA 430, 448), acquire ill-gotten wealth. And thirdly, there must
unless it is evident that the legislature intended a either be an ‘overall unlawful scheme’ or
technical or special legal meaning to those words (PLDT ‘conspiracy’ to achieve said common goal. As
v. Eastern Telecommunications Phil., Inc., G.R. No. commonly understood, the term ‘overall unlawful
943774, 27 August 1992, 213 SCRA 16, 26). The scheme’ indicates a ‘general plan of action or method’
intention of the lawmakers – who are, ordinarily, which the principal accused and public officer and others
untrained philologists and lexicographers – to use conniving with him follow to achieve the aforesaid
statutory phraseology in such a manner is always common goal. In the alternative, if there is no such
presumed. Thus, Webster’s New Collegiate Dictionary overall scheme or where the schemes or methods used
contains the following commonly accepted definition of by multiple accused vary, the overt or criminal acts must
the words “combination” and “series.” form part of a conspiracy to attain a common goal.

Combination – the result or product of combining; the Xxx


act or process of combining. To combine is to bring into
such close relationship as to obscure individual Hence, it cannot plausibly be contended that the law
characters. does not give a fair warning and sufficient notice of what
it seeks to penalize. Under the circumstances,
Series – a number of things or events of the same class petitioner’s reliance on the “void-for-vagueness” doctrine
coming one after another in spatial and temporal is manifestly misplaced.
succession.
Xxx
That Congress intended the words
“combination” and “series” to be understood in their Moreover, we agree with, hence we adopt, the
popular meanings is pristinely evident from the observations of Mr. Justice Vicente V. Mendoza during
legislative deliberations on the bill which eventually the deliberations of the Court that the allegations that
became RA 7080 or the Plunder Law x x x.
the Plunder Law is vague and overbroad do not justify a curtailed when invoked against ordinary criminal laws
facial review of its validity – that are sought to be applied to protected conduct.” For
this reason, it has been held that “a facial challenge to a
The void-for-vagueness doctrine states that “a statute legislative act is the most difficult challenge to mount
which either forbids or requires the doing of an act in successfully, since the challenger must establish that no
terms so vague that men of common intelligence must set of circumstances exists under which the Act would
necessarily guess at its meaning and differ as to its be valid.” (United States v. Salerno, supra.) As for the
application violates the first essential of due process of vagueness doctrine, it is said that a litigant may
law.” (Connally v. General Constr. Co., 269 U.S. 385, challenge a statute on its face only if it is vague in all its
391, 70 L. Ed. 328 [1926] cited in Ermita-Malate Hotel possible applications. “A plaintiff who engages in some
and Motel Operators Ass’n. v. City Mayor, 20 SCRA 849, conduct that is clearly proscribed cannot complain of the
867 [1967]) The overbreadth doctrine, on the other vagueness of the law as applied to the conduct of
hand, decrees that “a governmental purpose may not be others.” (Village of Hoffman Estates v. Flipside, Hoffman
achieved by means which sweep unnecessarily broadly Estates, Inc., 455 U.S. 489, 494-95, 71 L Ed. 2d 362,
and thereby invade the area of protected freedoms.” 369 [1982])
(NAACP v. Alabama, 377 U.S. 288, 307, 12, 2 L. Ed 325,
338 [1958]; Shelton v. Tucker, 364 U.S. 479, 5 L. Ed. 2d In sum, the doctrines of strict scrutiny, overbreadth, and
231 [1960]) vagueness are analytical tools developed for testing “on
their faces” statutes in free speech cases or, as they are
A facial challenge is allowed to be made to a vague called in American law, First Amendment cases. They
statute and to one which is overbroad because of cannot be made to do service when what is involved is a
possible “chilling effect” upon protected speech. The criminal statute. With respect to such statute, the
theory is that “[w]hen statutes regulate or proscribe established rule is that “one to whom application of a
speech and no readily apparent construction suggests statute is constitutional will not be heard to attack the
itself as a vehicle for rehabilitating the statutes in a statute on the ground that impliedly it might also be
single prosecution, the transcendent value to all society taken as applying to other persons or other situations in
of constitutionally protected expression is deemed to which its application might be unconstitutional.” (United
justify allowing attacks on overly broad statutes with no States v. Raines, 362 U.S. 17, 21, 4 L. Ed. 2d 524, 529
requirement that the person making the attack [1960]. The paradigmatic case is Yazoo & Mississippi
demonstrate that his own conduct could not be Valley RR. v. Jackson Vinegar Co., 226 U.S. 217, 57 l.
regulated by a statute drawn with narrow specificity.” Ed. 193 [1912]) As has been pointed out, “vagueness
(Gooding v. Wilson, 405 U.S. 518, 521, 31 L. Ed. 2d 408, challenges in the First Amendment context, like
413 [1972] [internal quotation marks omitted]) The overbreadth challenges typically produce facial
possible harm to society in permitting some unprotected invalidation, while statutes found to be vague as a
speed to go unpunished is outweighed by the possibility matter of due process typically are invalidated [only] ‘as
that the protected speech of others may be deterred and applied’ to a particular defendant.” (G. Gunther & K.
perceived grievances left to fester because of possible Sullivan, Constitutional Law 1299 [2001]) Consequently,
inhibitory effects of overly broad statutes. there is no basis for petitioner’s claim that this Court
review the Anti-Plunder Law on its face and in its
This rationale does not apply to penal statutes. Criminal entirety.
statutes have general in terrorem effect resulting from
their very existence, and, if facial challenge is allowed Indeed, “on its face” invalidation of statutes results in
for this reason alone, the State may well be prevented striking them down entirely on the ground that they
from enacting laws against socially harmful conduct. In might be applied to parties not before the Court whose
the area of criminal law, the law cannot take chances as activities are constitutionally protected (Id. at 1328). It
in the area of free speech. constitutes a departure from the case and controversy
requirement of the Constitution and permits decisions to
The overbreadth and vagueness doctrine then have be made without concrete factual settings and in sterile
special application only to free speech cases. They are abstract contexts (Constitution, Art. VIII, Sections 1 and
inapt for testing the validity of penal statutes. As the 5. Compare Angara v. Electoral Commission, 63 Phil.
U.S. Supreme Court put it, in an opinion by Chief Justice 139, 158 [1936]). But, as the U.S. Supreme Court
Rehnquist, “we have not recognized an ‘overbreadth’ pointed out in Younger v. Harris (401 U.S. 37, 52-53, 27
doctrine outside the limited context of the First L. Ed. 2d 669, 680 [1971]; others omitted.)
Amendment.” In Broadwick v. Oklahoma (413 U.S. 601,
612-613, 37 L Ed. 2d 830, 840-841 [1973]), the Court [T]he task of analyzing a proposed statute, pinpointing
ruled that “claims of facial overbreadth have been its deficiencies, and requiring correction of these
entertained in cases involving statutes which, by their deficiencies before the statute is put into effect, is rarely
terms, seek to regulate only spoken words” and, again, if ever an appropriate task for the judiciary. The
that “overbreadth claims, if entertained at all, have been combination of the relative remoteness of the
controversy, the impact on the legislative process of the not repugnant to the Constitution, as they shall judge to
relief sought, and above all the speculative and be for the good and welfare of the commonwealth, and
amorphous nature of the required line-by-line analysis of for the subjects of the same. The power is plenary and
detailed statutes, x x x ordinarily results in a kind of case its scope is vast and pervasive, reaching and justifying
that is wholly unsatisfactory for deciding constitutional measures for public health, public safety, public morals,
questions, whichever way they might be decided. and the general welfare.

For these reasons, “on its face” invalidation of statutes It bears stressing that police power is lodged primarily in
has been described as “manifestly strong medicine,” to the National Legislature. It cannot be exercised by any
be employed “sparingly and only as a last resort,” group or body of individuals not possessing legislative
(Broadwick v. Oklahoma, 413 U.S. at 613, 37 L.Ed.2d at power. The National Legislature, however, may delegate
841; National Endowment for the Arts v. Finley, 524 U.S. this power to the President and administrative boards as
569, 580 [1998]) and is generally disfavored (FW/PBS, well as the lawmaking bodies of municipal corporations or
Inc. v. City of Dallas, 493 U.S. 223, 107 L.Ed.2d 603 local government units. Once delegated, the agents can
[1990]; Cruz v. Secretary of Environment and Natural exercise only such legislative powers as are conferred on
Resources, G.R. No. 135385, 6 December 2000 them by the national lawmaking body . (Metropolitan
[Mendoza, J., Separate Opinion]). In determining the Manila Development Authority v. Bel-Air Village
constitutionality of a statute, therefore, its provisions Association, Inc., 328 SCRA 836, 843-844, March
which are alleged to have been violated in a case must 27, 2000, 1st Div. [Puno])
be examined in the light of the conduct with which the
defendant is charged (United States v. National Dairy 2. The scope of police power has been held to be so
Prod. Corp., 372 U.S. 29, 32-33, 9 L.Ed.2d 561, 565-6 comprehensive as to encompass almost all matters
[1963]) affecting the health, safety, peace, order, morals, comfort
and convenience of the community. Police power is
In light of the foregoing disquisition, it is evident essentially regulatory in nature and the power to issue
that the purported ambiguity of the Plunder Law, so licenses or grant business permits, if exercised for a
tenaciously claimed and argued at length by petitioner, regulatory and not revenue-raising purpose, is within the
is more imagined than real. Ambiguity, where none ambit of this power.
exists, cannot be created by dissecting parts and words
in the statute to furnish support to critics who cavil at Xxx
the want of scientific precision in the law. Every
provision of the law should be construed in relation and [T]he issuance of business licenses and permits by a
with reference to every other part. To be sure, it will municipality or city is essentially regulatory in nature. The
take more than nitpicking to overturn the well- authority, which devolved upon local government units to
entrenched presumption of constitutionality and validity issue or grant such licenses or permits, is essentially in
of the Plunder Law. A fortiori, petitioner cannot feign the exercise of the police power of the State within the
ignorance of what the Plunder Law is all about. Being contemplation of the general welfare clause of the Local
one of the Senators who voted for its passage, petitioner Government Code. (Acebedo Optical Company, Inc.
must be aware that the law was extensively deliberated v. Court of Appeals, 329 SCRA 314, March 31,
upon by the Senate and its appropriate committees by 2000, En Banc [Purisima])
reason of which he even registered his affirmative vote
with full knowledge of its legal implications and sound 127. Discuss the nature of the authority of local
constitutional anchorage. (Joseph Ejercito Estrada v. government units to issue or grant licenses or permits.
Sandiganbayan [Third Division], G.R. No. 148560,
Nov. 19, 2001, En Banc [Bellosillo]) Held: The issuance of business licenses and permits by
a municipality or city is essentially regulatory in nature.
The authority, which devolved upon local government
units to issue or grant such licenses or permits, is
A. THE INHERENT POWERS OF THE STATE essentially in the exercise of the police power of the State
within the contemplation of the general welfare clause of
Police Power the Local Government Code. (Acebedo Optical
Company, Inc. v. Court of Appeals, 329 SCRA 314,
126. Define Police Power and clarify its scope. March 31, 2000, En Banc [Purisima])

Held: 1. Police power is an inherent attribute of 128. How should laws that grant the right to exercise
sovereignty. It has been defined as the power vested by a part of the police power of the State be construed?
the Constitution in the legislature to make, ordain, and
establish all manner of wholesome and reasonable laws, Held: Lest the idea gets lost in the shoals of our
statutes and ordinances, either with penalties or without, subconsciousness, let us not forget that PAGCOR is
engaged in business affected with public interest. The In regulating rates charged by public utilities,
phrase “affected with public interest” means that an the State protects the public against arbitrary and
industry is subject to control for the public good (Nebbia excessive rates while maintaining the efficiency and
v. New York, 291 U.S. 502); it has been considered as the quality of services rendered. However, the power to
equivalent of “subject to the exercise of the police power.” regulate rates does not give the State the right to
(Bernas, The 1987 Constitution of the Republic of the prescribe rates which are so low as to deprive the public
Philippines, A Commentary, 1996 ed., p. 1053) Perforce, utility of a reasonable return on investment. Thus, the
a legislative franchise to operate jai-alai is imbued with rates prescribed by the State must be one that
public interest and involves an exercise of police power. yields a fair return on the public utility upon the
The familiar rule is that laws which grant the right to value of the property performing the service and
exercise a part of the police power of the state are to be one that is reasonable to the public for the
construed strictly and any doubt must be resolved against service rendered (IV A.F. Agbayani, Commentaries
the grant (People v. Chicago, 103 N.E. 609; Slaughter v. and Jurisprudence on the Commercial Laws of the
O’Berry, 35 S.E. 241, 48 L.R.A. 442). The legislature is Philippines 500 [1993]). The fixing of just and
regarded as the guardian of society, and therefore is not reasonable rates involves a balancing of the investor
presumed to disable itself or abandon the discharge of its and the consumer interests (Federal Power Commission
duty. Thus, courts do not assume that the legislature v. Hope Natural Gas Co., 320 U.S. 591). (Republic of
intended to part away with its power to regulate public the Philippines v. Manila Electric Company, G.R.
morals (Stone v. Mississippi, 101 U.S. 814). The No. 141314, Nov. 15, 2002, 3rd Div. [Puno])
presumption is influenced by constitutional
considerations. Constitutions are widely understood to 130. What powers of the State are involved in the
withhold from legislatures any authority to bargain away implementation of the Comprehensive Agrarian Reform
their police power (Sutherland Statutory Construction, Law (CARL)? Discuss.
Vol. 3, 5th ed., p. 244) for the power to protect the public
interest is beyond abnegation. Held: The implementation of the CARL is an exercise of
the State’s police power and the power of eminent
It is stressed that the case at bar does not involve domain. To the extent that the CARL prescribes retention
a franchise to operate a public utility (such as water, limits to the landowners, there is an exercise of police
transportation, communication or electricity) – the power for the regulation of private property in accordance
operation of which undoubtedly redounds to the benefit with the Constitution (Association of Small Landowners in
of the general public. What is claimed is an alleged the Philippines v. Secretary of Agrarian Reform, 175 SCRA
legislative grant of a gambling franchise – a franchise to 343, 373-374 [1989]). But where, to carry out such
operate jai-alai. A statute which legalizes a gambling regulation, the owners are deprived of lands they own in
activity or business should be strictly construed and every excess of the maximum area allowed, there is also a
reasonable doubt must be resolved to limit the powers taking under the power of eminent domain. The taking
and rights claimed under its authority (Aicardi v. Alabama, contemplated is not a mere limitation of the use of the
22 L.Ed. 215; West Indies, Inc. v. First National Bank, 214 land. What is required is the surrender of the title to and
P.2d 144). (Del Mar v. Philippine Amusement and physical possession of the said excess and all beneficial
Gaming Corporation, 346 SCRA 485, Nov. 29, rights accruing to the owner in favor of the farmer
2000, En Banc [Puno]) beneficiary (Id.). The Bill of Rights provides that “[n]o
person shall be deprived of life, liberty or property
129. Discuss why rates to be charged by public utilities without due process of law.” (Section 1, Article III, 1987
like MERALCO are subject to State regulation. Constitution) The CARL was not intended to take away
property without due process of law (Development Bank
Held: The regulation of rates to be charged by public of the Philippines v. Court of Appeals, 262 SCRA 245, 253
utilities is founded upon the police power of the State [1996]). The exercise of the power of eminent domain
and statutes prescribing rules for the control and requires that due process be observed in the taking of
regulations of public utilities are a valid exercise thereof. private property. (Roxas & Co., Inc. v. Court of
When private property is used for a public purpose and Appeals, 321 SCRA 106, Dec. 17, 1999, En Banc
is affected with public interest, it ceases to be juris [Puno])
privati only and becomes subject to regulation. The
regulation is to promote the common good. Submission
to regulation may be withdrawn by the owner by 131. Does Article 263(g) of the Labor Code (vesting
discontinuing use; but as long as the use of the property upon the Secretary of Labor the discretion to determine
is continued, the same is subject to public regulation what industries are indispensable to the national interest
(Munn v. People of the State of Illinois, 94 U.S. 113, 126 and thereafter, assume jurisdiction over disputes in said
[1877]). industries) violate the workers’ constitutional right to
strike?
Held: Said article does not interfere with the workers’ It does not follow, therefore, from the constitutional
right to strike but merely regulates it, when in the exercise guarantees of the free exercise of religion that everything
of such right, national interests will be affected. The which may be so called can be tolerated. It has been said
rights granted by the Constitution are not absolute. They that a law advancing a legitimate governmental interest
are still subject to control and limitation to ensure that is not necessarily invalid as one interfering with the “free
they are not exercised arbitrarily. The interests of both exercise” of religion merely because it also incidentally
the employers and the employees are intended to be has a detrimental effect on the adherents of one or more
protected and not one of them is given undue preference. religion. Thus, the general regulation, in the public
interest, of solicitation, which does not involve any
The Labor Code vests upon the Secretary of Labor the religious test and does not unreasonably obstruct or delay
discretion to determine what industries are indispensable the collection of funds, is not open to any constitutional
to national interest. Thus, upon the determination of the objection, even though the collection be for a religious
Secretary of Labor that such industry is indispensable to purpose. Such regulation would not constitute a
the national interest, it will assume jurisdiction over the prohibited previous restraint on the free exercise of
labor dispute of said industry. The assumption of religion or interpose an inadmissible obstacle to its
jurisdiction is in the nature of police power measure. This exercise.
is done for the promotion of the common good
considering that a prolonged strike or lockout can be Even with numerous regulative laws in existence, it is
inimical to the national economy. The Secretary of Labor surprising how many operations are carried on by persons
acts to maintain industrial peace. Thus, his certification and associations who, secreting their activities under the
for compulsory arbitration is not intended to impede the guise of benevolent purposes, succeed in cheating and
workers’ right to strike but to obtain a speedy settlement defrauding a generous public. It is in fact amazing how
of the dispute. (Philtread Workers Union [PTWU] v. profitable the fraudulent schemes and practices are to
Confesor, 269 SCRA 393, March 12, 1997) people who manipulate them. The State has authority
under the exercise of its police power to determine
132. May solicitation for religious purposes be subject whether or not there shall be restrictions on soliciting by
to proper regulation by the State in the exercise of police unscrupulous persons or for unworthy causes or for
power? fraudulent purposes. That solicitation of contributions
under the guise of charitable and benevolent purposes is
Held: The constitutional inhibition of legislation on the grossly abused is a matter of common knowledge.
subject of religion has a double aspect. On the one hand, Certainly the solicitation of contributions in good faith for
it forestalls compulsion by law of the acceptance of any worthy purposes should not be denied, but somewhere
creed or the practice of any form of worship. Freedom of should be lodged the power to determine within
conscience and freedom to adhere to such religious reasonable limits the worthy from the unworthy . The
organization or form of worship as the individual may objectionable practices of unscrupulous persons are
choose cannot be restricted by law. On the other hand, prejudicial to worthy and proper charities which naturally
it safeguards the free exercise of the chosen form of suffer when the confidence of the public in campaigns for
religion. Thus, the Constitution embraces two concepts, the raising of money for charity is lessened or destroyed.
that is, freedom to believe and freedom to act. The first Some regulation of public solicitation is, therefore, in the
is absolute but, in the nature of things, the second cannot public interest.
be. Conduct remains subject to regulation for the
protection of society. The freedom to act must have To conclude, solicitation for religious purposes may be
appropriate definitions to preserve the enforcement of subject to proper regulation by the State in the exercise
that protection. In every case, the power to regulate of police power. (Centeno v. Villalon-Pornillos, 236
must be so exercised, in attaining a permissible end, as SCRA 197, Sept. 1, 1994 [Regalado])
not to unduly infringe on the protected freedom.

Whence, even the exercise of religion may be regulated, The Power of Eminent Domain
at some slight inconvenience, in order that the State may
protect its citizens from injury. Without doubt, a State 132. What is Eminent Domain?
may protect its citizens from fraudulent solicitation by
requiring a stranger in the community, before permitting Held: 1. Eminent domain is the right or power of a
him publicly to solicit funds for any purpose, to establish sovereign state to appropriate private property to
his identity and his authority to act for the cause which particular uses to promote public welfare. It is an
he purports to represent. The State is likewise free to indispensable attribute of sovereignty; a power grounded
regulate the time and manner of solicitation generally, in in the primary duty of government to serve the common
the interest of public safety, peace, comfort, or need and advance the general welfare. Thus, the right of
convenience. eminent domain appertains to every independent
government without the necessity for constitutional
recognition. The provisions found in modern constitutions appropriate, in the nature of a compulsory sale to the
of civilized countries relating to the taking of property for State, private property for public use or purpose.
the public use do not by implication grant the power to Inherently possessed by the national legislature, the
the government, but limit a power which would otherwise power of eminent domain may be validly delegated to
be without limit. Thus, our own Constitution provides that local governments, other public entities and public
“[p]rivate property shall not be taken for public use utilities. For the taking of private property by the
without just compensation.” (Art. III, Sec. 9). government to be valid, the taking must be for public
Furthermore, the due process and equal protection purpose and there must be just compensation. (Moday
clauses (1987 Constitution, Art. III, Sec. 1) act as v. Court of Appeals, 268 SCRA 586, February 20,
additional safeguards against the arbitrary exercise of this 1997)
governmental power.
133. State some limitations on the exercise of the
Since the exercise of the power of eminent domain affects power of Eminent Domain.
an individual’s right to private property, a constitutionally-
protected right necessary for the preservation and Held: The limitations on the power of eminent domain
enhancement of personal dignity and intimately are that the use must be public, compensation must be
connected with the rights to life and liberty, the need for made and due process of law must be observed. The
its circumspect operation cannot be overemphasized. In Supreme Court, taking cognizance of such issues as the
City of Manila v. Chinese Community of Manila we said adequacy of compensation, necessity of the taking and
(40 Phil. 349 [1919): the public use character or the purpose of the taking, has
ruled that the necessity of exercising eminent domain
The exercise of the right of eminent domain, whether must be genuine and of a public character. Government
directly by the State, or by its authorized agents, is may not capriciously choose what private property should
necessarily in derogation of private rights, and the rule in be taken. (Moday v. Court of Appeals, 268 SCRA
that case is that the authority must be strictly construed. 586, February 20, 1997)
No species of property is held by individuals with greater
tenacity, and none is guarded by the Constitution and the 134. Discuss the expanded notion of public use in
laws more sedulously, than the right to the freehold of eminent domain proceedings.
inhabitants. When the legislature interferes with that
right, and, for greater public purposes, appropriates the Held: The City of Manila, acting through its legislative
land of ah individual without his consent, the plain branch, has the express power to acquire private lands in
meaning of the law should not be enlarged by doubt[ful] the city and subdivide these lands into home lots for sale
interpretation. (Bensley v. Mountainlake Water Co., 13 to bona fide tenants or occupants thereof, and to laborers
Cal., 306 and cases cited [73 Am. Dec., 576]) and low-salaried employees of the city.

The statutory power of taking property from the owner That only a few could actually benefit from the
without his consent is one of the most delicate exercise expropriation of the property does not diminish its public
of governmental authority. It is to be watched with character. It is simply not possible to provide all at once
jealous scrutiny. Important as the power may be to the land and shelter for all who need them.
government, the inviolable sanctity which all free
constitutions attach to the right of property of the citizens, Corollary to the expanded notion of public use,
constrains the strict observance of the substantial expropriation is not anymore confined to vast tracts of
provisions of the law which are prescribed as modes of land and landed estates. It is therefore of no moment
the exercise of the power, and to protect it from abuse x that the land sought to be expropriated in this case is less
x x. than half a hectare only.

The power of eminent domain is essentially legislative in Through the years, the public use requirement in eminent
nature. It is firmly settled, however, that such power may domain has evolved into a flexible concept, influenced by
be validly delegated to local government units, other changing conditions. Public use now includes the broader
public entities and public utilities, although the scope of notion of indirect public benefit or advantage, including in
this delegated legislative power is necessarily narrower particular, urban land reform and housing. (Filstream
than that of the delegating authority and may only be International Incorporated v. CA, 284 SCRA 716,
exercised in strict compliance with the terms of the Jan. 23, 1998 [Francisco])
delegating law. (Heirs of Alberto Suguitan v. City of
Mandaluyong, 328 SCRA 137, 144-146, March 14, 135. The constitutionality of Sec. 92 of B.P. Blg. 881
2000, 3rd Div. [Gonzaga-Reyes]) (requiring radio and television station owners and
operators to give to the Comelec radio and television time
2. Eminent domain is a fundamental State power that is free of charge) was challenged on the ground, among
inseparable from sovereignty. It is government’s right to others, that it violated the due process clause and the
eminent domain provision of the Constitution by taking
airtime from radio and television broadcasting stations The claim that petitioner would be losing P52,380,000.00
without payment of just compensation. Petitioners claim in unrealized revenue from advertising is based on the
that the primary source of revenue of radio and television assumption that airtime is “finished product” which, it is
stations is the sale of airtime to advertisers and that to said, become the property of the company, like oil
require these stations to provide free airtime is to produced from refining or similar natural resources after
authorize a taking which is not “a de minimis temporary undergoing a process for their production. As held in Red
limitation or restraint upon the use of private property.” Lion Broadcasting Co. v. F.C.C. (395 U.S. at 394, 23 L. Ed.
Will you sustain the challenge? 2d at 391, quoting 47 U.S.C. Sec. 301), which upheld the
right of a party personally attacked to reply, “licenses to
Held: All broadcasting, whether by radio or by television broadcast do not confer ownership of designated
stations, is licensed by the government. Airwave frequencies, but only the temporary privilege of using
frequencies have to be allocated as there are more them.” Consequently, “a license permits broadcasting,
individuals who want to broadcast than there are but the licensee has no constitutional right to be the one
frequencies to assign. A franchise is thus a privilege who holds the license or to monopolize a radio frequency
subject, among other things, to amendment by Congress to the exclusion of his fellow citizens. There is nothing in
in accordance with the constitutional provision that “any the First Amendment which prevents the government
such franchise or right granted x x x shall be subject to from requiring a licensee to share his frequency with
amendment, alteration or repeal by the Congress when others and to conduct himself as a proxy or fiduciary with
the common good so requires.” (Art. XII, Sec. 11) obligations to present those views and voices which are
representative of his community and which would
Indeed, provisions for Comelec Time have been made by otherwise, by necessity, be barred from the airwaves.” As
amendment of the franchises of radio and television radio and television broadcast stations do not own the
broadcast stations and such provisions have not been airwaves, no private property is taken by the requirement
thought of as taking property without just compensation. that they provide airtime to the Comelec. (TELEBAP,
Art. XII, Sec. 11 of the Constitution authorizes the Inc. v. COMELEC, 289 SCRA 337, April 21, 1998
amendment of franchises for “the common good.” What [Mendoza])
better measure can be conceived for the common good
than one for free airtime for the benefit not only of 136. May eminent domain be barred by "res judicata"
candidates but even more of the public, particularly the or "law of the case"?
voters, so that they will be fully informed of the issues in
an election? “[I]t is the right of the viewers and listeners, Held: The principle of res judicata, which finds
not the right of the broadcasters, which is paramount.” application in generally all cases and proceedings, cannot
bar the right of the State or its agents to expropriate
Nor indeed can there be any constitutional objection to private property. The very nature of eminent domain, as
the requirement that broadcast stations give free airtime. an inherent power of the State, dictates that the right to
Even in the United States, there are responsible scholars exercise the power be absolute and unfettered even by a
who believe that government controls on broadcast media prior judgment or res judicata. The scope of eminent
can constitutionally be instituted to ensure diversity of domain is plenary and, like police power, can “reach every
views and attention to public affairs to further the system form of property which the State might need for public
of free expression. For this purpose, broadcast stations use.” All separate interests of individuals in property are
may be required to give free airtime to candidates in an held of the government under this tacit agreement or
election. implied reservation. Notwithstanding the grant to
individuals, the eminent domain, the highest and most
In truth, radio and television broadcasting companies, exact idea of property, remains in the government, or in
which are given franchises, do not own the airwaves and the aggregate body of the people in their sovereign
frequencies through which they transmit broadcast capacity; and they have the right to resume the
signals and images. They are merely given the temporary possession of the property whenever the public interest
privilege of using them. Since a franchise is a mere requires it.” Thus, the State or its authorized agent
privilege, the exercise of the privilege may reasonably be cannot be forever barred from exercising said right by
burdened with the performance by the grantee of some reason alone of previous non-compliance with any legal
form of public service. requirement.

In the granting of the privilege to operate broadcast While the principle of res judicata does not denigrate the
stations and thereafter supervising radio and television right of the State to exercise eminent domain, it does
stations, the State spends considerable public funds in apply to specific issues decided in a previous case. For
licensing and supervising such stations. It would be example, a final judgment dismissing an expropriation
strange if it cannot even require the licensees to render suit on the ground that there was no prior offer precludes
public service by giving free airtime. another suit raising the same issue; it cannot, however,
bar the State or its agent from thereafter complying with of the exercise of the power, for instance, in view of
this requirement, as prescribed by law, and subsequently allegations of partiality and prejudice by those adversely
exercising its power of eminent domain over the same affected, and the just compensation for the subject
property. (Municipality of Paranaque v. V.M. Realty property is provided in our constitutional system.
Corporation, 292 SCRA 678, July 20, 1998
[Panganiban]) We see no point in distinguishing between judicial and
legislative expropriation as far as the two stages
137. Discuss how expropriation may be initiated, and mentioned above are concerned. Both involve these
the two stages in expropriation. stages and in both the process is not completed until
payment of just compensation is made. The Court of
Held: Expropriation may be initiated by court action or Appeals was correct in saying that B.P. Blg. 340 did not
by legislation. In both instances, just compensation is effectively expropriate the land of the De la Ramas. As a
determined by the courts (EPZA v. Dulay, 149 SCRA 305 matter of fact, it merely commenced the expropriation of
[1987]). the subject property.

The expropriation of lands consists of two stages. As Xxx


explained in Municipality of Binan v. Garcia (180 SCRA
576, 583-584 [1989], reiterated in National Power Corp. The De la Ramas make much of the fact that ownership
v. Jocson, 206 SCRA 520 [1992]): of the land was transferred to the government because
the equitable and the beneficial title was already acquired
The first is concerned with the determination of the by it in 1983, leaving them with only the naked title.
authority of the plaintiff to exercise the power of eminent However, as this Court held in Association of Small
domain and the propriety of its exercise in the context of Landowners in the Phil., Inc. v. Secretary of Agrarian
the facts involved in the suit. It ends with an order, if not Reform (175 SCRA 343, 389 [1989]):
dismissal of the action, "of condemnation declaring that
the plaintiff has a lawful right to take the property sought The recognized rule, indeed, is that title to the property
to be condemned, for the public use or purpose declared expropriated shall pass from the owner to the
in the complaint, upon the payment of just compensation expropriator only upon full payment of the just
to be determined as of the date of the filing of the compensation. Jurisprudence on this settled principle is
complaint" x x x. consistent both here and in other democratic jurisdictions.
Xxx
The second phase of the eminent domain action is (Republic v. Salem Investment Corporation, et. al.,
concerned with the determination by the court of "the just G.R. No. 137569, June 23, 2000, 2nd Div.
compensation for the property sought to be taken." This [Mendoza])
is done by the court with the assistance of not more than
three (3) commissioners x x x. 139. Is prior unsuccessful negotiation a condition
precedent for the exercise of eminent domain?
It is only upon the completion of these two stages that
expropriation is said to have been completed. Moreover, Held: Citing Iron and Steel Authority v. Court of
it is only upon payment of just compensation that title Appeals (249 SCRA 538, October 25, 1995), petitioner
over the property passes to the government. Therefore, insists that before eminent domain may be exercised by
until the action for expropriation has been completed and the state, there must be a showing of prior unsuccessful
terminated, ownership over the property being negotiation with the owner of the property to be
expropriated remains with the registered owner. expropriated.
Consequently, the latter can exercise all rights pertaining
to an owner, including the right to dispose of his property, This contention is not correct. As pointed out by
subject to the power of the State ultimately to acquire it the Solicitor General the current effective law on
through expropriation. (Republic v. Salem delegated authority to exercise the power of eminent
Investment Corporation, et. al., G.R. No. 137569, domain is found in Section 12, Book III of the Revised
June 23, 2000, 2nd Div. [Mendoza]) Administrative Code, which provides:

138. Does the two (2) stages in expropriation apply “SEC. 12. Power of Eminent Domain – The President shall
only to judicial, and not to legislative, expropriation? determine when it is necessary or advantageous to
exercise the power of eminent domain in behalf of the
Held: The De la Ramas are mistaken in arguing that the National Government, and direct the Solicitor General,
two stages of expropriation x x x only apply to judicial, whenever he deems the action advisable, to institute
and not to legislative, expropriation. Although Congress expropriation proceedings in the proper court.”
has the power to determine what land to take, it can not
do so arbitrarily. Judicial determination of the propriety
The foregoing provision does not require prior It is also useful to view the matter as being subject to constant
unsuccessful negotiation as a condition precedent for the growth, which is to say that as society advances, its demands
exercise of eminent domain. In Iron and Steel Authority upon the individual so increases, and each demand is a new
v. Court of Appeals, the President chose to prescribe this use to which the resources of the individual may be devoted
condition as an additional requirement instead. In the (Visayan Refining, supra). (Republic of the Philippines v.
instant case, however, no such voluntary restriction was The Hon. Court of Appeals, G.R. No. 146587, July 2, 2002,
imposed. (SMI Development Corporation v. 1st Div. [Vitug])
Republic, 323 SCRA 862, Jan. 28, 2000, 3rd Div.
[Panganiban]) 141. What is the meaning of “public use” in eminent
domain proceedings? Illustrative case.

Held: This Court holds that respondent


140. Discuss the nature of the right of eminent (Philippine Export Processing Zone) has the legal
domain and the limitations thereof. authority to expropriate the subject Lot 1406-B and that
the same was for a valid public purpose. In Sumulong v.
Held: The right of eminent domain is usually Guerrero (154 SCRA 461, 467-468 [1987]), this Court has
understood to be an ultimate right of the sovereign power to ruled that,
appropriate any property within its territorial sovereignty for a
public purpose (Bernas, 1987 Edition, p. 276, quoting Justice The “public use” requirement for a valid exercise of the
Story in Charles River Bridge v. Warren Bridge). power of eminent domain is a flexible and evolving
Fundamental to the independent existence of a State, it concept influenced by changing conditions. In this
requires no recognition by the Constitution, whose provisions jurisdiction, the statutory and judicial trend has been
are taken as being merely confirmatory of its presence and as summarized as follows:
being regulatory, at most, in the due exercise of the power. In
the hands of the legislature, the power is inherent, its scope This Court has ruled that the taking to be valid must be
matching that of taxation, even that of police power itself, in for public use. There was a time when it was felt that a
many respects. It reaches to every form of property the State literal meaning should be attached to such a requirement.
needs for public use and, as an old case so puts it, all separate Whatever project is undertaken must be for the public to
interests of individuals in property are held under a tacit enjoy, as in the case of streets or parks. Otherwise,
agreement or implied reservation vesting upon the sovereign expropriation is not allowable. It is not anymore. As long
the right to resume the possession of the property whenever
as the purpose of the taking is public, then the power of
the public interest so requires it (US v. Certain Lands in
eminent domain comes into play . . . It is accurate to state
Highlands [DY NY] 48 F Supp 306).
then that at present whatever may be beneficially
The ubiquitous character of eminent domain is employed for the general welfare satisfies the
manifest in the nature of the expropriation proceedings. requirement of public use. (Heirs of Juancho Ardona v.
Expropriation proceedings are not adversarial in the Reyes, 125 SCRA 220 [1983] at 234-235 quoting E.
conventional sense, for the condemning authority is not Fernando, the Constitution of the Philippines 523-4 [2nd
required to assert any conflicting interest in the property. Ed. 1977])
Thus, by filing the action, the condemnor in effect merely
serves notice that it is taking title and possession of the The term “public use” has acquired a more
property, and the defendant asserts title or interest in the comprehensive coverage. To the literal import of the
property, not to prove a right to possession, but to prove a term signifying strict use or employment by the public has
right to compensation for the taking (US v. Certain Lands in been added the broader notion of indirect public benefit
Highlands [DY NY] 48 F Supp 306; San Bernardino Valley or advantage.
Municipal Water District v. Gage Canal Co. [4th Dist] Cal
App 2d 206, 37 Cal Rptr 856). In Manosca v. Court of Appeals, this Court has
also held that what ultimately emerged is a concept of
Obviously, however, the power is not without its public use which is just as broad as “public welfare.” (252
limits: first, the taking must be for public use, and second, that SCRA 412, 422 [1996], quoting Joaquin Bernas, The
just compensation must be given to the private owner of the Constitution of the Republic of the Philippines, Vol. 1,
property (Sena v. Manila Railroad Co., 42 Phil. 102). These
1987 ed., p. 282)
twin proscriptions have their origin in the recognition of the
necessity for achieving balance between the State interests, on
the one hand, and private rights, upon the other hand, by Respondent PEZA expropriated the subject parcel
effectively restraining the former and affording protection to of land pursuant to Proclamation No. 1980 x x x issued by
the latter (Visayan Refining Co. v. Camus, 40 Phil. 550). In former President Ferdinand Marcos. Meanwhile, the
determining “public use,” two approaches are utilized – the power of eminent domain of respondent is contained in
first is public employment or the actual use by the public, and its original charter, Presidential Decree No. 66 x x x.
the second is public advantage or benefit (Thornton
Development authority v. Upah [DC Colo] 640 F Supp 1071). Accordingly, subject Lot 1406-B was expropriated
“for the construction . . . of terminal facilities, structures
and approaches thereto.” The authority is broad enough lease of certain portions of the expropriated lot to banks,
to give the respondent substantial leeway in deciding for as well as the construction of terminal facilities.
what public use the expropriated property would be
utilized. Pursuant to this broad authority, respondent Petitioner contends that respondent is bound by
leased a portion of the lot to commercial banks while the the representations of its Chief Civil Engineer when the
rest was made a transportation terminal. Said public latter testified before the trial court that the lot was to be
purposes were even reaffirmed by Republic Act No. 7916, devoted for the construction of government offices.
a law amending respondent PEZA’s original charter x x x. Anent this issue, suffice it to say that PEZA can vary the
purpose for which a condemned lot will be devoted to,
In Manila Railroad Co. v. Mitchel (50 Phil. 832, provided that the same is for public use. Petitioner cannot
837-838 [1927]), this Court has ruled that in the exercise impose or dictate on the respondent what facilities to
of eminent domain, only as much land can be taken as is establish for as long as the same are for public purpose.
necessary for the legitimate purpose of the (Estate of Salud Jimenez v. PEZA, 349 SCRA 240,
condemnation. The term “necessary,” in this connection, Jan. 16, 2001, 2nd Div. [De Leon])
does not mean absolutely indispensable but requires only
a reasonable necessity of the taking for the stated 142. Discuss the meaning of “just compensation” in
purpose, growth and future needs of the enterprise. The eminent domain proceedings. Does it include the
respondent cannot attain a self-sustaining and viable payment of “interest” and, if so, how is it to be
ECOZONE if inevitable needs in the expansion in the computed?
surrounding areas are hampered by the mere refusal of
the private landowners to part with their properties. The Held: 1. The constitutional limitation of “just
purpose of creating an ECOZONE and other facilities is compensation” is considered to be the sum equivalent to the
better served if respondent directly owns the areas market value of the property, broadly described to be the price
subject of the expansion program. fixed by the seller in open market in the usual and ordinary
course of legal action and competition or the fair value of the
property as between one who receives, and one who desires to
X x x. The expropriation of Lot 1406-B for the
sell, it fixed at the time of the actual taking by the government
purpose of being leased to banks and for the construction
(Manila Railway Co. v. Fabie, 17 Phil. 206). Thus, if property
of a terminal has the purpose of making banking and
is taken for public use before compensation is deposited with
transportation facilities easily accessible to the persons the court having jurisdiction over the case, the final
working at the industries located in PEZA. The compensation must include interests on its just value to be
expropriation of adjacent areas therefore comes as a computed from the time the property is taken to the time when
matter of necessity to bring life to the purpose of the law. compensation is actually paid or deposited with the court
In such a manner, PEZA’s goal of being a major force in (Philippine Railway Co. v. Solon, 13 Phil. 34). In fine,
the economic development of the country would be between the taking of the property and the actual payment,
realized. Furthermore, this Court has already ruled that: legal interests accrue in order to place the owner in a position
as good as (but not better than) the position he was in before
X x x [T]he Legislature may directly determine the the taking occurred (Commissioner of Public Highways v.
necessity for appropriating private property for a Burgos, 96 SCRA 831). (Republic of the Philippines v. The
particular improvement for public use, and it may select Hon. Court of Appeals, G.R. No. 146587, July 2, 2002, 1st
the exact location of the improvement. In such a case, it Div. [Vitug])
is well-settled that the utility of the proposed
improvement, the existence of the public necessity for its 2. We have ruled that the concept of just
construction, the expediency of constructing it, the compensation embraces not only the correct
suitableness of the location selected, are all questions determination of the amount to be paid to the owners of
exclusively for the legislature to determine, and the courts the land, but also the payment of the land within a
have no power to interfere or to substitute their own reasonable time from its taking. Without prompt
views for those of the representatives of the people. payment, compensation cannot be considered “just”
inasmuch as the property owner is made to suffer the
In the absence of some constitutional or statutory consequences of being immediately deprived of his land
provisions to the contrary, the necessity and expediency while being made to wait for a decade or more before
of exercising the right of eminent domain are questions actually receiving the amount necessary to cope with his
essentially political and not judicial in their character. (City loss (Land Bank of the Philippines v. Court of Appeals, 258
of Manila v. Chinese Community of Manila, 40 Phil. 349 SCRA 404, 408-409 [1996] quoting Municipality of Makati
[1919]) v. Court of Appeals, 190 SCRA 207, 213 [1990]). Payment
of just compensation should follow as a matter of right
Inasmuch as both Presidential Decree No. 66 and immediately after the order of expropriation is issued.
Republic Act No. 7916, bestow respondent with Any delay in payment must be counted from said order.
authority to develop terminal facilities and banking However, the delay to constitute a violation of due
centers, this Court will not question the respondent’s process must be unreasonable and inexcusable; it must
be deliberately done by a party in order to defeat the ends the real authority would want it to be (City of Manila v.
of justice. Chinese Cemetery of Manila, 40 Phil. 349).

We find that respondent capriciously evaded its duty of Thus, in Valdehueza v. Republic (17 SCRA 107)
giving what is due to petitioner. In the case at bar, the where the private landowners had remained unpaid ten years
expropriation order was issued by the trial court in 1991. after the termination of the expropriation proceedings, this
The compromise agreement between the parties was Court ruled –
approved by the trial court in 1993. However, from 1993
“The points in dispute are whether such payment can still be
up to the present, respondent has failed in its obligation
made and, if so, in what amount. Said lots have been the
to pay petitioner to the prejudice of the latter.
subject of expropriation proceedings. By final and executory
Respondent cause damage to petitioner in making the judgment in said proceedings, they were condemned for public
latter to expect that it had a good title to the property to use, as part of an airport, and ordered sold to the government.
be swapped with Lot 1406-B; and meanwhile, respondent X x x It follows that both by virtue of the judgment, long
has been reaping benefits from the lease or rental income final, in the expropriation suit, as well as the annotations upon
of the said expropriated lot. We cannot tolerate this their title certificates, plaintiffs are not entitled to recover
oppressive exercise of the power of eminent domain by possession of their expropriated lots – which are still devoted
respondent. As we have ruled in Cosculluela v. Court of to the public use for which they were expropriated – but only
Appeals (164 SCRA 393, 401 [1988]): to demand the fair market value of the same.

In the present case, the irrigation project was completed Said relief may be granted under plaintiffs’ prayer for: ‘such
and has been in operation since 1976. The project is other remedies, which may be deemed just and equitable
benefiting the farmers specifically and the community in under the premises’.” (At p. 112)
general. Obviously, petitioner’s land cannot be returned
to him. However, it is high time that the petitioner be The Court proceeded to reiterate its pronouncement in Alfonso
paid what has been due him eleven years ago. It is v. Pasay City (106 Phil. 1017) where the recovery of
arbitrary and capricious for a government agency to possession of property taken for public use prayed for by the
initiate expropriation proceedings, seize a person’s unpaid landowner was denied even while no requisite
property, allow the judgment of the court to become final expropriation proceedings were first instituted. The
landowner was merely given the relief of recovering
and executory and then refuse to pay on the ground that
compensation for his property computed at its market value at
there was no appropriations for the property earlier taken
the time it was taken and appropriated by the State.
and profitably used. We condemn in the strongest
possible terms the cavalier attitude of government The judgment rendered by the Bulacan RTC in 1979
officials who adopt such a despotic and irresponsible on the expropriation proceedings provides not only for the
stance. payment of just compensation to herein respondents but
(Estate of Salud Jimenez v. PEZA, 349 SCRA 240, likewise adjudges the property condemned in favor of
Jan. 16, 2001, 2nd Div. [De Leon]) petitioner over which parties, as well as their privies, are
bound (Mines v. Canal Authority of the State [Fla] 467 So2d
143. When may the property owner be entitled to 989, 10 FLW 230). Petitioner has occupied, utilized and, for
the return of the expropriated property in eminent all intents and purposes, exercised dominion over the property
domain cases? pursuant to the judgment. The exercise of such rights vested
to it as the condemnee indeed has amounted to at least a
Held: 1. In insisting on the return of the partial compliance or satisfaction of the 1979 judgment,
expropriated property, respondents would exhort on the thereby preempting any claim of bar by prescription on
pronouncement in Provincial Government of Sorsogon v. Vda. grounds of non-execution. In arguing for the return of their
De Villaroya (153 SCRA 291) where the unpaid landowners property on the basis of non-payment, respondents ignore the
were allowed the alternative remedy of recovery of the fact that the right of the expropriatory authority is far from that
property there in question. It might be borne in mind that the of an unpaid seller in ordinary sales, to which the remedy of
case involved the municipal government of Sorsogon, to rescission might perhaps apply. An in rem proceeding,
which the power of eminent domain is not inherent, but condemnation acts upon the property (Cadorette v. US CCA
merely delegated and of limited application. The grant of the [Mass] 988 F2d 215). After condemnation, the paramount
power of eminent domain to local governments under title is in the public under a new and independent title (Ibid.);
Republic Act No. 7160 (See Local Government Code of 1991) thus, by giving notice to all claimants to a disputed title,
cannot be understood as being the pervasive and all- condemnation proceedings provide a judicial process for
encompassing power vested in the legislative branch of securing better title against all the world than may be obtained
government. For local governments to be able to wield the by voluntary conveyance (Ibid.). (Republic of the Philippines
power, it must, by enabling law, be delegated to it by the v. The Hon. Court of Appeals, G.R. No. 146587, July 2,
national legislature, but even then, this delegated power of 2002, 1st Div. [Vitug])
eminent domain is not, strictly speaking, a power of eminent,
but only of inferior, domain or only as broad or confined as 2. Though the respondent has committed a misdeed to
petitioner, we cannot, however, grant the petitioner’s
prayer for the return of the expropriated Lot No. 1406-B. Held: The debates, interpellations and expressions of
The Order of expropriation dated July 11, 1991, has long opinion of the framers of the Constitution reveal their
become final and executory. Petitioner cited Provincial intent that which, in turn, may have guided the people in
Government of Sorsogon v. Rosa E. Vda. De Villaroya ratifying the Charter. Such intent must be effectuated.
(153 SCRA 291, 302 [1987]) to support its contention that
it is entitled to a return of the lot where this Court ruled Accordingly, Justice Hilario G. Davide, Jr., a former
that “under ordinary circumstances, immediate return to constitutional commissioner, who is now a member of this
the owners of the unpaid property is the obvious remedy.” Court, stressed during the Concom debates that "x x x
However, the said statement was not the ruling in that what is exempted is not the institution itself x x x; those
case. As in other cases where there was no prompt exempted from real estate taxes are lands, buildings and
payment by the government, this Court declared in improvements actually, directly and exclusively used for
Sorsogon that “the Provincial Government of Sorsogon is religious, charitable or educational purposes. Father
expected to immediately pay as directed. Should any Joaquin G. Bernas, an eminent authority on the
further delay be encountered, the trial court is directed to Constitution and also a member of the Concom, adhered
seize any patrimonial property or cash savings of the to the same view that the exemption created by said
province in the amount necessary to implement this provision pertained only to property taxes.
decision.” However, this Court also stressed and declared
in that case that “in cases where land is taken for public In his treatise on taxation, Mr. Justice Jose C. Vitug
use, public interest, however, must be considered.” concurs, stating that "[t]he tax exemption covers property
(Estate of Salud Jimenez v. PEZA, 349 SCRA 240, taxes only." (Commissioner of Internal Revenue v.
Jan. 16, 2001, 2nd Div. [De Leon]) CA, 298 SCRA 83, Oct. 14, 1998 [Panganiban])

146. Under Article XIV, Section 4, paragraph 3 of the


The Power of Taxation 1987 Constitution, "[A]ll revenues and assets of non-
stock, non-profit educational institutions used actually,
144. Can taxes be subject to off-setting or directly, and exclusively for educational purposes shall be
compensation? exempt from taxes and duties." YMCA alleged that it "is a
non-profit educational institution whose revenues and
Held: Taxes cannot be subject to compensation for the assets are used actually, directly and exclusively for
simple reason that the government and the taxpayer are educational purposes so it is exempt from taxes on its
not creditors and debtors of each other. There is a properties and income."
material distinction between a tax and debt. Debts are
due to the Government in its corporate capacity, while Held: We reiterate that private respondent is exempt
taxes are due to the Government in its sovereign capacity. from the payment of property tax, but not income tax on
It must be noted that a distinguishing feature of a tax is the rentals from its property. The bare allegation alone
that it is compulsory rather than a matter of bargain. that it is a non-stock, non-profit educational institution is
Hence, a tax does not depend upon the consent of the insufficient to justify its exemption from the payment of
taxpayer. If any taxpayer can defer the payment of taxes income tax.
by raising the defense that it still has a pending claim for
refund or credit, this would adversely affect the [L]aws allowing tax exemption are construed strictissimi
government revenue system. A taxpayer cannot refuse juris. Hence, for the YMCA to be granted the exemption
to pay his taxes when they fall due simply because he has it claims under the abovecited provision, it must prove
a claim against the government or that the collection of a with substantial evidence that (1) it falls under the
tax is contingent on the result of the lawsuit it filed against classification non-stock, non-profit educational institution;
the government. (Philex Mining Corporation v. and (2) the income it seeks to be exempted from taxation
Commissioner of Internal Revenue, 294 SCRA 687, is used actually, directly, and exclusively for educational
Aug. 28, 1998 [Romero]) purposes. However, the Court notes that not a scintilla
of evidence was submitted by private respondent to prove
145. Under Article VI, Section 28, paragraph 3 of the that it met the said requisites. (Commissioner of
1987 Constitution, "[C]haritable institutions, churches and Internal Revenue v. CA, 298 SCRA 83, Oct. 14,
parsonages or convents appurtenant thereto, mosques, 1998 [Panganiban])
non-profit cemeteries, and all lands, buildings, and
improvements, actually, directly and exclusively used for 147. Is the YMCA an educational institution within the
religious, charitable or educational purposes shall be purview of Article XIV, Section 4, par. 3 of the
exempt from taxation." YMCA claims that the income Constitution?
earned by its building leased to private entities and that
of its parking space is likewise covered by said exemption. Held: We rule that it is not. The term "educational
Resolve. institution" or "institution of learning" has acquired a well-
known technical meaning, of which the members of the
Constitutional Commission are deemed cognizant. Under drafted with a view towards the elimination of
the Education Act of 1982, such term refers to schools. international juridical double taxation x x x.
The school system is synonymous with formal education, (Commissioner of Internal Revenue v. S.C.
which "refers to the hierarchically structured and Johnson and Son, Inc., 309 SCRA 87, 101-102,
chronologically graded learnings organized and provided June 25, 1999, 3rd Div. [Gonzaga-Reyes])
by the formal school system and for which certification is
required in order for the learner to progress through the 150. What is "international juridical double taxation"?
grades or move to the higher levels." The Court has
examined the "Amended Articles of Incorporation" and Held: It is defined as the imposition of comparable taxes
"By-Laws" of the YMCA, but found nothing in them that in two or more states on the same taxpayer in respect of
even hints that it is a school or an educational institution. the same subject matter and for identical periods.
(Commissioner of Internal Revenue v. S.C.
Furthermore, under the Education Act of 1982, even non- Johnson and Son, Inc., 309 SCRA 87, 102, June 25,
formal education is understood to be school-based and 1999)
"private auspices such as foundations and civic-spirited
organizations" are ruled out. It is settled that the term 151. What is the rationale for doing away with
"educational institution," when used in laws granting tax international juridical double taxation? What are the
exemptions, refers to a "x x x school seminary, college or methods resorted to by tax treaties to eliminate double
educational establishment x x x." (84 CJS 566) Therefore, taxation?
the private respondent cannot be deemed one of the
educational institutions covered by the constitutional Held: The apparent rationale for doing away with double
provision under consideration. (Commissioner of taxation is to encourage the free flow of goods and
Internal Revenue v. CA, 298 SCRA 83, Oct. 14, services and the movement of capital, technology and
1998 [Panganiban]) persons between countries, conditions deemed vital in
creating robust and dynamic economies. Foreign
148. May the PCGG validly commit to exempt from all investments will only thrive in a fairly predictable and
forms of taxes the properties to be retained by the Marcos reasonable international investment climate and the
heirs in a Compromise Agreement between the former protection against double taxation is crucial in creating
and the latter? such a climate.

Held: The power to tax and to grant exemptions is Double taxation usually takes place when a person is
vested in the Congress and, to a certain extent, in the resident of a contracting state and derives income from,
local legislative bodies. Section 28(4), Article VI of the or owns capital in, the other contracting state and both
Constitution, specifically provides: “No law granting any states impose tax on that income or capital. In order to
tax exemption shall be passed without the concurrence of eliminate double taxation, a tax treaty resorts to several
a majority of all the members of the Congress.” The methods. First, it sets out the respective rights to tax of
PCGG has absolutely no power to grant tax exemptions, the state of source or situs and of the state of residence
even under the cover of its authority to compromise ill- with regard to certain classes of income or capital. In
gotten wealth cases. some cases, an exclusive right to tax is conferred on one
of the contracting states; however, for other items of
Even granting that Congress enacts a law exempting the income or capital, both states are given the right to tax,
Marcoses from paying taxes on their properties, such law although the amount of tax that may be imposed by the
will definitely not pass the test of the equal protection state of source is limited.
clause under the Bill of Rights. Any special grant of tax
exemption in favor only of the Marcos heirs will constitute The second method for the elimination of double taxation
class legislation. It will also violate the constitutional rule applies whenever the state of source is given a full or
that “taxation shall be uniform and equitable .” (Chavez limited right to tax together with the state of residence.
v. PCGG, 299 SCRA 744, Dec. 9, 1998 In this case, the treaties make it incumbent upon the state
[Panganiban]) of residence to allow relief in order to avoid double
taxation. There are two methods of relief - the exemption
149. Discuss the purpose of tax treaties? method and the credit method. In the exemption
method, the income or capital which is taxable in the state
Held: The RP-US Tax Treaty is just one of a number of of source or situs is exempted in the state of residence,
bilateral treaties which the Philippines has entered into for although in some instances it may be taken into account
the avoidance of double taxation. The purpose of these in determining the rate of tax applicable to the taxpayer's
international agreements is to reconcile the national fiscal remaining income or capital. On the other hand, in the
legislations of the contracting parties in order to help the credit method, although the income or capital which is
taxpayer avoid simultaneous taxation in two different taxed in the state of source is still taxable in the state of
jurisdictions. More precisely, the tax conventions are residence, the tax paid in the former is credited against
the tax levied in the latter. The basic difference between or shortened” by observing the constitutional mandate of
the two methods is that in the exemption method, the due process of law.
focus is on the income or capital itself, whereas the credit
method focuses upon the tax. (Commissioner of Held: They are obviously referring to the procedural
Internal Revenue v. S.C. Johnson and Son, Inc., aspect of the enactment. Fortunately, the Court has
309 SCRA 87, 102-103, June 25, 1999) maintained a clear position in this regard, a stance it has
stressed in the recent case of Lumiqued v. Hon. Exevea
152. What is the rationale for reducing the tax rate in (G.R. No. 117565, November 18, 1997), where it declared
negotiating tax treaties? that “(a)s long as a party was given the opportunity to
defend his interests in due course, he cannot be said to
Held: In negotiating tax treaties, the underlying rationale have been denied due process of law, for this opportunity
for reducing the tax rate is that the Philippines will give to be heard is the very essence of due process. Moreover,
up a part of the tax in the expectation that the tax given this constitutional mandate is deemed satisfied if a person
up for this particular investment is not taxed by the other is granted an opportunity to seek reconsideration of the
country. (Commissioner of Internal Revenue v. S.C. action or ruling complained of.”
Johnson and Son, Inc., 309 SCRA 87, 103, June 25,
1999) In the case at bar, respondents questioned PPA-AO No.
04-92 no less than four times before the matter was
finally elevated to this Tribunal. Their arguments on this
B. THE BILL OF RIGHTS score, however, failed to persuade. X x x

The Due Process Clause Neither does the fact that the pilots themselves were not
consulted in any way taint the validity of the
153. Discuss the Due Process Clause. Distinguish administrative order. As a general rule, notice and
substantive due process from procedural due process. hearing, as the fundamental requirements of procedural
due process, are essential only when an administrative
Held: Section 1 of the Bill of Rights lays down what is body exercises its quasi-judicial function. In the
known as the "due process clause" of the Constitution. performance of its executive or legislative functions, such
as issuing rules and regulations, an administrative body
In order to fall within the aegis of this provision, two need not comply with the requirements of notice and
conditions must concur, namely, that there is a hearing.
deprivation and that such deprivation is done without
proper observance of due process. When one speaks of Upon the other hand, it is also contended that the sole
due process of law, however, a distinction must be made and exclusive right to the exercise of harbor pilotage by
between matters of procedure and matters of substance. pilots is a settled issue. Respondents aver that said right
In essence, procedural due process "refers to the method has become vested and can only be “withdrawn or
or manner by which the law is enforced," while shortened” by observing the constitutional mandate of
substantive due process "requires that the law itself, not due process of law. Their argument has thus shifted from
merely the procedures by which the law would be the procedural to one of substance. It is here where PPA-
enforced, is fair, reasonable, and just." (Corona v. AO No. 04-92 fails to meet the condition set by the
United Harbor Pilots Association of the Phils., 283 organic law.
SCRA 31, Dec. 12, 1997 [Romero])
Pilotage, just like other professions, may be practiced only
154. Respondents United Harbor Pilots Association of by duly licensed individuals. Licensure is “the granting of
the Philippines argue that due process was not observed license especially to practice a profession.” It is also “the
in the adoption of PPA-AO No. 04-92 which provides that: system of granting licenses (as for professional practice)
“(a)ll existing regular appointments which have been in accordance with established standards.” A license is a
previously issued by the Bureau of Customs or the PPA right or permission granted by some competent authority
shall remain valid up to 31 December 1992 only,” and to carry on a business or do an act which, without such
“(a)ll appointments to harbor pilot positions in all pilotage license, would be illegal.
districts shall, henceforth, be only for a term of one (1)
year from date of effectivity subject to renewal or Before harbor pilots can earn a license to practice their
cancellation by the Philippine Ports Authority after profession, they literally have to pass through the
conduct of a rigid evaluation of performance,” allegedly proverbial eye of a needle by taking, not one but five
because no hearing was conducted whereby “relevant examinations, each followed by actual training and
government agencies” and the harbor pilots themselves practice. X x x
could ventilate their views. They also contended that the
sole and exclusive right to the exercise of harbor pilotage Their license is granted in the form of an appointment
by pilots has become vested and can only be “withdrawn which allows them to engage in pilotage until they retire
at the age of 70 years. This is a vested right. Under the counsel, irrespective of the nature of the charges and of
terms of PPA-AO No. 04-92, “[a]ll existing regular the respondent's capacity to represent himself, and no
appointments which have been previously issued by the duty rests on such a body to furnish the person being
Bureau of Customs or the PPA shall remain valid up to 31 investigated with counsel. In an administrative
December 1992 only,” and “(a)ll appointments to harbor proceeding x x x a respondent x x x has the option of
pilot positions in all pilotage districts shall, henceforth, be engaging the services of counsel or not. x x x Thus, the
only for a term of one (1) year from date of effectivity right to counsel is not imperative in administrative
subject to renewal or cancellation by the Authority after investigations because such inquiries are conducted
conduct of a rigid evaluation of performance.” merely to determine whether there are facts that merit
disciplinary measures against erring public officers and
It is readily apparent that PPA-AO No. 04-92 unduly employees, with the purpose of maintaining the dignity of
restricts the right of harbor pilots to enjoy their profession government service.
before their compulsory retirement. In the past, they
enjoyed a measure of security knowing that after passing The right to counsel is not indispensable to due process
five examinations and undergoing years of on-the-job unless required by the Constitution or the law. X x x.
training, they would have a license which they could use (Lumiqued v. Exevea, 282 SCRA 125, Nov. 18,
until their retirement, unless sooner revoked by the PPA 1997 [Romero])
for mental or physical unfitness. Under the new issuance,
they have to contend with an annual cancellation of their 156. Does an extraditee have the right to notice and
license which can be temporary or permanent depending hearing during the evaluation stage of an extradition
on the outcome of their performance evaluation. Veteran proceeding?
pilots and neophytes alike are suddenly confronted with
one-year terms which ipso facto expire at the end of that Held: Considering that in the case at bar, the extradition
period. Renewal of their license is now dependent on a proceeding is only at its evaluation stage, the nature of
“rigid evaluation of performance” which is conducted only the right being claimed by the private respondent is
after the license has already been cancelled. Hence, the nebulous and the degree of prejudice he will allegedly
use of the term “renewal.” It is this pre-evaluation suffer is weak, we accord greater weight to the interests
cancellation which primarily makes PPA-AO No. 04-92 espoused by the government thru the petitioner Secretary
unreasonable and constitutionally infirm. In a real sense, of Justice. X x x
it is a deprivation of property without due process of law.
(Corona v. United Harbor Pilots Association of the In tilting the balance in favor of the interests of the State,
Phils., 283 SCRA 31, December 12, 1997 the Court stresses that it is not ruling that the private
[Romero]) respondent has no right to due process at all throughout
the length and breadth of the extradition proceedings.
155. Does the due process clause encompass the right Procedural due process requires a determination of what
to be assisted by counsel during an administrative process is due, when it is due, and the degree of what is
inquiry? due. Stated otherwise, a prior determination should be
made as to whether procedural protections are at all due
Held: The right to counsel, which cannot be waived and when they are due, which in turn depends on the
unless the waiver is in writing and in the presence of extent to which an individual will be "condemned to suffer
counsel, is a right afforded a suspect or an accused during grievous loss." We have explained why an extraditee has
custodial investigation. It is not an absolute right and no right to notice and hearing during the evaluation stage
may, thus, be invoked or rejected in a criminal proceeding of the extradition process. As aforesaid, P.D. No. 1069
and, with more reason, in an administrative inquiry. In which implements the RP-US Extradition Treaty affords an
the case at bar, petitioners invoke the right of an accused extraditee sufficient opportunity to meet the evidence
in criminal proceedings to have competent and against him once the petition is filed in court. The time
independent counsel of his own choice. Lumiqued, for the extraditee to know the basis of the request for his
however, was not accused of any crime in the extradition is merely moved to the filing in court of the
proceedings below. The investigation conducted by the formal petition for extradition. The extraditee's right to
committee x x x was for the sole purpose of determining know is momentarily withheld during the evaluation stage
if he could be held administratively liable under the law of the extradition process to accommodate the more
for the complaints filed against him. x x x As such, the compelling interest of the State to prevent escape of
hearing conducted by the investigating committee was potential extraditees which can be precipitated by
not part of a criminal prosecution. X x x premature information of the basis of the request for his
extradition. No less compelling at that stage of the
While investigations conducted by an administrative body extradition proceedings is the need to be more deferential
may at times be akin to a criminal proceeding, the fact to the judgment of a co-equal branch of the government,
remains that under existing laws, a party in an the Executive, which has been endowed by our
administrative inquiry may or may not be assisted by Constitution with greater power over matters involving
our foreign relations. Needless to state, this balance of hindered from continuing with the due processes
interests is not a static but a moving balance which can prescribed under its laws. His invocation of due process
be adjusted as the extradition process moves from the now had thus become hollow. He already had that
administrative stage to the judicial stage and to the opportunity in the requesting state; yet, instead of taking
execution stage depending on factors that will come into it, he ran away.
play. In sum, we rule that the temporary hold on private
respondent's privilege of notice and hearing is a soft In this light, would it be proper and just for the
restraint on his right to due process which will not deprive government to increase the risk of violating its treaty
him of fundamental fairness should he decide to resist the obligations in order to accord Respondent Jimenez his
request for his extradition to the United States. There is personal liberty in the span of time that it takes to resolve
no denial of due process as long as fundamental fairness the Petition for Extradition? His supposed immediate
is assured a party. (Secretary of Justice v. Hon. deprivation of liberty without due process that he had
Ralph C. Lantion, G.R. No. 139465, Oct. 17, 2000, previously shunned pales against the government’s
En Banc [Puno]) interest in fulfilling its Extradition Treaty obligations and
in cooperating with the world community in the
suppression of crime. Indeed, “[c]onstitutional liberties
157. Will Mark Jimenez’s detention prior to the do not exist in a vacuum; the due process rights accorded
conclusion of the extradition proceedings not amount to to individuals must be carefully balanced against exigent
a violation of his right to due process? and palpable government interest.” (Coquia, “On the
Implementation of the US-RP Extradition Treaty,” supra;
Held: Contrary to his contention, his detention prior to citing Kelso v. US Department of State, 13 F Supp. 291
the conclusion of the extradition proceedings does not [DDC 1998])
amount to a violation of his right to due process. We
iterate the familiar doctrine that the essence of due Too, we cannot allow our country to be a haven
process is the opportunity to be heard (Garcia v. NLRC, for fugitives, cowards and weaklings who, instead of
GR No. 110494, November 18, 1996; Paat v. Court of facing the consequences of their actions, choose to run
Appeals, January 10, 1997) but, at the same time, point and hide. Hence, it would not be good policy to
out that the doctrine does not always call for a prior increase the risk of violating our treaty obligations if,
opportunity to be heard (See Central Bank of the through overprotection or excessively liberal treatment,
Philippines v. Court of Appeals, 220 SCRA 536, March 20, persons sought to be extradited are able to evade arrest
1993). Where the circumstances – such as those present or escape from our custody. In the absence of any
in an extradition case – call for it, a subsequent provision – in the Constitution, the law or the treaty –
opportunity to be heard is enough (Ibid. See also expressly guaranteeing the right to bail in extradition
Busuego v. Court of Appeals, 304 SCRA 473, March 11, proceedings, adopting the practice of not granting them
1999). In the present case, respondent will be given full bail, as a general rule, would be a step towards
opportunity to be heard subsequently, when the deterring fugitives from coming to the Philippines to hide
extradition court hears the Petition for Extradition. from or evade their prosecutors.
Hence, there is no violation of his right to due process and
fundamental fairness. The denial of bail as a matter of course in
extradition cases falls into place with and gives life to
Contrary to the contention of Jimenez, we find no Article 14 (It states: “If the person sought consents in
arbitrariness, either, in the immediate deprivation of his writing to surrender to the Requesting State, the
liberty prior to his being heard. That his arrest and Requested State may surrender the person as
detention will not be arbitrary is sufficiently ensured by expeditiously as possible without further proceedings.”)
(1) the DOJ’s filing in court the Petition with its supporting of the Treaty, since this practice would encourage the
documents after a determination that the extradition accused to voluntarily surrender to the requesting state
request meets the requirements of the law and the to cut short their detention here. Likewise, their
relevant treaty; (2) the extradition judge’s independent detention pending the resolution of extradition
prima facie determination that his arrest will best serve proceedings would fall into place with the emphasis of
the ends of justice before the issuance of a warrant for the Extradition Law on the summary nature of
his arrest; and (3) his opportunity, once he is under the extradition cases and the need for their speedy
court’s custody, to apply for bail as an exception to the disposition. (Government of the United States of
no-initial-bail rule. America v. Hon. Guillermo Purganan, G.R. No.
148571, Sept. 24, 2002, En Banc [Panganiban])
It is also worth noting that before the US
government requested the extradition of respondent,
proceedings had already been conducted in that country. 158. Is respondent in an Extradition Proceeding
But because he left the jurisdiction of the requesting state entitled to notice and hearing before the issuance of a
before those proceedings could be completed, it was warrant of arrest?
Held: Both parties cite Section 6 of PD 1069 in support Xxx
of their arguments. X x x
Verily x x x sending to persons sought to be
Does this provision sanction RTC Judge extradited a notice of the request for their arrest and
Purganan’s act of immediately setting for hearing the setting it for hearing at some future date would give them
issuance of a warrant of arrest? We rule in the negative. ample opportunity to prepare and execute an escape.
Neither the Treaty nor the Law could have intended that
1. On the Basis of the Extradition Law consequence, for the very purpose of both would have
been defeated by the escape of the accused from the
It is significant to note that Section 6 of PD 1069, requested state.
our Extradition Law, uses the word “immediate” to qualify
the arrest of the accused. This qualification would be 2. On the Basis of the Constitution
rendered nugatory by setting for hearing the issuance of
the arrest warrant. Hearing entails sending notices to the Even Section 2 of Article III of our Constitution, which is
opposing parties, receiving facts and arguments from invoked by Jimenez, does not require a notice or a hearing
them, and giving them time to prepare and present such before the issuance of a warrant of arrest. X x x
facts and arguments. Arrest subsequent to a hearing can
no longer be considered “immediate.” The law could not To determine probable cause for the issuance of arrest
have intended the word as a mere superfluity but, on the warrants, the Constitution itself requires only the
whole, as a means of impairing a sense of urgency and examination – under oath or affirmation – of complainants
swiftness in the determination of whether a warrant of and the witnesses they may produce. There is no
arrest should be issued. requirement to notify and hear the accused before the
issuance of warrants of arrest.
By using the phrase “if it appears,” the law further
conveys that accuracy is not as important as speed at In Ho v. People (280 SCRA 365, October 9, 1997) and in
such early stage. The trial court is not expected to make all the cases cited therein, never was a judge required to
an exhaustive determination to ferret out the true and go to the extent of conducting a hearing just for the
actual situation, immediately upon the filing of the purpose of personally determining probable cause for the
petition. From the knowledge and the material then issuance of a warrant of arrest. All we required was that
available to it, the court is expected merely to get a good the “judge must have sufficient supporting documents
first impression – a prima facie finding – sufficient to make upon which to make his independent judgment, or at the
a speedy initial determination as regards the arrest and very least, upon which to verify the findings of the
detention of the accused. prosecutor as to the existence of probable cause.”

Xxx In Webb v. De Leon (247 SCRA 652, 680, per Puno, J.),
the Court categorically stated that a judge was not
We stress that the prima facie existence of supposed to conduct a hearing before issuing a warrant
probable cause for hearing the petition and, a priori, for of arrest x x x.
issuing an arrest warrant was already evident from the
Petition itself and its supporting documents. Hence, after At most, in cases of clear insufficiency of evidence on
having already determined therefrom that a prima facie record, judges merely further examine complainants and
finding did exist, respondent judge gravely abused his their witnesses (Ibid; citing Allado v. Diokno, 233 SCRA
discretion when he set the matter for hearing upon 192, May 5, 1994). In the present case, validating the act
motion of Jimenez. of respondent judge and instituting the practice of hearing
the accused and his witnesses at this early stage would
Moreover, the law specifies that the court sets a be discordant with the rationale for the entire system. If
hearing upon receipt of the answer or upon failure of the the accused were allowed to be heard and necessarily to
accused to answer after receiving the summons. In present evidence during the prima facie determination for
connection with the matter of immediate arrest, however, the issuance of a warrant of arrest, what would stop him
the word “hearing” is notably absent from the provision. from presenting his entire plethora of defenses at this
Evidently, had the holding of a hearing at that stage been stage – if he so desires – in his effort to negate a prima
intended, the law could have easily so provided. It also facie finding? Such a procedure could convert the
bears emphasizing at this point that extradition determination of a prima facie case into a full-blown trial
proceedings are summary (See Sec. 9, PD 1069) in of the entire proceedings and possibly make trial of the
nature. Hence, the silence of the Law and the Treaty main case superfluous. This scenario is also anathema to
leans to the more reasonable interpretation that there is the summary nature of extraditions. (Government of
no intention to punctuate with a hearing every little step the United States of America v. Hon. Guillermo
in the entire proceedings.
Purganan, G.R. No. 148571, Sept. 24, 2002, En capricious nor unfounded. (Himagan v. People, 237
Banc [Panganiban]) SCRA 538, Oct. 7, 1994, En Banc [Kapunan])

The Equal Protection Clause 160. Congress enacted R.A. No. 8189 which provides,
in Section 44 thereof, that "No Election Officer shall hold
159. Explain and discuss the equal protection of the office in a particular city or municipality for more than four
law clause. (4) years. Any election officer who, either at the time of
the approval of this Act or subsequent thereto, has served
Held: 1. The equal protection of the law is embraced in for at least four (4) years in a particular city or
the concept of due process, as every unfair discrimination municipality shall automatically be reassigned by the
offends the requirements of justice and fair play. It has Commission to a new station outside the original
nonetheless been embodied in a separate clause in Article congressional district." Petitioners, who are City and
III, Sec. 1, of the Constitution to provide for a more Municipal Election Officers, theorize that Section 44 of RA
specific guaranty against any form of undue favoritism or 8189 is violative of the "equal protection clause" of the
hostility from the government. Arbitrariness in general 1987 Constitution because it singles out the City and
may be challenged on the basis of the due process clause. Municipal Election Officers of the COMELEC as prohibited
But if the particular act assailed partakes of an from holding office in the same city or municipality for
unwarranted partiality or prejudice, the sharper weapon more than four (4) years. They maintain that there is no
to cut it down is the equal protection clause. substantial distinction between them and other COMELEC
officials, and therefore, there is no valid classification to
According to a long line of decisions, equal protection justify the objective of the provision of law under attack.
simply requires that all persons or things similarly situated Resolve.
should be treated alike, both as to rights conferred and
responsibilities imposed. Similar subjects, in other words, Held: The Court is not persuaded by petitioners'
should not be treated differently, so as to give undue arguments. The "equal protection clause" of the 1987
favor to some and unjustly discriminate against others. Constitution permits a valid classification under the
following conditions:
The equal protection clause does not require the universal
application of the laws on all persons or things without 1) The classification must rest on substantial
distinction. This might in fact sometimes result in unequal distinction;
protection, as where, for example, a law prohibiting 2) The classification must be germane to the
mature books to all persons, regardless of age, would purpose of the law;
benefit the morals of the youth but violate the liberty of 3) The classification must not be limited to
adults. What the clause requires is equality among equals existing conditions only; and
as determined according to a valid classification. By 4) The classification must apply equally to all
classification is meant the grouping of persons or things members of the same class.
similar to each other in certain particulars and different
from all others in these same particulars. (Philippine After a careful study, the ineluctable conclusion is that the
Judges Association v. Prado, 227 SCRA 703, 711- classification under Section 44 of RA 8189 satisfies the
712, Nov. 11, 1993, En Banc [Cruz]) aforestated requirements.

2. The equal protection clause exists to prevent undue The singling out of election officers in order to "ensure
favor or privilege. It is intended to eliminate the impartiality of election officials by preventing them
discrimination and oppression based on inequality. from developing familiarity with the people of their place
Recognizing the existence of real difference among men, of assignment" does not violate the equal protection
the equal protection clause does not demand absolute clause of the Constitution.
equality. It merely requires that all persons shall be
treated alike, under like circumstances and conditions In Lutz v. Araneta (98 Phil. 148, 153 [1955]), it was held
both as to the privileges conferred and liabilities enforced . that "the legislature is not required by the Constitution to
Thus, the equal protection clause does not absolutely adhere to a policy of 'all or none'". This is so for
forbid classifications x x x. If the classification is based underinclusiveness is not an argument against a valid
on real and substantial differences; is germane to the classification. It may be true that all other officers of
purpose of the law; applies to all members of the same COMELEC referred to by petitioners are exposed to the
class; and applies to current as well as future conditions, same evils sought to be addressed by the statute.
the classification may not be impugned as violating the However, in this case, it can be discerned that the
Constitution's equal protection guarantee. A distinction legislature thought the noble purpose of the law would be
based on real and reasonable considerations related to a sufficiently served by breaking an important link in the
proper legislative purpose x x x is neither unreasonable, chain of corruption than by breaking up each and every
link thereof. Verily, under Section 3(n) of RA 8189,
election officers are the highest officials or authorized this presumption. The mere allegation that appellant, a
representatives of the COMELEC in a city or municipality. Cebuana, was charged with the commission of a crime,
It is safe to say that without the complicity of such while a Zamboanguena, the guilty party in appellant’s
officials, large-scale anomalies in the registration of voters eyes, was not, is insufficient to support a conclusion that
can hardly be carried out. (Agripino A. De Guzman, Jr., the prosecution officers denied appellant equal
et al. v. COMELEC (G.R. No. 129118, July 19, 2000, protection of the laws.
en Banc [Purisima])
There is also common sense practicality in sustaining
appellant’s prosecution.
161. Appellant, who was charged with Illegal
Recruitment in the RTC of Zamboanga City, invokes the While all persons accused of crime are to be treated on
equal protection clause in her defense. She points out a basis of equality before the law, it does not follow that
that although the evidence purportedly shows that they are to be protected in the commission of crime. It
Jasmine Alejandro handed out application forms and would be unconscionable, for instance, to excuse a
even received Lourdes Modesto’s payment, appellant defendant guilty of murder because others have
was the only one criminally charged. Alejandro, on the murdered with impunity. The remedy for unequal
other hand, remained scot-free. From this, appellant enforcement of the law in such instances does not lie in
concludes that the prosecution discriminated against her the exoneration of the guilty at the expense of society x
on grounds of regional origins. Appellant is a Cebuana x x. Protection of the law will be extended to all persons
while Alejandro is a Zamboanguena, and the alleged equally in the pursuit of their lawful occupations, but no
crime took place in Zamboanga City. person has the right to demand protection of the law in
the commission of a crime (People v. Montgomery, 117
Held: The argument has no merit. P.2d 437 [1941]).

The prosecution of one guilty while others equally guilty Likewise,


are not prosecuted, however, is not, by itself, a denial of
the equal protection of the laws (Application of Finn, 356 [i]f the failure of prosecutors to enforce the criminal
P.2d 685 [1960]). Where the official action purports to laws as to some persons should be converted into a
be in conformity to the statutory classification, an defense for others charged with crime, the result would
erroneous or mistaken performance of the statutory be that the trial of the district attorney for nonfeasance
duty, although a violation of the statute, is not without would become an issue in the trial of many persons
more a denial of the equal protection of the laws charged with heinous crimes and the enforcement of law
(Snowden v. Hughes, 321 US 1, 88 L Ed 497, 64 S Ct would suffer a complete breakdown (State v. Hicks, 325
397 [1943]). The unlawful administration by officers of P.2d 794 [1958]).
a statute fair on its face, resulting in its unequal (People v. Dela Piedra, 350 SCRA 163, Jan. 24,
application to those who are entitled to be treated alike, 2001, 1st Div. [Kapunan])
is not a denial of equal protection, unless there is shown
to be present in it an element of intentional or
purposeful discrimination. This may appear on the face
of the action taken with respect to a particular class or 162. Are there substantial distinctions between print
person, or it may only be shown by extrinsic evidence media and broadcast media to justify the requirement for
showing a discriminatory design over another not to be the latter to give free airtime to be used by the Comelec
inferred from the action itself. But a discriminatory to inform the public of qualifications and program of
purpose is not presumed, there must be a showing of government of candidates and political parties during the
“clear and intentional discrimination.” (Ibid.) Appellant campaign period? Discuss.
has failed to show that, in charging appellant in court,
that there was a “clear and intentional discrimination” on Held: There are important differences in the
the part of the prosecuting officials. characteristics of the two media which justify their
differential treatment for free speech purposes. Because
The discretion of who to prosecute depends on the of the physical limitations of the broadcast spectrum, the
prosecution’s sound assessment whether the evidence government must, of necessity, allocate broadcast
before it can justify a reasonable belief that a person frequencies to those wishing to use them. There is no
has committed an offense (Tan, Jr. v. Sandiganbayan similar justification for government allocation and
[Third Division], 292 SCRA 452 [1998]). The regulation of the print media.
presumption is that the prosecuting officers regularly
performed their duties (Rules of Court, Rule 131, Sec. 5 In the allocation of limited resources, relevant conditions
[m]), and this presumption can be overcome only by may validly be imposed on the grantees or licensees. The
proof to the contrary, not by mere speculation. Indeed, reason for this is that the government spends public funds
appellant has not presented any evidence to overcome for the allocation and regulation of the broadcast industry,
which it does not do in the case of print media. To require International law, which springs from general principles
radio and television broadcast industry to provide free of law, likewise proscribes discrimination x x x. The
airtime for the Comelec Time is a fair exchange for what Universal Declaration of Human Rights, the International
the industry gets. Covenant on Economic, Social and Cultural Rights, the
International Convention on the Elimination of All Forms
From another point of view, the SC has also held that of Racial Discrimination, the Convention against
because of the unique and pervasive influence of the Discrimination in Education, the Convention (No. 111)
broadcast media, “[n]ecessarily x x x the freedom of Concerning Discrimination in Respect of Employment and
television and radio broadcasting is somewhat lesser in Occupation - all embody the general principle against
scope than the freedom accorded to newspaper and print discrimination, the very antithesis of fairness and justice.
media.” (TELEBAP, Inc. v. COMELEC, 289 SCRA 337, The Philippines, through its Constitution, has incorporated
April 21, 1998 [Mendoza]) this principle as part of its national laws.

163. Does the death penalty law (R.A. No. 7659) [I]t would be an affront to both the spirit and letter of
violate the equal protection clause considering that, in these provisions if the State, in spite of its primordial
effect, it punishes only people who are poor, uneducated, obligation to promote and ensure equal employment
and jobless? opportunities, closes its eyes to unequal and
discriminatory terms and conditions of employment x x x.
Held: R.A. No. 7659 specifically provides that “[T]he
death penalty shall be imposed if the crime of rape is Discrimination, particularly in terms of wages, is frowned
committed x x x when the victim is a religious or a child upon by the Labor Code. Article 135, for example,
below seven (7) years old.” Apparently, the death penalty prohibits and penalizes the payment of lesser
law makes no distinction. It applies to all persons and to compensation to a female employee as against a male
all classes of persons – rich or poor, educated or employee for work of equal value. Article 248 declares it
uneducated, religious or non-religious. No particular an unfair labor practice for an employer to discriminate in
person or classes of persons are identified by the law regards to wages in order to encourage or discourage
against whom the death penalty shall be exclusively membership in any labor organization. X x x
imposed. The law punishes with death a person who shall
commit rape against a child below seven years of age. The foregoing provisions impregnably institutionalize in
Thus, the perpetration of rape against a 5-year old girl this jurisdiction the long honored legal truism of "Equal
does not absolve or exempt an accused from the pay for equal work." Persons who work with substantially
imposition of the death penalty by the fact that he is poor, equal qualifications, skill, effort and responsibility, under
uneducated, jobless, and lacks catechetical instruction. similar conditions, should be paid similar salaries. This
To hold otherwise will not eliminate but promote rule applies to the School (International School, Inc.), its
inequalities. "international character" notwithstanding.

In Cecilleville Realty and Service Corporation v. CA, 278 The School contends that petitioner has not adduced
SCRA 819 [1997]), the SC clarified that compassion for evidence that local-hires perform work equal to that of
the poor is an imperative of every humane society but foreign-hires. The Court finds this argument a little
only when the recipient is not a rascal claiming an cavalier. If an employer accords employees the same
undeserved privilege. (People v. Jimmy Mijano y position and rank, the presumption is that these
Tamora, G.R. No. 129112, July 23, 1999, En Banc employees perform equal work. This presumption is
[Per Curiam]) borne by logic and human experience. If the employer
pays one employee less than the rest, it is not for that
164. The International School Alliance of Educators employee to explain why he receives less or why the
(ISAE) questioned the point-of-hire classification others receive more. That would be adding insult to
employed by International School, Inc. to justify injury. The employer has discriminated against that
distinction in salary rates between foreign-hires and local- employee; it is for the employer to explain why the
hires, i.e., salary rates of foreign-hires are higher by 25% employee is treated unfairly.
than their local counterparts, as discriminatory and,
therefore, violates the equal protection clause. The The employer in this case failed to discharge this burden.
International School contended that this is necessary in There is no evidence here that foreign-hires perform 25%
order to entice foreign-hires to leave their domicile and more efficiently or effectively than the local-hires. Both
work here. Resolve. groups have similar functions and responsibilities, which
they perform under similar working conditions.
Held: That public policy abhors inequality and
discrimination is beyond contention. Our Constitution and The School cannot invoke the need to entice foreign-hires
laws reflect the policy against these evils. X x x to leave their domicile to rationalize the distinction in
salary rates without violating the principle of equal work The performance of legitimate and even essential duties
for equal pay. by public officers has never been an excuse to free a
person validly in prison. The duties imposed by the
Xxx “mandate of the people” are multifarious. The accused-
appellant asserts that the duty to legislate ranks highest
While we recognize the need of the School to attract in the hierarchy of government. The accused-appellant is
foreign-hires, salaries should not be used as an only one of 250 members of the House of
enticement to the prejudice of local-hires. The local-hires Representatives, not to mention the 24 members of the
perform the same services as foreign-hires and they Senate, charged with the duties of legislation. Congress
ought to be paid the same salaries as the latter. For the continues to function well in the physical absence of one
same reason, the "dislocation factor" and the foreign- or a few of its members. Depending on the exigency of
hires' limited tenure also cannot serve as valid bases for Government that has to be addressed, the President or
the distinction in salary rates. The dislocation factor and the Supreme Court can also be deemed the highest for
limited tenure affecting foreign-hires are adequately that particular duty. The importance of a function
compensated by certain benefits accorded them which depends on the need for its exercise. The duty of a
are not enjoyed by local-hires, such as housing, mother to nurse her infant is most compelling under the
transportation, shipping costs, taxes and home leave law of nature. A doctor with unique skills has the duty to
travel allowances. save the lives of those with a particular affliction. An
elective governor has to serve provincial constituents. A
The Constitution enjoins the State to "protect the rights police officer must maintain peace and order. Never had
of workers and promote their welfare", "to afford labor the call of a particular duty lifted a prisoner into a different
full protection." The State, therefore, has the right and classification from those others who are validly restrained
duty to regulate the relations between labor and capital . by law.
These relations are not merely contractual but are so
impressed with public interest that labor contracts, A strict scrutiny of classifications is essential lest wittingly
collective bargaining agreements included, must yield to or otherwise, insidious discriminations are made in favor
the common good. Should such contracts contain of or against groups or types of individuals.
stipulations that are contrary to public policy, courts will
not hesitate to strike down these stipulations. The Court cannot validate badges of inequality. The
necessities imposed by public welfare may justify exercise
In this case, we find the point-of-hire classification of government authority to regulate even if thereby
employed by respondent School to justify the distinction certain groups may plausibly assert that their interests are
in the salary rates of foreign-hires and local-hires to be an disregarded.
invalid classification. There is no reasonable distinction
between the services rendered by foreign-hires and local- We, therefore, find that election to the position of
hires. The practice of the School of according higher Congressman is not a reasonable classification in criminal
salaries to foreign-hires contravenes public policy and, law enforcement. The functions and duties of the office
certainly, does not deserve the sympathy of this Court. are not substantial distinctions which lift him from the
(International School Alliance of Educators (ISAE) class of prisoners interrupted in their freedom and
v. Hon. Leonardo A. Quisumbing, G.R. No. 128845, restricted in liberty of movement. Lawful arrest and
June 1, 2000, 1st Div. [Kapunan]) confinement are germane to the purposes of the law and
apply to all those belonging to the same class.
165. Accused-appellant Romeo G. Jalosjos filed a
motion before the Court asking that he be allowed to fully Xxx
discharge the duties of a Congressman, including
attendance at legislative sessions and committee It can be seen from the foregoing that incarceration, by
meetings despite his having been convicted in the first its nature, changes an individual’s status in society .
instance of a non-bailable offense. Does being an elective Prison officials have the difficult and often thankless job
official result in a substantial distinction that allows of preserving the security in a potentially explosive
different treatment? Is being a Congressman a setting, as well as of attempting to provide rehabilitation
substantial differentiation which removes the accused- that prepare inmates for re-entry into the social
appellant as a prisoner from the same class as all persons mainstream. Necessarily, both these demands require
validly confined under law? the curtailment and elimination of certain rights.

Held: In the ultimate analysis, the issue before us boils Premises considered, we are constrained to rule against
down to a question of constitutional equal protection. the accused-appellant’s claim that re-election to public
office gives priority to any other right or interest, including
Xxx the police power of the State. (People v. Jalosjos, 324
SCRA 689, Feb. 3, 2000, En Banc [Ynares- of arrest. Obviously and understandably, the contents of
Santiago]) the prosecutor’s report will support his own conclusion
that there is reason to charge the accused for an offense
and hold him for trial. However, the judge must decide
The Right against Unreasonable Searches and Seizures independently. Hence, he must have supporting
evidence, other than the prosecutor’s bare report, upon
166. Discuss the constitutional requirement that a which to legally sustain his own findings on the existence
judge, in issuing a warrant of arrest, must determine (or nonexistence) of probable cause to issue an arrest
probable cause “personally.” Distinguish determination of order. This responsibility of determining personally and
probable cause by the prosecutor and determination of independently the existence or nonexistence of probable
probable cause by the judge. cause is lodged in him by no less than the most basic law
of the land. Parenthetically, the prosecutor could ease
Held: It must be stressed that the 1987 Constitution the burden of the judge and speed up the litigation
requires the judge to determine probable cause process by forwarding to the latter not only the
“personally,” a requirement which does not appear in the information and his bare resolution finding probable
corresponding provisions of our previous constitutions. cause, but also so much of the records and the evidence
This emphasis evinces the intent of the framers to place on hand as to enable the His Honor to make his personal
a greater degree of responsibility upon trial judges than and separate judicial finding on whether to issue a
that imposed under previous Constitutions . warrant of arrest.

In Soliven v. Makasiar, this Court pronounced: Lastly, it is not required that the complete or entire
records of the case during the preliminary investigation
“What the Constitution underscores is the exclusive and be submitted to and examined by the judge. We do not
personal responsibility of the issuing judge to satisfy intend to unduly burden trial courts by obliging them to
himself of the existence of probable cause. In satisfying examine the complete records of every case all the time
himself of the existence of probable cause for the simply for the purpose of ordering the arrest of an
issuance of a warrant of arrest, the judge is not required accused. What is required, rather, is that the judge must
to personally examine the complainant and his witnesses. have sufficient supporting documents (such as the
Following established doctrine and procedure, he shall: complaint, affidavits, counter-affidavits, sworn
(1) personally evaluate the report and the supporting statements of witnesses or transcript of stenographic
documents submitted by the fiscal regarding the notes, if any) upon which to make his independent
existence of probable cause and, on the basis thereof, judgment or, at the very least, upon which to verify the
issue a warrant of arrest; or (2) if in the basis thereof he findings of the prosecutor as to the existence of probable
finds no probable cause, he may disregard the fiscal’s cause. The point is: he cannot rely solely and entirely on
report and require the submission of supporting affidavits the prosecutor’s recommendation, as Respondent Court
of witnesses to aid him in arriving at a conclusion as to did in this case. Although the prosecutor enjoys the legal
the existence of probable cause.” presumption of regularity in the performance of his official
duties and functions, which in turn gives his report the
Ho v. People (Ibid.) summarizes existing jurisprudence on presumption of accuracy, the Constitution, we repeat,
the matter as follows: commands the judge to personally determine probable
cause in the issuance of warrants of arrest. This Court
“Lest we be too repetitive, we only wish to emphasize has consistently held that a judge fails in his bounden
three vital matters once more: First, as held in Inting, the duty if he relies merely on the certification or the report
determination of probable cause by the prosecutor is for of the investigating officer.” (Citations omitted)
a purpose different from that which is to be made by the
judge. Whether there is reasonable ground to believe In the case at bench, respondent admits that he issued
that the accused is guilty of the offense charged and the questioned warrant as there was “no reason for (him)
should be held for trial is what the prosecutor passes to doubt the validity of the certification made by the
upon. The judge, on the other hand, determines whether Assistant Prosecutor that a preliminary investigation was
a warrant of arrest should be issued against the accused, conducted and that probable cause was found to exist as
i.e., whether there is a necessity for placing him under against those charged in the information filed.” The
immediate custody in order not to frustrate the ends of statement is an admission that respondent relied solely
justice. Thus, even if both should base their findings on and completely on the certification made by the fiscal that
one and the same proceeding or evidence, there should probable cause exists as against those charged in the
be no confusion as to their distinct objectives. information and issued the challenged warrant of arrest
on the sole basis of the prosecutor’s findings and
Second, since their objectives are different, the judge recommendations. He adopted the judgment of the
cannot rely solely on the report of the prosecutor in prosecutor regarding the existence of probable cause as
finding probable cause to justify the issuance of a warrant
his own. (Abdula v. Guiani, 326 SCRA 1, Feb. 18, address of the compound which is 516 San Jose de la
2000, 3rd Div. [Gonzaga-Reyes]) Montana St., Mabolo, Cebu City. Did this satisfy the
constitutional requirement under Section 2, Article III that
the place to be searched must be particularly described?
167. Accused-appellant assails the validity of his
arrest and his subsequent convictions for the two Held: This Court has held that the applicant should
crimes. Both the trial court and the Court of Appeals particularly describe the place to be searched and the
found that the arrest and subsequent seizure were legal. person or things to be seized, wherever and whenever it
is feasible. In the present case, it must be noted that the
Held: A review of the records at bar shows no reason application for a search warrant was accompanied by a
to depart therefrom. sketch of the compound at 516 San Jose de la Montana
St., Mabolo, Cebu City. The sketch indicated the 2-storey
The constitutional proscription, that no person residential house of private respondent with a large "X"
shall be arrested without any warrant of arrest having enclosed in a square. Within the same compound are
been issued prior thereto, is not a hard-and-fast rule. X residences of other people, workshops, offices, factories
x x (Citations omitted) and warehouse. With this sketch as the guide, it could
have been very easy to describe the residential house of
In the cases at bar, the police saw the gun private respondent with sufficient particularity so as to
tucked in appellant’s waist when he stood up. The gun segregate it from the other buildings or structures inside
was plainly visible. No search was conducted as none the same compound. But the search warrant merely
was necessary. Accused-appellant could not show any indicated the address of the compound which is 516 San
license for the firearm, whether at the time of his arrest Jose de la Montana St., Mabolo, Cebu City. This
or thereafter. Thus, he was in effect committing a crime description of the place to be searched is too general and
in the presence of the police officers. No warrant of does not pinpoint the specific house of private
arrest was necessary in such a situation, it being one of respondent. Thus, the inadequacy of the description of
the recognized exceptions under the Rules. the residence of private respondent sought to be
searched has characterized the questioned search
As a consequence of appellant’s valid warrant as a general warrant, which is violative of the
warrantless arrest, he may be lawfully searched for constitutional requirement. (People v. Estrada, 296
dangerous weapons or anything which may be used as SCRA 383, 400, [Martinez])
proof of the commission of an offense, without a search
warrant, as provided in Rule 126, Section 12. This is a 169. Can the place to be searched, as set out in the
valid search incidental to a lawful arrest. The warrant, be amplified or modified by the officers’ own
subsequent discovery in his car of drug paraphernalia personal knowledge of the premises, or the evidence they
and the crystalline substance, which, was later identified adduce in support of their application for the warrant?
as shabu, though in a distant place from where the
illegal possession of firearm was committed, cannot be Held: Such a change is proscribed by the Constitution
said to have been made during an illegal search. As which requires inter alia the search warrant to particularly
such, the seized items do not fall within the exclusionary describe the place to be searched as well as the persons
clause x x x. Hence, not being fruits of the poisonous or things to be seized. It would concede to police officers
tree x x x the objects found at the scene of the crime, the power of choosing the place to be searched, even if it
such as the firearm, the shabu and the drug not be that delineated in the warrant. It would open wide
paraphernalia, can be used as evidence against the door to abuse of the search process, and grant to
appellant. Besides, it has been held that drugs officers executing a search warrant that discretion which
discovered as a result of a consented search is the Constitution has precisely removed from them. The
admissible in evidence. (Citations omitted.) (People v. particularization of the description of the place to be
Go, 354 SCRA 338, Mar. 14, 2001, 1st Div. searched may properly be done only by the Judge, and
[Ynares-Santiago]) only in the warrant itself; it cannot be left to the discretion
of the police officers conducting the search.

It is neither fair nor licit to allow police officers to search


168. In an application for search warrant, the a place different from that stated in the warrant on the
application was accompanied by a sketch of the claim that the place actually searched – although not that
compound at 516 San Jose de la Montana St., Mabolo, specified in the warrant – is exactly what they had in view
Cebu City, indicating the 2-storey residential house of when they applied for the warrant and had demarcated in
private respondent with a large “X” enclosed in a square. their supporting evidence. What is material in
Within the same compound are residences of other determining the validity of a search is the place stated in
people, workshops, offices, factories and warehouse. The the warrant itself, not what applicants had in their
search warrant issued, however, merely indicated the thoughts, or had represented in the proofs they submitted
to the court issuing the warrant. (People v. Court of the items that they observe may be evidence of a crime,
Appeals, 291 SCRA 400, June 26, 1998 [Narvasa]) contraband or otherwise subject to seizure. (People v.
Doria, 301 SCRA 668, Jan. 22, 1999, En Banc
170. What is “search incidental to a lawful arrest”? [Puno, J.])
Discuss.
2. For the doctrine to apply, the following elements must
Held: While a contemporaneous search of a person be present:
arrested may be effected to discover dangerous weapons
or proofs or implements used in the commission of the a) a prior valid intrusion based on the valid
crime and which search may extend to the area within his warrantless arrest in which the police are legally present
immediate control where he might gain possession of a in the pursuit of their official duties;
weapon or evidence he can destroy, a valid arrest must b) the evidence was inadvertently discovered by
precede the search. The process cannot be reversed. the police who have the right to be where they are; and
c) the evidence must be immediately apparent;
In a search incidental to a lawful arrest, as the precedent and
arrest determines the validity of the incidental search, the d) plain view justified mere seizure of evidence
legality of the arrest is questioned in a large majority of without further search.
these cases, e.g., whether an arrest was merely used as
a pretext for conducting a search. In this instance, the In the instant case, recall that PO2 Balut testified that
law requires that there be first a lawful arrest before a they first located the marijuana plants before appellant
search can be made – the process cannot be reversed. was arrested without a warrant. Hence, there was no
(Malacat v. Court of Appeals, 283 SCRA 159, 175 [1997]) valid warrantless arrest which preceded the search of
(People v. Chua Ho San, 308 SCRA 432, June 17, appellant’s premises. Note further that the police team
1999, En Banc [Davide, Jr., C.J.]) was dispatched to appellant’s kaingin precisely to search
for and uproot the prohibited flora. The seizure of
171. What is the “plain view” doctrine? What are its evidence in “plain view” applies only where the police
requisites? Discuss. officer is not searching for evidence against the accused,
but inadvertently comes across an incriminating object .
Held: 1. Objects falling in plain view of an officer who Clearly, their discovery of the cannabis plants was not
has a right to be in the position to have that view are inadvertent. We also note the testimony of SPO2 Tipay
subject to seizure even without a search warrant and may that upon arriving at the area, they first had to “look
be introduced in evidence . The “plain view” doctrine around the area” before they could spot the illegal plants.
applies when the following requisites concur: (a) the law Patently, the seized marijuana plants were not
enforcement officer in search of the evidence has a prior “immediately apparent” and “further search” was needed.
justification for an intrusion or is in a position from which In sum, the marijuana plants in question were not in
he can view a particular area; (b) the discovery of the “plain view” or “open to eye and hand.” The “plain view”
evidence in plain view is inadvertent; (c) it is immediately doctrine, thus, cannot be made to apply.
apparent to the officer that the item he observes may be
evidence of a crime, contraband or otherwise subject to Nor can we sustain the trial court’s conclusion that just
seizure. The law enforcement officer must lawfully make because the marijuana plants were found in an unfenced
an initial intrusion or properly be in a position from which lot, appellant could not invoke the protection afforded by
he can particularly view the area. In the course of such the Charter against unreasonable searches by agents of
lawful intrusion, he came inadvertently across a piece of the State. The right against unreasonable searches and
evidence incriminating the accused. The object must be seizures is the immunity of one’s person, which includes
open to eye and hand and its discovery inadvertent. his residence, his papers, and other possessions. The
guarantee refers to “the right of personal security” of the
It is clear that an object is in plain view if the object itself individual. X x x, what is sought to be protected against
is plainly exposed to sight. The difficulty arises when the the State’s unlawful intrusion are persons, not places . To
object is inside a closed container. Where the object conclude otherwise would not only mean swimming
seized was inside a closed package, the object itself is not against the stream, it would also lead to the absurd logic
in plain view and therefore cannot be seized without a that for a person to be immune against unreasonable
warrant. However, if the package proclaims its contents, searches and seizures, he must be in his home or office,
whether by its distinctive configuration, its transparency, within a fenced yard or a private place. The Bill of Rights
or if its contents are obvious to an observer, then the belongs as much to the person in the street as to the
contents are in plain view and may be seized. In other individual in the sanctuary of his bedroom. (People v.
words, if the package is such that an experienced Abe Valdez, G.R. No. 129296, Sept. 25, 2000, En
observer could infer from its appearance that it contains Banc [Quisumbing])
the prohibited article, then the article is deemed in plain
view. It must be immediately apparent to the police that
3. Considering its factual milieu, this case falls squarely In allowing such a search, the United States Supreme
under the plain view doctrine. X x x. Court held that the interest of effective crime prevention
and detection allows a police officer to approach a person,
When Spencer wrenched himself free from the grasp of in appropriate circumstances and manner, for purposes of
PO2 Gaviola, he instinctively ran towards the house of investigating possible criminal behavior even though
appellant. The members of the buy-bust team were there is insufficient probable cause to make an actual
justified in running after him and entering the house arrest.
without a search warrant for they were hot in the heels
of a fleeing criminal. Once inside the house, the police In admitting in evidence two guns seized during the stop-
officers cornered Spencer and recovered the buy-bust and-frisk, the US Supreme Court held that what justified
money from him. They also caught appellant in flagrante the limited search was the more immediate interest of the
delicto repacking the marijuana bricks which were in full police officer in taking steps to assure himself that the
view on top of a table. X x x. person with whom he was dealing was not armed with a
weapon that could unexpectedly and fatally be used
Hence, appellant’s subsequent arrest was likewise lawful, against him.
coming as it is within the purview of Section 5(a) of Rule
113 of the 1985 Rules on Criminal Procedure x x x. It did not, however, abandon the rule that the police
must, whenever practicable, obtain advance judicial
Section 5(a) is commonly referred to as the rule on in approval of searches and seizures through the warrant
flagrante delicto arrests. Here two elements must concur: procedure, excused only by exigent circumstances.
(1) the person to be arrested must execute an overt act (Manalili v. CA, 280 SCRA 400, Oct. 9, 1997
indicating that he has just committed, is actually [Panganiban])
committing, or is attempting to commit a crime; and (2)
such overt act is done in the presence or within the view 2. We now proceed to the justification for and allowable
of the arresting officer. Thus, when appellant was seen scope of a “stop-and-frisk” as a “limited protective search
repacking the marijuana, the police officers were not only of outer clothing for weapons,” as laid down in Terry,
authorized but also duty-bound to arrest him even thus:
without a warrant. (People v. Elamparo, 329 SCRA
404, 414-415, March 31, 2000, 2nd Div. We merely hold today that where a police officer observes
[Quisumbing]) unusual conduct which leads him reasonably to conclude
in light of his experience that criminal activity may be
172. What is a “stop-and-frisk” search? afoot and that the persons with whom he is dealing may
be armed and presently dangerous, where in the course
Held: 1. In the landmark case of Terry v. Ohio (20 L Ed of investigating this behavior he identifies himself as a
2d 889; 88 S Ct 1868, 392 US 1, 900, June 10, 1968), a policeman and makes reasonable inquiries, and where
stop-and-frisk was defined as the vernacular designation nothing in the initial stages of the encounter serves to
of the right of a police officer to stop a citizen on the dispel his reasonable fear for his own or others’ safety, he
street, interrogate him, and pat him for weapon(s): is entitled for the protection of himself and others in the
area to conduct a carefully limited search of the outer
“x x x (W)here a police officer observes an unusual clothing of such persons in an attempt to discover
conduct which leads him reasonably to conclude in light weapons which might be used to assault him. Such a
of his experience that criminal activity may be afoot and search is a reasonable search under the Fourth
that the persons with whom he is dealing may be armed Amendment (Terry, at 911. In fact, the Court noted that
and presently dangerous, where in the course of the ‘sole justification’ for a stop-and-frisk was the
investigating this behavior he identified himself as a ‘protection of the police officer and others nearby’; while
policeman and make reasonable inquiries, and where the scope of the search conducted in the case was limited
nothing in the initial stages of the encounter serves to to patting down the outer clothing of petitioner and his
dispel his reasonable fear for his own or others’ safety, he companions, the police officer did not place his hands in
is entitled for the protection of himself or others in the their pockets nor under the outer surface of their
area to conduct a carefully limited search of the outer garments until he had felt weapons, and then he merely
clothing of such persons in an attempt to discover reached for and removed the guns. This did not
weapons which might be used to assault him. Such a constitute a general exploratory search, Id.)
search is a reasonable search under the Fourth
Amendment, and any weapon seized may properly be Other notable points of Terry are that while probable
introduced in evidence against the person from whom cause is not required to conduct a “stop-and-frisk,” it
they were taken.” (Herrera, A Handbook on Arrest, nevertheless holds that mere suspicion or a hunch will not
Search and Seizure and Custodial Investigation, 1995 ed., validate a “stop-and-frisk.” A genuine reason must exist,
p. 185; and Terry v. Ohio, supra, p. 911) in light of the police officer’s experience and surrounding
conditions, to warrant the belief that the person detained
has weapons concealed about him. Finally, a “stop-and- The facts adduced do not constitute a ground for
frisk” serves a two-fold interest: (1) the general interest a violation of the constitutional rights of the accused
of effective crime prevention and detection, which against illegal search and seizure. PO3 Suba admitted
underlies the recognition that a police officer may, under that they were merely stopping cars they deemed
appropriate circumstances and in an appropriate manner, suspicious, such as those whose windows are heavily
approach a person for purposes of investigating possible tinted just to see if the passengers thereof were carrying
criminal behavior even without probable cause; and (2) guns. At best they would merely direct their flashlights
the more pressing interest of safety and self-preservation inside the cars they would stop, without opening the car’s
which permit the police officer to take steps to assure doors or subjecting its passengers to a body search.
himself that the person with whom he deals is not armed There is nothing discriminatory in this as this is what the
with a deadly weapon that could unexpectedly and fatally situation demands.
be used against the police officer. (Malacat v. Court of
Appeals, 283 SCRA 159, Dec. 12, 1997 [Davide]) We see no need for checkpoints to be announced
x x x. Not only would it be impractical, it would also
173. Are searches at checkpoints valid? Discuss. forewarn those who intend to violate the ban. Even so,
badges of legitimacy of checkpoints may still be inferred
Held: Accused-appellants assail the manner by which the from their fixed location and the regularized manner in
checkpoint in question was conducted. They contend that which they are operated. (People v. Usana, 323 SCRA
the checkpoint manned by elements of the Makati Police 754, Jan. 28, 2000, 1st Div. [Davide, CJ])
should have been announced. They also complain of its
having been conducted in an arbitrary and discriminatory 174. Do the ordinary rights against unreasonable
manner. searches and seizures apply to searches conducted at the
airport pursuant to routine airport security procedures?
We take judicial notice of the existence of the
COMELEC resolution imposing a gun ban during the Held: Persons may lose the protection of the search and
election period issued pursuant to Section 52(c) in relation seizure clause by exposure of their persons or property to
to Section 26(q) of the Omnibus Election Code (Batas the public in a manner reflecting a lack of subjective
Pambansa Blg. 881). The national and local elections in expectation of privacy, which expectation society is
1995 were held on 8 May, the second Monday of the prepared to recognize as reasonable. Such recognition is
month. The incident, which happened on 5 April 1995, implicit in airport security procedures. With increased
was well within the election period. concern over airplane hijacking and terrorism has come
increased security at the nation’s airports. Passengers
This Court has ruled that not all checkpoints are attempting to board an aircraft routinely pass through
illegal. Those which are warranted by the exigencies of metal detectors; their carry-on baggage as well as
public order and are conducted in a way least intrusive to checked luggage are routinely subjected to x-ray scans.
motorists are allowed. For, admittedly, routine Should these procedures suggest the presence of
checkpoints do intrude, to a certain extent, on motorists’ suspicious objects, physical searches are conducted to
right to “free passage without interruption,” but it cannot determine what the objects are. There is little question
be denied that, as a rule, it involves only a brief detention that such searches are reasonable, given their minimal
of travelers during which the vehicle’s occupants are intrusiveness, the gravity of the safety interests involved,
required to answer a brief question or two. For as long and the reduced privacy expectations associated with
as the vehicle is neither searched nor its occupants airline travel. Indeed, travelers are often notified through
subjected to a body search, and the inspection of the airport public address systems, signs, and notices in their
vehicle is limited to a visual search, said routine checks airline tickets that they are subject to search and, if any
cannot be regarded as violative of an individual’s right prohibited materials or substances are found, such would
against unreasonable search. In fact, these routine be subject to seizure. These announcements place
checks, when conducted in a fixed area, are even less passengers on notice that ordinary constitutional
intrusive. protections against warrantless searches and seizures do
not apply to routine airport procedures.
The checkpoint herein conducted was in
pursuance of the gun ban enforced by the COMELEC. The The packs of methamphetamine hydrochloride having
COMELEC would be hard put to implement the ban if its thus been obtained through a valid warrantless search,
deputized agents were limited to a visual search of they are admissible in evidence against the accused-
pedestrians. It would also defeat the purpose for which appellant herein. Corollarily, her subsequent arrest,
such ban was instituted. Those who intend to bring a gun although likewise without warrant, was justified since it
during said period would know that they only need a car was effected upon the discovery and recovery of “shabu”
to be able to easily perpetrate their malicious designs. in her person in flagrante delicto. (People v. Leila
Johnson, G.R. No. 138881, Dec. 18, 2000, 2nd Div.
[Mendoza])
assistance to the local health centers or through outreach
175. May the constitutional protection against and charity programs. Only with the proper government
unreasonable searches and seizures be extended to acts sanctions can medicines and drugs circulate the market.
committed by private individuals? We cannot afford to take any risk, for the life and health
of the citizenry are as precious as the existence of the
Held: As held in People v. Marti (193 SCRA 57 [1991]), State. (People v. Judge Estrella T. Estrada, G.R No.
the constitutional protection against unreasonable 124461, June 26, 2000, Spcl. 2nd Div. [Ynares-
searches and seizures refers to the immunity of one's Santiago])
person from interference by government and it cannot be
extended to acts committed by private individuals so as 177. Do Regional Trial Courts have competence to
to bring it within the ambit of alleged unlawful intrusion. pass upon the validity or regularity of seizure and
(People v. Mendoza, 301 SCRA 66, Jan. 18, 1999, forfeiture proceedings conducted by the Bureau of
1st Div. [Melo]) Customs and to enjoin or otherwise interfere with these
proceedings?
176. Should the seized drugs which are
pharmaceutically correct but not properly documented Held: In Jao v. Court of Appeals (249 SCRA 35, 42-43
subject of an illegal search because the applicant “failed [1995]), this Court, reiterating its rulings x x x said:
to allege in the application for search warrant that the
subject drugs for which she was applying for search There is no question that Regional Trial Courts are devoid
warrant were either fake, misbranded, adulterated, or of any competence to pass upon the validity or regularity
unregistered,” be returned to the owner? of seizure and forfeiture proceedings conducted by the
Bureau of Customs and to enjoin or otherwise interfere
Held: With the State's obligation to protect and promote with these proceedings. The Collector of Customs sitting
the right to health of the people and instill health in seizure and forfeiture proceedings has exclusive
consciousness among them (Article II, Section 15, 1987 jurisdiction to hear and determine all questions touching
Constitution), in order to develop a healthy and alert on the seizure and forfeiture of dutiable goods. The
citizenry (Article XIV, Section 19[1]), it became Regional Trial Courts are precluded from assuming
mandatory for the government to supervise and control cognizance over such matters even through petitions of
the proliferation of drugs in the market. The certiorari, prohibition or mandamus.
constitutional mandate that "the State shall adopt an
integrated and comprehensive approach to health It is likewise well-settled that the provisions of the Tariff
development which shall endeavor to make essential and Customs Code and that of Republic Act No. 1125, as
goods, health and other social services available to all amended, otherwise known as “An Act Creating the Court
people at affordable cost" (Article XIII, Section 11) cannot of Tax Appeals,” specify the proper fora and procedure
be neglected. This is why "the State shall establish and for the ventilation of any legal objections or issues raised
maintain an effective food and drug regulatory system." concerning these proceedings. Thus, actions of the
(Article XIII, Section 12) The BFAD is the government Collector of Customs are appealable to the Commissioner
agency vested by law to make a mandatory and of Customs, whose decision, in turn, is subject to the
authoritative determination of the true therapeutic effect exclusive appellate jurisdiction of the Court of Tax Appeals
of drugs because it involves technical skill which is within and from there to the Court of Appeals.
its special competence. The health of the citizenry should
never be compromised. To the layman, medicine is a cure The rule that Regional Trial Courts have no review powers
that may lead to better health. over such proceedings is anchored upon the policy of
placing no unnecessary hindrance on the government’s
If the seized 52 boxes of drugs are pharmaceutically drive, not only to prevent smuggling and other frauds
correct but not properly documented, they should be upon Customs, but more importantly, to render effective
promptly disposed of in the manner provided by law in and efficient the collection of import and export duties
order to ensure that the same do not fall into the wrong due the State, which enables the government to carry out
hands who might use the drugs underground. Private the functions it has been instituted to perform.
respondent cannot rely on the statement of the trial court
that the applicant "failed to allege in the application for Even if the seizure by the Collector of Customs were
search warrant that the subject drugs for which she was illegal, x x x we have said that such act does not deprive
applying for search warrant were either fake, misbranded, the Bureau of Customs of jurisdiction thereon.
adulterated, or unregistered" in order to obtain the return
of the drugs. The policy of the law enunciated in R.A. No. Respondents cite the statement of the Court of Appeals
8203 is to protect the consumers as well as the licensed that regular courts still retain jurisdiction “where, as in
businessmen. Foremost among these consumers is the this case, for lack of probable cause, there is serious
government itself which procures medicines and doubt as to the propriety of placing the articles under
distributes them to the local communities through direct Customs jurisdiction through seizure/forfeiture
proceedings.” They overlook the fact, however, that suspicion. The grounds of suspicion are reasonable
under the law, the question of whether probable cause when, in the absence of actual belief of the arresting
exists for the seizure of the subject sacks of rice is not for officers, the suspicion that the person to be arrested is
the Regional Trial Court to determine. The customs probably guilty of committing the offense, is based on
authorities do not have to prove to the satisfaction of the actual facts, i.e., supported by circumstances sufficiently
court that the articles on board a vessel were imported strong in themselves to create the probable cause of guilt
from abroad or are intended to be shipped abroad before of the person to be arrested. A reasonable suspicion
they may exercise the power to effect customs’ searches, therefore must be founded on probable cause, coupled
seizures, or arrests provided by law and continue with the with good faith on the part of the peace officers making
administrative hearings. As the Court held in Ponce Enrile the arrest.
v. Vinuya (37 SCRA 381, 388-389 [1971], reiterated in Jao
v. Court of Appeals, supra and Mison v. Natividad, 213 As applied to in flagrante delicto arrests, it is
SCRA 734 [1992]): settled that “reliable information” alone, absent any overt
act indicative of a felonious enterprise in the presence and
The governmental agency concerned, the Bureau of within the view of the arresting officers, are not sufficient
Customs, is vested with exclusive authority. Even if it be to constitute probable cause that would justify an in
assumed that in the exercise of such exclusive flagrante delicto arrest. Thus, in People v. Aminnudin
competence a taint of illegality may be correctly imputed, (163 SCRA 402, 409-410 [1988]), it was held that “the
the most that can be said is that under certain accused-appellant was not, at the moment of his arrest,
circumstances the grave abuse of discretion conferred committing a crime nor was it shown that he was about
may oust it of such jurisdiction. It does not mean to do so or that he had just done so. What he was doing
however that correspondingly a court of first instance is was descending the gangplank of the M/V Wilcon 9 and
vested with competence when clearly in the light of the there was no outward indication that called for his arrest.
above decisions the law has not seen fit to do so. The To all appearances, he was like any of the other
proceeding before the Collector of Customs is not final. passengers innocently disembarking from the vessel. It
An appeal lies to the Commissioner of Customs and was only when the informer pointed to him as the carrier
thereafter to the Court of Tax Appeals. It may even reach of the marijuana that he suddenly became suspect and so
this Court through the appropriate petition for review. subject to apprehension.”
The proper ventilation of the legal issues raised is thus
indicated. Certainly a court of first instance is not therein Likewise, in People v. Mengote (210 SCRA 174,
included. It is devoid of jurisdiction. 179-180 [1992]), the Court did not consider “eyes . . .
(Bureau of Customs v. Ogario, 329 SCRA 289, 296- darting from side to side . . . [while] holding . . . [one’s]
298, March 30, 2000, 2nd Div. [Mendoza]) abdomen,” in a crowded street at 11:30 in the morning,
as overt acts and circumstances sufficient to arouse
suspicion and indicative of probable cause. According to
178. Discuss the nature of an in flagrante delicto the Court, “[b]y no stretch of the imagination could it
warrantless arrest. Illustrative case. have been inferred from these acts that an offense had
just been committed, or was actually being committed, or
Held: In the case at bar, the court a quo was at least being attempted in [the arresting officers’]
anchored its judgment of conviction on a finding that the presence.” So also, in People v. Encinada (280 SCRA 72,
warrantless arrest of accused-appellants, and the 86-87 [1997]), the Court ruled that no probable cause is
subsequent search conducted by the peace officers, are gleanable from the act of riding a motorela while holding
valid because accused-appellants were caught in two plastic baby chairs.
flagrante delicto in possession of prohibited drugs. This
brings us to the issue of whether or not the warrantless Then, too, in Malacat v. Court of Appeals (283
arrest, search and seizure in the present case fall within SCRA 159 [1997]), the trial court concluded that
the recognized exceptions to the warrant requirement. petitioner was attempting to commit a crime as he was
“’standing at the corner of Plaza Miranda and Quezon
In People v. Chua Ho San, the Court held that in Boulevard’ with his eyes ‘moving very fast’ and ‘looking at
cases of in flagrante delicto arrests, a peace officer or a every person that come (sic) nearer (sic) to them.’” In
private person may, without a warrant, arrest a person declaring the warrantless arrest therein illegal, the Court
when, in his presence, the person to be arrested has said:
committed, is actually committing, or is attempting to
commit an offense. The arresting office, therefore, must Here, there could have been no valid in flagrante delicto
have personal knowledge of such fact or, as a recent case … arrest preceding the search in light of the lack of
law adverts to, personal knowledge of facts or personal knowledge on the part of Yu, the arresting
circumstances convincingly indicative or constitutive of officer, or an overt physical act, on the part of petitioner,
probable cause. As discussed in People v. Doria, probable indicating that a crime had just been committed, was
cause means an actual belief or reasonable grounds of
being committed or was going to be committed. (Id., at while they were on the side of the road. These
175) circumstances could not have afforded SPO1 Paguidopon
a closer look at accused-appellant Mula, considering that
It went on to state that – the latter was then driving a motorcycle when SPO1
Paguidopon caught a glimpse of him. With respect to
Second, there was nothing in petitioner’s behavior or accused-appellant Molina, SPO1 Paguidopon admitted
conduct which could have reasonably elicited even mere that he had never seen him before the arrest.
suspicion other than that his eyes were “moving very fast”
– an observation which leaves us incredulous since Yu and This belies the claim of SPO1 Pamplona that he knew
his teammates were nowhere near petitioner and it was the name of accused-appellants even before the arrest x
already 6:60 p.m., thus presumably dusk. Petitioner and x x.
his companions were merely standing at the corner and
were not creating any commotion or trouble . . . The aforesaid testimony of SPO1 Pamplona, therefore, is
entirely baseless. SPO1 Pamplona could not have
Third, there was at all no ground, probable or otherwise, learned the name of accused-appellants from SPO1
to believe that petitioner was armed with a deadly Paguidopon because Paguidopon himself, who allegedly
weapon. None was visible to Yu, for as he admitted, the conducted the surveillance, was not even aware of
alleged grenade was “discovered” “inside the front accused-appellants’ name and address prior to the
waistline” of petitioner, and from all indications as to the arrest.
distance between Yu and petitioner, any telltale bulge,
assuming that petitioner was indeed hiding a grenade, Evidently, SPO1 Paguidopon, who acted as informer of
could not have been visible to Yu. (Id., at 178). the arresting officers, more so the arresting officers
themselves, could not have been certain of accused-
Clearly, to constitute a valid in flagrante delicto arrest, appellants’ identity, and were, from all indications,
two requisites must concur: (1) the person to be merely fishing for evidence at the time of the arrest.
arrested must execute an overt act indicating that he
has just committed, is actually committing, or is Compared to People v. Encinada, the arresting officer in
attempting to commit a crime; and (2) such overt act is the said case knew appellant Encinada even before the
done in the presence or within the view of the arresting arrest because of the latter’s illegal gambling activities,
officer (Concurring Opinion of Justice Artemio V. thus, lending at least a semblance of validity on the
Panganiban in People v. Doria, 301 SCRA 668, 720 arrest effected by the peace officers. Nevertheless, the
[1999]). Court declared in said case that the warrantless arrest
and the consequent search were illegal, holding that
In the case at bar, accused-appellants manifested no “[t]he prosecution’s evidence did not show any
outward indication that would justify their arrest. In suspicious behavior when the appellant disembarked
holding a bag on board a trisikad, accused-appellants from the ship or while he rode the motorela. No act or
could not be said to be committing, attempting to fact demonstrating a felonious enterprise could be
commit or have committed a crime. It matters not that ascribed to appellant under such bare circumstances.”
accused-appellant Molina responded “Boss, if possible (People v. Encinada, supra.)
we will settle this” to the request of SPO1 Pamplona to
open the bag. Such response which allegedly reinforced Moreover, it could not be said that accused-appellants
the “suspicion” of the arresting officers that accused- waived their right against unreasonable searches and
appellants were committing a crime, is an equivocal seizure. Implied acquiescence to the search, if there
statement which standing alone will not constitute was any, could not have been more than mere passive
probable cause to effect an in flagrante delicto arrest. conformity given under intimidating or coercive
Note that were it not for SPO1 Marino Paguidopon (who circumstances and is thus considered no consent at all
did not participate in the arrest but merely pointed within the purview of the constitutional guarantee (Id.,
accused-appellants to the arresting officers), accused- at 91; citing Aniag v. Commission on Elections, 237
appellants could not be subject of any suspicion, SCRA 424, 436-437 [1994]).
reasonable or otherwise.
Withal, the Court holds that the arrest of accused-
While SPO1 Paguidopon claimed that he and his appellants does not fall under the exceptions allowed by
informer conducted a surveillance of accused-appellant the rules. Hence, the search conducted on their person
Mula, SPO1 Paguidopon, however, admitted that he only was likewise illegal. Consequently, the marijuana seized
learned Mula’s name and address after the arrest. What by the peace officers could not be admitted as evidence
is more, it is doubtful if SPO1 Paguidopon indeed against accused-appellants, and the Court is thus, left
recognized accused-appellant Mula. It is worthy to note with no choice but to find in favor of accused-appellants.
that, before the arrest, he was able to see Mula in (People v. Molina, 352 SCRA 174, Feb. 19, 2001, En
person only once, pinpointed to him by his informer Banc [Ynares-Santiago])
The SC clarified that the right of privacy is recognized and
The Privacy of Communications and Correspondence enshrined in several provisions of our Constitution. It is
expressly recognized in Section 3(1) of the Bill of Rights.
179. Private respondent Rafael S. Ortanez filed with Other facets of the right to privacy are protected in
the Regional Trial Court of Quezon City a complaint for various provisions of the Bill of Rights, i.e., Secs. 1, 2, 6,
annulment of marriage with damages against petitioner 8, and 17. (Ople v. Torres, G.R. No. 127685, July
Teresita Salcedo-Ortanez, on grounds of lack of marriage 23, 1998 [Puno])
license and/or psychological incapacity of the petitioner.
Among the exhibits offered by private respondent were 135.Identify the zones of privacy recognized and
three (3) cassette tapes of alleged telephone protected in our laws.
conversations between petitioner and unidentified
persons. The trial court issued the assailed order Held: The Civil Code provides that “[e]very person shall
admitting all of the evidence offered by private respect the dignity, personality, privacy and peace of
respondent, including tape recordings of telephone mind of his neighbors and other persons” and punishes
conversations of petitioner with unidentified persons. as actionable torts several acts by a person of meddling
These tape recordings were made and obtained when and prying into the privacy of another. It also holds a
private respondent allowed his friends from the military public officer or employee or any private individual liable
to wire tap his home telephone. Did the trial court act for damages for any violation of the rights and liberties of
properly when it admitted in evidence said tape another person, and recognizes the privacy of letters and
recordings? other private communications. The Revised Penal Code
makes a crime the violation of secrets by an officer, the
Held: Republic Act No. 4200 entitled "An Act to Prohibit revelation of trade and industrial secrets, and trespass to
and Penalize Wire Tapping and Other Related Violations dwelling. Invasion of privacy is an offense in special laws
of the Privacy of Communication, and For Other Purposes" like the Anti-Wiretapping Law (R.A. 4200), the Secrecy of
expressly makes such tape recordings inadmissible in Bank Deposits (R.A. 1405) and the Intellectual Property
evidence. x x x. Code (R.A. 8293). The Rules of Court on privileged
communication likewise recognize the privacy of certain
Clearly, respondent trial court and Court of Appeals failed information (Section 24, Rule 130[c], Revised Rules on
to consider the afore-quoted provisions of the law in Evidence). (Ople v. Torres, G.R. No. 127685, July
admitting in evidence the cassette tapes in question. 23, 1998 [Puno])
Absent a clear showing that both parties to the telephone
conversations allowed the recording of the same, the 180. Discuss why Administrative Order No. 308 (issued
inadmissibility of the subject tapes is mandatory under by the President prescribing for a National ID system for
Rep. Act No. 4200. all citizens to facilitate business transactions with
government agencies engaged in the delivery of basic
Additionally, it should be mentioned that the above- services and social security provisions) should be declared
mentioned Republic Act in Section 2 thereof imposes a unconstitutional.
penalty of imprisonment of not less than six (6) months
and up to six (6) years for violation of said Act. Held: We prescind from the premise that the right to
(Salcedo-Ortanez v. Court of Appeals, 235 SCRA privacy is a fundamental right guaranteed by the
111, Aug. 4, 1994 [Padilla]) Constitution, hence, it is the burden of government to
show that A.O. No. 308 is justified by some compelling
state interest and that it is narrowly drawn. A.O. No. 308
The Right to Privacy is predicated on two considerations: (1) the need to
provide our citizens and foreigners with the facility to
180. Is there a constitutional right to privacy? conveniently transact business with basic service and
social security providers and other government
Held: The essence of privacy is the “right to be let instrumentalities and (2) the need to reduce, if not totally
alone.” In the 1965 case of Griswold v. Connecticut (381 eradicate, fraudulent transactions and misrepresentations
U.S. 479, 14 l. ed. 2D 510 [1965]), the United States by persons seeking basic services. It is debatable
Supreme Court gave more substance to the right of whether these interests are compelling enough to warrant
privacy when it ruled that the right has a constitutional the issuance of A.O. No. 308. But what is not arguable is
foundation. It held that there is a right of privacy which the broadness, the vagueness, the overbreadth of A.O.
can be found within the penumbras of the First, Third, No. 308 which if implemented will put our people’s right
Fourth, Fifth and Ninth Amendments. In the 1968 case to privacy in clear and present danger.
of Morfe v. Mutuc (22 SCRA 424, 444-445), we adopted
the Griswold ruling that there is a constitutional right to The heart of A.O. No. 308 lies in its Section 4 which
privacy. provides for a Population Reference Number (PRN) as a
“common reference number to establish a linkage among with other data on the subject. Once extracted, the
concerned agencies” through the use of “Biometrics information is putty in the hands of any person. The end
Technology” and “computer application designs.” of privacy begins.

It is noteworthy that A.O. No. 308 does not state what [T]he Court will not be true to its role as the ultimate
specific biological characteristics and what particular guardian of the people’s liberty if it would not immediately
biometrics technology shall be used to identify people smother the sparks that endanger their rights but would
who will seek its coverage. Considering the banquet of rather wait for the fire that could consume them.
options available to the implementors of A.O. No. 308, the
fear that it threatens the right to privacy of our people is [A]nd we now hold that when the integrity of a
not groundless. fundamental right is at stake, this Court will give the
challenged law, administrative order, rule or regulation a
A.O. No. 308 should also raise our antennas for a further stricter scrutiny. It will not do for the authorities to invoke
look will show that it does not state whether encoding of the presumption of regularity in the performance of
data is limited to biological information alone for official duties. Nor is it enough for the authorities to prove
identification purposes. X x x. Clearly, the indefiniteness that their act is not irrational for a basic right can be
of A.O. No. 308 can give the government the roving diminished, if not defeated, even when the government
authority to store and retrieve information for a purpose does not act irrationally. They must satisfactorily show
other than the identification of the individual through his the presence of compelling state interest and that the law,
PRN. rule, or regulation is narrowly drawn to preclude abuses.
This approach is demanded by the 1987 Constitution
The potential for misuse of the data to be gathered under whose entire matrix is designed to protect human rights
A.O. No. 308 cannot be underplayed x x x. The more and to prevent authoritarianism. In case of doubt, the
frequent the use of the PRN, the better the chance of least we can do is to lean towards the stance that will not
building a huge and formidable information base through put in danger the rights protected by the Constitution.
the electronic linkage of the files. The data may be
gathered for gainful and useful government purposes; but The right to privacy is one of the most threatened rights
the existence of this vast reservoir of personal information of man living in a mass society. The threats emanate
constitutes a covert invitation to misuse, a temptation from various sources – governments, journalists,
that may be too great for some of our authorities to resist. employers, social scientists, etc. In the case at bar, the
threat comes from the executive branch of government
It is plain and we hold that A.O. No. 308 falls short of which by issuing A.O. No. 308 pressures the people to
assuring that personal information which will be gathered surrender their privacy by giving information about
about our people will only be processed for unequivocally themselves on the pretext that it will facilitate delivery of
specified purposes. The lack of proper safeguards in this basic services. Given the record-keeping power of the
regard of A.O. No. 308 may interfere with the individual’s computer, only the indifferent will fail to perceive the
liberty of abode and travel by enabling authorities to track danger that A.O. No. 308 gives the government the power
down his movement; it may also enable unscrupulous to compile a devastating dossier against unsuspecting
persons to access confidential information and circumvent citizens. X x x [W]e close with the statement that the
the right against self-incrimination; it may pave the way right to privacy was not engraved in our Constitution for
for “fishing expeditions” by government authorities and flattery. (Ople v. Torres, G.R. No. 127685, July 23,
evade the right against unreasonable searches and 1998 [Puno])
seizures. The possibilities of abuse and misuse of the
PRN, biometrics and computer technology are 181. Should in camera inspection of bank accounts be
accentuated when we consider that the individual lacks allowed? If in the affirmative, under what circumstances
control over what can be read or placed on his ID, much should it be allowed?
less verify the correctness of the data encoded. They
threaten the very abuses that the Bill of Rights seeks to Held: The issue is whether petitioner may be cited for
prevent. indirect contempt for her failure to produce the
documents requested by the Ombudsman. And whether
The ability of a sophisticated data center to generate a the order of the Ombudsman to have an in camera
comprehensive cradle-to-grave dossier on an individual inspection of the questioned account is allowed as an
and transmit it over a national network is one of the most exception to the law on secrecy of bank deposits (R.A.
graphic threats of the computer revolution. The computer No. 1405).
is capable of producing a comprehensive dossier on
individuals out of information given at different times and An examination of the secrecy of bank deposits law (R.A.
for varied purposes. X x x. Retrieval of stored data is No. 1405) would reveal the following exceptions:
simple. When information of a privileged character finds
its way into the computer, it can be extracted together 1) Where the depositor consents in writing;
2) Impeachment cases;
3) By court order in bribery or dereliction of duty Zones of privacy are recognized and protected in our
cases against public officials; laws. The Civil Code provides that “[e]very person shall
4) Deposit is subject of litigation; respect the dignity, personality, privacy and peace of
5) Sec. 8, R.A. No. 3019, in cases of unexplained mind of his neighbors and other persons” and punishes
wealth as held in the case of PNB v. Gancayco (122 Phil. as actionable torts several acts for meddling and prying
503, 508 [1965]). into the privacy of another. It also holds public officer or
employee or any private individual liable for damages for
The order of the Ombudsman to produce for in camera any violation of the rights and liberties of another person,
inspection the subject accounts with the Union Bank of and recognizes the privacy of letters and other private
the Philippines, Julia Vargas Branch, is based on a communications. The Revised Penal Code makes a crime
pending investigation at the Office of the Ombudsman of the violation of secrets by an officer, revelation of trade
against Amado Lagdameo, et. al. for violation of R.A. No. and industrial secrets, and trespass to dwelling. Invasion
3019, Sec. 3 (e) and (g) relative to the Joint Venture of privacy is an offense in special laws like the anti-
Agreement between the Public Estates Authority and Wiretapping Law, the Secrecy of Bank Deposits Act, and
AMARI. the Intellectual Property Code. (Lourdes T. Marquez v.
Hon. Aniano A. Desierto, G.R. No. 135882, June 27,
We rule that before an in camera inspection may be 2001, En Banc [Pardo])
allowed, there must be a pending case before a court of
competent jurisdiction. Further, the account must be
clearly identified, the inspection limited to the subject Freedom of Expression
matter of the pending case before the court of competent
jurisdiction. The bank personnel and the account holder 182. Distinguish “content-based restrictions” on free
must be notified to be present during the inspection, and speech from “content-neutral restrictions,” and give
such inspection may cover only the account identified in example of each.
the pending case.
Held: Content-based restrictions are imposed because of
In Union Bank of the Philippines v. Court of Appeals, we the content of the speech and are, therefore, subject to
held that “Section 2 of the Law on Secrecy of Bank the clear-and-present danger test. For example, a rule
Deposits, as amended, declares bank deposits to be such as that involved in Sanidad v. Comelec (181 SCRA
‘absolutely confidential’ except: 529 [1990]), prohibiting columnists, commentators, and
announcers from campaigning either for or against an
1) In an examination made in the course of a issue in a plebiscite must have compelling reason to
special or general examination of a bank that is support it, or it will not pass muster under strict scrutiny.
specifically authorized by the Monetary Board after being These restrictions are censorial and therefore they bear a
satisfied that there is reasonable ground to believe that a heavy presumption of constitutional invalidity. In addition,
bank fraud or serious irregularity has been or is being they will be tested for possible overbreadth and
committed and that it is necessary to look into the deposit vagueness.
to establish such fraud or irregularity,
2) In an examination made by an independent Content-neutral restrictions, on the other hand, like Sec.
auditor hired by the bank to conduct its regular audit 11(b) of R.A. No. 6646, which prohibits the sale or
provided that the examination is for audit purposes only donation of print space and air time to political candidates
and the results thereof shall be for the exclusive use of during the campaign period, are not concerned with the
the bank, content of the speech. These regulations need only a
3) Upon written permission of the depositor, substantial governmental interest to support them. A
4) In cases of impeachment, deferential standard of review will suffice to test their
5) Upon order of a competent court in cases of validity. The clear-and-present danger rule is
bribery or dereliction of duty of public officials, or inappropriate as a test for determining the constitutional
6) In cases where the money deposited or validity of laws, like Sec. 11(b) of R.A. No. 6646, which
invested is the subject matter of the litigation”. are not concerned with the content of political ads but
only with their incidents. To apply the clear-and-present
In the case at bar, there is yet no pending litigation before danger test to such regulatory measures would be like
any court of competent authority. What is existing is an using a sledgehammer to drive a nail when a regular
investigation by the Office of the Ombudsman. In short, hammer is all that is needed.
what the Office of the Ombudsman would wish to do is to
fish for additional evidence to formally charge Amado The test for this difference in the level of justification for
Lagdameo, et. al., with the Sandiganbayan. Clearly, there the restriction of speech is that content-based restrictions
was no pending case in court which would warrant the distort public debate, have improper motivation, and are
opening of the bank account for inspection. usually imposed because of fear of how people will react
to a particular speech. No such reasons underlie content- hand, there are other valid and reasonable ways and
neutral regulations, like regulation of time, place and means to achieve the Comelec end of avoiding or
manner of holding public assemblies under B.P. Blg. 880, minimizing disorder and confusion that may be brought
the Public Assembly Act of 1985. (Osmena v. about by exit surveys.
COMELEC, 288 SCRA 447, March 31, 1998
[Mendoza]) With foregoing premises, it is concluded that the interest
of the state in reducing disruption is outweighed by the
183. Does the conduct of exit poll by ABS CBN present drastic abridgment of the constitutionally guaranteed
a clear and present danger of destroying the credibility rights of the media and the electorate. Quite the
and integrity of the electoral process as it has the contrary, instead of disrupting elections, exit polls –
tendency to sow confusion considering the randomness properly conducted and publicized – can be vital tools for
of selecting interviewees, which further makes the exit the holding of honest, orderly, peaceful and credible
poll highly unreliable, to justify the promulgation of a elections; and for the elimination of election-fixing, fraud
Comelec resolution prohibiting the same? and other electoral ills. (ABS-CBN Broadcasting
Corporation v. COMELEC, G.R. No. 133486, Jan. 28,
Held: Such arguments are purely speculative and clearly 2000, En Banc [Panganiban])
untenable. First, by the very nature of a survey, the
interviewees or participants are selected at random, so 184. Section 5.4 of R.A. No. 9006 (Fair Election Act)
that the results will as much as possible be representative which provides: “Surveys affecting national candidates
or reflective of the general sentiment or view of the shall not be published fifteen (15) days before an election
community or group polled. Second, the survey result is and surveys affecting local candidates shall not be
not meant to replace or be at par with the official Comelec published seven (7) days before an election.” The Social
count. It consists merely of the opinion of the polling Weather Stations, Inc. (SWS), a private non-stock, non-
group as to who the electorate in general has probably profit social research institution conducting surveys in
voted for, based on the limited data gathered from polled various fields; and Kamahalan Publishing Corporation,
individuals. Finally, not at stake are the credibility and publisher of the Manila Standard, a newspaper of general
the integrity of the elections, which are exercises that are circulation, which features newsworthy items of
separate and independent from the exit polls. The information including election surveys, challenged the
holding and the reporting of the results of exit polls constitutionality of aforesaid provision as it constitutes a
cannot undermine those of the elections, since the former prior restraint on the exercise of freedom of speech
is only part of the latter. If at all, the outcome of one can without any clear and present danger to justify such
only be indicative of the other. restraint. Should the challenge be sustained?

The COMELEC’s concern with the possible Held: For reason hereunder given, we hold that Section
noncommunicative effect of exit polls – disorder and 5.4 of R.A. No. 9006 constitutes an unconstitutional
confusion in the voting centers – does not justify a total abridgment of freedom of speech, expression, and the
ban on them. Undoubtedly, the assailed Comelec press.
Resolution is too broad, since its application is without
qualification as to whether the polling is disruptive or not. To be sure, Section 5.4 lays a prior restraint on freedom
There is no showing, however, that exit polls or the of speech, expression, and the press by prohibiting the
means to interview voters cause chaos in voting centers. publication of election survey results affecting candidates
Neither has any evidence been presented proving that the within the prescribed periods of fifteen (15) days
presence of exit poll reporters near an election precinct immediately preceding a national election and seven (7)
tends to create disorder or confuse the voters. days before a local election. Because of the preferred
status of the constitutional rights of speech, expression,
Moreover, the prohibition incidentally prevents the and the press, such a measure is vitiated by a weighty
collection of exit poll data and their use for any purpose. presumption of invalidity. Indeed, “any system of prior
The valuable information and ideas that could be derived restraints of expression comes to this Court bearing a
from them, based on the voters’ answers to the survey heavy presumption against its constitutional validity x x x.
questions will forever remain unknown and unexplored. The Government ‘thus carries a heavy burden of showing
Unless the ban is restrained, candidates, researchers, justification for the enforcement of such restraint.’” There
social scientists and the electorate in general would be is thus a reversal of the normal presumption of validity
deprived of studies on the impact of current events and that inheres in every legislation.
of election-day and other factors on voters’ choices.
Nor may it be argued that because of Art. IX-C, Sec. 4 of
The absolute ban imposed by the Comelec cannot, the Constitution, which gives the Comelec supervisory
therefore, be justified. It does not leave open any power to regulate the enjoyment or utilization of franchise
alternative channel of communication to gather the type for the operation of media of communication, no
of information obtained through exit polling. On the other presumption of invalidity attaches to a measure like Sec.
5.4. For as we have pointed out in sustaining the ban on asserted governmental interest makes such interest “not
media political advertisements, the grant of power to the unrelated to the suppression of free expression.” By
Comelec under Art. IX-C, Sec. 4 is limited to ensuring prohibiting the publication of election survey results
“equal opportunity, time, space, and the right to reply” as because of the possibility that such publication might
well as uniform and reasonable rates of charges for the undermine the integrity of the election, Sec. 5.4 actually
use of such media facilities for “public information suppresses a whole class of expression, while allowing the
campaigns and forums among candidates.” expression of opinion concerning the same subject matter
by newspaper columnists, radio and TV commentators,
Xxx armchair theorists, and other opinion makers. In effect,
Sec. 5.4 shows a bias for a particular subject matter, if
Nor can the ban on election surveys be justified on the not viewpoint, by preferring personal opinion to statistical
ground that there are other countries x x x which similarly results. The constitutional guarantee of freedom of
impose restrictions on the publication of election surveys. expression means that “the government has no power to
At best this survey is inconclusive. It is noteworthy that restrict expression because of its message, its ideas, its
in the United States no restriction on the publication of subject matter, or its contents.” The inhibition of speech
election survey results exists. It cannot be argued that should be upheld only if the expression falls within one of
this is because the United States is a mature democracy. the few unprotected categories dealt with in Chaplinsky
Neither are there laws imposing an embargo on survey v. New Hampshire (315 U.S. 568, 571-572, 86 L. Ed.
results, even for a limited period, in other countries. X x 1031, 1035 [1942]), thus:
x.
There are certain well-defined and narrowly limited
What test should then be employed to determine the classes of speech, the prevention and punishment of
constitutional validity of Section 5.4? The United States which have never been thought to raise any Constitutional
Supreme Court x x x held in United States v. O’ Brien: problem. These include the lewd and obscene, the
profane, the libelous, and the insulting or ‘fighting’ words
[A] government regulation is sufficiently justified (1) if it – those which by their very utterance inflict injury or tend
is within the constitutional power of the government; (2) to incite an immediate breach of the peace. [S]uch
if it furthers an important or substantial governmental utterances are no essential part of any exposition of ideas,
interest; (3) if the governmental interest is unrelated to and are of such slight social value as a step to truth that
the suppression of free expression; and (4) if the any benefit that may be derived from them is clearly
incidental restriction on alleged First Amendment outweighed by the social interest in order and morality.
freedoms (of speech, expression and press) is no greater
than is essential to the furtherance of that interest (391 Nor is there justification for the prior restraint which Sec.
U.S. 367, 20 L. Ed. 2d 692, 680 [1968] [bracketed 5.4 lays on protected speech. In Near v. Minnesota (283
numbers added]). U.S. 697, 715-716, 75 l. Ed. 1357, 1367 [1931]), it was
held:
This is so far the most influential test for distinguishing
content-based from content-neutral regulations and is [T]he protection even as to previous restraint is not
said to have “become canonical in the review of such absolutely unlimited. But the limitation has been
laws.” It is noteworthy that the O’ Brien test has been recognized only in exceptional cases x x x. No one would
applied by this Court in at least two cases (Adiong v. question but that a government might prevent actual
Comelec, 207 SCRA 712 [1992]; Osmena v. Comelec, obstruction to its recruiting service or the publication of
supra.). the sailing dates of transports or the number and location
of troops. On similar grounds, the primary requirements
Under this test, even if a law furthers an important or of decency may be enforced against obscene publications.
substantial governmental interest, it should be invalidated The security of the community life may be protected
if such governmental interest is “not unrelated to the against incitements to acts of violence and the overthrow
suppression of free expression.” Moreover, even if the by force of orderly government x x x.
purpose is unrelated to the suppression of free speech,
the law should nevertheless be invalidated if the Thus, x x x the prohibition imposed by Sec. 5.4 cannot be
restriction on freedom of expression is greater than is justified on the ground that it is only for a limited period
necessary to achieve the governmental purpose in and is only incidental. The prohibition may be for a
question. limited time, but the curtailment of the right of expression
is direct, absolute, and substantial. It constitutes a total
Our inquiry should accordingly focus on these two suppression of a category of speech and is not made less
considerations as applied to Sec. 5.4. so because it is only for a period of fifteen (15) days
immediately before a national election and seven (7) days
First. Sec. 5.4 fails to meet criterion (3) of the O’ Brien immediately before a local election.
test because the causal connection of expression to the
This sufficiently distinguishes Sec. 5.4 from R.A. No. 6646, interest sought to be promoted can be achieved by means
Sec. 11(b), which this Court found to be valid in National other than the suppression of freedom of expression.
Press Club v. Comelec (supra.), and Osmena v. Comelec (Social Weather Stations, Inc., v. COMELEC, G.R.
(supra.). For the ban imposed by R.A. No. 6646, Sec. No. 147571, May 5, 2001, En Banc [Mendoza])
11(b) is not only authorized by a specific constitutional
provision (Art. IX-C, Sec. 4), but it also provided an 185. Discuss the "doctrine of fair comment" as a valid
alternative so that, as this Court pointed out in Osmena, defense in an action for libel or slander.
there was actually no ban but only a substitution of media
advertisements by the Comelec space, and Comelec hour. Held: Fair commentaries on matters of public interest
are privileged and constitute a valid defense in an action
Second. Even if the governmental interest sought to be for libel or slander. The doctrine of fair comment means
promoted is unrelated to the suppression of speech and that while in general every discreditable imputation
the resulting restriction of free expression is only publicly made is deemed false, because every man is
incidental, Sec. 5.4 nonetheless fails to meet criterion (4) presumed innocent until his guilt is judicially proved, and
of the O’ Brien test, namely, that the restriction be not every false imputation is deemed malicious, nevertheless,
greater than is necessary to further the governmental when the discreditable imputation is directed against a
interest. As already stated, Sec. 5.4. aims at the public person in his public capacity, it is not necessarily
prevention of last-minute pressure on voters, the creation actionable. In order that such discreditable imputation to
of bandwagon effect, “junking” of weak or “losing” a public official may be actionable, it must either be a
candidates, and resort to the form of election cheating false allegation of fact or a comment based on a false
called “dagdag-bawas.” Praiseworthy as these aims of supposition. If the comment is an expression of opinion,
the regulation might be, they cannot be attained at the based on established facts, then it is immaterial that the
sacrifice of the fundamental right of expression, when opinion happens to be mistaken, as long as it might
such aim can be more narrowly pursued by punishing reasonably be inferred from the facts. (Borjal v. CA,
unlawful acts, rather than speech because of 301 SCRA 1, Jan. 14, 1999, 2nd Div. [Bellosillo])
apprehension that such speech creates the danger of
such evils. Thus, under the Administrative Code of 1987 186. What is the “raison d’etre” for the New York
(Bk. V, Tit. I, Subtit. C, Ch 1, Sec. 3[1]), the Comelec is Times v. Sullivan (376 US 254) holding that honest
given the power: criticisms on the conduct of public officials and public
figures are insulated from libel judgments?
To stop any illegal activity, or confiscate, tear down, and
stop any unlawful, libelous, misleading or false election Held: The guarantees of freedom of speech and press
propaganda, after due notice and hearing. prohibit a public official or public figure from recovering
damages for a defamatory falsehood relating to his official
This is surely a less restrictive means than the prohibition conduct unless he proves that the statement was made
contained in Sec. 5.4. Pursuant to this power of the with actual malice, i.e., with knowledge that it was false
Comelec, it can confiscate bogus survey results calculated or with reckless disregard of whether it was false or not.
to mislead voters. Candidates can have their own surveys
conducted. No right of reply can be invoked by others. The raison d’etre for the New York Times doctrine was
No principle of equality is involved. It is a free market to that to require critics of official conduct to guarantee the
which each candidate brings his ideas. As for the purpose truth of all their factual assertions on pain of libel
of the law to prevent bandwagon effects, it is doubtful judgments would lead to self-censorship, since would-be
whether the Government can deal with this natural- critics would be deterred from voicing out their criticisms
enough tendency of some voters. Some voters want to even if such were believed to be true, or were in fact true,
be identified with the “winners.” Some are susceptible to because of doubt whether it could be proved or because
the herd mentality. Can these be legitimately prohibited of fear of the expense of having to prove it . (Borjal v.
by suppressing the publication of survey results which are CA, 301 SCRA 1, Jan. 14, 1999, 2nd Div. [Bellosillo])
a form of expression? It has been held that “[mere]
legislative preferences or beliefs respecting matters of 187. Who is a “public figure,” and therefore subject to
public convenience may well support regulation directed public comment?
at other personal activities, but be insufficient to justify
such as diminishes the exercise of rights so vital to the Held: [W]e deem private respondent a public figure
maintenance of democratic institutions.” within the purview of the New York Times ruling. At any
rate, we have also defined “public figure” in Ayers
To summarize then, we hold that Sec. 5.4. is invalid Production Pty., Ltd. v. Capulong (G.R. Nos. 82380 and
because (1) it imposes a prior restraint on the freedom of 82398, 29 April 1988, 160 SCRA 861) as –
expression, (2) it is a direct and total suppression of a
category of expression even though such suppression is X x x a person who, by his accomplishments, fame, mode
only for a limited period, and (3) the governmental of living, or by adopting a profession or calling which gives
the public a legitimate interest in his doings, his affairs inconsistent with any requirement placing on him the
and his character, has become a ‘public personage.’ He burden of proving that he acted with good motives and
is, in other words, a celebrity. Obviously, to be included for justifiable ends.
in this category are those who have achieved some
degree of reputation by appearing before the public, as in For that matter, even if the defamatory statement is false,
the case of an actor, a professional baseball player, a no liability can attach if it relates to official conduct, unless
pugilist, or any other entertainer. The list is, however, the public official concerned proves that the statement
broader than this. It includes public officers, famous was made with actual malice – that is, with knowledge
inventors and explorers, war heroes and even ordinary that it was false or with reckless disregard of whether it
soldiers, infant prodigy, and no less a personage than the was false or not. This is the gist of the ruling in the
Great Exalted Ruler of the lodge. It includes, in short, landmark case of New York Times v. Sullivan (376 U.S.
anyone who has arrived at a position where the public 254, 11 L. Ed. 2d [1964]. For a fascinating account of
attention is focused upon him as a person. this case, see Anthony Lewis, Make No Law – The Sullivan
Case and the First Amendment [1991]), which this Court
The FNCLT (First National Conference on Land has cited with approval in several of its own decisions
Transportation) was an undertaking infused with public (Lopez v. Court of Appeals, 145 Phil. 219 [1970], others
interest. It was promoted as a joint project of the omitted). This is the rule of “actual malice.” In this case,
government and the private sector, and organized by top the prosecution failed to prove not only that the charges
government officials and prominent businessmen. For made by petitioner were false but also that petitioner
this reason, it attracted media mileage and drew public made them with knowledge of their falsity or with reckless
attention not only to the conference itself but to the disregard of whether they were false or not.
personalities behind as well. As its Executive Director and
spokesman, private respondent consequently assumed A rule placing on the accused the burden of showing the
the status of a public figure. truth of allegations of official misconduct and/or good
motives and justifiable ends for making such allegations
But even assuming ex-gratia argumenti that private would not only be contrary to Art. 361 of the Revised
respondent, despite the position he occupied in the Penal Code. It would, above all, infringe on the
FNCLT, would not qualify as a public figure, it does not constitutionally guaranteed freedom of expression. Such
necessarily follow that he could not validly be the subject a rule would deter citizens from performing their duties
of a public comment even if he was not a public official or as members of a self-governing community. Without free
at least a public figure, for he could be, as long as he was speech and assembly, discussions of our most abiding
involved in a public issue. If a matter is a subject of public concerns as a nation would be stifled. As Justice Brandies
or general interest, it cannot suddenly become less so has said, “public discussion is a political duty” and the
merely because a private individual is involved or because “greatest menace to freedom is an inert people.”
in some sense the individual did not voluntarily choose to (Whitney v. California, 247 U.S. 357, 375, 71 L. Ed. 1095,
become involved. The public’s primary interest is in the 1105 [1927] [concurring]) (Vasquez v. Court of
event; the public focus is on the conduct of the participant Appeals, 314 SCRA 460, Sept. 15, 1999, En Banc
and the content, effect and significance of the conduct, [Mendoza])
not the participant’s prior anonymity or notoriety.
(Borjal v. CA, 301 SCRA 1, Jan. 14, 1999, 2nd Div.
[Bellosillo])
189. The Office of the Mayor of Las Pinas refused to
188. The question for determination in this case is the issue permit to petitioners to hold rally a rally in front of
liability for libel of a citizen who denounces a barangay the Justice Hall of Las Pinas on the ground that it was
official for misconduct in office. The Regional Trial Court prohibited under Supreme Court En Banc Resolution
of Manila x x x found petitioner guilty x x x on the ground dated July 7,1998 in A.M. No. 98-7-02-SC, entitled, "Re:
that petitioner failed to prove the truth of the charges and Guidelines on the Conduct of Demonstrations, Pickets,
that he was “motivated by vengeance in uttering the Rallies and Other Similar Gatherings in the Vicinity of the
defamatory statement.” Supreme Court and All Other Courts." Petitioners thus
initiated the instant proceedings. They submit that the
Held: The decision appealed from should be reversed. Supreme Court gravely abused its discretion and/or acted
without or in excess of jurisdiction in promulgating those
In denouncing the barangay chairman in this case, guidelines.
petitioner and the other residents of the Tondo Foreshore
Area were not only acting in their self-interest but Held: We shall first dwell on the critical argument made
engaging in the performance of a civic duty to see to it by petitioners that the rules constitute an abridgment of
that public duty is discharged faithfully and well by those the people's aggregate rights of free speech, free
on whom such duty is incumbent. The recognition of this expression, peaceful assembly and petitioning
right and duty of every citizen in a democracy is government for redress of grievances citing Sec. 4, Article
III of the 1987 Constitution that "no law shall be passed within the context, in other words, of viable independent
abridging" them. institutions for delivery of justice which are accepted by
the general community. x x x" (In Re: Emil P. Jurado,
It is true that the safeguarding of the people's freedom of 243 SCRA 299, 323-324 [1995])
expression to the end that individuals may speak as they
think on matters vital to them and that falsehoods may It is sadly observed that judicial independence and the
be exposed through the processes of education and orderly administration of justice have been threatened not
discussion, is essential to free government. But freedom only by contemptuous acts inside, but also by irascible
of speech and expression despite its indispensability has demonstrations outside, the courthouses. They wittingly
its limitations. It has never been understood as the or unwittingly, spoil the ideal of sober, non-partisan
absolute right to speak whenever, however, and proceedings before a cold and neutral judge. Even in the
wherever one pleases, for the manner, place, and time of United States, a prohibition against picketing and
public discussion can be constitutionally controlled . [T]he demonstrating in or near courthouses, has been ruled as
better policy is not liberty untamed but liberty regulated valid and constitutional notwithstanding its limiting effect
by law where every freedom is exercised in accordance on the exercise by the public of their liberties. X x x
with law and with due regard for the rights of others .
The administration of justice must not only be fair but
Conventional wisdom tells us that the realities of life in a must also appear to be fair and it is the duty of this Court
complex society preclude an absolutist interpretation of to eliminate everything that will diminish if not destroy
freedom of expression where it does not involve pure this judicial desideratum. To be sure, there will be
speech but speech plus physical actions like picketing. grievances against our justice system for there can be no
There are other significant societal values that must be perfect system of justice but these grievances must be
accommodated and when they clash, they must all be ventilated through appropriate petitions, motions or other
weighed with the promotion of the general welfare of the pleadings. Such a mode is in keeping with the respect
people as the ultimate objective. In balancing these due to the courts as vessels of justice and is necessary if
values, this Court has accorded freedom of expression a judges are to dispose their business in a fair fashion. It
preferred position in light of its more comparative is the traditional conviction of every civilized society that
importance. Hence, our rulings now musty in years hold courts must be insulated from every extraneous influence
that only the narrowest time, place and manner in their decisions. The facts of a case should be
regulations that are specifically tailored to serve an determined upon evidence produced in court, and should
important governmental interest may justify the be uninfluenced by bias, prejudice or sympathies. (In Re:
application of the balancing of interests test in derogation Petition to Annul En Banc Resolution A.M. 98-7-02-
of the people's right of free speech and expression. SC - Ricardo C. Valmonte and Union of Lawyers and
Where said regulations do not aim particularly at the evils Advocates for Transparency in Government
within the allowable areas of state control but, on the [ULAT], G.R. No. 134621, Sept. 29, 1998)
contrary, sweep within their ambit other activities as to
operate as an overhanging threat to free discussion, or 190. Did the Supreme Court commit an act of judicial
where upon their face they are so vague, indefinite, or legislation in promulgating En Banc Resolution A.M. 98-7-
inexact as to permit punishment of the fair use of the right 02-SC, entitled, "Re: Guidelines on the Conduct of
of free speech, such regulations are void. Demonstrations, Pickets, Rallies and Other Similar
Gatherings in the Vicinity of the Supreme Court and All
Prescinding from this premise, the Court reiterates that Other Courts?"
judicial independence and the fair and orderly
administration of justice constitute paramount Held: Petitioners also claim that this Court committed an
governmental interests that can justify the regulation of act of judicial legislation in promulgating the assailed
the public's right of free speech and peaceful assembly in resolution. They charge that this Court amended
the vicinity of courthouses. In the case of In Re: Emil P. provisions of Batas Pambansa (B.P.) Blg. 880, otherwise
Jurado, the Court pronounced in no uncertain terms that: known as "the Public Assembly Act," by converting the
sidewalks and streets within a radius of two hundred
"x x x freedom of expression needs on occasion to be (200) meters from every courthouse from a public forum
adjusted to and accommodated with the requirements of place into a "no rally" zone. Thus, they accuse this Court
equally important public interests. One of these of x x x violating the principle of separation of powers.
fundamental public interests is the maintenance of the
integrity and orderly functioning of the administration of We reject these low watts arguments. Public places
justice. There is no antinomy between free expression historically associated with the free exercise of expressive
and the integrity of the system of administering justice. activities, such as streets, sidewalks, and parks, are
For the protection and maintenance of freedom of considered, without more, to be public fora. In other
expression itself can be secured only within the context words, it is not any law that can imbue such places with
of a functioning and orderly system of dispensing justice, the public nature inherent in them. But even in such
public fora, it is settled jurisprudence that the government media, particularly television and its role in society, and
may restrict speech plus activities and enforce reasonable of the impact of new technologies on law.
time, place, and manner regulations as long as the
restrictions are content-neutral, are narrowly tailored to The records of the Constitutional Commission are bereft
serve a significant governmental interest, and leave open of discussion regarding the subject of cameras in the
ample alternative channels of communication. courtroom. Similarly, Philippine courts have not had the
opportunity to rule on the question squarely.
Contrary therefore to petitioners’ impression, B.P. Blg.
880 did not establish streets and sidewalks, among other While we take notice of the September 1990 report of the
places, as public fora. A close look at the law will reveal United States Judicial Conference Ad Hoc Committee on
that it in fact prescribes reasonable time, place, and Cameras in the Courtroom, still the current rule obtaining
manner regulations. Thus, it requires a written permit for in the Federal Courts of the United States prohibits the
the holding of public assemblies in public places subject, presence of television cameras in criminal trials. Rule 53
even, to the right of the mayor to modify the place and of the Federal Rules of Criminal Procedure forbids the
time of the public assembly, to impose a rerouting of the taking of photographs during the progress of judicial
parade or street march, to limit the volume of loud proceedings or radio broadcasting of such proceedings
speakers or sound system and to prescribe other from the courtroom. A trial of any kind or in any court is
appropriate restrictions on the conduct of the public a matter of serious importance to all concerned and
assembly. should not be treated as a means of entertainment. To
so treat it deprives the court of the dignity which pertains
The existence of B.P. Blg. 880, however, does not to it and departs from the orderly and serious quest for
preclude this Court from promulgating rules regulating truth for which our judicial proceedings are formulated.
conduct of demonstrations in the vicinity of courts to
assure our people of an impartial and orderly Courts do not discriminate against radio and television
administration of justice as mandated by the Constitution. media by forbidding the broadcasting or televising of a
To insulate the judiciary from mob pressure, friendly or trial while permitting the newspaper reporter access to
otherwise, and isolate it from public hysteria, this Court the courtroom, since a television or news reporter has the
merely moved away the situs of mass actions within a same privilege, as the news reporter is not permitted to
200-meter radius from every courthouse. In fine, B.P. bring his typewriter or printing press into the courtroom.
Blg. 880 imposes general restrictions to the time, place
and manner of conducting concerted actions. On the In Estes v. Texas (381 U.S. 532), the United States
other hand, the resolution of this Court regulating Supreme Court held that television coverage of judicial
demonstrations adds specific restrictions as they involve proceedings involves an inherent denial of due process
judicial independence and the orderly administration of rights of a criminal defendant. Voting 5-4, the Court
justice. There is thus no discrepancy between the two through Mr. Justice Clark, identified four (4) areas of
sets of regulatory measures. Simply put, B.P. Blg. 880 potential prejudice which might arise from the impact of
and the assailed resolution complement each other. We the cameras on the jury, witnesses, the trial judge and
so hold following the rule in legal hermeneutics that an the defendant. The decision in part pertinently stated:
apparent conflict between a court rule and a statutory
provision should be harmonized and both should be given "Experience likewise has established the prejudicial effect
effect if possible. (In Re: Petition to Annul En Banc of telecasting on witnesses. Witnesses might be
Resolution A.M. 98-7-02-SC - Ricardo C. Valmonte frightened, play to the camera, or become nervous. They
and Union of Lawyers and Advocates for are subject to extraordinary out-of-court influences which
Transparency in Government [ULAT], G.R. No. might affect their testimony. Also, telecasting not only
134621, Sept. 29, 1998) increases the trial judge's responsibility to avoid actual
prejudice to the defendant; it may as well affect his own
191. Should live media coverage of court proceedings performance. Judges are human beings also and are
be allowed? subject to the same psychological reactions as laymen.
For the defendant, telecasting is a form of mental
Held: The propriety of granting or denying permission to harassment and subjects him to excessive public
the media to broadcast, record, or photograph court exposure and distracts him from the effective
proceedings involves weighing the constitutional presentation of his defense.
guarantees of freedom of the press, the right of the public
to information and the right to public trial, on the one "The television camera is a powerful weapon which
hand, and on the other hand, the due process rights of intentionally or inadvertently can destroy an accused and
the defendant and the inherent and constitutional power his case in the eyes of the public."
of the courts to control their proceedings in order to
permit the fair and impartial administration of justice. Representatives of the press have no special standing to
Collaterally, it also raises issues on the nature of the apply for a writ of mandate to compel a court to permit
them to attend a trial, since within the courtroom a an unprejudiced mind, unbridled by running emotions or
reporter's constitutional rights are no greater than those passions.
of any other member of the public. Massive intrusion of
representatives of the news media into the trial itself can Due process guarantees the accused a presumption of
so alter or destroy the constitutionally necessary judicial innocence until the contrary is proved in a trial that is not
atmosphere and decorum that the requirements of lifted above its individual settings nor made an object of
impartiality imposed by due process of law are denied the public’s attention and where the conclusions reached are
defendant and a defendant in a criminal proceeding induced not by any outside force or influence but only by
should not be forced to run a gauntlet of reporters and evidence and argument given in open court, where fitting
photographers each time he enters or leaves the dignity and calm ambiance is demanded.
courtroom.
Witnesses and judges may very well be men and women
Considering the prejudice it poses to the defendant's right of fortitude, able to thrive in hardy climate, with every
to due process as well as to the fair and orderly reason to presume firmness of mind and resolute
administration of justice, and considering further that the endurance, but it must also be conceded that “television
freedom of the press and the right of the people to can work profound changes in the behavior of the people
information may be served and satisfied by less it focuses on.” Even while it may be difficult to quantify
distracting, degrading and prejudicial means, live radio the influence, or pressure that media can bring to bear on
and television coverage of court proceedings shall not be them directly and through the shaping of public opinion,
allowed. Video footages of court hearings for news it is a fact, nonetheless, that, indeed, it does so in so many
purposes shall be restricted and limited to shots of the ways and in varying degrees. The conscious or
courtroom, the judicial officers, the parties and their unconscious effect that such a coverage may have on the
counsel taken prior to the commencement of official testimony of witnesses and the decision of judges cannot
proceedings. No video shots or photographs shall be be evaluated but, it can likewise be said, it is not at all
permitted during the trial proper. (Supreme Court En unlikely for a vote of guilt or innocence to yield to it . It
Banc Resolution Re: Live TV and Radio Coverage of might be farcical to build around them an impregnable
the Hearing of President Corazon C. Aquino's Libel armor against the influence of the most powerful media
Case, dated Oct. 22, 1991) of public opinion.

192. Should the Court allow live media coverage of the To say that actual prejudice should first be present would
anticipated trial of the plunder and other criminal cases leave to near nirvana the subtle threats to justice that a
filed against former President Joseph E. Estrada before disturbance of the mind so indispensable to the calm and
the Sandiganbayan in order “to assure the public of full deliberate dispensation of justice can create . The effect
transparency in the proceedings of an unprecedented of television may escape the ordinary means of proof, but
case in our history” as requested by the Kapisanan ng it is not far-fetched for it to gradually erode our basal
mga Brodkaster ng Pilipinas? conception of a trial such as we know it now.

Held: The propriety of granting or denying the instant An accused has a right to a public trial but it is a right that
petition involve the weighing out of the constitutional belongs to him, more than anyone else, where his life or
guarantees of freedom of the press and the right to public liberty can be held critically in balance. A public trial aims
information, on the one hand, and the fundamental rights to ensure that he is fairly dealt with and would not be
of the accused, on the other hand, along with the unjustly condemned and that his rights are not
constitutional power of a court to control its proceedings compromised in secret conclaves of long ago. A public
in ensuring a fair and impartial trial. trial is not synonymous with publicized trial; it only implies
that the court doors must be open to those who wish to
When these rights race against one another, come, sit in the available seats, conduct themselves with
jurisprudence tells us that the right of the accused must decorum and observe the trial process. In the
be preferred to win. constitutional sense, a courtroom should have enough
facilities for a reasonable number of the public to observe
With the possibility of losing not only the precious liberty the proceedings, not too small as to render the openness
but also the very life of an accused, it behooves all to negligible and not too large as to distract the trial
make absolutely certain that an accused receives a verdict participants from their proper functions, who shall then
solely on the basis of a just and dispassionate judgment, be totally free to report what they have observed during
a verdict that would come only after the presentation of the proceedings.
credible evidence testified to by unbiased witnesses
unswayed by any kind of pressure, whether open or The courts recognize the constitutionally embodied
subtle, in proceedings that are devoid of histrionics that freedom of the press and the right to public information.
might detract from its basic aim to ferret veritable facts It also approves of media’s exalted power to provide the
free from improper influence, and decreed by a judge with most accurate and comprehensive means of conveying
the proceedings to the public and in acquainting the in the Administrative Code of 1987, the present Court
public with the judicial process in action; nevertheless, believes that the time has come to reexamine it. The idea
within the courthouse, the overriding consideration is still that one may be compelled to salute the flag, sing the
the paramount right of the accused to due process which national anthem, and recite the patriotic pledge, during a
must never be allowed to suffer diminution in its flag ceremony on pain of being dismissed from one’s job
constitutional proportions. Justice Clark thusly or of being expelled from school, is alien to the conscience
pronounced, “while a maximum freedom must be allowed of the present generation of Filipinos who cut their teeth
the press in carrying out the important function of on the Bill of Rights which guarantees their rights to free
informing the public in a democratic society, its exercise speech (The flag salute, singing the national anthem and
must necessarily be subject to the maintenance of reciting the patriotic pledge are all forms of utterances.)
absolute fairness in the judicial process.” and the free exercise of religious profession and worship.

Xxx Religious freedom is a fundamental right which is entitled


to the highest priority and the amplest protection among
The Integrated Bar of the Philippines x x x expressed its human rights, for it involves the relationship of man to his
own concern on the live television and radio coverage of Creator (Chief Justice Enrique M. Fernando’s separate
the criminal trials of Mr. Estrada; to paraphrase: Live opinion in German v. Barangan, 135 SCRA 514, 530-531).
television and radio coverage can negate the rule on
exclusion of witnesses during the hearings intended to “The right to religious profession and worship has a two-
assure a fair trial; at stake in the criminal trial is not only fold aspect, viz., freedom to believe and freedom to act
the life and liberty of the accused but the very credibility on one’s belief. The first is absolute as long as the belief
of the Philippine criminal justice system, and live is confined within the realm of thought. The second is
television and radio coverage of the trial could allow the subject to regulation where the belief is translated into
“hooting throng” to arrogate unto themselves the task of external acts that affect the public welfare” (J. Cruz,
judging the guilt of the accused, such that the verdict of Constitutional Law, 1991 Ed., pp. 176-177).
the court will be acceptable only if popular; and live
television and radio coverage of the trial will not subserve Petitioners stress x x x that while they do not take part in
the ends of justice but will only pander to the desire for the compulsory flag ceremony, they do not engage in
publicity of a few grandstanding lawyers. “external acts” or behavior that would offend their
countrymen who believe in expressing their love of
Xxx country through the observance of the flag ceremony.
They quietly stand at attention during the flag ceremony
Unlike other government offices, courts do not express to show their respect for the rights of those who choose
the popular will of the people in any sense which, instead, to participate in the solemn proceedings. Since they do
are tasked to only adjudicate controversies on the basis not engage in disruptive behavior, there is no warrant for
of what alone is submitted before them. A trial is not a their expulsion.
free trade of ideas. Nor is a competing market of
thoughts the known test of truth in a courtroom. (Re: “The sole justification for a prior restraint or limitation on
Request Radio-TV coverage of the Trial in the the exercise of religious freedom (according to the late
Sandiganbayan of the Plunder Cases against the Chief Justice Claudio Teehankee in his dissenting opinion
former President Joseph E. Estrada, A.M. No. 01-4- in German v. Barangan, 135 SCRA 514, 517) is the
03-SC, June 29, 2001, En Banc [Vitug]) existence of a grave and present danger of a character
both grave and imminent, of a serious evil to public
safety, public morals, public health or any other legitimate
Freedom of Religion public interest, that the State has a right (and duty) to
prevent.” Absent such a threat to public safety, the
193. Discuss why the Gerona ruling (justifying the expulsion of the petitioners from the schools is not
expulsion from public schools of children of Jehovah’s justified.
Witnesses who refuse to salute the flag and sing the
national anthem during flag ceremony as prescribed by The situation that the Court directly predicted in Gerona
the Flag Salute Law) should be abandoned. that:

Held: Our task here is extremely difficult, for the 30-year “[T]he flag ceremony will become a thing of the past or
old decision of this court in Gerona upholding the flag perhaps conducted with very few participants, and the
salute law and approving the expulsion of students who time will come when we would have citizens untaught and
refuse to obey it, is not lightly to be trifled with. uninculcated in and not imbued with reverence for the
flag and love of country, admiration for national heroes,
It is somewhat ironic however, that after the Gerona and patriotism – a pathetic, even tragic situation, and all
ruling had received legislative cachet by its incorporation
because a small portion of the school population imposed In Victoriano v. Elizalde Rope Workers’ Union, 59 SCRA
its will, demanded and was granted an exemption.” 54, 72-75, we upheld the exemption of members of the
Iglesia Ni Cristo, from the coverage of a closed shop
has not come to pass. We are not persuaded that by agreement between their employer and a union because
exempting the Jehovah’s Witnesses from saluting the it would violate the teaching of their church not to join
flag, singing the national anthem and reciting the patriotic any labor group:
pledge, this religious group which admittedly comprises a
“small portion of the school population” will shake up our “x x x It is certain that not every conscience can be
part of the globe and suddenly produce a nation accommodated by all the laws of the land; but when
“untaught and uninculcated in and unimbued with general laws conflict with scruples of conscience,
reverence for the flag, patriotism, love of country and exemptions ought to be granted unless some ‘compelling
admiration for national heroes. After all, what the state interests’ intervenes. (Sherbert v. Berner, 374 U.S.
petitioners seek only is exemption from the flag 398, 10 L. Ed. 2d 965, 970, 83 S. Ct. 1790).”
ceremony, not exclusion from the public schools where
they may study the Constitution, the democratic way of We hold that a similar exemption may be accorded to the
life and form of government, and learn not only the arts, Jehovah’s Witnesses with regard to the observance of the
sciences, Philippine history and culture but also receive flag ceremony out of respect for their religious beliefs,
training for a vocation or profession and be taught the however “bizarre” those beliefs may seem to others.
virtues of “patriotism, respect for human rights, Nevertheless, their right not to participate in the flag
appreciation for national heroes, the rights and duties of ceremony does not give them a right to disrupt such
citizenship, and moral and spiritual values (Sec. 3[2], Art. patriotic exercises. Paraphrasing the warning cited by this
XIV, 1987 Constitution) as part of the curricula. Expelling Court in Non v. Dames II, 185 SCRA 523, 535, while the
or banning the petitioners from Philippine schools will highest regard must be afforded their right to the free
bring about the very situation that this Court had feared exercise of their religion, “this should not be taken to
in Gerona. Forcing a small religious group, through the mean that school authorities are powerless to discipline
iron hand of the law, to participate in a ceremony that them” if they should commit breaches of the peace by
violates their religious beliefs, will hardly be conducive to actions that offend the sensibilities, both religious and
love of country or respect for duly constituted authorities. patriotic, of other persons. If they quietly stand at
attention during the flag ceremony while their classmates
As Mr. Justice Jackson remarked in West Virginia v. and teachers salute the flag, sing the national anthem and
Barnette, 319 U.S. 624 (1943): recite the patriotic pledge, we do not see how such
conduct may possibly disturb the peace, or pose “a grave
“x x x To believe that patriotism will not flourish if patriotic and present danger of a serious evil to public safety,
ceremonies are voluntary and spontaneous instead of a public morals, public health or any other legitimate public
compulsory routine is to make an unflattering statement interest that the State has a right (and duty) to prevent .”
of the appeal of our institutions to free minds. x x x When (Ebralinag v. The Division Superintendent of
they (diversity) are so harmless to others or to the State Schools of Cebu, 219 SCRA 256, 269-273, March 1,
as those we deal with here, the price is not too great. But 1993, En Banc [Grino-Aquino])
freedom to differ is not limited to things that do not
matter much. That would be a mere shadow of freedom. 194. A pre-taped TV program of the Iglesia Ni Cristo
The test of its substance is the right to differ as to things (INC) was submitted to the MTRCB for review. The latter
that touch the heart of the existing order.” classified it as “rated X” because it was shown to be
attacking another religion. The INC protested by claiming
“Furthermore, let it be noted that coerced unity and that its religious freedom is per se beyond review by the
loyalty even to the country, x x x – assuming that such MTRCB. Should this contention be upheld?
unity and loyalty can be attained through coercion – is not
a goal that is constitutionally obtainable at the expense of Held: The right to religious profession and worship has
religious liberty. A desirable end cannot be promoted by a two-fold aspect, viz., freedom to believe and freedom
prohibited means.” (Meyer v. Nebraska, 262 U.S. 390, 67 to act on one's belief. The first is absolute as long as the
L. ed. 1042, 1046) belief is confined within the realm of thought. The second
is subject to regulation where the belief is translated into
Moreover, the expulsion of members of Jehovah’s external acts that affect the public welfare.
Witnesses from the schools where they are enrolled will
violate their right as Philippine citizens, under the 1987 The Iglesia Ni Cristo's postulate that its religious freedom
Constitution, to receive free education, for it is the duty is per se beyond review by the MTRCB should be rejected.
of the State to “protect and promote the right of all Its public broadcast on TV of its religious programs brings
citizens to quality education x x x and to make such it out of the bosom of internal belief. Television is a
education accessible to all” (Sec. 1, Art. XIV). medium that reaches even the eyes and ears of children.
The exercise of religious freedom can be regulated by the
State when it will bring about the clear and present to religious magazines, it is part of the propagation of
danger of a substantive evil which the State is duty-bound religious faith or evangelization. Such solicitation calls
to prevent, i.e., serious detriment to the more overriding upon the virtue of faith, not of charity, save as those
interest of public health, public morals, or public welfare. solicited for money or aid may not belong to the same
A laissez faire policy on the exercise of religion can be religion as the solicitor. Such solicitation does not engage
seductive to the liberal mind but history counsels the the philanthropic as much as the religious fervor of the
Court against its blind adoption as religion is and person who is solicited for contribution.
continues to be a volatile area of concern in our society
today. "For sure, we shall continue to subject any act Second. The purpose of the Decree is to protect the
pinching the space for the free exercise of religion to a public against fraud in view of the proliferation of fund
heightened scrutiny but we shall not leave its rational campaigns for charity and other civic projects. On the
exercise to the irrationality of man. For when religion other hand, since religious fund drives are usually
divides and its exercise destroys, the State should not conducted among those belonging to the same religion,
stand still." (Iglesia Ni Cristo v. CA, 259 SCRA 529, the need for public protection against fraudulent
July 26, 1996 [Puno]) solicitations does not exist in as great a degree as does
the need for protection with respect to solicitations for
195. Did the MTRCB act correctly when it rated “X” the charity or civic projects as to justify state regulation.
Iglesia Ni Cristo's pre-taped TV program simply because
it was found to be "attacking" another religion? Third. To require a government permit before solicitation
for religious purpose may be allowed is to lay a prior
Held: The MTRCB may disagree with the criticisms of restraint on the free exercise of religion. Such restraint,
other religions by the Iglesia Ni Cristo but that gives it no if allowed, may well justify requiring a permit before a
excuse to interdict such criticisms, however unclean they church can make Sunday collections or enforce tithing.
may be. Under our constitutional scheme, it is not the But in American Bible Society v. City of Manila (101 Phil.
task of the State to favor any religion by protecting it 386 [1957]), we precisely held that an ordinance requiring
against an attack by another religion. Religious dogma payment of a license fee before one may engage in
and beliefs are often at war and to preserve peace among business could not be applied to the appellant's sale of
their followers, especially the fanatics, the establishment bibles because that would impose a condition on the
clause of freedom of religion prohibits the State from exercise of a constitutional right. It is for the same reason
leaning towards any religion. Vis-à-vis religious that religious rallies are exempted from the requirement
differences, the State enjoys no banquet of options. of prior permit for public assemblies and other uses of
Neutrality alone is its fixed and immovable stance. In public parks and streets (B.P. Blg. 880, Sec. 3[a]). To
fine, the MTRCB cannot squelch the speech of the INC read the Decree, therefore, as including within its reach
simply because it attacks another religion. In a State solicitations for religious purposes would be to construe it
where there ought to be no difference between the in a manner that it violates the Free Exercise of Religion
appearance and the reality of freedom of religion, the Clause of the Constitution x x x. (Concurring Opinion,
remedy against bad theology is better theology. The Mendoza, V.V., J., in Centeno v. Villalon-Pornillos,
bedrock of freedom of religion is freedom of thought and 236 SCRA 197, Sept. 1, 1994)
it is best served by encouraging the marketplace of
dueling ideas. When the luxury of time permits, the 197. What is a purely ecclesiastical affair to which the
marketplace of ideas demands that speech should be met State can not meddle?
by more speech for it is the spark of opposite speech, the
heat of colliding ideas, that can fan the embers of truth. Held: An ecclesiastical affair is “one that concerns
(Iglesia Ni Cristo v. CA, 259 SCRA 529, July 26, doctrine, creed, or form of worship of the church, or the
1996 [Puno]) adoption and enforcement within a religious association
of needful laws and regulations for the government of the
196. Is solicitation for the construction of a church membership, and the power of excluding from such
covered by P.D. No. 1564 and, therefore, punishable if associations those deemed not worthy of membership.”
done without the necessary permit for solicitation from Based on this definition, an ecclesiastical affair involves
the DSWD? the relationship between the church and its members and
relate to matters of faith, religious doctrines, worship and
Held: First. Solicitation of contributions for the governance of the congregation. To be concrete,
construction of a church is not solicitation for "charitable examples of this so-called ecclesiastical affairs to which
or public welfare purpose" but for a religious purpose, and the State cannot meddle are proceedings for
a religious purpose is not necessarily a charitable or public excommunication, ordinations of religious ministers,
welfare purpose. A fund campaign for the construction administration of sacraments and other activities with
or repair of a church is not like fund drives for needy attached religious significance. (Pastor Dionisio V.
families or victims of calamity or for the construction of a Austria v. NLRC, G.R. No. 124382, Aug. 16, 1999,
civic center and the like. Like solicitation of subscription 1st Div. [Kapunan])
conduct of citizens, as held in Tanada. Likewise did the
198. Petitioner is a religious minister of the Seventh “public nature of the loanable funds of the GSIS and the
Day Adventist (SDA). He was dismissed because of public office held by the alleged borrowers (members of
alleged misappropriation of denominational funds, willful the defunct Batasang Pambansa)” qualify the information
breach of trust, serious misconduct, gross and habitual sought in Valmonte as matters of public interest and
neglect of duties and commission of an offense against concern. In Aquino-Sarmiento v. Morato (203 SCRA 515,
the person of his employer’s duly authorized 522-23, November 13, 1991), the Court also held that
representative. He filed an illegal termination case official acts of public officers done in pursuit of their
against the SDA before the labor arbiter. The SDA filed a official functions are public in character; hence, the
motion to dismiss invoking the doctrine of separation of records pertaining to such official acts and decisions are
Church and State. Should the motion be granted? within the ambit of the constitutional right of access to
public records.
Held: Where what is involved is the relationship of the
church as an employer and the minister as an employee Under Republic Act No. 6713, public officials and
and has no relation whatsoever with the practice of faith, employees are mandated to “provide information on their
worship or doctrines of the church, i.e., the minister was policies and procedures in clear and understandable
not excommunicated or expelled from the membership of language, [and] ensure openness of information, public
the congregation but was terminated from employment, consultations and hearing whenever appropriate x x x,”
it is a purely secular affair. Consequently, the suit may except when “otherwise provided by law or when required
not be dismissed invoking the doctrine of separation of by the public interest.” In particular, the law mandates
church and the state. (Pastor Dionisio V. Austria v. free public access, at reasonable hours, to the annual
NLRC, G.R. No. 124382, Aug. 16, 1999, 1st Div. performance reports of offices and agencies of
[Kapunan]) government and government-owned or controlled
corporations; and the statements of assets, liabilities and
financial disclosures of all public officials and employees.
The Right of the People to Information on Matters
of Public Concern In general, writings coming into the hands of public
officers in connection with their official functions must be
199. Discuss the scope of the right to information on accessible to the public, consistent with the policy of
matters of public concern. transparency of governmental affairs. This principle is
aimed at affording the people an opportunity to determine
Held: In Valmonte v. Belmonte, Jr., the Court whether those to whom they have entrusted the affairs of
emphasized that the information sought must be “matters the government are honestly, faithfully and competently
of public concern,” access to which may be limited by law. performing their functions as public servants.
Similarly, the state policy of full public disclosure extends Undeniably, the essence of democracy lies in the free-flow
only to “transactions involving public interest” and may of thought; but thoughts and ideas must be well-informed
also be “subject to reasonable conditions prescribed by so that the public would gain a better perspective of vital
law.” As to the meanings of the terms “public interest” issues confronting them and, thus, be able to criticize as
and “public concern,” the Court, in Legaspi v. Civil Service well as participate in the affairs of the government in a
Commission, elucidated: responsible, reasonable and effective manner. Certainly,
it is by ensuring an unfettered and uninhibited exchange
“In determining whether or not a particular information is of ideas among a well-informed public that a government
of public concern there is no rigid test which can be remains responsive to the changes desired by the people .
applied. ‘Public concern’ like ‘public interest’ is a term that (Chavez v. PCGG, 299 SCRA 744, Dec. 9, 1998,
eludes exact definition. Both terms embrace a broad [Panganiban])
spectrum of subjects which the public may want to know,
either because these directly affect their lives, or simply 200. What are some of the recognized restrictions to
because such matters naturally arouse the interest of an the right of the people to information on matters of public
ordinary citizen. In the final analysis, it is for the courts concern?
to determine on a case by case basis whether the matter
at issue is of interest or importance, as it relates to or Held:
affects the public.”
1) National security matters and intelligence
Considered a public concern in the above-mentioned case information. This jurisdiction recognizes the common law
was the “legitimate concern of citizens to ensure that holding that there is a governmental privilege against
government positions requiring civil service eligibility are public disclosure with respect to state secrets regarding
occupied only by persons who are eligibles.” So was the military, diplomatic and other national security matters.
need to give the general public adequate notification of Likewise, information on inter-government exchanges
various laws that regulate and affect the actions and prior to the conclusion of treaties and executive
agreements may be subject to reasonable safeguards for
the sake of national interest;
2) Trade or industrial secrets (pursuant to the Freedom of Association
Intellectual Property Code [R.A. No. 8293, approved on
June 6, 1997] and other related laws) and banking 202. Does the right of civil servants to organize include
transactions (pursuant to the Secrecy of Bank Deposits their right to strike? Clarify.
Act [R.A. No. 1405, as amended]);
3) Criminal matters, such as those relating to Held: Specifically, the right of civil servants to organize
the apprehension, the prosecution and the detention of themselves was positively recognized in Association of
criminals, which courts may not inquire into prior to such Court of Appeals Employees (ACAE) v. Ferrer-Calleja (203
arrest, detention and prosecution; SCRA 596, November 15, 1991). But, as in the exercise
4) Other confidential information. The Ethical of the rights of free expression and of assembly, there are
Standards Act (R.A. No. 6713, enacted on February 20, standards for allowable limitations such as the legitimacy
1989) further prohibits public officials and employees of the purposes of the association, the overriding
from using or divulging “confidential or classified considerations of national security and the preservation
information officially known to them by reason of their of democratic institutions (People v. Ferrer, 48 SCRA 382,
office and not made available to the public.” (Sec. 7[c], December 27, 1972, per Castro, J., where the Court, while
ibid.) Other acknowledged limitations to information upholding the validity of the Anti-Subversion Act which
access include diplomatic correspondence, closed door outlawed the Communist Party of the Philippines and
Cabinet meetings and executive sessions of either house other "subversive" organizations, clarified, "Whatever
of Congress, as well as the internal deliberations of the interest in freedom of speech and freedom of association
Supreme Court. is infringed by the prohibition against knowing
(Chavez v. PCGG, 299 SCRA 744, Dec. 9, 1998 membership in the Communist Party of the Philippines, is
[Panganiban]) so indirect and so insubstantial as to be clearly and heavily
outweighed by the overriding considerations of national
201. Is the alleged ill-gotten wealth of the Marcoses a security and the preservation of democratic institutions in
matter of public concern subject to this right? this country." It cautioned, though, that "the need for
prudence and circumspection [cannot be
Held: With such pronouncements of our government, overemphasized] in [the law's] enforcement, operating as
whose authority emanates from the people, there is no it does in the sensitive area of freedom of expression and
doubt that the recovery of the Marcoses' alleged ill-gotten belief.")
wealth is a matter of public concern and imbued with
public interest. We may also add that “ill-gotten wealth” As regards the right to strike, the Constitution itself
refers to assets and properties purportedly acquired, qualifies its exercise with the proviso "in accordance with
directly or indirectly, by former President Marcos, his law." This is a clear manifestation that the state may, by
immediate family, relatives and close associates through law, regulate the use of this right, or even deny certain
or as a result of their improper or illegal use of sectors such right. Executive Order No. 180 (Issued by
government funds or properties; or their having taken former President Corazon C. Aquino on June 1, 1987)
undue advantage of their public office; or their use of which provides guidelines for the exercise of the right of
powers, influences or relationships, “resulting in their government workers to organize, for instance, implicitly
unjust enrichment and causing grave damage and endorsed an earlier CSC circular which "enjoins under
prejudice to the Filipino people and the Republic of the pain of administrative sanctions, all government officers
Philippines.” Clearly, the assets and properties referred and employees from staging strikes, demonstrations,
to supposedly originated from the government itself. To mass leaves, walkouts and other forms of mass action
all intents and purposes, therefore, they belong to the which will result in temporary stoppage or disruption of
people. As such, upon reconveyance they will be returned public service" (CSC Memorandum Circular No. 6, s. 1987,
to the public treasury, subject only to the satisfaction of dated April 21, 1987) by stating that the Civil Service law
positive claims of certain persons as may be adjudged by and rules governing concerted activities and strikes in the
competent courts. Another declared overriding government service shall be observed.
consideration for the expeditious recovery of ill-gotten
wealth is that it may be used for national economic It is also settled in jurisprudence that, in general, workers
recovery. in the public sector do not enjoy the right to strike.
Alliance of Concerned Government Workers v. Minister of
We believe the foregoing disquisition settles the question Labor and Employment (124 SCRA 1, August 3, 1983, also
of whether petitioner has a right to respondents' per Gutierrez, Jr., J.) rationalized the proscription thus:
disclosure of any agreement that may be arrived at
concerning the Marcoses’ purported ill-gotten wealth. "The general rule in the past and up to the present is that
(Chavez v. PCGG, 299 SCRA 744, Dec. 9, 1998 the 'terms and conditions of employment in the
[Panganiban]) Government, including any political subdivision or
instrumentality thereof are governed by law.' X x x. Since conduct prejudicial to the best interests of the service for
the terms and conditions of government employment are having absented themselves without proper authority,
fixed by law, government workers cannot use the same from their schools during regular school days, and
weapons employed by the workers in the private sector penalized. They denied that they engaged in “strike” but
to secure concessions from their employers. The principle claimed that they merely exercised a constitutionally
behind labor unionism in private industry is that industrial guaranteed right – the right to peaceably assemble and
peace cannot be secured through compulsion by law. petition the government for redress of grievances - and,
Relations between private employers and their employees therefore, should not have been penalized. Should their
rest on an essentially voluntary basis. Subject to the contention be upheld?
minimum requirements of wage laws and other labor and
welfare legislation, the terms and conditions of Held: Petitioners, who are public schoolteachers and
employment in the unionized private sector are settled thus government employees, do not seek to establish that
through the process of collective bargaining. In they have a right to strike. Rather, they tenaciously insist
government employment, however, it is the legislature that their absences during certain dates in September
and, where properly given delegated power, the 1990 were a valid exercise of their constitutional right to
administrative heads of government which fix the terms engage in peaceful assembly to petition the government
and conditions of employment. And this is effected for a redress of grievances. They claim that their
through statutes or administrative circulars, rules, and gathering was not a strike, therefore, their participation
regulations, not through collective bargaining therein did not constitute any offense. MPSTA v. Laguio
agreements." (Ibid., p. 13) (Supra, per Narvasa, J., now CJ.) and ACT v. Carino
(Ibid.), in which this Court declared that "these 'mass
After delving into the intent of the framers of the actions' were to all intents and purposes a strike; they
Constitution, the Court affirmed the above rule in Social constituted a concerted and unauthorized stoppage of, or
Security System Employees Association (SSSEA) v. Court absence from, work which it was the teachers' duty to
of Appeals (175 SCRA 686, July 28, 1989) and explained: perform, undertaken for essentially economic reasons,"
should not principally resolve the present case, as the
"Government employees may, therefore, through their underlying facts are allegedly not identical.
unions or associations, either petition the Congress for the
betterment of the terms and conditions of employment Strike, as defined by law, means any temporary stoppage
which are within the ambit of legislation or negotiate with of work done by the concerted action of employees as a
the appropriate government agencies for the result of an industrial or labor dispute. A labor dispute
improvement of those which are not fixed by law. If there includes any controversy or matter concerning terms and
be any unresolved grievances, the dispute may be conditions of employment; or the association or
referred to the Public Sector Labor-Management Council representation of persons in negotiating, fixing,
for appropriate action. But employees in the civil service maintaining, changing or arranging the terms and
may not resort to strikes, walkouts and other temporary conditions of employment, regardless of whether the
work stoppages, like workers in the private sector, to disputants stand in the proximate relation of employers
pressure the Government to accede to their demands. As and employees. With these premises, we now evaluate
now provided under Sec. 4, Rule III of the Rules and the circumstances of the instant petition.
Regulations to Govern the Exercise of the Right of
Government Employees to Self-Organization, which took It cannot be denied that the mass action or assembly
effect after the instant dispute arose, '[t]he terms and staged by the petitioners resulted in the non-holding of
conditions of employment in the government, including classes in several public schools during the corresponding
any political subdivision or instrumentality thereof and period. Petitioners do not dispute that the grievances for
government-owned and controlled corporations with which they sought redress concerned the alleged failure
original charters are governed by law and employees of public authorities - essentially, their "employers" - to
therein shall not strike for the purpose of securing fully and justly implement certain laws and measures
changes [thereto].'' (Ibid., p. 698) intended to benefit them materially x x x. And probably
(Jacinto v. Court of Appeals, 281 SCRA 657, Nov. to clothe their action with permissible character (In
14, 1997, En Banc [Panganiban]) justifying their mass actions, petitioners liken their activity
to the pro-bases rally led by former President Corazon C.
203. Petitioners public school teachers walked out of Aquino on September 10, 1991, participated in, as well,
their classes and engaged in mass actions during certain by public school teachers who consequently absented
dates in September 1990 protesting the alleged unlawful themselves from their classes. No administrative charges
withholding of their salaries and other economic benefits. were allegedly instituted against any of the participants.) ,
They also raised national issues, such as the removal of they also raised national issues such as the removal of the
US bases and the repudiation of foreign debts, in their U.S. bases and the repudiation of foreign debt. In
mass actions. They refused to return to work despite Balingasan v. Court of Appeals (G.R. No. 124678, July 31,
orders to do so and subsequently were found guilty of 1997, per Regalado, J.), however, this Court said that the
fact that the conventional term "strike" was not used by powers of the State and involving the public rights and
the participants to describe their common course of action public welfare of the entire community affected by it. It
was insignificant, since the substance of the situation, and does not prevent a proper exercise by the State of its
not its appearance, was deemed controlling. police power by enacting regulations reasonably
necessary to secure the health, safety, morals, comfort,
Moreover, the petitioners here x x x were not penalized or general welfare of the community, even though
for the exercise of their right to assemble peacefully and contracts may thereby be affected, for such matters
to petition the government for a redress of grievances. cannot be placed by contract beyond the power of the
Rather, the Civil Service Commission found them guilty of State to regulate and control them.
conduct prejudicial to the best interest of the service for
having absented themselves without proper authority, Verily, the freedom to contract is not absolute; all
from their schools during regular school days, in order to contracts and all rights are subject to the police power of
participate in the mass protest, their absence ineluctably the State and not only may regulations which affect them
resulting in the non-holding of classes and in the be established by the State, but all such regulations must
deprivation of students of education, for which they were be subject to change from time to time, as the general
responsible. Had petitioners availed themselves of their well-being of the community may require, or as the
free time - recess, after classes, weekends or holidays - circumstances may change, or as experience may
to dramatize their grievances and to dialogue with the demonstrate the necessity. And under the Civil Code,
proper authorities within the bounds of law, no one - not contracts of labor are explicitly subject to the police power
the DECS, the CSC or even this Court - could have held of the State because they are not ordinary contracts but
them liable for the valid exercise of their constitutionally are impressed with public interest. Article 1700 thereof
guaranteed rights. As it was, the temporary stoppage of expressly provides:
classes resulting from their activity necessarily disrupted
public services, the very evil sought to be forestalled by Art. 1700. The relations between capital and labor are
the prohibition against strikes by government workers. not merely contractual. They are so impressed with
Their act by their nature was enjoined by the Civil Service public interest that labor contracts must yield to the
law, rules and regulations, for which they must, therefore, common good. Therefore, such contracts are subject to
be made answerable. (Jacinto v. CA, 281 SCRA 657, the special laws on labor unions, collective bargaining,
Nov. 14, 1997, En Banc [Panganiban]) strikes and lockouts, closed shop, wages, working
conditions, hours of labor and similar subjects.

The Non-Impairment Clause The challenged resolution and memorandum circular


being valid implementations of E.O. No. 797 (Creating the
204. Is the constitutional prohibition against POEA), which was enacted under the police power of the
impairing contractual obligations absolute? State, they cannot be struck down on the ground that
they violate the contract clause. To hold otherwise is to
Held: 1. Nor is there merit in the claim that the alter long-established constitutional doctrine and to
resolution and memorandum circular violate the contract subordinate the police power to the contract clause. (The
clause of the Bill of Rights. Conference of Maritime Manning Agencies, Inc. v.
POEA, 243 SCRA 666, April 21, 1995 [Davide, Jr.])
The executive order creating the POEA was enacted to
further implement the social justice provisions of the 1973 2. Petitioners pray that the present action should be
Constitution, which have been greatly enhanced and barred, because private respondents have voluntarily
expanded in the 1987 Constitution by placing them under executed quitclaims and releases and received their
a separate Article (Article XIII). The Article on Social separation pay. Petitioners claim that the present suit is
Justice was aptly described as the "heart of the new a "grave derogation of the fundamental principle that
Charter" by the President of the 1986 Constitutional obligations arising from a valid contract have the force of
Commission, retired Justice Cecilia Munoz Palma . Social law between the parties and must be complied with in
justice is identified with the broad scope of the police good faith."
power of the state and requires the extensive use of such
power. X x x. The Court disagrees. Jurisprudence holds that the
constitutional guarantee of non-impairment of contract is
The constitutional prohibition against impairing subject to the police power of the state and to reasonable
contractual obligations is not absolute and is not to be legislative regulations promoting health, morals, safety
read with literal exactness. It is restricted to contracts and welfare. Not all quitclaims are per se invalid or
with respect to property or some object of value and against public policy, except (1) where there is clear proof
which confer rights that may be asserted in a court of that the waiver was wangled from an unsuspecting or
justice; it has no application to statutes relating to public gullible person, or (2) where the terms of settlement are
subjects within the domain of the general legislative unconscionable on their face. In these cases, the law will
step in to annul the questionable transactions. Such In short, the non-impairment clause must yield to the
quitclaim and release agreements are regarded as police power of the state.
ineffective to bar the workers from claiming the full
measure of their legal rights. Finally, it is difficult to imagine x x x how the non-
impairment clause could apply with respect to the prayer
In the case at bar, the private respondents agreed to the to enjoin the respondent Secretary from receiving,
quitclaim and release in consideration of their separation accepting, processing, renewing or approving new timber
pay. Since they were dismissed allegedly for business license for, save in cases of renewal, no contract would
losses, they are entitled to separation pay under Article have as yet existed in the other instances. Moreover, with
283 of the Labor Code. And since there was thus no extra respect to renewal, the holder is not entitled to it as a
consideration for the private respondents to give up their matter of right. (Oposa v. Factoran, Jr., 224 SCRA
employment, such undertakings cannot be allowed to bar 792 [1993])
the action for illegal dismissal. (Bogo-Medellin
Sugarcane Planters Association, Inc. v. NLRC, 296 5. Anent petitioners' contention that the forcible refund
SCRA 108, 124, [Panganiban]) of incentive benefits is an unconstitutional impairment of
a contractual obligation, suffice it to state that "[n]ot all
3. Only slightly less abstract but nonetheless hypothetical contracts entered into by the government will operate as
is the contention of CREBA that the imposition of the VAT a waiver of its non-suability; distinction must be made
on the sales and leases of real estate by virtue of between its sovereign and proprietary acts. The acts
contracts entered prior to the effectivity of the law would involved in this case are governmental. Besides, the
violate the constitutional provision that "No law impairing Court is in agreement with the Solicitor General that the
the obligation of contracts shall be passed." It is enough incentive pay or benefit is in the nature of a bonus which
to say that the parties to a contract cannot, through the is not a demandable or enforceable obligation.
exercise of prophetic discernment, fetter the exercise of (Blaquera v. Alcala, 295 SCRA 366, 446, Sept. 11,
the taxing power of the State. For not only are existing 1998, En Banc [Purisima])
laws read into contracts in order to fix obligations as
between parties, but the reservation of essential
attributes of sovereign power is also read into contracts The In-Custodial Investigation Rights of an Accused
as a basic postulate of the legal order. The policy of Person
protecting contracts against impairment presupposes the
maintenance of a government which retains adequate 205. State the procedure, guidelines and duties which
authority to secure the peace and good order of society. the arresting, detaining, inviting, or investigating officer
or his companions must do and observe at the time of
In truth, the Contract Clause has never been thought as making an arrest and again at and during the time of the
a limitation on the exercise of the State's power of custodial interrogation.
taxation save only where a tax exemption has been
granted for a valid consideration. X x x. (Tolentino v. Held: Lastly, considering the heavy penalty of death and
Secretary of Finance, 235 SCRA 630, 685-686, in order to ensure that the evidence against an accused
Aug. 25, 1994, En Banc [Mendoza]) were obtained through lawful means, the Court, as
guardian of the rights of the people lays down the
4. Since timber licenses are not contracts, the non- procedure, guidelines and duties which the arresting,
impairment clause x x x cannot be invoked. detaining, inviting, or investigating officer or his
companions must do and observe at the time of making
X x x, even if it is to be assumed that the same are an arrest and again at and during the time of the custodial
contracts, the instant case does not involve a law or even interrogation in accordance with the Constitution,
an executive issuance declaring the cancellation or jurisprudence and Republic Act No. 7438 (An Act Defining
modification of existing timber licenses. Hence, the non- Certain Rights of Person Arrested, Detained or Under
impairment clause cannot as yet be invoked. Custodial Investigation as well as the Duties of the
Nevertheless, granting further that a law has actually Arresting, Detaining, and Investigating Officers and
been passed mandating cancellations or modifications, Providing Penalties for Violations Thereof). It is high-time
the same cannot still be stigmatized as a violation of the to educate our law-enforcement agencies who neglect
non-impairment clause. This is because by its very nature either by ignorance or indifference the so-called Miranda
and purpose, such a law could have only been passed in rights which had become insufficient and which the Court
the exercise of the police power of the state for the must update in the light of new legal developments:
purpose of advancing the right of the people to a balanced
and healthful ecology, promoting their health and 1) The person arrested, detained, invited or
enhancing their general welfare. X x x. under custodial investigation must be informed in a
language known to and understood by him of the reason
for the arrest and he must be shown the warrant of arrest,
if any. Every other warnings, information or (People v. Mahinay, 302 SCRA 455, Feb. 1, 1999,
communication must be in a language known to and En Banc [Per Curiam])
understood by said person;
2) He must be warned that he has a right to 206. Explain the kind of information that is required to
remain silent and that any statement he makes may be be given by law enforcement officers to suspect during
used as evidence against him; custodial investigation.
3) He must be informed that he has the right to
be assisted at all times and have the presence of an Held: [I]t is settled that one’s right to be informed of the
independent and competent lawyer, preferably of his own right to remain silent and to counsel contemplates the
choice; transmission of meaningful information rather just the
4) He must be informed that if he has no lawyer ceremonial and perfunctory recitation of an abstract
or cannot afford the services of a lawyer, one will be constitutional principle. It is not enough for the
provided for him; and that a lawyer may also be engaged interrogator to merely repeat to the person under
by any person in his behalf, or may be appointed by the investigation the provisions of Section 12, Article III of the
court upon petition of the person arrested or one acting 1987 Constitution; the former must also explain the
on his behalf; effects of such provision in practical terms – e.g., what
5) That whether or not the person arrested has the person under investigation may or may not do – and
a lawyer, he must be informed that no custodial in a language the subject fairly understands. The right to
investigation in any form shall be conducted except in the be informed carries with it a correlative obligation on the
presence of his counsel of after a valid waiver has been part of the police investigator to explain, and
made; contemplates effective communication which results in
6) The person arrested must be informed that, the subject’s understanding of what is conveyed. Since it
at any time, he has the right to communicate or confer is comprehension that is sought to be attained, the
by the most expedient means - telephone, radio, letter or degree of explanation required will necessarily vary and
messenger - with his lawyer (either retained or depend on the education, intelligence, and other relevant
appointed), any member of his immediate family, or any personal circumstances of the person undergoing
medical doctor, priest or minister chosen by him or by any investigation. In further ensuring the right to counsel, it
one from his immediate family or by his counsel, or be is not enough that the subject is informed of such right;
visited by/confer with duly accredited national or he should also be asked if he wants to avail of the same
international non-government organization. It shall be and should be told that he could ask for counsel if he so
the responsibility of the officer to ensure that this is desired or that one could be provided him at his request.
accomplished; If he decides not to retain a counsel of his choice or avail
7) He must be informed that he has the right to of one to be provided for him and, therefore, chooses to
waive any of said rights provided it is made voluntarily, waive his right to counsel, such waiver, to be valid and
knowingly and intelligently and ensure that he understood effective, must still be made with the assistance of
the same; counsel, who, under prevailing jurisprudence, must be a
8) In addition, if the person arrested waives his lawyer. (People v. Canoy, 328 SCRA 385, March 17,
right to a lawyer, he must be informed that it must be 2000, 1st Div. [Davide, CJ])
done in writing and in the presence of counsel, otherwise,
he must be warned that the waiver is void even if he insist 207. What is the meaning of “competent counsel”
on his waiver and chooses to speak; under Section 12 of the Bill of Rights?
9) That the person arrested must be informed
that he may indicate in any manner at any time or stage Held: The meaning of “competent counsel” was
of the process that he does not wish to be questioned explained in People v. Deniega (251 SCRA 626, 637) as
with warning that once he makes such indication, the follows:
police may not interrogate him if the same had not yet
commenced, or the interrogation must cease if it has “x x x [T]he lawyer called to be present during such
already begun; investigation should be as far as reasonably possible, the
10) The person arrested must be informed that choice of the individual undergoing questioning. If the
his initial waiver of his right to remain silent, the right to lawyer were one furnished in the accused’s behalf, it is
counsel or any of his rights does not bar him from important that he should be competent and independent,
invoking it at any time during the process, regardless of i.e., that he is willing to fully safeguard the constitutional
whether he may have answered some questions or rights of the accused, as distinguished from one who
volunteered some statements; would merely be giving a routine, peremptory and
11) He must also be informed that any statement meaningless recital of the individual’s rights. In People v.
or evidence, as the case may be, obtained in violation of Basay (219 SCRA 404, 418), this Court stressed that an
any of the foregoing, whether inculpatory or exculpatory, accused’s right to be informed of the right to remain silent
in whole or in part, shall be admissible in evidence. and to counsel ‘contemplates the transmission of
meaningful information rather than just the ceremonial
and perfunctory recitation of an abstract constitutional appellant was going to confess his guilt to him. When
principle.’ appellant talked with the mayor as a confidant and not as
a law enforcement officer, his uncounselled confession to
“Ideally therefore, a lawyer engaged for an individual him did not violate his constitutional rights. Thus, it has
facing custodial investigation (if the latter could not afford been held that the constitutional procedures on custodial
one) ‘should be engaged by the accused (himself), or by investigation do not apply to a spontaneous statement,
the latter’s relative or person authorized by him to engage not elicited through questioning by the authorities, but
an attorney or by the court, upon proper petition of the given in an ordinary manner whereby appellant orally
accused or person authorized by the accused to file such admitted having committed the crime. What the
petition.’ Lawyers engaged by the police, whatever Constitution bars is the compulsory disclosure of
testimonials are given as proof of their probity and incriminating facts or confessions. The rights under
supposed independence, are generally suspect, as in Section 12 are guaranteed to preclude the slightest use
many areas, the relationship between lawyers and law of coercion by the State as would lead the accused to
enforcement authorities can be symbiotic. admit something false, not to prevent him from freely and
voluntarily telling the truth. (People v. Andan, 269
“x x x The competent or independent lawyer so engaged SCRA 95, March 3, 1997)
should be present from the beginning to end, i.e., at all
stages of the interview, counseling or advising caution 210. Are confessions made in response to questions by
reasonably at every turn of the investigation, and news reporters admissible in evidence?
stopping the interrogation once in a while either to give
advice to the accused that he may either continue, choose Answer: Yes. Confessions made in response to
to remain silent or terminate the interview.” questions by news reporters, not by the police or any
(People v. Espiritu, 302 SCRA 533, Feb. 2, 1999, 3rd other investigating officer, are admissible. In People v.
Div. [Panganiban]) Vizcarra, 115 SCRA 743, 752 [1982], where the accused,
under custody, gave spontaneous answers to a televised
208. Can a PAO lawyer be considered an independent interview by several press reporters in the office of the
counsel within the contemplation of Section 12, Article III, chief of the CIS, it was held that statements
1987 Constitution? spontaneously made by a suspect to news reporters on a
televised interview are deemed voluntary and are
Held: In People v. Oracoy, 224 SCRA 759 [1993]; People admissible in evidence. In People v. Andan, 269 SCRA
v. Bandula, 232 SCRA 566 [1994], the SC has held that a 95, March 3, 1997, it was held that appellant’s
PAO lawyer can be considered an independent counsel confessions to the news reporters were given free from
within the contemplation of the Constitution considering any undue influence from the police authorities. The
that he is not a special counsel, public or private news reporters acted as news reporters when they
prosecutor, counsel of the police, or a municipal attorney interviewed appellant. They were not acting under the
whose interest is admittedly adverse to that of the direction and control of the police. They did not force
accused-appellant. Thus, the assistance of a PAO lawyer appellant to grant them an interview and reenact the
satisfies the constitutional requirement of a competent commission of the crime. In fact, they asked his
and independent counsel for the accused. (People v. permission before interviewing him. The Supreme Court
Bacor, 306 SCRA 522, April 30, 1999, 2nd Div. further ruled that appellant’s verbal confessions to the
[Mendoza]) newsmen are not covered by Section 12(1) and (3) of
Article III of the Constitution and, therefore, admissible in
209. Is the confession of an accused given evidence.
spontaneously, freely and voluntarily to the Mayor
admissible in evidence, considering that the Mayor has
“operational supervision and control” over the local police 211. Discuss why lower court’s should act with
and may arguably be deemed a law enforcement officer? extreme caution in admitting in evidence accused’s
videotaped media confessions.
Held: While it is true that a municipal mayor has
“operational supervision and control” over the local police Held: Apropos the court a quo’s admission of accused-
and may arguably be deemed a law enforcement officer appellant’s videotaped confession, we find such
for purposes of applying Section 12(1) and (3) of Article admission proper. The interview was recorded on video
III of the Constitution, however, appellant’s confession to and it showed accused-appellant unburdening his guilt
the mayor was not made in response to any interrogation willingly, openly and publicly in the presence of
by the latter. In fact, the mayor did not question the newsmen. Such confession does not form part of
appellant at all. No police authority ordered appellant to custodial investigation as it was not given to police
talk to the mayor. It was appellant himself who officers but to media men in an attempt to elicit
spontaneously, freely and voluntarily sought the mayor sympathy and forgiveness from the public. Besides, if
for a private meeting. The mayor did not know that he had indeed been forced into confessing, he could
have easily sought succor from the newsmen who, in all violence on him. He did not seek medical treatment nor
likelihood, would have been sympathetic with him. X x x even a physical examination. His allegation that the fact
that he was made to sign the confession five times is
X x x However, because of the inherent danger proof that he refused to sign it.
in the use of television as a medium for admitting one’s
guilt, and the recurrence of this phenomenon in several Xxx
cases (People v. Vizcarra, No. L-38859, 30 July 1982, 115
SCRA 743; others omitted), it is prudent that trial courts We discern no sign that the confession was involuntarily
are reminded that extreme caution must be taken in executed from the fact that it was signed by accused-
further admitting similar confessions. For in all appellant five times.
probability, the police, with the connivance of
unscrupulous media practitioners, may attempt to Xxx
legitimize coerced extrajudicial confessions and place
them beyond the exclusionary rule by having an accused Extrajudicial confessions are presumed voluntary, and, in
admit an offense on television. Such a situation would be the absence of conclusive evidence showing the
detrimental to the guaranteed rights of the accused and declarant’s consent in executing the same has been
thus imperil our criminal justice system. vitiated, such confession will be sustained.

We do not suggest that videotaped confessions given Moreover, the confession contains details that only the
before media men by an accused with the knowledge of perpetrator of the crime could have given. X x x. It has
and in the presence of police officers are impermissible. been held that voluntariness of a confession may be
Indeed, the line between proper and invalid police inferred from its being replete with details which could
techniques and conduct is a difficult one to draw, possibly be supplied only by the accused, reflecting
particularly in cases such as this where it is essential to spontaneity and coherence which cannot be said of a
make sharp judgments in determining whether a mind on which violence and torture have been applied .
confession was given under coercive physical or When the details narrated in an extrajudicial confession
psychological atmosphere. are such that they could not have been concocted by one
who did not take part in the acts narrated, where the
A word of caution then to lower courts: we should never claim of maltreatment in the extraction of the confession
presume that all media confessions described as is unsubstantiated and where abundant evidence exists
voluntary have been freely given. This type of confession showing that the statement was voluntarily executed, the
always remains suspect and therefore should be confession is admissible against the declarant. There is
thoroughly examined and scrutinized. Detection of greater reason for finding a confession to be voluntary
coerced confessions is admittedly a difficult and arduous where it is corroborated by evidence aliunde which
task for the courts to make. It requires persistence and dovetails with the essential facts contained in such
determination in separating polluted confessions from confession.
untainted ones. We have a sworn duty to be vigilant and
protective of the rights guaranteed by the Constitution. But what renders the confession of accused-appellant
(People v. Endino, 353 SCRA 307, Feb. 20, 2001, inadmissible is the fact that accused-appellant was not
2nd Div. [Bellosillo]) given the Miranda warnings effectively. Under the
Constitution, an uncounseled statement, such as it is
called in the United States from which Article III, Section
212. Discuss the two kinds of involuntary or coerced 12(1) was derived, is presumed to be psychologically
confessions under Section 12, Article III of the 1987 coerced. Swept into an unfamiliar environment and
Constitution. Illustrate how the Court should appreciate surrounded by intimidating figures typical of the
said involuntary or coerced confessions. atmosphere of police interrogation, the suspect really
needs the guiding hand of counsel.
Held: There are two kinds of involuntary or coerced
confessions treated in this constitutional provision: (1) Now, under the first paragraph of this provision, it is
those which are the product of third degree methods such required that the suspect in custodial interrogation must
as torture, force, violence, threat, intimidation, which are be given the following warnings: (1) he must be informed
dealt with in paragraph 2 of Section 12, and (2) those of his right to remain silent; (2) he must be warned that
which are given without the benefit of Miranda warnings, anything he says can and will be used against him; and
which are the subject of paragraph 1 of the same Section (3) he must be told that he has a right to counsel, and
12. that if he is indigent, a lawyer will be appointed to
represent him.
Accused-appellant claims that his confession was
obtained by force and threat. Aside from this bare Xxx
assertion, he has shown no proof of the use of force and
There was thus only a perfunctory reading of the Miranda conscience. When all these requirements are met and the
rights to accused-appellant without any effort to find out confession is admitted in evidence, the burden of proof
from him whether he wanted to have counsel and, if so, that it was obtained by undue pressure, threat or
whether he had his own counsel or he wanted the police intimidation rests upon the accused. (People v. Fabro,
to appoint one for him. This kind of giving of warnings, 277 SCRA 19, Aug. 11, 1997 [Panganiban])
in several decisions of this Court, has been found to be
merely ceremonial and inadequate to transmit meaningful 2. Numerous decisions of this Court rule that for an
information to the suspect. Especially in this case, care extrajudicial confession to be admissible, it must be: 1)
should have been scrupulously observed by the police voluntary; 2) made with the assistance of competent and
investigator that accused-appellant was specifically asked independent counsel; 3) express; and 4) in writing.
these questions considering that he only finished the
fourth grade of the elementary school. X x x The mantle of protection afforded by the above-quoted
constitutional provision covers the period from the time a
Moreover, Article III, Section 12(1) requires that counsel person is taken into custody for the investigation of his
assisting suspects in custodial interrogations be possible participation in the commission of a crime or from
competent and independent. Here, accused-appellant the time he is singled out as a suspect in the commission
was assisted by Atty. De los Reyes, who, though of the offense although not yet in custody. The
presumably competent, cannot be considered an exclusionary rule is premised on the presumption that the
“independent counsel” as contemplated by the law for the defendant is thrust into an unfamiliar atmosphere running
reason that he was station commander of the WPD at the through menacing police interrogation procedures where
time he assisted accused-appellant. X x x. the potentiality for compulsion, physical or psychological
is forcefully apparent.
This is error. As observed in People v. Bandula (232 SCRA
566 [1994]), the independent counsel required by Article However, the rule is not intended as a deterrent to the
III, Section 12(1) cannot be special counsel, public or accused from confessing guilt if he voluntarily and
private prosecutor, municipal attorney, or counsel of the intelligently so desires but to protect the accused from
police whose interest is admittedly adverse to the admitting what he is coerced to admit although untrue .
accused. In this case, Atty. De los Reyes, as PC Captain (People v. Base, 329 SCRA 158, 169-171, March
and Station Commander of the WPD, was part of the 30, 2000, 1st Div. [Ynares-Santiago])
police force who could not be expected to have effectively
and scrupulously assisted accused-appellant in the 214. Is the choice of a lawyer by a person under
investigation. To allow such a happenstance would custodial investigation who cannot afford the services of
render illusory the protection given to the suspect during a counsel exclusive as to preclude other equally
custodial investigation. (People v. Obrero, 332 SCRA competent and independent attorneys from handling his
190, 220 – 208, May 17, 2000, 2nd Div. [Mendoza]) defense?

213. What are the requirements for an extra-judicial Held: It must be remembered in this regard that while
confession of an accused to be admissible in evidence? the right to counsel is immutable, the option to secure the
services of counsel de parte is not absolute. Indeed –
Held: 1. In jurisprudence, no confession can be
admitted in evidence unless it is given: The phrase “competent and independent” and “preferably
of his own choice” were explicit details which were added
1) Freely and voluntarily, without compulsion, upon the persistence of human rights lawyers in the 1986
inducement or trickery; Constitutional Commission who pointed out cases where,
2) Knowingly based on an effective during the martial law period, the lawyers made available
communication to the individual under custodial to the detainee would be one appointed by the military
investigation of his constitutional rights; and and therefore beholden to the military. (Citing I Record of
3) Intelligently with full appreciation of its the Constitutional Commission 731-734; I Bernas, The
importance and comprehension of its consequences. Constitution of the Republic of the Philippines, 1987 1st
ed., p. 347)
Once admitted, the confession must inspire credibility or
be one which the normal experience of mankind can Xxx xxx xxx
accept as being within the realm of probability.
Withal, the word “preferably” under Section 12(1), Article
A confession meeting all the foregoing requisites 3 of the 1987 Constitution does not convey the message
constitutes evidence of a high order since it is supported that the choice of a lawyer by a person under
by the strong presumption that no person of normal mind investigation is exclusive as to preclude other equally
will knowingly, freely and deliberately confess that he is competent and independent attorneys from handling his
the perpetrator of a crime unless prompted by truth and defense. If the rule were otherwise, then, the tempo of
a custodial investigation will be solely in the hands of the admission was allegedly made to the arresting officer
accused who can impede, nay, obstruct the progress of during an “informal talk” at the police station after his
the interrogation by simply selecting a lawyer who for one arrest as a prime suspect in the rape and killing of x x x.
reason or another, is not available to protect his interest. The arresting policeman testified that the appellant
This absurd scenario could not have been contemplated admitted that he was with the victim on the evening of
by the framers of the charter. January 12, 1994, the probable time of the commission of
the crime and that he carried her on his shoulder but that
While the initial choice in cases where a person under he was too drunk to remember what subsequently
custodial investigation cannot afford the services of a happened. The arresting policeman admitted that he did
lawyer is naturally lodged in the police investigators, the not inform the appellant of his constitutional rights to
accused really has the final choice as he may reject the remain silent and to counsel. We note that the alleged
counsel chosen for him and ask for another one. A lawyer admission is incriminating because it places the accused
provided by the investigators is deemed engaged by the in the company of the victim at the time the crime was
accused where he never raised any objection against the probably committed.
former’s appointment during the course of the
investigation and the accused thereafter subscribes to the The exclusionary rule applies.
veracity of his statement before the swearing officer.
The accused was under arrest for the rape and killing of
Verily, to be an effective counsel “[a] lawyer need not x x x and any statement allegedly made by him pertaining
challenge all the questions being propounded to his client. to his possible complicity in the crime without prior
The presence of a lawyer is not intended to stop an notification of his constitutional rights is inadmissible in
accused from saying anything which might incriminate evidence. The policeman’s apparent attempt to
him but, rather, it was adopted in our Constitution to circumvent the rule by insisting that the admission was
preclude the slightest coercion as would lead the accused made during an “informal talk” prior to custodial
to admit something false (People v. Layuso, 175 SCRA 47 investigation prior is not tenable. The appellant was not
[1989]). The counsel, however, should never prevent an invited to the police station as part of a general inquiry
accused from freely and voluntarily telling the truth.” for any possible lead to the perpetrators of the crime
(People v. Base, 329 SCRA 158, 169-171, March under investigation. At the time the alleged admission
30, 2000, 1st Div. [Ynares-Santiago]) was made the appellant was in custody and had been
arrested as the prime suspect in the rape and killing of x
215. Should courts be allowed to distinguish between x x. The exclusionary rule presumes that the alleged
preliminary questioning and custodial investigation proper admission was coerced, the very evil the rule stands to
when applying the exclusionary rule? avoid. Supportive of such presumption is the absence of
a written extra-judicial confession to that effect and the
Held: The exclusionary rule sprang from a recognition appellant’s denial in court of the alleged oral admission.
that police interrogatory procedures lay fertile grounds for The alleged admission should be struck down as
coercion, physical and psychological, of the suspect to inadmissible. (People v. Bravo, 318 SCRA 812, Nov.
admit responsibility for the crime under investigation. It 22, 1999, En Banc [Gonzaga-Reyes])
was not intended as a deterrent to the accused from
confessing guilt, if he voluntarily and intelligently so 216. Explain the procedure for out-of-court
desires but to protect the accused from admitting what identification of suspects and the test to determine the
he is coerced to admit although untrue . Law enforcement admissibility of such identification.
agencies are required to effectively communicate the
rights of a person under investigation and to insure that Held: 1. In People v. Teehankee, Jr. (249 SCRA 54,
it is fully understood. Any measure short of this October 6, 1995), the Court x x x explained the procedure
requirement is considered a denial of such right. Courts for out-of-court identification and the test to determine
are not allowed to distinguish between preliminary the admissibility of such identification. It listed the
questioning and custodial investigation proper when following ways of identifying the suspects during custodial
applying the exclusionary rule. Any information or investigation: show-up, mug shots and line-ups. The
admission given by a person while in custody which may Court there ruled:
appear harmless or innocuous at the time without the
competent assistance of an independent counsel should “x x x. Out-of-court identification is conducted by the
be struck down as inadmissible. It has been held, police in various ways. It is done thru show-ups where
however, that an admission made to news reporters or to the suspect alone is brought face to face with the witness
a confidant of the accused is not covered by the for identification. It is done thru mug shots where
exclusionary rule. photographs are shown to the witness to identify the
suspect. It is also done thru line ups where a witness
The admission allegedly made by the appellant is not in identifies the suspect from a group of persons lined up for
the form of a written extra-judicial confession; the the purpose. Since corruption of out-of-court
identification contaminates the integrity of in court in evidence since his urine sample was derived in effect
identification during the trial of the case, courts have from an uncounselled extra-judicial confession. Petitioner
fashioned out rules to assure its fairness and its claims that the taking of his urine sample allegedly
compliance with the requirements of constitutional due violates Article III, Section 2 of the Constitution x x x.”
process. In resolving the admissibility of and relying on Should his contentions be upheld?
out-of- court identification of suspects, courts have
adopted the totality of circumstances test where they Held: We are not persuaded. The right to counsel begins
consider the following factors, viz: (1) the witness’ from the time a person is taken into custody and placed
opportunity to view the criminal at the time of the crime; under investigation for the commission of a crime, i.e.,
(2) the witness’ degree of attention at that time; (3) the when the investigating officer starts to ask questions to
accuracy of any prior description given by the witness; (4) elicit information and/or confession or admissions from
the level of certainty demonstrated by the witness at the the accused. Such right is guaranteed by the Constitution
identification; (5) the length of time between the crime and cannot be waived except in writing and in the
and the identification; and (6) the suggestiveness of the presence of counsel. However, what the Constitution
identification procedure.” (Ibid., p. 95) (People v. prohibits is the use of physical or moral compulsion to
Timon, 281 SCRA 577, Nov. 12, 1997 extort communication from the accused, but not an
[Panganiban]) inclusion of his body in evidence, when it may be material.
In fact, an accused may validly be compelled to be
2. x x x. The totality test has been fashioned precisely photographed or measured, or his garments or shoes
to assure fairness as well as compliance with removed or replaced, or to move his body to enable the
constitutional requirements of due process in regard to foregoing things to be done, without running afoul of the
out-of-court identification. These cited factors must be proscription against testimonial compulsion . The
considered to prevent contamination of the integrity of in- situation in the case at bar falls within the exemption
court identifications better. (People v. Gamer, 326 under the freedom from testimonial compulsion since
SCRA 660, Feb. 29, 2000, 2nd Div. [Quisumbing]) what was sought to be examined came from the body of
the accused. This was a mechanical act the accused was
217. Does the prohibition for custodial investigation made to undergo which was not meant to unearth
conducted without the assistance of counsel extend to a undisclosed facts but to ascertain physical attributes
person in a police line-up? Consequently, is the determinable by simple observation. In fact, the record
identification by private complainant of accused who was shows that petitioner and his co-accused were not
not assisted by counsel during police line-up admissible in compelled to give samples of their urine but they in fact
evidence? voluntarily gave the same when they were requested to
undergo a drug test. (Gutang v. People, 335 SCRA
Held: The prohibition x x x does not extend to a person 479, July 11, 2000, 2nd Div. [De Leon])
in a police line-up because that stage of an investigation
is not yet a part of custodial investigation. It has been
repeatedly held that custodial investigation commences The Right to Bail
when a person is taken into custody and is singled out as
a suspect in the commission of the crime under 219. In bail application where the accused is charged
investigation and the police officers begin to ask questions with a capital offense, will it be proper for the judge to
on the suspect’s participation therein and which tend to grant bail without conducting hearing if the prosecutor
elicit an admission. The stage of an investigation wherein interposes no objection to such application? Why?
a person is asked to stand in a police line-up has been
held to be outside the mantle of protection of the right to Held: Jurisprudence is replete with decisions compelling
counsel because it involves a general inquiry into an judges to conduct the required hearings in bail
unsolved crime and is purely investigatory in nature. It applications, in which the accused stands charged with a
has also been held that an uncounseled identification at capital offense. The absence of objection from the
the police line-up does not preclude the admissibility of prosecution is never a basis for the grant of bail in such
an in-court identification. The identification made by the cases, for the judge has no right to presume that the
private complainant in the police line-up pointing to prosecutor knows what he is doing on account of
Pavillare as one of his abductors is admissible in evidence familiarity with the case. "Said reasoning is tantamount
although the accused-appellant was not assisted by to ceding to the prosecutor the duty of exercising judicial
counsel. X x x (People v. Pavillare, 329 SCRA 684, discretion to determine whether the guilt of the accused
694-695, April 5, 2000, En Banc [Per Curiam]) is strong. Judicial discretion is the domain of the judge
before whom the petition for provisional liberty will be
218. Petitioner in a case “x x x posits the theory that decided. The mandated duty to exercise discretion has
since he had no counsel during the custodial investigation never been reposed upon the prosecutor."
when his urine sample was taken and chemically
examined, Exhibits “L” and “M,” x x x are also inadmissible
Imposed in Baylon v. Sison (243 SCRA 284, April 6, 1995) otherwise, deprived of liberty. The purpose of bail is to
was this mandatory duty to conduct a hearing despite the secure one’s release and it would be incongruous to grant
prosecution's refusal to adduce evidence in opposition to bail to one who is free. Petitioners’ Compliance and
the application to grant and fix bail. (Joselito V. Motion x x x came short of an unconditional submission
Narciso v. Flor Marie Sta. Romana-Cruz, G.R. No. to respondent court’s lawful order and to its jurisdiction.
134504, March 17, 2000, 3rd Div. [Panganiban])
The trial court correctly denied petitioners’ motion that
220. What are the duties of the judge in cases of bail they be allowed provisional liberty after their conviction,
applications where the accused is charged with capital under their respective bail bonds. Apart from the fact that
offense? they were at large, Section 5, Rule 114 of the Rules of
Court, as amended by Supreme Court Administrative
Held: Basco v. Rapatalo (269 SCRA 220, March 5, 1997) Circular 12-94, provides that:
enunciated the following duties of the trial judge in such
petition for bail: Xxx

1) Notify the prosecutor of the hearing of the The Court, in its discretion, may allow the accused to
application for bail or require him to submit his continue on provisional liberty under the same bail bond
recommendation; during the period to appeal subject to the consent of the
2) Conduct a hearing of the application for bail bondsman.
regardless of whether or not the prosecution refuses to
present evidence to show that the guilt of the accused is The bail bond that the accused previously posted can only
strong for the purpose of enabling the court to exercise be used during the 15-day period to appeal (Rule 122)
its sound discretion; and not during the entire period of appeal. This is
3) Decide whether the evidence of guilt of the consistent with Section 2(a) of Rule 114 which provides
accused is strong based on the summary of evidence of that the bail “shall be effective upon approval and remain
the prosecution; in force at all stages of the case, unless sooner cancelled,
4) If the guilt of the accused is not strong, until the promulgation of the judgment of the Regional
discharge the accused upon the approval of the bailbond. Trial Court, irrespective of whether the case was originally
Otherwise, petition should be denied. filed in or appealed to it.” This amendment, introduced
by SC Administrative Circular 12-94 is a departure from
The Court added: "The above-enumerated procedure the old rules which then provided that bail shall be
should now leave no room for doubt as to the duties of effective and remain in force at all stages of the case until
the trial judge in cases of bail applications. So basic and its full determination, and thus even during the period of
fundamental is it to conduct a hearing in connection with appeal. Moreover, under the present rule, for the accused
the grant of bail in the proper cases that it would amount to continue his provisional liberty on the same bail bond
to judicial apostasy for any member of the judiciary to during the period to appeal, consent of the bondsman is
disclaim knowledge or awareness thereof." necessary. From the record, it appears that the
bondsman x x x filed a motion in the trial court x x x for
Additionally, the court's grant or refusal of bail must the cancellation of petitioners’ bail bond for the latter’s
contain a summary of the evidence for the prosecution, failure to renew the same upon its expiration. Obtaining
on the basis of which should be formulated the judge's the consent of the bondsman was, thus, foreclosed.
own conclusion on whether such evidence is strong (Maguddatu v. Court of Appeals, 326 SCRA 362,
enough to indicate the guilt of the accused. The summary Feb. 23, 2000, 1st Div. [Kapunan])
thereof is considered an aspect of procedural due process
for both the prosecution and the defense; its absence will 223. Is a condition in an application for bail that
invalidate the grant or the denial of the application for accused be first arraigned before he could be granted bail
bail. (Joselito V. Narciso v. Flor Marie Sta. Romana- valid?
Cruz, G.R. No. 134504, March 17, 2000, 3rd Div.
[Panganiban]) Held: In requiring that petitioner be first arraigned
before he could be granted bail, the trial court
221. Should the accused who remained at large after apprehended that if petitioner were released on bail he
their conviction be allowed provisional liberty? Can the could, by being absent, prevent his early arraignment and
bail bond that the accused previously posted be used thereby delay his trial until the complainants got tired and
during the entire period of appeal? lost interest in their cases. Hence, to ensure his presence
at the arraignment, approval of petitioner’s bail bonds
Held: Despite an order of arrest from the trial court and should be deferred until he could be arraigned. After that,
two warnings from the Court of Appeals, petitioners had even if petitioner does not appear, trial can proceed as
remained at large. It is axiomatic that for one to be long as he is notified of the date of the hearing and his
entitled to bail, he should be in the custody of the law, or failure to appear is unjustified, since under Art. III, Sec.
14(2) of the Constitution, trial in absencia is authorized. 3) To inform the court of the facts alleged, so
This seems to be the theory of the trial court in its x x x that it may decide whether they are sufficient in law to
order conditioning the grant of bail to petitioner on his support a conviction, if one should be had.
arraignment.
It is thus imperative that the Information filed with the
This theory is mistaken. In the first place x x x in cases trial court be complete – to the end that the accused may
where it is authorized, bail should be granted before suitably prepare for his defense. Corollary to this, an
arraignment, otherwise the accused may be precluded indictment must fully state the elements of the specific
from filing a motion to quash. For if the information is offense alleged to have been committed as it is the recital
quashed and the case is dismissed, there would then be of the essentials of a crime which delineates the nature
no need for the arraignment of the accused. In the and cause of accusation against the accused.
second place, the trial court could ensure the presence of
petitioner at the arraignment precisely by granting bail Xxx
and ordering his presence at any stage of the
proceedings, such as arraignment. Under Rule 114, Sec. In the case under scrutiny, the information does not
2(b) of the Rules on Criminal Procedure, one of the allege the minority of the victim x x x although the same
conditions of bail is that “the accused shall appear before was proven during the trial x x x. The omission is not
the proper court whenever so required by the court or merely formal in nature since doctrinally, an accused
these Rules,” while under Rule 116, Sec. 1(b) the cannot be held liable for more than what he is indicted
presence of the accused at the arraignment is required. for. It matters not how conclusive and convincing the
evidence of guilt may be, but an accused cannot be
On the other hand, to condition the grant of bail to an convicted of any offense, not charged in the Complaint or
accused on his arraignment would be to place him in a Information on which he is tried or therein necessarily
position where he has to choose between (1) filing a included. He has a right to be informed of the nature of
motion to quash and thus delay his release on bail the offense with which he is charged before he is put on
because until his motion to quash can be resolved, his trial. To convict an accused of an offense higher than that
arraignment cannot be held, and (2) foregoing the filing charged in the Complaint or Information on which he is
of a motion to quash so that he can be arraigned at once tried would constitute unauthorized denial of that right.
and thereafter be released on bail. These scenarios (People v. Bayya, 327 SCRA 771, March 10, 2000,
certainly undermine the accused’s constitutional right not En Banc [Purisima])
to be put on trial except upon valid complaint or
information sufficient to charge him with a crime and his
right to bail. (Lavides v. CA, 324 SCRA 321, Feb. 1, The Right to a Fair Trial
2000, 2 nd
Div. [Mendoza])
225. What is the purpose of the rule barring trial or
The Right to be Informed of the Nature and Cause sentence of an insane person? What are the reasons
of Accusation against the Accused underlying it?

224. What are the objectives of the right to be Held: The rule barring trial or sentence of an insane
informed of the nature and cause of accusations against person is for the protection of the accused, rather than of
the accused? the public. It has been held that it is inhuman to require
an accused disabled by God to make a just defense for
Held: Instructive in this regard is Section 6, Rule 110 of his life or liberty. To put a legally incompetent person on
the Rules of Court x x x. trial or to convict and sentence him is a violation of the
constitutional rights to a fair trial; and this has several
The purpose of the above-quoted rule is to inform the reasons underlying it. For one, the accuracy of the
accused of the nature and cause of the accusation against proceedings may not be assured, as an incompetent
him, a right guaranteed by no less than the fundamental defendant who cannot comprehend the proceedings may
law of the land (Article III, Section 14[2], 1987 not appreciate what information is relevant to the proof
Constitution). Elaborating on the defendant’s right to be of his innocence. Moreover, he is not in a position to
informed, the Court held in Pecho v. People (262 SCRA exercise many of the rights afforded a defendant in a
518) that the objectives of this right are: criminal case, e.g., the right to effectively consult with
counsel, the right to testify in his own behalf, and the
1) To furnish the accused with such a right to confront opposing witnesses, which rights are
description of the charge against him as will enable him safeguards for the accuracy of the trial result. Second,
to make the defense; the fairness of the proceedings may be questioned, as
2) To avail himself of his conviction or acquittal there are certain basic decisions in the course of a criminal
for protection against a further prosecution for the same proceeding which a defendant is expected to make for
cause; and himself, and one of these is his plea. Third, the dignity of
the proceedings may be disrupted, for an incompetent
defendant is likely to conduct himself in the courtroom in This is not the first time the issue of trial by
a manner which may destroy the decorum of the court. publicity has been raised in this Court to stop the trials or
Even if the defendant remains passive, his lack of annul convictions in high profile criminal cases. In People
comprehension fundamentally impairs the functioning of v. Teehankee, Jr. (249 SCRA 54 [1995]) , later reiterated
the trial process. A criminal proceeding is essentially an in the case of Larranaga v. Court of Appeals, et al. (287
adversarial proceeding. If the defendant is not a SCRA 581 at pp. 596-597 [1998]), we laid down the
conscious and intelligent participant, the adjudication doctrine that:
loses its character as a reasoned interaction between an
individual and his community and becomes and invective “We cannot sustain appellant’s claim that he was denied
against an insensible object. Fourth, it is important that the right to impartial trial due to prejudicial publicity. It
the defendant knows why he is being punished, a is true that the print and broadcast media gave the case
comprehension which is greatly dependent upon his at bar pervasive publicity, just like all high profile and high
understanding of what occurs at trial. An incompetent stake criminal trials. Then and now, we rule that the right
defendant may not realize the moral reprehensibility of of an accused to a fair trial is not incompatible to a free
his conduct. The societal goal of institutionalized press. To be sure, responsible reporting enhances an
retribution may be frustrated when the force of the state accused’s right to a fair trial for, as well pointed out, a
is brought to bear against one who cannot comprehend responsible press has always been regarded as the
its significance. (People v. Estrada, 333 SCRA 699, handmaiden of effective judicial administration, especially
718-719, June 19, 2000, En Banc [Puno]) in the criminal field x x x. The press does not simply
publish information about trials but guards against the
miscarriage of justice by subjecting the police,
The Right to an Impartial Trial prosecutors, and judicial processes to extensive public
scrutiny and criticism.
226. What are the two principal legal and philosophical
schools of thought on how to deal with the rain of Pervasive publicity is not per se prejudicial to the right of
unrestrained publicity during the investigation and trial of an accused to fair trial. The mere fact that the trial of
high profile cases? appellant was given a day-to-day, gavel-to-gavel
coverage does not by itself prove that the publicity so
Held: There are two (2) principal legal and philosophical permeated the mind of the trial judge and impaired his
schools of thought on how to deal with the rain of impartiality. For one, it is impossible to seal the minds of
unrestrained publicity during the investigation and trial of members of the bench from pre-trial and other off-court
high profile cases. The British approach the problem with publicity of sensational criminal cases. The state of the
the presumption that publicity will prejudice a jury. Thus, art of our communication system brings news as they
English courts readily stay and stop criminal trials when happen straight to our breakfast tables and right to our
the right of an accused to fair trial suffers a threat . The bedrooms. These news form part of our everyday menu
American approach is different. US courts assume a of the facts and fictions of life. For another, our idea of a
skeptical approach about the potential effect of pervasive fair and impartial judge is not that of a hermit who is out
publicity on the right of an accused to a fair trial. They of touch with the world. We have not installed the jury
have developed different strains of tests to resolve this system whose members are overly protected from
issue, i.e., substantial probability of irreparable harm, publicity lest they lose their impartiality. x x x. Our judges
strong likelihood, clear and present danger, etc. (Estrada are learned in the law and trained to disregard off-court
v. Desierto, G.R. Nos. 146710-15, March 2, 2001, evidence and on-camera performances of parties to a
En Banc [Puno]) litigation. Their mere exposure to publications and
publicity stunts does not per se fatally infect their
227. Should the Ombudsman be stopped from impartiality.
conducting the investigation of the cases filed against
petitioner (former President) Estrada due to the barrage At best, appellant can only conjure possibility of prejudice
of prejudicial publicity on his guilt? on the part of the trial judge due to the barrage of
publicity that characterized the investigation and trial of
Held: Petitioner x x x contends that the respondent the case. In Martelino, et al. v. Alejandro, et al., we
Ombudsman should be stopped from conducting the rejected this standard of possibility of prejudice and
investigation of the cases filed against him due to the adopted the test of actual prejudice as we ruled that to
barrage of prejudicial publicity on his guilt. He submits warrant a finding of prejudicial publicity, there must be
that the respondent Ombudsman has developed bias and allegation and proof that the judges have been unduly
is all set to file the criminal cases in violation of his right influenced, not simply that they might be, by the barrage
to due process. of publicity. In the case at bar, the records do not show
that the trial judge developed actual bias against
Xxx appellant as a consequence of the extensive media
coverage of the pre-trial and trial of his case. The totality reasons as valid today as in centuries past, it must be
of circumstances of the case does not prove that the trial concluded that a presumption of openness inheres in the
judge acquired a fixed opinion as a result of prejudicial very nature of a criminal trial under this Nation’s system
publicity which is incapable of change even by evidence of justice, Cf., e.g., Levine v. United States, 362 US 610,
presented during the trial. Appellant has the burden to 4 L Ed 2d 989, 80 S Ct 1038.
prove this actual bias and he has not discharged the
burden.” (b) The freedoms of speech, press, and assembly,
expressly guaranteed by the First Amendment, share a
We expounded further on this doctrine in the common core purpose of assuring freedom of
subsequent case of Webb v. Hon. Raul de Leon, etc. (247 communication on matters relating to the functioning of
SCRA 652 [1995]) and its companion cases, viz.: government. In guaranteeing freedoms such as those of
speech and press, the First Amendment can be read as
“Again, petitioners raise the effect of prejudicial publicity protecting the right of everyone to attend trials so as give
on their right to due process while undergoing preliminary meaning to those explicit guarantees; the First
investigation. We find no procedural impediment to its Amendment right to receive information and ideas means,
early invocation considering the substantial risk to their in the context of trials, that the guarantees of speech and
liberty whole undergoing a preliminary investigation. press, standing alone, prohibit government from
summarily closing courtroom doors which had long been
Xxx open to the public at the time the First Amendment was
adopted. Moreover, the right of assembly is also relevant,
The democratic settings, media coverage of trials of having been regarded not only as an independent right
sensational cases cannot be avoided and oftentimes, its but also as a catalyst to augment the free exercise of the
excessiveness has been aggravated by kinetic other First Amendment rights with which it was
developments in the telecommunications industry. For deliberately linked by the draftsmen. A trial courtroom is
sure, few cases can match the high volume and high a public place where the people generally – and
velocity of publicity that attended the preliminary representatives of the media – have a right to be present,
investigation of the case at bar. Our daily diet of facts and where their presence historically has been thought to
and fiction about the case continues unabated even enhance the integrity and quality of what takes place.
today. Commentators still bombard the public with views
not too many of which are sober and sublime. Indeed, (c) Even though the Constitution contains no provision
even the principal actors in the case – the NBI, the which by its terms guarantees to the public the right to
respondents, their lawyers and their sympathizers – have attend criminal trials, various fundamental rights, not
participated in this media blitz. The possibility of media expressly guaranteed, have been recognized as
abuses and their threat to a fair trial notwithstanding, indispensable to the enjoyment of enumerated rights.
criminal trials cannot be completely closed to the press The right to attend criminal trial is implicit in the
and public. In the seminal case of Richmond Newspapers, guarantees of the First Amendment: without the freedom
Inc. v. Virginia, it was wisely held: to attend such trials, which people have exercised for
centuries, important aspects of freedom of speech and of
‘x x x the press could be eviscerated.’

(a) The historical evidence of the evolution of the criminal Be that as it may, we recognize that pervasive and
trial in Anglo-American justice demonstrates conclusively prejudicial publicity under certain circumstances can
that at the time this Nation’s organic laws were adopted, deprive an accused of his due process right to fair trial.
criminal trials both here and in England had long been Thus, in Martelino, et al. v. Alejandro, et al., we held that
presumptively open, thus giving assurance that the to warrant a finding of prejudicial publicity there must be
proceedings were conducted fairly to all concerned and allegation and proof that the judges have been unduly
discouraging perjury, the misconduct of participants, or influenced, not simply that they might be, by the barrage
decisions based on secret bias or partiality. In addition, of publicity. In the case at bar, we find nothing in the
the significant community therapeutic value of public records that will prove that the tone and content of the
trials was recognized: when a shocking crime occurs, a publicity that attended the investigation of petitioners
community reaction of outrage and public protest often fatally infected the fairness and impartiality of the DOJ
follows, and thereafter the open processes of justice serve Panel. Petitioners cannot just rely on the subliminal
an important prophylactic purpose, providing an outlet for effects of publicity on the sense of fairness of the DOJ
community concern, hostility, and emotion. To work Panel, for these are basically unbeknown and beyond
effectively, it is important that society’s criminal process knowing. To be sure, the DOJ Panel is composed of an
‘satisfy the appearance of justice,’ Offutt v. United States, Assistant Chief State Prosecutor and Senior State
348 US 11, 14, 99 L Ed 11, 75 S Ct 11, which can best be Prosecutors. Their long experience in criminal
provided by allowing people to observe such process. investigation is a factor to consider in determining
From this unbroken, uncontradicted history, supported by whether they can easily be blinded by the klieg lights of
publicity. Indeed, their 26-page Resolution carries no Held: [W]e reject respondent court’s ruling that the
indubitable indicia of bias for it does not appear that they grant of section 5 immunity must be strictly construed
considered any extra-record evidence except evidence against the petitioners. It simplistically characterized the
properly adduced by the parties. The length of time the grant as a special privilege, as if it was gifted by the
investigation was conducted despite it summary nature government, ex gratia. In taking this posture, it misread
and the generosity with which they accommodated the the raison d’ etre and the long pedigree of the right
discovery motions of petitioners speak well of their against self-incrimination vis-à-vis immunity statutes.
fairness. At no instance, we note, did petitioners seek the
disqualification of any member of the DOJ Panel on the The days of inquisition brought about the most despicable
ground of bias resulting from their bombardment of abuses against human rights. Not the least of these
prejudicial publicity.” abuses is the expert use of coerced confessions to send
to the guillotine even the guiltless. To guard against the
Applying the above ruling, we hold that there is not recurrence of this totalitarian method, the right against
enough evidence to warrant this Court to enjoin the self-incrimination was ensconced in the fundamental laws
preliminary investigation of the petitioner by the of all civilized countries. Over the years, however, came
respondent Ombudsman. Petitioner needs to offer more the need to assist government in its task of containing
than hostile headlines to discharge his burden of proof. crime for peace and order is a necessary matrix of public
He needs to show more than weighty social science welfare. To accommodate the need, the right against
evidence to successfully prove the impaired capacity of a self-incrimination was stripped of its absoluteness.
judge to render a bias-free decision. Well to note, the Immunity statutes in varying shapes were enacted which
cases against the petitioner are still undergoing would allow government to compel a witness to testify
preliminary investigation by a special panel of prosecutors despite his plea of the right against self-incrimination. To
in the office of the respondent Ombudsman. No insulate these statutes from the virus of
allegation whatsoever has been made by the petitioner unconstitutionality, a witness is given what has come to
that the minds of the members of this special panel have be known as transactional or a use-derivative-use
already been infected by bias because of the pervasive immunity x x x. Quite clearly, these immunity statutes
prejudicial publicity against him. Indeed, the special are not a bonanza from government. Those given the
panel has yet to come out with its findings and the Court privilege of immunity paid a high price for it – the
cannot second guess whether its recommendation will be surrender of their precious right to be silent. Our
unfavorable to the petitioner. (Estrada v. Desierto, hierarchy of values demands that the right against self-
G.R. Nos. 146710-15, March 2, 2001, En Banc incrimination and the right to be silent should be accorded
[Puno]) greater respect and protection. Laws that tend to erode
the force of these preeminent rights must necessarily be
given a liberal interpretation in favor of the individual.
The Right against Self-Incrimination The government has a right to solve crimes but it must
do it, rightly. (Mapa, Jr. v. Sandiganbayan, 231
228. Discuss the types of immunity statutes. Which SCRA 783, 805-806, April 26, 1994, En Banc
has broader scope of protection? [Puno])

Held: Our immunity statutes are of American origin. In


the United States, there are two types of statutory 230. May the right against self-incrimination be validly
immunity granted to a witness. They are the invoked during inquiry in aid of legislation?
transactional immunity and the use-and-derivative-use
immunity. Transactional immunity is broader in the scope Held: Now to another matter. It has been held
of its protection. By its grant, a witness can no longer be that “a congressional committee’s right to inquire is
prosecuted for any offense whatsoever arising out of the ‘subject to all relevant limitations placed by the
act or transaction. In contrast, by the grant of use-and- Constitution on governmental action,’ including ‘the
derivative-use immunity, a witness is only assured that relevant limitations of the Bill of Rights’.” (Maurice A.
his or her particular testimony and evidence derived from Hutcheson v. U.S., 369 US 599)
it will not be used against him or her in a subsequent
prosecution. (Mapa, Jr. v. Sandiganbayan, 231 In another case –
SCRA 783, 797-798, April 26, 1994, En Banc
[Puno]) “x x x the mere semblance of legislative purpose would
not justify an inquiry in the face of the Bill of Rights. The
229. Is the grant of immunity to an accused willing to critical element is the existence of, and the weight to be
testify for the government a special privilege and ascribed to, the interest of the Congress in demanding
therefore must be strictly construed against the accused? disclosures from an unwilling witness. We cannot simply
assume, however, that every congressional investigation
is justified by a public need that over-balances any private
rights affected. To do so would be to abdicate the pursued, would be violative of the principle of separation
responsibility placed by the Constitution upon the of powers between the legislative and the judicial
judiciary to insure that the Congress does not unjustifiably departments of government, ordained by the
encroach upon an individual’s right to privacy nor abridge Constitution. (Bengzon, Jr. v. Senate Blue Ribbon
his liberty of speech, press, religion or assembly.” Committee, 203 SCRA 767, Nov. 20, 1991, En Banc
(Watkins v. US, 354 USS 178 citing US v. Rumely, 345 US [Padilla])
41)
The Right against Double Jeopardy
One of the basic rights guaranteed by the
Constitution to an individual is the right against self- 231. Discuss the two kinds of double jeopardy.
incrimination (Sec. 17, Art. III of the Constitution). This
right construed as the right to remain completely silent Held: Our Bill of Rights deals with two (2) kinds of double
may be availed of by the accused in a criminal case; but jeopardy. The first sentence of Clause 20, Section 1,
it may be invoked by other witnesses only as questions Article III of the Constitution ordains that “no person shall
are asked of them. be twice put in jeopardy of punishment for the same
offense.” The second sentence of said clause provides
This distinction is enunciated by the Court in that “if an act is punishable by a law and an ordinance,
Romeo Chavez v. The Honorable Court of Appeals, et al. conviction or acquittal under either shall constitute a bar
(G.R. No. L-29169, August 19, 1968, 24 SCRA 663) thus to another prosecution for the same act.” Thus, the first
– sentence prohibits double jeopardy of punishment for the
same offense whereas, the second contemplates double
“Petitioner, as accused, occupies a different tier of jeopardy of punishment for the same act. Under the first
protection from an ordinary witness. Whereas an sentence, one may be twice put in jeopardy of
ordinary witness may be compelled to take the witness punishment of the same act, provided that he is charged
stand and claim the privilege as each question requiring with different offenses, or the offense charged in one case
an incriminating answer is shot at him, an accused may is not included in, or does not include, the crime charged
altogether refuse to take the witness stand and refuse to in the other case. The second sentence applies, even if
answer any and all questions.” the offense charged are not the same, owing to the fact
that one constitutes a violation of an ordinance and the
Moreover, this right of the accused is extended to other a violation of statute. If the two charges are based
respondents in administrative investigations but only if on one and the same act, conviction or acquittal under
they partake of the nature of a criminal proceeding or either the law or the ordinance shall bar a prosecution
analogous to a criminal proceeding. In Galman v. under the other. Incidentally, such conviction or acquittal
Pamaran (G.R. Nos. 71208-09, August 30, 1985, 138 is not indispensable to sustain the plea of double jeopardy
SCRA 294), the Court reiterated the doctrine in Cabal v. of punishment or the same offense. So long as jeopardy
Kapunan (6 SCRA 1059) to illustrate the right of witnesses has been attached under one of the informations charging
to invoke the right against self-incrimination not only in said offense, the defense may be availed of in the other
criminal proceedings but also in all other types of suit. case involving the same offense, even if there has been
neither conviction nor acquittal in either case.
It was held that:
Elsewhere stated, where the offense charged are
“We did not therein state that since he is not an accused penalized either by different sections of the same statute
and the case is not a criminal case, Cabal cannot refuse or by different statutes, the important inquiry relates to
to take the witness stand and testify, and that he can the identity of offenses charged. The constitutional
invoke his right against self-incrimination only when a protection against double jeopardy is available only where
question which tends to elicit an answer that will an identity is shown to exist between the earlier and the
incriminate him is propounded to him. Clearly then, it is subsequent offenses charged. The question of identity or
not the character of the suit involved but the nature of lack of identity of offenses is addressed by examining the
the proceedings that controls. The privilege has essential elements of each of the two offenses charged,
consistently been held to extend to all proceedings as such elements are set out in the respective legislative
sanctioned by law and to all cases in which punishment is definitions of the offenses involved. (People v.
sought to be visited upon a witness, whether a party or Quijada, 259 SCRA 191, July 24, 1996)
not.”
232. What must be proved to substantiate a claim of
We do not here modify these doctrines. If we double jeopardy? When may legal jeopardy attach?
presently rule that petitioners may not be compelled by
the respondent Committee to appear, testify and produce Held: To substantiate a claim of double jeopardy, the
evidence before it, it is only because we hold that the following must be proven:
questioned inquiry is not in aid of legislation and, if
(1) A first jeopardy must have attached prior to the Held: In the cases at bar, the order of
second; (2) the first jeopardy must have been validly dismissal based on a violation of the right to speedy trial
terminated; (3) the second jeopardy must be for the same was made upon motion by counsel for petitioner before
offense, or the second offense includes or is necessarily the trial court. It was made at the instance of the
included in the offense charged in the first information, or accused before the trial court, and with his express
is an attempt to commit the same or is a frustration consent. Generally, the dismissal of a criminal case
thereof. resulting in acquittal made with the express consent of
the accused or upon his own motion will not place the
Legal jeopardy attaches only: (1) upon a valid indictment; accused in double jeopardy. However, this rule admits
(b) before a competent court; (c) after arraignment; (d) of two exceptions, namely: insufficiency of evidence and
when a valid plea has been entered; and (e) the case was denial of the right to speedy trial (People v. Bans, 239
dismissed or otherwise terminated without the express SCRA 48, 55 [1994]). Double jeopardy may attach
consent of the accused. (Cuison v. CA, 289 SCRA 159, when the proceedings have been prolonged
April 15, 1998 [Panganiban]) unreasonably, in violation of the accused’s right to
speedy trial (Commission on Elections v. Court of
233. In its decision in a criminal case, the Judge Appeals, 229 SCRA 501, 507 [1994]). (Almario v.
promulgated only the civil aspect of the case, but not the Court of Appeals, 355 SCRA 1, Mar. 22, 2001, 2nd
criminal. Will the promulgation of the criminal aspect Div. [Quisumbing]
later constitute double jeopardy?
235. If the criminal case was dismissed predicated on
Held: Petitioner contends that "the promulgation by the right of the accused to speedy trial, but later the trial
Judge Ramos on April 4, 1995 of the Respondent Court's court reconsidered its decision and allowed the case to be
decision of June 30, 1991 by reading its dispositive reinstated as it noted that the delay in the trial was due
portion has effectively terminated the criminal cases to circumstances beyond the control of the parties and of
against the petitioner x x x." In other words, petitioner the trial court, i.e., the presiding judge was promoted to
claims that the first jeopardy attached at that point. the Court of Appeals, and his successor as trial judge was
not immediately appointed, nor another judge detailed to
The Court is not persuaded. As a rule, a criminal his sala, is there violation of the accused’s right against
prosecution includes a civil action for the recovery of double jeopardy?
indemnity. Hence, a decision in such case disposes of
both the criminal as well as the civil liabilities of an Held: Here we must inquire whether there was
accused. Here, trial court promulgated only the civil unreasonable delay in the conduct of the trial so that
aspect of the case, but not the criminal. violation of the right to speedy trial of the accused x x x
resulted. For it must be recalled that in the application of
[T]he promulgation of the CA Decision was not complete. the constitutional guaranty of the right to speedy
In fact and in truth, the promulgation was not merely disposition of cases, particular regard must also be taken
incomplete; it was also void. In excess of its jurisdiction, of the facts and circumstances peculiar to each case
the trial judge rendered a substantially incomplete (Socrates v. Sandiganbayan, 253 SCRA 773, 788 [1996]).
promulgation on April 4, 1995, and he repeated his Both the trial court and the appellate court noted that
mistake in his April 12, 1996 Order. We emphasize that after pre-trial of petitioner’s case was terminated x x x
grave abuse of discretion rendered the aforementioned continuous trial was set x x x. The scheduled hearings,
act of the trial court void. Since the criminal cases have however, were cancelled when the presiding judge was
not yet been terminated, the first jeopardy has not yet promoted to the Court of Appeals, and his successor as
attached. Hence, double jeopardy cannot prosper as a trial judge was not immediately appointed, nor another
defense. judge detailed to his sala.

We must stress that Respondent Court's questioned Xxx


Decision did not modify or amend its July 30, 1991
Decision. It merely ordered the promulgation of the As observed by respondent appellate court, delay
judgment of conviction and the full execution of the in the trial was due to circumstances beyond the control
penalty it had earlier imposed on petitioner. (Cuison v. of the parties and of the trial court. X x x. Thus, after a
CA, 289 SCRA 159, April 15, 1998 [Panganiban]) closer analysis of these successive events, the trial court
realized that the dates of the hearings were transferred
for valid grounds. Hence, the trial court set aside its initial
234. What are the exceptions to the rule that the order and reinstated the cases against petitioner, which
dismissal of a criminal case resulting in acquittal made order the appellate court later sustained.
with the express consent of the accused or upon his own
motion will not place the accused in double jeopardy? That there was no unreasonable delay of the
proceedings is apparent from the chronology of the
hearings with the reasons for their postponements or reasonable doubt. He submits that in finding him liable
transfers. X x x for indemnity and damages, the appellate court not only
placed his acquittal in suspicion, but also put him in
There being no oppressive delay in the “double jeopardy.”
proceedings, and no postponements unjustifiably sought,
we concur with the conclusion reached by the Court of Private respondents contend that while the trial court
Appeals that petitioner’s right to speedy trial had not been found that petitioner’s guilt had not been proven beyond
infringed. Where the right of the accused to speedy trial reasonable doubt, it did not state in clear and
had not been violated, there was no reason to support the unequivocal terms that petitioner was not recklessly
initial order of dismissal. imprudent or negligent. Hence, impliedly the trial court
acquitted him on reasonable doubt. Since civil liability is
It follows that petitioner cannot invoke the not extinguished in criminal cases, if the acquittal is
constitutional right against double jeopardy when that based on reasonable doubt, the Court of Appeals had to
order was reconsidered seasonably (People v. Leviste, review the findings of the trial court to determine if
255 SCRA 238, 249 [1996]). For as petitioner’s right to there was a basis for awarding indemnity and damages.
speedy trial was not transgressed, this exception to the
fifth element of double jeopardy – that the defendant was Preliminarily, petitioner’s claim that the decision of the
acquitted or convicted, or the case was dismissed or appellate court awarding indemnity placed him in double
otherwise terminated without the express consent of the jeopardy is misplaced. X x x. When a person is charged
accused – was not met. The trial court’s initial order of with an offense and the case is terminated either by
dismissal was upon motion of petitioner’s counsel, hence acquittal or conviction or in any manner without the
made with the express consent of petitioner. That being consent of the accused, the latter cannot again be
the case, despite the reconsideration of said order, double charged with the same or identical offense (Melo v.
jeopardy did not attach. As this Court had occasion to People, 85 Phil. 766, 768 [1950]). This is double
rule in People v. Tampal (244 SCRA 202), reiterated in jeopardy. For double jeopardy to exist, the following
People v. Leviste (Ibid.), where we overturned an order elements must be established: (1) a first jeopardy must
of dismissal by the trial court predicated on the right to have attached prior to the second; (2) the first jeopardy
speedy trial – must have terminated; and (3) the second jeopardy
must be for the same offense as the first (People v.
It is true that in an unbroken line of cases, we have held Bocar, 138 SCRA 166, 171 [1985]). In the instant case,
that the dismissal of cases on the ground of failure to petitioner had once been placed in jeopardy by the filing
prosecute is equivalent to an acquittal that would bar of Criminal Case No. 066 and the jeopardy was
further prosecution of the accused for the same offense. terminated by his discharge. The judgment of acquittal
It must be stressed, however, that these dismissals were became immediately final. Note, however, that what
predicated on the clear right of the accused to speedy was elevated to the Court of Appeals by private
trial. These cases are not applicable to the petition at respondents was the civil aspect of Criminal Case No.
bench considering that the right of the private 066. Petitioner was not charged anew in CA-G.R. CV
respondents to speedy trial has not been violated by the No. 19240 with a second criminal offense identical to the
State. For this reason, private respondents cannot invoke first offense. The records clearly show that no second
their right against double jeopardy. criminal offense was being imputed to petitioner on
appeal. In modifying the lower court’s judgment, the
Both the trial court and the Court of Appeals appellate court did not modify the judgment of acquittal.
were thus not in error when they allowed reinstatement Nor did it order the filing of a second criminal cases
of the cases against petitioner. (Almario v. Court of against petitioner for the same offense. Obviously,
Appeals, 355 SCRA 1, Mar. 22, 2001, 2nd Div. therefore, there was no second jeopardy to speak of.
[Quisumbing] Petitioner’s claim of having been placed in double
jeopardy is incorrect.
236. Is there double jeopardy when an accused was
acquitted in a criminal case for reckless imprudence but Our law recognizes two kinds of acquittal, with different
the civil aspect of the case was elevated to the Court of effects on the civil liability of the accused. First is an
Appeals and the latter found him liable for indemnity acquittal on the ground that the accused is not the
and damages? author of the act or omission complained of. This
instance closes the door to civil liability, for a person
Held: Petitioner opines that the Court of Appeals who has been found to be not the perpetrator of any act
should not have disturbed the findings of the trial court or omission cannot and can never be held liable for such
on the lack of negligence or reckless imprudence under or omission (Almeida, et al. v. Abaroa, 8 Phil. 178, 181
the guise of determining his civil liability. He argues that [1907]; other citations omitted.) There being no delict,
the trial court’s finding that he was neither imprudent civil liability ex delicto is out of the question, and the civil
nor negligent was the basis for his acquittal, and not action, if any, which may be instituted must be based on
grounds other than the delict complained of. This is the that the qualifying circumstances attached to it has been
situation contemplated in Rule 111 of the Rules of Court. established also beyond reasonable doubt as the
The second instance is an acquittal based on reasonable Constitution and judicial precedents require. (Misolas v.
doubt on the guilt of the accused. In this case, even if Panga, 181 SCRA 648, 659-660, Jan. 30, 1990, En
the guilt of the accused has not been satisfactorily Banc [Cortes])
established, he is not exempt from civil liability which
may be proved by preponderance of evidence only 238. What is an ex post facto law? Is R.A. No. 8249
(Manahan, Jr. v. Court of Appeals, 255 SCRA 202, 214 an ex post facto law?
[1996], citing Padilla v. Court of Appeals, 129 SCRA 558
[1984]). This is the situation contemplated in Article 29 Held: Ex post facto law, generally, prohibits
of the Civil Code, where the civil action for damages is retrospectivity of penal laws. R.A. 8249 is not a penal law.
“for the same act or omission.” Although the two It is a substantive law on jurisdiction which is not penal in
actions have different purposes, the matters discussed in character. Penal laws are those acts of the Legislature
the civil case are similar to those discussed in the which prohibit certain acts and establish penalties for their
criminal case. However, the judgment in the criminal violations; or those that define crimes, treat of their
proceeding cannot be read in evidence in the civil action nature, and provide for their punishment. R.A. 7975,
to establish any fact there determined, even though which amended P.D. 1606 as regards the
both actions involve the same act or omission (Almeida Sandiganbayan’s jurisdiction, its mode of appeal and
Chantangco and Lete v. Abaroa, supra note 13, at other procedural matters, has been declared by the Court
1061). The reason for this rule is that the parties are as not a penal law, but clearly a procedural statute, i.e.,
not the same and secondarily, different rules of evidence one which prescribes rules of procedure by which courts
are applicable. Hence, notwithstanding herein applying laws of all kinds can properly administer justice.
petitioner’s acquittal, the Court of Appeals in Not being a penal law, the retroactive application of R.A.
determining whether Article 29 applied, was not 8249 cannot be challenged as unconstitutional.
precluded from looking into the question of petitioner’s
negligence or reckless imprudence. (Manantan v. Petitioner’s and intervenors’ contention that their right to
Court of Appeals, 350 SCRA 387, Jan. 29, 2001, a two-tiered appeal which they acquired under R.A. 7975
2nd Div. [Quisumbing]) has been diluted by the enactment of R.A. 8249, is
incorrect. The same contention has already been rejected
by the court several times considering that the right to
appeal is not a natural right but statutory in nature that
The Right against Ex Post Facto Laws and Bills of can be regulated by law. The mode of procedure
Attainder provided for in the statutory right of appeal is not included
in the prohibition against ex post facto laws. R.A. 8249
237. What is a bill of attainder? Is P.D. 1866 a bill of pertains only to matters of procedure, and being merely
attainder? an amendatory statute it does not partake the nature of
an ex post facto law. It does not mete out a penalty and,
Held: [T]he Court, in People v. Ferrer (G.R. Nos. L- therefore, does not come within the prohibition.
32613-14, December 27, 1972, 48 SCRA 382), defined a Moreover, the law did not alter the rules of evidence or
bill of attainder as a legislative act which inflicts the mode of trial. It has been ruled that adjective statutes
punishment on individuals or members of a particular may be made applicable to actions pending and
group without a judicial trial. Essential to a bill of unresolved at the time of their passage.
attainder are a specification of certain individuals or a
group of individuals, the imposition of a punishment, At any rate, R.A. 8249 has preserved the accused’s right
penal or otherwise, and the lack of judicial trial. This last to appeal to the Supreme Court to review questions of
element, the total lack of court intervention in the finding law. On the removal of the intermediate review of facts,
of guilt and the determination of the actual penalty to be the Supreme Court still has the power of review to
imposed, is the most essential. P.D. No. 1866 does not determine if the presumption of innocence has been
possess the elements of a bill of attainder. It does not convincingly overcome. (Panfilo M. Lacson v. The
seek to inflict punishment without a judicial trial. Executive Secretary, et. al., G.R. No. 128096, Jan.
Nowhere in the measure is there a finding of guilt and an 20, 1999 [Martinez])
imposition of a corresponding punishment. What the
decree does is to define the offense and provide for the
penalty that may be imposed, specifying the qualifying ADMINISTRATIVE LAW
circumstances that would aggravate the offense. There
is no encroachment on the power of the court to 239. Describe the Administrative Code of 1987
determine after due hearing whether the prosecution has
proved beyond reasonable doubt that the offense of Held: The Code is a general law and “incorporates in a
illegal possession of firearms has been committed and unified document the major structural, functional and
procedural principles of governance (Third Whereas Answer: A government instrumentality refers to any
Clause, Administrative Code of 1987) and “embodies agency of the national government, not integrated within
changes in administrative structures and procedures the department framework, vested with special functions
designed to serve the people.” (Fourth Whereas Clause, or jurisdiction by law, endowed with some if not all
Administrative Code of 1987) The Code is divided into corporate powers, administering special funds, enjoying
seven (7) books. These books contain provisions on the operational autonomy, usually through a charter. The
organization, powers and general administration of term includes regulatory agencies, chartered institutions
departments, bureaus and offices under the executive and government-owned or controlled corporations. (Sec.
branch, the organization and functions of the 2[10], Introductory Provisions, Executive Order
Constitutional Commissions and other constitutional No. 292)
bodies, the rules on the national government budget, as
well as guidelines for the exercise by administrative 244. What is a regulatory agency?
agencies of quasi-legislative and quasi-judicial powers.
The Code covers both the internal administration, i.e., Answer: A regulatory agency refers to any agency
internal organization, personnel and recruitment, expressly vested with jurisdiction to regulate, administer
supervision and discipline, and the effects of the functions or adjudicate matters affecting substantial rights and
performed by administrative officials on private interest of private persons, the principal powers of which
individuals or parties outside government. (Ople v. are exercised by a collective body, such as a commission,
Torres, G.R. No. 127685, July 23, 1998 [Puno]) board or council. (Sec. 2[11], Introductory
Provisions, Executive Order No. 292)
240. What is administrative power?
245. What is a chartered institution?
Held: Administrative power is concerned with the work
of applying policies and enforcing orders as determined Answer: A chartered institution refers to any agency
by proper governmental organs. It enables the President organized or operating under a special charter, and
to fix a uniform standard of administrative efficiency and vested by law with functions relating to specific
check the official conduct of his agents. To this end, he constitutional policies or objectives. This term includes
can issue administrative orders, rules and regulations. state universities and colleges and the monetary authority
(Ople v. Torres, G.R. No. 127685, July 23, 1998 of the State. (Section 2[12], Introductory
[Puno]) Provisions, Executive Order No. 292)

241. What is an administrative order? 246. When is a government-owned or controlled


corporation deemed to be performing proprietary
Held: An administrative order is an ordinance issued by function? When is it deemed to be performing
the President which relates to specific aspects in the governmental function?
administrative operation of government. It must be in
harmony with the law and should be for the sole purpose Held: Government-owned or controlled corporations
of implementing the law and carrying out the legislative may perform governmental or proprietary functions or
policy. (Ople v. Torres, G.R. No. 127685, July 23, both, depending on the purpose for which they have been
1998 [Puno]) created. If the purpose is to obtain special corporate
benefits or earn pecuniary profit, the function is
242. What is the Government of the Republic of the proprietary. If it is in the interest of health, safety and for
Philippines? the advancement of public good and welfare, affecting
the public in general, the function is governmental.
Answer: The Government of the Republic of the Powers classified as “proprietary” are those intended for
Philippines refers to the corporate governmental entity private advantage and benefit. (Blaquera v. Alcala,
through which the functions of the government are 295 SCRA 366, 425, Sept. 11, 1998, En Banc
exercised throughout the Philippines, including, save as [Purisima])
the contrary appears from the context, the various arms
through which political authority is made effective in the 247. Does the petition for annulment of proclamation
Philippines, whether pertaining to the autonomous of a candidate merely involve the exercise by the
regions, the provincial, city, municipal or barangay COMELEC of its administrative power to review, revise
subdivisions or other forms of local government. (Sec. and reverse the actions of the board of canvassers and,
2[1], Introductory Provisions, Executive Order No. therefore, justifies non-observance of procedural due
292) process, or does it involve the exercise of the COMELEC's
quasi-judicial function?
243. What is a government instrumentality? What are
included in the term government instrumentality? Held: Taking cognizance of private respondent's
petitions for annulment of petitioner's proclamation,
COMELEC was not merely performing an administrative In cases where the doctrine of primary jurisdiction is
function. The administrative powers of the COMELEC clearly applicable, the court cannot arrogate unto itself
include the power to determine the number and location the authority to resolve a controversy, the jurisdiction
of polling places, appoint election officials and inspectors, over which is lodged with an administrative body of
conduct registration of voters, deputize law enforcement special competence. (Villaflor v. CA, 280 SCRA 287)
agencies and governmental instrumentalities to ensure
free, orderly, honest, peaceful and credible elections, 249. Discuss the Doctrine of Exhaustion of
register political parties, organizations or coalition, Administrative Remedies. Enumerate exceptions thereto.
accredit citizen's arms of the Commission, prosecute
election offenses, and recommend to the President the Held: 1. Before a party is allowed to seek the
removal of or imposition of any other disciplinary action intervention of the court, it is a pre-condition that he
upon any officer or employee it has deputized for violation should have availed of all the means of administrative
or disregard of its directive, order or decision. In addition, processes afforded him. Hence, if a remedy within the
the Commission also has direct control and supervision administrative machinery can still be resorted to by giving
over all personnel involved in the conduct of election. the administrative officer concerned every opportunity to
However, the resolution of the adverse claims of private decide on a matter that comes within his jurisdiction then
respondent and petitioner as regards the existence of a such remedy should be exhausted first before the court’s
manifest error in the questioned certificate of canvass judicial power can be sought. The premature invocation
requires the COMELEC to act as an arbiter. It behooves of court’s jurisdiction is fatal to one’s cause of action.
the Commission to hear both parties to determine the Accordingly, absent any finding of waiver or estoppel the
veracity of their allegations and to decide whether the case is susceptible of dismissal for lack of cause of action.
alleged error is a manifest error. Hence, the resolution of This doctrine of exhaustion of administrative remedies
this issue calls for the exercise by the COMELEC of its was not without its practical and legal reasons, for one
quasi-judicial power. It has been said that where a power thing, availment of administrative remedy entails lesser
rests in judgment or discretion, so that it is of judicial expenses and provides for a speedier disposition of
nature or character, but does not involve the exercise of controversies. It is no less true to state that the courts of
functions of a judge, or is conferred upon an officer other justice for reasons of comity and convenience will shy
than a judicial officer, it is deemed quasi-judicial. The away from a dispute until the system of administrative
COMELEC therefore, acting as quasi-judicial tribunal, redress has been completed and complied with so as to
cannot ignore the requirements of procedural due process give the administrative agency concerned every
in resolving the petitions filed by private respondent. opportunity to correct its error and to dispose of the case.
(Federico S. Sandoval v. COMELEC, G.R. No.
133842, Jan. 26, 2000 [Puno]) This doctrine is disregarded:

248. Discuss the Doctrine of Primary Jurisdiction (or 1) when there is a violation of due process;
Prior Resort). 2) when the issue involved is purely a legal
question;
Held: Courts cannot and will not resolve a controversy 3) when the administrative action is patently
involving a question which is within the jurisdiction of an illegal amounting to lack or excess of jurisdiction;
administrative tribunal, especially where the question 4) when there is estoppel on the part of the
demands the exercise of sound administrative discretion administrative agency concerned;
requiring the special knowledge, experience and services 5) when there is irreparable injury;
of the administrative tribunal to determine technical and 6) when the respondent is a department
intricate matters of fact. secretary whose acts as an alter ego of the President
bears the implied and assumed approval of the latter;
In recent years, it has been the jurisprudential trend to 7) when to require exhaustion of administrative
apply this doctrine to cases involving matters that remedies would be unreasonable;
demand the special competence of administrative 8) when it would amount to a nullification of a
agencies even if the question involved is also judicial in claim;
character. It applies “where a claim is originally 9) when the subject matter is a private land in
cognizable in the courts, and comes into play whenever land case proceeding;
enforcement of the claim requires the resolution of issues 10) when the rule does not provide a plain,
which, under a regulatory scheme, have been placed speedy and adequate remedy, and
within the special competence of an administrative body; 11) when there are circumstances indicating the
in such case, the judicial process is suspended pending urgency of judicial intervention.
referral of such issues to the administrative body for its (Paat v. CA, 266 SCRA 167 [1997])
view.”
2. Non-exhaustion of administrative remedies is not
jurisdictional. It only renders the action premature, i.e.,
claimed cause of action is not ripe for judicial governmental or constituent and proprietary or
determination and for that reason a party has no cause of ministrant. While public benefit and public welfare,
action to ventilate in court. (Carale v. Abarintos, 269 particularly, the promotion of the economic and social
SCRA 132) development of Central Luzon, may be attributable to the
operation of the BCDA, yet it is certain that the functions
performed by the BCDA are basically proprietary in
250. When may the Government not validly invoke the nature. The promotion of economic and social
rule that prescription does not run against the State? development of Central Luzon, in particular, and the
country’s goal for enhancement, in general, do not make
Held: While it is true that prescription does not run the BCDA equivalent to the Government. Other
against the State, the same may not be invoked by the corporations have been created by government to act as
government in this case since it is no longer interested in its agents for the realization of its programs, the SSS,
the subject matter. While Camp Wallace may have GSIS, NAWASA and the NIA, to count a few, and yet, the
belonged to the government at the time Rafael Galvez’s Court has ruled that these entities, although performing
title was ordered cancelled in Land Registration Case No. functions aimed at promoting public interest and public
N-361, the same no longer holds true today. welfare, are not government-function corporations
invested with governmental attributes. It may thus be
Republic Act No. 7227, otherwise known as the Base said that the BCDA is not a mere agency of the
Conversion and Development Act of 1992, created the Government but a corporate body performing proprietary
Bases Conversion and Development Authority. X x x functions.

Xxx Xxx

With the transfer of Camp Wallace to the BCDA, the Having the capacity to sue or be sued, it should thus be
government no longer has a right or interest to protect. the BCDA which may file an action to cancel petitioner’s
Consequently, the Republic is not a real party in interest title, not the Republic, the former being the real party in
and it may not institute the instant action. Nor may it interest. One having no right or interest to protect cannot
raise the defense of imprescriptibility, the same being invoke the jurisdiction of the court as a party plaintiff in
applicable only in cases where the government is a party an action (Ralla v. Ralla, 199 SCRA 495 [1991]). A suit
in interest. X x x. Being the owner of the areas covered may be dismissed if the plaintiff or the defendant is not a
by Camp Wallace, it is the Bases Conversion and real party in interest. X x x.
Development Authority, not the Government, which
stands to be benefited if the land covered by TCT No. T- However, E.B. Marcha Transport Co., Inc. v. IAC (147
5710 issued in the name of petitioner is cancelled. SCRA 276 [1987]) is cited as authority that the Republic
is the proper party to sue for the recovery of possession
Nonetheless, it has been posited that the transfer of of property which at the time of the installation of the suit
military reservations and their extensions to the BCDA is was no longer held by the national government body but
basically for the purpose of accelerating the sound and by the Philippine Ports Authrotiy. In E.B. Marcha, the
balanced conversion of these military reservations into Court ruled:
alternative productive uses and to enhance the benefits
to be derived from such property as a measure of It can be said that in suing for the recovery of the rentals,
promoting the economic and social development, the Republic of the Philippines, acted as principal of the
particularly of Central Luzon and, in general, the country’s Philippine Ports Authority, directly exercising the
goal for enhancement (Section 2, Republic Act No. 7227). commission it had earlier conferred on the latter as its
It is contended that the transfer of these military agent. We may presume that, by doing so, the Republic
reservations to the Conversion Authority does not amount of the Philippines did not intend to retain the said rentals
to an abdication on the part of the Republic of its for its own use, considering that by its voluntary act it had
interests, but simply a recognition of the need to create a transferred the land in question to the Philippine Ports
body corporate which will act as its agent for the Authority effective July 11, 1974. The Republic of the
realization of its program. It is consequently asserted that Philippines had simply sought to assist, not supplant, the
the Republic remains to be the real party in interest and Philippine Ports Authority, whose title to the disputed
the Conversion Authority merely its agent. property it continues to recognize. We may expect the
that the said rentals, once collected by the Republic of the
We, however, must not lose sight of the fact that the Philippines, shall be turned over by it to the Philippine
BCDA is an entity invested with a personality separate and Ports Authority conformably to the purposes of P.D. No.
distinct from the government. X x x 857.

It may not be amiss to state at this point that the E.B. Marcha is, however, not on all fours with the case at
functions of government have been classified into bar. In the former, the Court considered the Republic a
proper party to sue since the claims of the Republic and attached agency of the Department of Transportation
the Philippine Ports Authority against the petitioner and Communications.
therein were the same. To dismiss the complaint in E.B.
Marcha would have brought needless delay in the In the regulatory communications industry, the
settlement of the matter since the PPA would have to NTC has the sole authority to issue Certificates of Public
refile the case on the same claim already litigated upon. Convenience and Necessity (CPCN) for the installation,
Such is not the case here since to allow the government operation, and maintenance of communications facilities
to sue herein enables it to raise the issue of and services, radio communications systems, telephone
imprescriptibility, a claim which is not available to the and telegraph systems. Such power includes the
BCDA. The rule that prescription does not run against the authority to determine the areas of operations of
State does not apply to corporations or artificial bodies applicants for telecommunications services. Specifically,
created by the State for special purposes, it being said Section 16 of the Public Service Act authorizes the then
that when the title of the Republic has been divested, its PSC, upon notice and hearing, to issue Certificates of
grantees, although artificial bodies of its own creation, are Public Convenience for the operation of public services
in the same category as ordinary persons (Kingston v. within the Philippines “whenever the Commission finds
LeHigh Valley Coal Co., 241 Pa 469). By raising the claim that the operation of the public service proposed and the
of imprescriptibility, a claim which cannot be raised by the authorization to do business will promote the public
BCDA, the Government not only assists the BCDA, as it interests in a proper and suitable manner.”
did in E.B. Marcha, it even supplants the latter, a course (Commonwealth Act No. 146, Section 16[a]) The
of action proscribed by said case. procedure governing the issuance of such authorizations
is set forth in Section 29 of the said Act x x x.
Moreover, to recognize the Government as a proper party (Republic v. Express Telecommunication Co., Inc.,
to sue in this case would set a bad precedent as it would 373 SCRA 316, Jan. 15, 2002, 1st Div. [Ynares-
allow the Republic to prosecute, on behalf of government- Santiago])
owned or controlled corporations, causes of action which
have already prescribed, on the pretext that the 252. Is the filing of the administrative rules and
Government is the real party in interest against whom regulations with the UP Law Center the operative act
prescription does not run, said corporations having been that gives the rules force and effect?
created merely as agents for the realization of
government programs. Held: In granting Bayantel the provisional
authority to operate a CMTS, the NTC applied Rule 15,
It should also be noted that petitioner is unquestionably Section 3 of its 1978 Rules of Practice and Procedure,
a buyer in good faith and for value, having acquired the which provides:
property in 1963, or 5 years after the issuance of the
original certificate of title, as a third transferee. If only Sec. 3. Provisional Relief. – Upon the filing of an
not to do violence and to give some measure of respect application, complaint or petition or at any stage
to the Torrens System, petitioner must be afforded some thereafter, the Board may grant on motion of the
measure of protection. (Shipside Incorporated v. pleader or on its own initiative, the relief prayed for,
Court of Appeals, 352 SCRA 334, Feb. 20, 2001, 3rd based on the pleading, together with the affidavits and
Div. [Melo]) supporting documents attached thereto, without
prejudice to a final decision after completion of the
251. Discuss the nature and functions of the NTC, hearing which shall be called within thirty (30) days from
and analyze its powers and authority as well as the laws, grant of authority asked for. (italics ours)
rules and regulations that govern its existence and
operations. Respondent Extelcom, however, contends that
the NTC should have applied the Revised Rules which
Held: The NTC was created pursuant to were filed with the Office of the National Administrative
Executive Order No. 546, promulgated on July 23, 1979. Register on February 3, 1993. These Revised Rules
It assumed the functions formerly assigned to the Board deleted the phrase “on its own initiative”; accordingly, a
of Communications and the Communications Control provisional authority may be issued only upon filing of
Bureau, which were both abolished under the said the proper motion before the Commission.
Executive Order. Previously, the NTC’s function were
merely those of the defunct Public Service Commission In answer to this argument, the NTC, through
(PSC), created under Commonwealth Act No. 146, as the Secretary of the Commission, issued a certification to
amended, otherwise known as the Public Service Act, the effect that inasmuch as the 1993 Revised Rules have
considering that the Board of Communications was the not been published in a newspaper of general
successor-in-interest of the PSC. Under Executive Order circulation, the NTC has been applying the 1978 Rules.
No. 125-A, issued in April 1987, the NTC became an
The absence of publication, coupled with the
certification by the Commissioner of the NTC stating that “We hold therefore that all statutes, including those of
the NTC was still governed by the 1987 Rules, clearly local application and private laws, shall be published as
indicate that the 1993 Revised Rules have not taken a condition for their effectivity, which shall begin fifteen
effect at the time of the grant of the provisional days after publication unless a different effectivity is
authority to Bayantel. The fact that the 1993 Revised fixed by the legislature.
Rules were filed with the UP Law Center on February 3,
1993 is of no moment. There is nothing in the Covered by this rule are presidential decrees and
Administrative Code of 1987 which implies that the filing executive orders promulgated by the President in the
of the rules with the UP Law Center is the operative act exercise of legislative power or, at present, directly
that gives the rules force and effect. Book VII, Chapter conferred by the Constitution. Administrative Rules and
2, Section 3 thereof merely states: Regulations must also be published if their purpose is to
enforce or implement existing law pursuant also to a
Filing. – (1) Every agency shall file with the University of valid delegation.
the Philippines Law Center three (3) certified copies of
every rule adopted by it. Rules in force on the date of Interpretative regulations and those merely internal in
effectivity of this Code which are not filed within three nature, that is, regulating only the personnel of the
(3) months from the date shall not thereafter be the administrative agency and not the public, need not be
basis of any sanction against any party or persons. published. Neither is publication required of the so-
called letters of instructions issued by administrative
(2) The records officer of the agency, or his equivalent superiors concerning the rules or guidelines to be
functionary, shall carry out the requirements of this followed by their subordinates in the performance of
section under pain of disciplinary action. their duties.

(3) A permanent register of all rules shall be kept by the Xxx


issuing agency and shall be open to public inspection.
We agree that the publication must be in full or it is no
The National Administrative Register is merely a publication at all since its purpose is to inform the public
bulletin of codified rules and it is furnished only to the of the contents of the laws.”
Office of the President, Congress, all appellate courts,
the National Library, other public offices or agencies as The Administrative Order under consideration is one of
the Congress may select, and to other persons at a price those issuances which should be published for its
sufficient to cover publication and mailing or distribution effectivity, since its purpose is to enforce and implement
costs (Administrative Code of 1987, Book VII, Chapter 2, an existing law pursuant to a valid delegation, i.e., P.D.
Section 7). In a similar case, we held: 1071, in relation to LOI 444 and EO 133 (Philippine
International Trading Corp. v. Angeles, 263 SCRA 421,
This does not imply, however, that the subject 446-447 [1996]).
Administrative Order is a valid exercise of such quasi-
legislative power. The original Administrative Order Thus, publication in the Official Gazette or a
issued on August 30, 1989, under which the newspaper of general circulation is a condition sine qua
respondents filed their applications for importations, was non before statutes, rules or regulations can take effect.
not published in the Official Gazette or in a newspaper This is explicit from Executive Order No. 200, which
of general circulation. The questioned Administrative repealed Article 2 of the Civil Code, and which states
Order, legally, until it is published, is invalid within the that:
context of Article 2 of Civil Code, which reads:
Laws shall take effect after fifteen days following the
“Article 2. Laws shall take effect after fifteen days completion of their publication either in the Official
following the completion of their publication in the Gazette or in a newspaper of general circulation in the
Official Gazette (or in a newspaper of general circulation Philippines, unless it is otherwise provided (E.O. 200,
in the Philippines), unless it is otherwise provided. X x x” Section 1).

The fact that the amendments to Administrative Order The Rules of Practice and Procedure of the NTC,
No. SOCPEC 89-08-01 were filed with, and published by which implements Section 29 of the Public Service Act
the UP Law Center in the National Administrative (C.A. 146, as amended), fall squarely within the scope of
Register, does not cure the defect related to the these laws, as explicitly mentioned in the case of Tanada
effectivity of the Administrative Order. v. Tuvera (146 SCRA 446 [1986]).

This Court, in Tanada v. Tuvera (G.R. No. L-63915, Our pronouncement in Tanada v. Tuvera is clear and
December 29, 1986, 146 SCRA 446) stated, thus: categorical. Administrative rules and regulations must
be published if their purpose is to enforce or implement
existing law pursuant to a valid delegation. The only Applying this doctrine, we have previously
exception are interpretative regulations, those merely declared as having no force and effect the following
internal in nature, or those so-called letters of administrative issuances: a) Rules and Regulations
instructions issued by administrative superiors issued by the Joint Ministry of Health-Ministry of Labor
concerning the rules and guidelines to be followed by and Employment Accreditation Committee regarding the
their subordinates in the performance of their duties accreditation of hospitals, medical clinics and
(PHILSA International Placement & Services Corp. v. laboratories (Joint Ministry of Health-Ministry of Labor
Secretary of Labor, G.R. No. 103144, April 4, 2001, 356 and Employment Accreditation Committee v. Court of
SCRA 174). Appeals, 196 SCRA 263 [1991]); b) Letter of Instruction
No. 416 ordering the suspension of payments due and
Hence, the 1993 Revised Rules should be payable by distressed copper mining companies to the
published in the Official Gazette or in a newspaper of national government (Caltex Philippines, Inc. v. Court of
general circulation before it can take effect. Even the Appeals, 208 SCRA 726 [1992]); c) Memorandum
1993 Revised Rules itself mandates that said Rules shall Circulars issued by the POEA regulating the recruitment
take effect only after their publication in a newspaper of of domestic helpers to Hong Kong (Phil. Association of
general circulation (Section 20 thereof). In the absence Service Exporters v. Torres, 212 SCRA 298 [1992]); d)
of such publication, therefore, it is the 1978 Rules that Administrative Order No. SOCPEC 89-08-01 issued by
governs. (Republic v. Express Telecommunication the Philippine International Trading Corporation
Co., Inc., 373 SCRA 316, Jan. 15, 2002, 1st Div. regulating applications for importation from the People’s
[Ynares-Santiago]) Republic of China (Philippine International Trading
Corporation v. Angeles, 263 SCRA 421 [1996]); and e)
253. May a person be held liable for violation of an Corporate Compensation Circular No. 10 issued by the
administrative regulation which was not published? Department of Budget and Management discontinuing
the payment of other allowances and fringe benefits to
Held: Petitioner insists, however, that it cannot government officials and employees (De Jesus v.
be held liable for illegal exaction as POEA Memorandum Commission on Audit, 294 SCRA 152 [1998). In all
Circular No. II, Series of 1983, which enumerated the these cited cases, the administrative issuances
allowable fees which may be collected from applicants, questioned therein were uniformly struck down as they
is void for lack of publication. were not published or filed with the National
Administrative Register as required by the Administrative
There is merit in the argument. Code of 1987 (Administrative Code of 1987, Book VII,
chapter 2, Section 3).
In Tanada v. Tuvera (136 SCRA 27 [1985]), the
Court held, as follows: POEA memorandum Circular No. 2, Series of
1983 must likewise be declared ineffective as the same
“We hold therefore that all statutes, including those of was never published or filed with the National
local application and private laws, shall be published as Administrative Register.
a condition for their effectivity, which shall begin fifteen
days after publication unless a different effectivity date POEA Memorandum Circular No. 2, Series of
is fixed by the legislature. 1983 provides for the applicable schedule of placement
and documentation fees for private employment
Covered by this rule are presidential decrees and agencies or authority holders. Under the said Order, the
executive orders promulgated by the President in the maximum amount which may be collected from
exercise of legislative powers whenever the same are prospective Filipino overseas workers is P2,500.00. The
validly delegated by the legislature or, at present, said circular was apparently issued in compliance with
directly conferred by the Constitution. Administrative the provisions of Article 32 of the Labor Code x x x.
rules and regulations must also be published if their
purpose is to enforce or implement existing law pursuant It is thus clear that the administrative circular
to a valid delegation. under consideration is one of those issuances which
should be published for its effectivity, since its purpose
Interpretative regulations and those merely internal in is to enforce and implement an existing law pursuant to
nature, that is, regulating only the personnel of the a valid delegation (Philippine International Trading
administrative agency and the public, need not be Corporation v. Angeles, supra.). Considering that POEA
published. Neither is publication required of the so- Administrative Circular No. 2, Series of 1983 has not as
called letter of instructions issued by the administrative yet been published or filed with the National
superiors concerning the rules or guidelines to be Administrative Register, the same is ineffective and may
followed by their subordinates in the performance of not be enforced. (Philsa International Placement
their duties.” and Services Corporation v. Secretary of Labor
and Employment, 356 SCRA 174, April 4, 2001, 3rd To summarize, petitioner should be absolved from the
Div., [Gonzaga-Reyes]) three (3) counts of exaction as POEA Administrative
Circular No. 2, Series of 1983 could not be the basis of
254. Does the publication requirement apply as well administrative sanctions against petitioner for lack of
to administrative regulations addressed only to a specific publication. (Philsa International Placement and
group and not to the general public? Services Corporation v. Secretary of Labor and
Employment, 356 SCRA 174, April 4, 2001, 3rd
Held: The Office of the Solicitor General Div., [Gonzaga-Reyes])
likewise argues that the questioned administrative
circular is not among those requiring publication 255. May a successful bidder compel a government
contemplated by Tanada v. Tuvera as it is addressed agency to formalize a contract with it notwithstanding
only to a specific group of persons and not to the that its bid exceeds the amount appropriated by
general public. Congress for the project?

Again, there is no merit in this argument. Held: Enshrined in the 1987 Philippine Constitution is
the mandate that “no money shall be paid out of the
The fact that the said circular is addressed only Treasury except in pursuance of an appropriation made
to a specified group, namely private employment by law.” (Sec. 29[1], Article VI of the 1987 Constitution)
agencies or authority holders, does not take it away Thus, in the execution of government contracts, the
from the ambit of our ruling in Tanada v. Tuvera. In the precise import of this constitutional restriction is to
case of Phil. Association of Service Exporters v. Torres require the various agencies to limit their expenditures
((212 SCRA 298 [1992]), the administrative circulars within the appropriations made by law for each fiscal
questioned therein were addressed to an even smaller year.
group, namely Philippine and Hong Kong agencies
engaged in the recruitment of workers for Hong Kong, Xxx
and still the Court ruled therein that, for lack of proper
publication, the said circulars may not be enforced or It is quite evident from the tenor of the
implemented. language of the law that the existence of appropriations
and the availability of funds are indispensable pre-
Our pronouncement in Tanada v. Tuvera is clear requisites to or conditions sine qua non for the execution
and categorical. Administrative rules and regulations of government contracts. The obvious intent is to
must be published if their purpose is to enforce or impose such conditions as a priori requisites to the
implement existing law pursuant to a valid delegation. validity of the proposed contract (Fernandez, A Treatise
The only exceptions are interpretative regulations, those on Government Contracts Under Philippine Law, 2001,
merely internal in nature, or those so-called letters of pp. 40-41). Using this as our premise, we cannot
instructions issued by administrative superiors accede to PHOTOKINA’s contention that there is already
concerning the rules and guidelines to be followed by a perfected contract. While we held in Metropolitan
their subordinates in the performance of their duties. Manila Development Authority v. Jancom Environmental
Administrative Circular No. 2, Series of 1983 has not Corporation (Supra) that “the effect of an unqualified
been shown to fall under any of these exceptions. acceptance of the offer or proposal of the bidder is to
perfect a contract, upon notice of the award to the
In this regard, the Solicitor General’s reliance on the bidder,” however, such statement would be
case of Yaokasin v. Commissioner of Customs (180 inconsequential in a government where the acceptance
SCRA 599 [1989]) is misplaced. In the said case, the referred to is yet to meet certain conditions. To hold
validity of certain Customs Memorandum Orders were otherwise is to allow a public officer to execute a binding
upheld despite their lack of publication as they were contract that would obligate the government in an
addressed to a particular class of persons, the customs amount in excess of the appropriations for the purpose
collectors, who were also the subordinates of the for which the contract was attempted to be made (64
Commissioner of the Bureau of Customs. As such, the Am Jur 2d Sec. 11). This is a dangerous precedent.
said Memorandum Orders clearly fall under one of the
exceptions to the publication requirement, namely those In the case at bar, there seems to be an
dealing with instructions from an administrative superior oversight of the legal requirements as early as the
to a subordinate regarding the performance of their bidding stage. The first step of a Bids and Awards
duties, a circumstance which does not obtain in the case Committee (BAC) is to determine whether the bids
at bench. comply with the requirements. The BAC shall rate a bid
“passed” only if it complies with all the requirements and
Xxx the submitted price does not exceed the approved
budget for the contract.” (Implementing Rules and
Regulations [IRR] for Executive Order No. 262, supra.)
Petitioners are justified in refusing to formalize the
Extant on the record is the fact that the VRIS contract with PHOTOKINA. Prudence dictated them not
Project was awarded to PHOTOKINA on account of its to enter into a contract not backed up by sufficient
bid in the amount of P6.588 Billion Pesos. However, appropriation and available funds. Definitely, to act
under Republic Act No. 8760 (General Appropriations otherwise would be a futile exercise for the contract
Act, FY 2000, p. 1018, supra.), the only fund would inevitably suffer the vice of nullity. X x x
appropriated for the project was P1 Billion Pesos and
under the Certification of Available Funds (CAF) only Xxx
P1.2 Billion Pesos was available. Clearly, the amount
appropriated is insufficient to cover the cost of the entire Verily, the contract, as expressly declared by
VRIS Project. There is no way that the COMELEC could law, is inexistent and void ab initio (Article 1409 of the
enter into a contract with PHOTOKINA whose accepted Civil Code of the Philippines). This is to say that the
bid was way beyond the amount appropriated by law for proposed contract is without force and effect from the
the project. This being the case, the BAC should have very beginning or from its incipiency, as if it had never
rejected the bid for being excessive or should have been entered into, and hence, cannot be validated either
withdrawn the Notice of Award on the ground that in the by lapse of time or ratification (Manila Lodge v. Court of
eyes of the law, the same is null and void. Appeals, 73 SCRA 162 [1976]; See also Tongoy v. Court
of Appeals, 123 SCRA 99 [1983]).
Xxx
Xxx
Even the draft contract submitted by
Commissioner Sadain, that provides for a contract price In fine, we rule that PHOTOKINA, though the winning
in the amount of P1.2 Billion Pesos is unacceptable. X x bidder, cannot compel the COMELEC to formalize the
x While the contract price under the draft contract is contract. Since PHOTOKINA’s bid is beyond the amount
only P1.2 Billion and, thus, within the certified available appropriated by Congress for the VRIS Project, the
funds, the same covers only Phase I of the VRIS Project, proposed contract is not binding upon the COMELEC and
i.e., the issuance of identification cards for only is considered void x x x. (Commission on Elections
1,000,000 voters in specified areas (Ibid., p. 382). In v. Judge Ma. Luisa Quijano-Padilla, G.R. No.
effect, the implementation of the VRIS Project will be 151992, Sept. 18, 2002, En Banc [Sandoval-
“segmented” or “chopped” into several phases. Not only Gutierrez])
is such arrangement disallowed by our budgetary laws
and practices, it is also disadvantageous to the 256. What is the remedy available to a party who
COMELEC because of the uncertainty that will loom over contracts with the government contrary to the
its modernization project for an indefinite period of time. requirements of the law and, therefore, void ab initio?
Should Congress fail to appropriate the amount
necessary for the completion of the entire project, what Held: Of course, we are not saying that the party who
good will the accomplished Phase I serve? As expected, contracts with the government has no other recourse in
the project failed “to sell” with the Department of law. The law itself affords him the remedy. Section 48
Budget and Management. Thus, Secretary Benjamin of E.O. No. 292 explicitly provides that any contract
Diokno, per his letter of December 1, 2000, declined the entered into contrary to the above-mentioned
COMELEC’s request for the issuance of the Notice of requirements shall be void, and “the officers entering
Cash Availability (NCA) and a multi-year obligatory into the contract shall be liable to the Government or
authority to assume payment of the total VRIS Project other contracting party for any consequent damage to
for lack of legal basis. Corollarily, under Section 33 of the same as if the transaction had been wholly between
R.A. No. 8760, no agency shall enter into a multi-year private parties.” So when the contracting officer
contract without a multi-year obligational authority, transcends his lawful and legitimate powers by acting in
thus: excess of or beyond the limits of his contracting
authority, the Government is not bound under the
“SECTION 33. Contracting Multi-Year Projects. - In the contract. It would be as if the contract in such case
implementation of multi-year projects, no agency shall were a private one, whereupon, he binds himself, and
enter into a multi-year contract without a multi-year thus, assumes personal liability thereunder. (Fernandez,
Obligational Authority issued by the Department of a Treatise on Government Contracts Under Philippine
Budget and Management for the purpose. Law, 2001, supra., pp. 22-23). Otherwise stated, the
Notwithstanding the issuance of the multi-year proposed contract is unenforceable as to the
Obligational Authority, the obligation to be incurred in Government.
any given calendar year, shall in no case exceed the
amount programmed for implementation during said While this is not the proceeding to determine
calendar year.” where the culpability lies, however, the constitutional
mandate cited above constrains us to remind all public
officers that public office is a public trust and all public Held: In Export Processing Zone Authority v.
officers must at all times be accountable to the people. Commission on Human rights (208 SCRA 125, 131), the
The authority of public officers to enter into government Court x x x explained:
contracts is circumscribed with a heavy burden of
responsibility. In the exercise of their contracting “The constitutional provision directing the CHR to ‘provide
prerogative, they should be the first judges of the for preventive measures and legal aid services to the
legality, propriety and wisdom of the contract they underprivileged whose human rights have been violated
entered into. They must exercise a high degree of or need protection’ may not be construed to confer
caution so that the Government may not be the victim of jurisdiction on the Commission to issue a restraining order
ill-advised or improvident action (Rivera v. Maclang, 7 or writ of injunction for, if that were the intention, the
SCRA 57 [1963]). (Commission on Elections v. Constitution would have expressly said so. ‘Jurisdiction is
Judge Ma. Luisa Quijano-Padilla, G.R. No. 151992, conferred only by the Constitution or by law.’ It is never
Sept. 18, 2002, En Banc [Sandoval-Gutierrez]) derived by implication.”

257. Does the Commission on Human Rights have the “Evidently, the ‘preventive measures and legal aid
power to adjudicate? services’ mentioned in the Constitution refer to
extrajudicial and judicial remedies (including a writ of
Held: In its Order x x x denying petitioners’ preliminary injunction) which the CHR may seek from the
motion to dismiss, the CHR theorizes that the intention of proper courts on behalf of the victims of human rights
the members of the Constitutional Commission is to make violations. Not being a court of justice, the CHR itself
CHR a quasi-judicial body. This view, however, has not has no jurisdiction to issue the writ, for a writ of
heretofore been shared by this Court. In Carino v. preliminary injunction may only be issued ‘by the judge of
Commission on Human Rights (204 SCRA 483, 492), the any court in which the action is pending [within his
Court x x x has observed that it is “only the first of the district], or by a Justice of the Court of Appeals, or of the
enumerated powers and functions that bears any Supreme Court. x x x. A writ of preliminary injunction is
resemblance to adjudication of adjudgment,” but that an ancillary remedy. It is available only in a pending
resemblance can in no way be synonymous to the principal action, for the preservation or protection of the
adjudicatory power itself. The Court explained: rights and interest of a party thereto, and for no other
purpose.”
“x x x [T]he Commission on Human Rights x x x was not
meant by the fundamental law to be another court or The Commission does have legal standing to indorse, for
quasi-judicial agency in this country, or duplicate much appropriate action, its findings and recommendations to
less take over the functions of the latter. any appropriate agency of government (See Export
Processing Zone Authority v. Commission on Human
“The most that may be conceded to the Commission in Rights, 208 SCRA 125). (Simon, Jr. v. Commission on
the way of adjudicative power is that it may investigate, Human Rights, 229 SCRA 117, 134-135, Jan. 5,
i.e., receive evidence and make findings of fact as regards 1994, En Banc [Vitug, J.])
claimed human rights violations involving civil and political
rights. But fact finding is not adjudication, and cannot be 259. Discuss the contempt power of the Commission
likened to the judicial function of a court of justice, or on Human Rights (CHR). When may it be validly
even a quasi-judicial agency or official. The function of exercised?
receiving evidence and ascertaining therefrom the facts
of a controversy is not a judicial function, properly Held: On its contempt powers, the CHR is
speaking. To be considered such, the faculty of receiving constitutionally authorized to “adopt its operational
evidence and making factual conclusions in a controversy guidelines and rules of procedure, and cite for contempt
must be accompanied by the authority of applying the law for violations thereof in accordance with the Rules of
to those factual conclusions to the end that the Court.” Accordingly, the CHR acted within its authority in
controversy may be decided or determined providing in its revised rules, its power “to cite or hold any
authoritatively, finally and definitively, subject to such person in direct or indirect contempt, and to impose the
appeals or modes of review as may be provided by law. appropriate penalties in accordance with the procedure
This function, to repeat, the Commission does not have. and sanctions provided for in the Rules of Court.” That
(Simon, Jr. v. Commission on Human Rights, 229 power to cite for contempt, however, should be
SCRA 117, 125, Jan. 5, 1994, En Banc [Vitug, J.]) understood to apply only to violations of its adopted
operational guidelines and rules of procedure essential to
258. Does the Commission on Human Rights have carry out its investigatorial powers. To exemplify, the
jurisdiction to issue TRO or writ of preliminary injunction? power to cite for contempt could be exercised against
persons who refuse to cooperate with the said body, or
who unduly withhold relevant information, or who decline
to honor summons, and the like, in pursuing its
investigative work. The “order to desist” (a semantic
interplay for a restraining order) in the instance before us, Indeed, it may rightly be said that the right of choice is
however, is not investigatorial in character but prescinds the heart of the power to appoint. In the exercise of the
from an adjudicative power that it does not possess. x x power of appointment, discretion is an integral thereof.
x (Simon, Jr. v. Commission on Human Rights, 229 (Bermudez v. Torres, 311 SCRA 733, Aug. 4, 1999,
SCRA 117, 134, Jan. 5, 1994, En Banc [Vitug, J.]) 3rd Div. [Vitug])

262. May the Civil Service Commission, or the


Supreme Court, validly nullify an appointment on the
THE LAW OF PUBLIC OFFICERS ground that somebody else is better qualified?

260. How are positions in the Civil Service classified? Held: The head of an agency who is the appointing
Discuss the characteristics of each. power is the one most knowledgeable to decide who can
best perform the functions of the office. Appointment is
Ans.: Positions in the Civil Service may be an essentially discretionary power and must be performed
classified into: 1) Career Positions, and 2) Non-Career by the officer vested with such power according to his
Positions. best lights, the only condition being that the appointee
should possess the qualifications required by law. If he
Career Positions are characterized by (1) entrance based does, then the appointment cannot be faulted on the
on merit and fitness to be determined as far as ground that there are others better qualified who should
practicable by competitive examination, or based on have been preferred. Indeed, this is a prerogative of the
highly technical qualifications; (2) opportunity for appointing authority which he alone can decide. The
advancement to higher career positions; and (3) security choice of an appointee from among those who possess
of tenure (Sec. 7, Chap. 2, Subtitle A, Title I, Bk. V, the required qualifications is a political and administrative
E.O. No. 292). decision calling for considerations of wisdom,
convenience, utility and the interests of the service which
The Non-Career Service shall be characterized by (1) can best be made by the head of the office concerned,
entrance on bases other than of the usual tests of merit the person most familiar with the organizational structure
or fitness utilized for the career service; and (2) tenure and environmental circumstances within which the
which is limited to a period specified by law, or which is appointee must function.
coterminous with that of the appointing authority or
subject to his pleasure, or which is limited to the As long as the appointee is qualified the Civil Service
duration of a particular project for which purpose Commission has no choice but to attest to and respect the
employment was made (Sec. 9, Chap. 2, Subtitle A, appointment even if it be proved that there are others
Title I, Bk. V, E.O. No. 292). with superior credentials. The law limits the Commission’s
authority only to whether or not the appointees possess
the legal qualifications and the appropriate civil service
eligibility, nothing else. If they do then the appointments
261. Define Appointment. Discuss its nature. are approved because the Commission cannot exceed its
power by substituting its will for that of the appointing
Held: An “appointment” to a public office is the authority. Neither can we. (Rimonte v. CSC, 244
unequivocal act of designating or selecting by one having SCRA 504-505, May 29, 1995, En Banc [Bellosillo,
the authority therefor of an individual to discharge and J.])
perform the duties and functions of an office or trust. The
appointment is deemed complete once the last act 263. Does the “next-in-rank” rule import any
required of the appointing authority has been complied mandatory or peremptory requirement that the person
with and its acceptance thereafter by the appointee in next-in-rank must be appointed to the vacancy?
order to render it effective. Appointment necessarily calls
for an exercise of discretion on the part of the appointing Held: The “next-in-rank rule is not absolute; it only
authority. In Pamantasan ng Lungsod ng Maynila v. applies in cases of promotion, a process which denotes a
Intermediate Appellate Court (140 SCRA 22), reiterated in scalar ascent of an officer to another position higher
Flores v. Drilon (223 SCRA 568), this Court has held: either in rank or salary. And even in promotions, it can
be disregarded for sound reasons made known to the
“The power to appoint is, in essence, discretionary. The next-in-rank, as the concept does not import any
appointing power has the right of choice which he may mandatory or peremptory requirement that the person
exercise freely according to his judgment, deciding for next-in-rank must be appointed to the vacancy. The
himself who is best qualified among those who have the appointing authority, under the Civil Service Law, is
necessary qualifications and eligibilities. It is a allowed to fill vacancies by promotion, transfer of present
prerogative of the appointing power x x x.” (At p. 579) employees, reinstatement, reemployment, and
appointment of outsiders who have appropriate civil
service eligibility, not necessarily in that order. There is The mere fact that a position belongs to the Career
no legal fiat that a vacancy must be filled only by Service does not automatically confer security of tenure
promotion; the appointing authority is given wide in its occupant even if he does not possess the required
discretion to fill a vacancy from among the several qualifications. Such right will have to depend on the
alternatives provided by law. nature of his appointment, which in turn depends on his
eligibility or lack of it. A person who does not have the
What the Civil Service Law provides is that if a vacancy is requisite qualifications for the position cannot be
filled by promotion, the person holding the position next appointed to it in the first place or, only as an exception
in rank thereto “shall be considered for promotion.” to the rule, may be appointed to it merely in an acting
capacity in the absence of appropriate eligibles. The
In Taduran v. Civil Service Commission (131 SCRA 66 appointment extended to him cannot be regarded as
[1984]), the Court construed that phrase to mean that the permanent even if it may be so designated.
person next-in-rank “would be among the first to be
considered for the vacancy, if qualified.” In Santiago, Jr. Evidently, private respondent’s appointment did
v. Civil Service Commission (178 SCRA 733 [1989]), the not attain permanency. Not having taken the necessary
Court elaborated the import of the rule in the following Career Executive Service examination to obtain the
manner: requisite eligibility, he did not at the time of his
appointment and up to the present, possess the needed
“One who is next-in-rank is entitled to preferential eligibility for a position in the Career Executive Service.
consideration for promotion to the higher vacancy but it Consequently, his appointment as Ministry Legal
does not necessarily follow that he and no one else can Counsel-CESO IV/Department Legal Counsel and/or
be appointed. The rule neither grants a vested right to Director III, was merely temporary. Such being the
the holder nor imposes a ministerial duty on the case, he could be transferred or reassigned without
appointing authority to promote such person to the next violating the constitutionally guaranteed right to security
higher position x x x” of tenure.
(Abila v. CSC, 198 SCRA 102, June 3, 1991, En Banc
[Feliciano]) Private respondent capitalizes on his lack of CES
eligibility by adamantly contending that the mobility and
263. Can a person who lacks the necessary flexibility concepts in the assignment of personnels
qualifications for a public position be appointed to it in a under the Career Executive Service do not apply to him
permanent capacity? Illustrative case. because he s not a Career Executive Service Officer.
Obviously, the contention is without merit. As correctly
Held: At the outset, it must be stressed that pointed out by the Solicitor General, non-eligibles
the position of Ministry Legal Counsel-CESO IV is holding permanent appointments to CES positions were
embraced in the Career Executive Service. X x x never meant to remain immobile in their status.
Otherwise, their lack of eligibility would be a premium
In the case at bar, there is no question that vesting them with permanency in the CES positions, a
private respondent does not have the required CES privilege even their eligible counterparts do not enjoy.
eligibility. As admitted by private respondent in his
Comment, he is “not a CESO or a member of the Career Then too, the cases on unconsented transfer
Executive Service.” invoked by private respondent find no application in the
present case. To reiterate, private respondent’s
In the case of Achacoso v. Macaraig, et al. (195 appointment is merely temporary; hence, he could be
SCRA 235, 239-240 [1991]), the Court held: transferred or reassigned to other positions without
violating his right to security of tenure. (De Leon v.
It is settled that a permanent appointment can be issued Court of Appeals, 350 SCRA 1, Jan. 22, 2001, En
only “to a person who meets all the requirements for the Banc [Ynares-Santiago])
position to which he s being appointed, including the
appropriate eligibility prescribed.” Achacoso did not. At 264. In the career executive service, is a career
best, therefore, his appointment could be regarded only executive service (CES) eligibility all that an employee
as temporary. And being so, it could be withdrawn at needs to acquire security of tenure? Is appointment to
will by the appointing authority and “at a moment’s a CES rank necessary for the acquisition of such security
notice,” conformably to established jurisprudence. of tenure?

The Court, having considered these submissions and the Held: The petitions are impressed with merit.
additional arguments of the parties in the petitioner’s
Reply and of the Solicitor-General’s Rejoinder, must find In the career executive service, the acquisition
for the respondents. of security of tenure which presupposes a permanent
appointment is governed by the rules and regulations 350 SCRA 528, Jan. 29, 2001, 1st Div. [Ynares-
promulgated by the CES Board x x x. Santiago])

As clearly set forth in the foregoing provisions, two 265. May an elective public official be validly
requisites must concur in order that an employee in the appointed or designated to any public office or position
career executive service may attain security of tenure, to during his tenure?
wit:
Ans.: No elective official shall be eligible for
a) CES eligibility; and appointment or designation in any capacity to any public
b) Appointment to the appropriate CES rank. office or position during his tenure. (Sec. 7, 1st par.,
Art. IX-B, 1987 Constitution)
In addition, it must be stressed that the security of
tenure of employees in the career executive service 266. May an appointive public official hold any other
(except first and second level employees in the civil office or employment?
service), pertains only to rank and not to the office or to
the position to which they may be appointed. Thus, a Ans.: Unless otherwise allowed by law or by the
career executive service officer may be transferred or primary functions of his position, no appointive official
reassigned from one position to another without losing shall hold any other office or employment in the
his rank which follows him wherever he is transferred or Government or any subdivision, agency or instrumentality
reassigned. In fact, a CESO suffers no diminution of thereof, including government-owned or controlled
salary even if assigned to a CES position with lower corporation. (Sec. 7, 2nd par., Art. IX-B, 1987
salary grade, as he is compensated according to his CES Constitution)
rank and not on the basis of the position or office he
occupies. 267. May the President, Vice-President, Members of
the Cabinet, their deputies or assistants hold any other
In the case at bar, there is no question that respondent office or employment?
Ramon S. Roco, though a CES eligible, does not possess
the appropriate CES rank, which is – CES rank level V, Ans.: The President, Vice-President, the
for the position of Regional Director of the LTO (Region Members of the Cabinet, and their deputies or assistants
V). Falling short of one of the qualifications that would shall not, unless otherwise provided in this Constitution,
complete his membership in the CES, respondent cannot hold any other office or employment during their tenure.
successfully interpose violation of security of tenure. (Sec. 13, Art. VII, 1987 Constitution)
Accordingly, he could be validly reassigned to other
positions in the career executive service. X x x 268. Does the prohibition in Section 13, Article VII of
the 1987 Constitution insofar as Cabinet members, their
Moreover, under the mobility and flexibility principles of deputies or assistants are concerned admit of the broad
the Integrated Reorganization Plan, CES personnel may exceptions made for appointive officials in general under
be reassigned or transferred from one position to Section 7, par. (2), Article IX-B?
another x x x.
Held: The threshold question therefore is: does
One last point. Respondent capitalizes on the fact that the prohibition in Section 13, Article VII of the 1987
petitioner Luis Mario M. General is not a CES eligible. Constitution insofar as Cabinet members, their deputies
The absence, however, of such CES eligibility is of no or assistants are concerned admit of the broad exceptions
moment. As stated in Part III, Chapter I, Article IV, made for appointive officials in general under Section 7,
paragraph 5(c), of the Integrated Reorganization Plan – par. (2), Article IX-B which, for easy reference is quoted
anew, thus: “Unless otherwise allowed by law or by the
“x x x the President may, in exceptional cases, appoint primary functions of his position, no appointive official
any person who is not a Career Executive Service shall hold any other office or employment in the
eligible; provided that such appointee shall subsequently government or any subdivision, agency or instrumentality
take the required Career Executive Service examination thereof, including government-owned or controlled
and that he shall not be promoted to a higher class until corporation or their subsidiaries.”
he qualified in such examination.”
We rule in the negative.
Evidently, the law allows appointment of those who are
not CES eligible, subject to the obtention of said eligibility, Xxx
in the same manner that the appointment of respondent
who does not possess the required CES rank (CES rank The practice of designating members of the
level V) for the position of Regional Director of the LTO, Cabinet, their deputies and assistants as members of the
is permitted in a temporary capacity. (General v. Roco, governing bodies or boards of various government
agencies and instrumentalities, including government- question, the intent of the framers of the Constitution was
owned and controlled corporations, became prevalent to impose a stricter prohibition on the President and his
during the time legislative powers in this country were official family in so far as holding other offices or
exercised by former President Ferdinand E. Marcos employment in the government or elsewhere is
pursuant to his martial law authority. There was a concerned.
proliferation of newly-created agencies, instrumentalities
and government-owned and controlled corporations Moreover, such intent is underscored by a
created by presidential decrees and other modes of comparison of Section 13, Article VII with other provisions
presidential issuances where Cabinet members, their of the Constitution on the disqualifications of certain
deputies or assistants were designated to head or sit as public officials or employees from holding other offices or
members of the board with the corresponding salaries, employment. Under Section 13, Article VI, “[N]o Senator
emoluments, per diems, allowances and other perquisites or Member of the House of Representatives may hold any
of office. X x x other office or employment in the Government x x x.”
Under section 5(4), Article XVI, “[N]o member of the
This practice of holding multiple offices or armed forces in the active service shall, at any time, be
positions in the government soon led to abuses by appointed in any capacity to a civilian position in the
unscrupulous public officials who took advantage of this Government, including government-owned or controlled
scheme for purposes of self-enrichment. X x x corporations or any of their subsidiaries.” Even Section
7(2), Article IX-B, relied upon by respondents provides
Particularly odious and revolting to the people’s “[U]nless otherwise allowed by law or by the primary
sense of propriety and morality in government service functions of his position, no appointive official shall hold
were the data contained therein that Roberto v. Ongpin any other office or employment in the Government.”
was a member of the governing boards of twenty-nine
(29) governmental agencies, instrumentalities and It is quite notable that in all these provisions on
corporations; Imelda R. Marcos of twenty-three (23); disqualifications to hold other office or employment, the
Cesar E.A. Virata of twenty-two (22); Arturo R. Tanco, Jr. prohibition pertains to an office or employment in the
of fifteen (15); Jesus S. Hipolito and Geronimo Z. Velasco, government and government-owned or controlled
of fourteen each (14); Cesar C. Zalamea of thirteen (13); corporations or their subsidiaries. In striking contrast is
Ruben B. Ancheta and Jose A. Rono of twelve (12) each; the wording of Section 13, Article VII which states that
Manuel P. Alba, Gilberto O. Teodoro, and Edgardo “[T]he President, Vice-President, the Members of the
Tordesillas of eleven (11) each; and Lilia Bautista and Cabinet, and their deputies or assistants shall not, unless
Teodoro Q. Pena of ten (10) each. otherwise provided in this Constitution, hold any other
office or employment during their tenure.” In the latter
The blatant betrayal of public trust evolved into provision, the disqualification is absolute, not being
one of the serious causes of discontent with the Marcos qualified by the phrase “in the Government.” The
regime. It was therefore quite inevitable and in prohibition imposed on the President and his official
consonance with the overwhelming sentiment of the family is therefore all-embracing and covers both public
people that the 1986 Constitutional Commission, and private office or employment.
convened as it was after the people successfully unseated
former President Marcos, should draft into its proposed Going further into Section 13, Article VII, the
Constitution the provisions under consideration which are second sentence provides: “They shall not, during said
envisioned to remedy, if not correct, the evils that flow tenure, directly or indirectly, practice any other
from the holding of multiple governmental offices and profession, participate in any business, or be financially
employment. X x x interested in any contract with, or in any franchise, or
special privilege granted by the Government or any
But what is indeed significant is the fact that subdivision, agency or instrumentality thereof, including
although Section 7, Article IX-B already contains a blanket government-owned or controlled corporations or their
prohibition against the holding of multiple offices or subsidiaries.” These sweeping, all-embracing prohibitions
employment in the government subsuming both elective imposed on the President and his official family, which
and appointive public officials, the Constitutional prohibitions are not similarly imposed on other public
Commission should see it fit to formulate another officials or employees such as the Members of Congress,
provision, Sec. 13, Article VII, specifically prohibiting the members of the civil service in general and members of
President, Vice-President, members of the Cabinet, their the armed forces, are proof of the intent of the 1987
deputies and assistants from holding any other office or Constitution to treat the President and his official family
employment during their tenure, unless otherwise as a class by itself and to impose upon said class stricter
provided in the Constitution itself. prohibitions.

Evidently, from this move as well as in the Xxx


different phraseologies of the constitutional provisions in
Thus, while all other appointive officials in the
civil service are allowed to hold other office or Xxx
employment in the government during their tenure when
such is allowed by law or by the primary functions of their Since the evident purpose of the framers of the
positions, members of the Cabinet, their deputies and 1987 Constitution is to impose a stricter prohibition on the
assistants may do so only when expressly authorized by President, Vice-President, members of the Cabinet, their
the Constitution itself. In other words, Section 7, Article deputies and assistants with respect to holding multiple
IX-B is meant to lay down the general rule applicable to offices or employment in the government during their
all elective and appointive public officials and employees, tenure, the exception to this prohibition must be read with
while Section 13, Article VII is meant to be the exception equal severity. On its face, the language of Section 13,
applicable only to the President, the Vice-President, Article VII is prohibitory so that it must be understood as
Members of the Cabinet, their deputies and assistants. intended to be a positive and unequivocal negation of the
privilege of holding multiple government offices and
This being the case, the qualifying phrase “unless employment. Verily, wherever the language used in the
otherwise provided in this Constitution” in Section 13, constitution is prohibitory, it is to be understood as
Article VII cannot possibly refer to the broad exceptions intended to be a positive and unequivocal negation
provided under Section 7, Article IX-B of the 1987 (Varney v. Justice, 86 Ky 596; 6 S.W. 457; Hunt v. State,
Constitution. To construe said qualifying phrase as 22 Tex. App. 396, 3 S.W. 233). The phrase “unless
respondents would have us to do, would render nugatory otherwise provided in this Constitution” must be given a
and meaningless the manifest intent and purpose of the literal interpretation to refer only to those particular
framers of the Constitution to impose a stricter prohibition instances cited in the Constitution itself, to wit: the Vice-
on the President, Vice-President, Members of the Cabinet, President being appointed as a member of the Cabinet
their deputies and assistants with respect to holding other under Section 3, par. (2), Article VII; or acting as
offices or employment in the government during their President in those instances provided under Section 7,
tenure. Respondents’ interpretation that Section 13 of pars. (2) and (3), Article VII; and, the Secretary of Justice
Article VII admits of the exceptions found in Section 7, being ex-officio member of the Judicial and Bar Council
par. (2) of Article IX-B would obliterate the distinction so by virtue of Section 8 (1), Article VIII.
carefully set by the framers of the Constitution as to when
the high-ranking officials of the Executive Branch from the Xxx
President to assistant Secretary, on the one hand, and the
generality of civil servants from the rank immediately It being clear x x x that the 1987 Constitution
below Assistant Secretary downwards, on the other, may seeks to prohibit the President, Vice-President, members
hold any other office or position in the government during of the Cabinet, their deputies or assistants from holding
their tenure. during their tenure multiple offices or employment in the
government, except in those cases specified in the
Moreover, respondents’ reading of the provisions Constitution itself and as above clarified with respect to
in question would render certain parts of the Constitution posts held without additional compensation in an ex-
inoperative. This observation applies particularly to the officio capacity as provided by law and as required by the
Vice-President who, under Section 13 of Article VII is primary functions of their office, the citation of Cabinet
allowed to hold other office or employment when so members (then called Ministers) as examples during the
authorized by the Constitution, but who as an elective debate and deliberation on the general rule laid down for
public official under Sec. 7, par. (1) of Article IX-B is all appointive officials should be considered as mere
absolutely ineligible “for appointment or designation in personal opinions which cannot override the constitution’s
any capacity to any public office or position during his manifest intent and the people’s understanding thereof.
tenure.” Surely, to say that the phrase “unless otherwise
provided in this Constitution” found in Section 13, Article In the light of the construction given to Section
VII has reference to Section 7, par. (1) of Article IX-B 13, Article VII in relation to Section 7, par. (2), Article IX-
would render meaningless the specific provisions of the B of the 1987 Constitution, Executive Order No. 284 dated
Constitution authorizing the Vice-President to become a July 23, 1987 is unconstitutional. Ostensibly restricting
member of the Cabinet (Sec. 3, Ibid.), and to act as the number of positions that Cabinet members,
President without relinquishing the Vice-Presidency where undersecretaries or assistant secretaries may hold in
the President shall not have been chosen or fails to qualify addition to their primary position to not more than two
(Sec. 7, Article VII). Such absurd consequence can be (2) positions in the government and government
avoided only by interpreting the two provisions under corporations, Executive Order No. 284 actually allows
consideration as one, i.e., Section 7, par. (1) of Article IX- them to hold multiple offices or employment in direct
B providing the general rule and the other, i.e., Section contravention of the express mandate of Section 13,
13, Article VII as constituting the exception thereto. In Article VII of the 1987 Constitution prohibiting them from
the same manner must Section 7, par. (2) of Article IX-B doing so, unless otherwise provided in the 1987
be construed vis-à-vis Section 13, Article VII. Constitution itself.
opposed to the term “allowed” used in Section 7, par. (2),
The Court is alerted by respondents to the Article IX-B of the Constitution, which is permissive.
impractical consequences that will result from a strict “Required” suggests an imposition, and therefore,
application of the prohibition mandated under Section 13, obligatory in nature) by the primary functions of said
Article VII on the operations of the Government, officials’ office. The reason is that these posts do not
considering that Cabinet members would be stripped of comprise “any other office” within the contemplation of
their offices held in an ex-officio capacity, by reason of the constitutional prohibition but are properly an
their primary positions or by virtue of legislation. As imposition of additional duties and functions on said
earlier clarified in this decision, ex-officio posts held by officials (Martin v. Smith, 140 A.L.R. 1073; Ashmore v.
the executive official concerned without additional Greater Greenville Sewer District, 173 A.L.R. 407). To
compensation as provided by law and as required by the characterize these posts otherwise would lead to absurd
primary functions of his office do not fall under the consequences, among which are: The President of the
definition of “any other office” within the contemplation Philippines cannot chair the National Security Council
of the constitutional prohibition. With respect to other reorganized under Executive Order No. 115 (December
offices or employment held by virtue of legislation, 24, 1986). Neither can the Vice-President, the Executive
including chairmanships or directorships in government- Secretary, and the Secretaries of National Defense,
owned or controlled corporations and their subsidiaries, Justice, Labor and Employment and Local Government sit
suffice it to say that the feared impractical consequences in this Council, which would then have no reason to exist
are more apparent than real. Being head of an executive for lack of a chairperson and members. The respective
department is no mean job. It is more than a full-time undersecretaries and assistant secretaries, would also be
job, requiring full attention, specialized knowledge, skills prohibited.
and expertise. If maximum benefits are to be derived
from a department head’s ability and expertise, he should Xxx
be allowed to attend to his duties and responsibilities
without the distraction of other governmental offices or Indeed, the framers of our Constitution could not
employment. He should be precluded from dissipating his have intended such absurd consequences. A
efforts, attention and energy among too many positions Constitution, viewed as a continuously operative charter
and responsibility, which may result in haphazardness and of government, is not to be interpreted as demanding the
inefficiency. Surely the advantages to be derived from impossible or the impracticable; and unreasonable or
this concentration of attention, knowledge and expertise, absurd consequences, if possible, should be avoided
particularly at this stage of our national and economic (Hirabayashi v. United States, 320 U.S. 81, 87 L. Ed. 1774,
development, far outweigh the benefits, if any, that may 63 S. Ct. 1375; others omitted).
be gained from a department head spreading himself too
thin and taking in more than what he can handle. To reiterate, the prohibition under Section 13,
Article VII is not to be interpreted as covering positions
Finding Executive Order No. 284 to be held without additional compensation in ex-officio
constitutionally infirm, the Court hereby orders capacities as provided by law and as required by the
respondents x x x to immediately relinquish their other primary functions of the concerned official’s office. The
offices or employment, as herein defined, in the term ex-officio means “from office; by virtue of office.” It
government, including government-owned or controlled refers to an “authority derived from official character
corporations and their subsidiaries. (Civil Liberties merely, not expressly conferred upon the individual
Union v. Executive Secretary, 194 SCRA 317, Feb. character, but rather annexed to the official position.” Ex
22, 1991, En Banc [Fernan, CJ]) officio likewise denotes an “act done in an official
character, or as a consequence of office, and without any
other appointment or authority than that conferred by the
269. Does the prohibition against holding dual or office.” (Black’s Law Dictionary, p. 516; 15A Words and
multiple offices or employment under Section 13, Article Phrases, p. 392) An ex-officio member of a board is one
VII of the Constitution apply to posts occupied by the who is a member by virtue of his title to a certain office,
Executive officials specified therein without additional and without further warrant or appointment (15A Words
compensation in an ex-officio capacity as provided by law and Phrases, p. 392). To illustrate, by express provision
and as required by the primary functions of said officials’ of law, the Secretary of Transportation and
office? Communications is the ex-officio Chairman of the Board
of the Philippine Ports Authority (Sec. 7, E.O. 778), and
Held: The prohibition against holding dual or the Light Rail Transit Authority (Sec. 1, E.O. 210).
multiple offices or employment under Section 13, Article
VII of the Constitution must not, however, be construed The Court had occasion to explain the meaning of
as applying to posts occupied by the Executive officials an ex-officio position in Rafael v. Embroidery and Apparel
specified therein without additional compensation in an Control and Inspection Board (21 SCRA 336 [1967]),
ex-officio capacity as provided by law and as required (As thus: “An examination of Section 2 of the questioned
statute (R.A. 3137) reveals that for the chairman and the President and his alter egos, the Cabinet members, to
members of the Board to qualify they need only be have control of all the executive departments, bureaus
designated by the respective department heads. With the and offices and to ensure that the laws are faithfully
exception of the representative from the private sector, executed (Section 17, Article VII). Without these
they sit ex-officio. I order to be designated they must additional duties and functions being assigned to the
already be holding positions in the offices mentioned in President and his official family to sit in the governing
the law. Thus, for instance, one who does not hold a bodies or boards of governmental agencies or
previous appointment in the Bureau of Customs, cannot, instrumentalities in an ex-officio capacity as provided by
under the act, be designated a representative from that law and as required by their primary functions, they would
office. The same is true with respect to the be deprived of the means for control and supervision,
representatives from the other offices. No new thereby resulting in an unwieldy and confused
appointments are necessary. This is as it should be, bureaucracy.
because the representatives so designated merely
perform duties in the Board in addition to those already It bears repeating though that in order that such
performed under their original appointments.” (Italics additional duties or functions may not transgress the
supplied) prohibition embodied in Section 13, Article VII of the 1987
Constitution, such additional duties or functions must be
The term “primary” used to describe “functions” required by the primary functions of the official
refers to the order of importance and thus means chief or concerned, who is to perform the same in an ex-officio
principal function. The term is not restricted to the capacity as provided by law, without receiving any
singular but may refer to the plural (33A Words and additional compensation therefor.
Phrases, p. 210, citing Collector of Revenue v. Louisiana
Ready Mix Co., La. App., 197 S. 2d 141, 145). The The ex-officio position being actually and in legal
additional duties must not only be closely related to, but contemplation part of the principal office, it follows that
must be required by the official’s primary functions. the official concerned has no right to receive additional
Examples of designations to positions by virtue of one’s compensation for his services in the said position. The
primary functions are the Secretaries of Finance and reason is that these services are already paid for and
Budget sitting as members of the Monetary Board, and covered by the compensation attached to his principal
the Secretary of Transportation and Communications office. It should be obvious that if, say, the Secretary of
acting as Chairman of the Maritime Industry Authority Finance attends a meeting of the Monetary Board as an
(Sec. 7, P.D. No. 474) and the Civil Aeronautics Board. ex-officio member thereof, he is actually and in legal
contemplation performing the primary function of his
If the functions to be performed are merely principal office in defining policy in monetary and banking
incidental, remotely related, inconsistent, incompatible, or matters, which come under the jurisdiction of his
otherwise alien to the primary function of a cabinet department. For such attendance, therefore, he is not
official, such additional functions would fall under the entitled to collect any extra compensation, whether it be
purview of “any other office” prohibited by the in the form of a per diem or an honorarium or an
Constitution. An example would be the Press allowance, or some other such euphemism. By whatever
Undersecretary sitting as a member of the Board of the name it is designated, such additional compensation is
Philippine Amusement and Gaming Corporation. The prohibited by the Constitution. (Civil Liberties Union v.
same rule applies to such positions which confer on the Executive Secretary, 194 SCRA 317, Feb. 22, 1991,
cabinet official management functions and/or monetary En Banc [Fernan, CJ])
compensation, such as but not limited to chairmanships
or directorships in government-owned or controlled 270. Should members of the Cabinet appointed to
corporations and their subsidiaries. other positions in the government pursuant to Executive
Order No. 284 which later was declared unconstitutional
Mandating additional duties and functions to the by the SC for being violative of Section 13, Article VII of
President, Vice-President, Cabinet Members, their the Constitution be made to reimburse the government
deputies or assistants which are not inconsistent with for whatever pay and emoluments they received from
those already prescribed by their offices or appointments holding such other positions?
by virtue of their special knowledge, expertise and skill in
their respective executive offices is a practice long- Held: During their tenure in the questioned
recognized in many jurisdictions. It is a practice justified positions, respondents may be considered de facto
by the demands of efficiency, policy direction, continuity officers and as such entitled to emoluments for actual
and coordination among the different offices in the services rendered (Castillo v.
Arrieta, G.R. No. L-31444,
Executive Branch in the discharge of its multifarious tasks November 13, 1974, 61 SCRA 55). It has been held that
of executing and implementing laws affecting national “in cases where there is no de jure officer, a de facto
interest and general welfare and delivering basic services officer, who, in good faith has had possession of the office
to the people. It is consistent with the power vested on and has discharged the duties pertaining thereto, is
legally entitled to the emoluments of the office, and may neither does she hold Career Executive Service Eligibility,
in an appropriate action recover the salary, fees and other which are necessary qualifications for holding the
compensations attached to the office. This doctrine is, position of Director IV as prescribed in the Qualifications
undoubtedly, supported on equitable grounds since it Standards (Revised 1987) issued by the Civil Service
seems unjust that the public should benefit by the Commission. Obviously, petitioner does not enjoy
services of an officer de facto and then be freed from all security of tenure as Director IV. X x x
liability to pay any one for such services (Patterson v.
Benson, 112 Pac. 801, 32 L.R.A. [NS] 949). Any per diem, Xxx
allowances or other emoluments received by the
respondents by virtue of actual services rendered in the Having been appointed merely in a temporary or
questioned positions may therefore be retained by them. acting capacity, and not possessed of the necessary
(Civil Liberties Union v. Executive Secretary, 194 qualifications to hold the position of Director IV,
SCRA 317, Feb. 22, 1991, En Banc [Fernan, CJ]) petitioner has no legal basis in claiming that her
reassignment was contrary to the Civil Service Law. X x
271. May a Senator or Congressman hold any other x
office or employment?
Still, petitioner assails her reassignment, carried out
Ans.: No Senator or Member of the House of during the election period, as a prohibited act under
Representatives may hold any other office or Section 261 (h) of the Omnibus Election Code x x x.
employment in the government, or any subdivision,
agency, or instrumentality thereof, including Xxx
government-owned or controlled corporations or their
subsidiaries, during his term without forfeiting his seat. Petitioner claims that Benipayo failed to secure the
Neither shall he be appointed to any office which may approval of the COMELEC en banc to effect transfers or
have been created or the emoluments thereof increased reassignments of COMELEC personnel during the
during the term for which he was elected (Sec. 13, Art. election period. Moreover, petitioner insists that the
VI, 1987 Constitution). The first sentence is referred to COMELEC en banc must concur to every transfer or
as an incompatible office; the second is a forbidden reassignment of COMELEC personnel during the election
office. period.

Contrary to petitioner’s allegation, the COMELEC did in


fact issue COMELEC Resolution No. 3300 dated
272. Petitioner claims that Benipayo has no authority November 6, 2000, exempting the COMELEC from
to remove her as Director IV of the EID and reassign her Section 261 (h) of the Omnibus Election Code. X x x
to the Law Department. Petitioner further argues that
only the COMELEC, acting as a collegial body, can Xxx
authorize such reappointment. Moreover, petitioner
maintains that a reassignment without her consent The proviso in COMELEC Resolution No. 3300, requiring
amounts to removal from office without due process and due notice and hearing before any transfer or
therefore illegal. reassignment can be made within thirty days prior to
election day, refers only to COMELEC field personnel and
Held: Petitioner’s posturing will hold water if not to head office personnel like the petitioner. Under
Benipayo does not possess any color of title to the office the Revised Administrative Code (see Section 7 [4],
of Chairman of the COMELEC. We have ruled, however, Chapter 2, Subtitle C, Book V of the Revised
that Benipayo is the de jure COMELEC Chairman, and Administrative Code), the COMELEC Chairman is the sole
consequently he has full authority to exercise all the officer specifically vested with the power to transfer or
powers of that office for so long as his ad interim reassign COMELEC personnel. The COMELEC Chairman
appointment remains effective. X x x. The Chairman, will logically exercise the authority to transfer or
as the Chief Executive of the COMELEC, is expressly reassign COMELEC personnel pursuant to COMELEC
empowered on his own authority to transfer or reassign Resolution No. 3300. The COMELEC en banc cannot
COMELEC personnel in accordance with the Civil Service arrogate unto itself this power because that will mean
Law. In the exercise of this power, the Chairman is not amending the Revised Administrative Code, an act the
required by law to secure the approval of the COMELEC COMELEC en banc cannot legally do.
en banc.
COMELEC Resolution No. 3300 does not require that
Petitioner’s appointment papers x x x every transfer or reassignment of COMELEC personnel
indisputably show that she held her Director IV position should carry the concurrence of the COMELEC as a
in the EID only in an acting or temporary capacity. collegial body. Interpreting Resolution No. 3300 to
Petitioner is not a Career Executive Service (CES), and require such concurrence will render the resolution
meaningless since the COMELEC en banc will have to specifically provides that “an appointment accepted by
approve every personnel transfer or reassignment, the appointee cannot be withdrawn or revoked by the
making the resolution utterly useless. Resolution No. appointing authority and shall remain in force and in
3300 should be interpreted for what it is, an approval to effect until disapproved by the Commission.” Thus, it is
effect transfers and reassignments of personnel, without the CSC that is authorized to recall an appointment
need of securing a second approval from the COMELEC initially approved, but only when such appointment and
en banc to actually implement such transfer or approval are proven to be in disregard of applicable
reassignment. provisions of the civil service law and regulations
(Debulgado v. Civil Service Commission, 237 SCRA
The COMELEC Chairman is the official expressly 184, 200 [1994]).
authorized by law to transfer or reassign COMELEC
personnel. The person holding that office, in a de jure Moreover, Section 10 of the same rule provides:
capacity, is Benipayo. The COMELEC en banc, in
COMELEC Resolution No. 3300, approved the transfer or Sec. 10. An appointment issued in accordance with
reassignment of COMELEC personnel during the election pertinent laws and rules shall take effect immediately
period. Thus, Benipayo’s order reassigning petitioner upon its issuance by the appointing authority, and if the
from the EID to the Law Department does not violate appointee has assumed the duties of the position, he shall
Section 261 (h) of the Omnibus Election Code. For the be entitled to receive his salary at once without awaiting
same reason, Benipayo’s order designating Cinco the approval of his appointment by the Commission. The
Officer-in-Charge of the EID is legally unassailable. appointment shall remain effective until disapproved by
(Matibag v. Benipayo, 380 SCRA 49, April 2, 2002, the Commission. In no case shall an appointment take
En Banc [Carpio]) effect earlier than the date of its issuance.

273. May the appointment of a person assuming a Section 20 of Rule VI also provides:
position in the civil service under a completed
appointment be validly recalled or revoked? Sec. 20. Notwithstanding the initial approval of an
appointment, the same may be recalled on any of the
Held: It has been held that upon the issuance of an following grounds:
appointment and the appointee’s assumption of the
position in the civil service, “he acquires a legal right (a) Non-compliance with the
which cannot be taken away either by revocation of the procedures/criteria provided in the agency’s Merit
appointment or by removal except for cause and with Promotion Plan;
previous notice and hearing.” (Mauna v. Civil Service (b) Failure to pass through the agency’s
Commission, 232 SCRA 388, 398 [1994]) Moreover, it is Selection/Promotion Board;
well-settled that the person assuming a position in the (c) Violation of the existing collective
civil service under a completed appointment acquires a agreement between management and employees relative
legal, not just an equitable, right to the position. This to promotion; or
right is protected not only by statute, but by the (d) Violation of other existing civil
Constitution as well, which right cannot be taken away by service law, rules and regulations.
either revocation of the appointment, or by removal,
unless there is valid cause to do so, provided that there is Accordingly, the appointments of the private respondents
previous notice and hearing. (Aquino v. Civil Service may only be recalled on the above-cited grounds. And
Commission, 208 SCRA 240, 248 [1992]) yet, the only reason advanced by the petitioner to justify
the recall was that these were “midnight appointments.”
Petitioner admits that his very first official act upon The CSC correctly ruled, however, that the constitutional
assuming the position of town mayor was to issue Office prohibition on so-called “midnight appointments,”
Order No. 95-01 which recalled the appointments of the specifically those made within two (2) months
private respondents. There was no previous notice, much immediately prior to the next presidential elections,
less a hearing accorded to the latter. Clearly, it was applies only to the President or Acting President. (De
petitioner who acted in undue haste to remove the private Rama v. Court of Appeals, 353 SCRA 94, Feb. 28,
respondents without regard for the simple requirements 2001, En Banc [Ynares-Santiago])
of due process of law. While he argues that the
appointing power has the sole authority to revoke said
appointments, there is no debate that he does not have 274. The Philippine National Red Cross (PNRC) is a
blanket authority to do so. Neither can he question the government-owned and controlled corporation with an
CSC’s jurisdiction to affirm or revoke the recall. original charter under R.A. No. 95, as amended. Its
charter, however, was amended to vest in it the authority
Rule V, Section 9 of the Omnibus Implementing to secure loans, be exempted from payment of all duties,
Regulations of the Revised Administrative Code taxes, fees and other charges, etc. With the amendment
of its charter, has it been “impliedly converted to a private Commission, 204 SCRA 965, 968) In other words, we
corporation”? overrule prior decisions holding that the Civil Service Law
“does not contemplate a review of decisions exonerating
Held: The test to determine whether a corporation is officers or employees from administrative charges”
government owned or controlled, or private in nature is enunciated in Paredes v. Civil Service Commission (192
simple. Is it created by its own charter for the exercise SCRA 84); Mendez v. Civil Service Commission (204 SCRA
of a public function, or by incorporation under the general 965); Magpale v. Civil Service Commission (215 SCRA
corporation law? Those with special charters are 398); Navarro v. Civil Service Commission and Export
government corporations subject to its provisions, and its Processing Zone Authority (226 SCRA 207) and more
employees are under the jurisdiction of the Civil Service recently Del Castillo v. Civil Service Commission (237
Commission. The PNRC was not “impliedly converted to SCRA 184). (CSC v. Pedro O. Dacoycoy, G.R. No.
a private corporation” simply because its charter was 135805, April 29, 1999, En Banc [Pardo])
amended to vest in it the authority to secure loans, be
exempted from payment of all duties, taxes, fees and 277. What is preventive suspension? Discuss its
other charges, etc. (Camporedondo v. NLRC, G.R. nature.
No. 129049, Aug. 6, 1999, 1st Div. [Pardo])
Held: Imposed during the pendency of an administrative
275. What is a primarily confidential position? What is investigation, preventive suspension is not a penalty in
the test to determine whether a position is primarily itself. It is merely a measure of precaution so that the
confidential or not? employee who is charged may be separated, for obvious
reasons, from the scene of his alleged misfeasance while
Held: A primarily confidential position is one which the same is being investigated. Thus preventive
denotes not only confidence in the aptitude of the suspension is distinct from the administrative penalty of
appointee for the duties of the office but primarily close removal from office such as the one mentioned in Sec.
intimacy which ensures freedom from intercourse without 8(d) of P.D. No. 807. While the former may be imposed
embarrassment or freedom from misgivings or betrayals on a respondent during the investigation of the charges
of personal trust or confidential matters of state. (De los against him, the latter is the penalty which may only be
Santos v. Mallare, 87 Phil. 289 [1950]) meted upon him at the termination of the investigation or
the final disposition of the case. (Beja, Sr. v. CA, 207
Under the proximity rule, the occupant of a particular SCRA 689, March 31, 1992 [Romero])
position could be considered a confidential employee if
the predominant reason why he was chosen by the 278. Discuss the kinds of preventive suspension under
appointing authority was the latter’s belief that he can the Civil Service Law. When may a civil service employee
share a close intimate relationship with the occupant placed under preventive suspension be entitled to
which ensures freedom of discussion without fear or compensation?
embarrassment or misgivings of possible betrayal of
personal trust or confidential matters of state. Withal, Held: There are two kinds of preventive suspension of
where the position occupied is more remote from that of civil service employees who are charged with offenses
the appointing authority, the element of trust between punishable by removal or suspension: (1) preventive
them is no longer predominant. (CSC v. Salas, 274 suspension pending investigation (Sec. 51, Civil Service
SCRA 414, June 19, 1997) Law, EO No. 292) and (2) preventive suspension pending
appeal if the penalty imposed by the disciplining authority
276. Does the Civil Service Law contemplate a review is suspension or dismissal and, after review, the
of decisions exonerating officers or employees from respondent is exonerated (Section 47, par. 4, Civil Service
administrative charges? Law, EO No. 292).

Held: By this ruling, we now expressly abandon and Preventive suspension pending investigation is not a
overrule extant jurisprudence that “the phrase ‘party penalty. It is a measure intended to enable the
adversely affected by the decision’ refers to the disciplining authority to investigate charges against
government employee against whom the administrative respondent by preventing the latter from intimidating or
case is filed for the purpose of disciplinary action which in any way influencing witnesses against him. If the
may take the form of suspension, demotion in rank or investigation is not finished and a decision is not rendered
salary, transfer, removal or dismissal from office” and not within that period, the suspension will be lifted and the
included are “cases where the penalty imposed is respondent will automatically be reinstated. If after
suspension for not more than thirty (30) days or fine in investigation respondent is found innocent of the charges
an amount not exceeding thirty days salary” (Paredes v. and is exonerated, he should be reinstated. However, no
Civil Service Commission, 192 SCRA 84, 85) or “when compensation was due for the period of preventive
respondent is exonerated of the charges, there is no suspension pending investigation. The Civil Service Act of
occasion for appeal.” (Mendez v. Civil Service 1959 (R.A. No. 2260) providing for compensation in such
a case once the respondent was exonerated was revised We have previously interpreted the phrase “under his
in 1975 and the provision on the payment of salaries authority” to mean that the Ombudsman can preventively
during suspension was deleted. suspend all officials under investigation by his office,
regardless of the branch of government in which they are
But although it is held that employees who are employed (Buenaseda v. Flavier, 226 SCRA 645, 654
preventively suspended pending investigation are not [1993]), excepting of course those removable by
entitled to the payment of their salaries even if they are impeachment, members of Congress and the Judiciary.
exonerated, they are entitled to compensation for the
period of their suspension pending appeal if eventually The power to preventively suspend is available not only
they are found innocent. to the Ombudsman but also to the Deputy Ombudsman.
This is the clear import of Section 24 of R.A. 6770
Preventive suspension pending investigation x x x is not a abovecited.
penalty but only a means of enabling the disciplining
authority to conduct an unhampered investigation. On There can be no question in this case as to the power and
the other hand, preventive suspension pending appeal is authority of respondent Deputy Ombudsman to issue an
actually punitive although it is in effect subsequently order of preventive suspension against an official like the
considered illegal if respondent is exonerated and the petitioner, to prevent that official from using his office to
administrative decision finding him guilty is reversed. intimidate or influence witnesses (Gloria v. CA, et al., G.R.
Hence, he should be reinstated with full pay for the period No. 131012, April 21, 1999, p. 7, 306 SCRA 287) or to
of the suspension. (Gloria v. CA, G.R. No. 131012, tamper with records that might be vital to the prosecution
April 21, 1999, En Banc [Mendoza]) of the case against him (Yasay, Jr. v. Desierto, et al., G.R.
No. 134495, December 28, 1998, p. 9, 300 SCRA 494).
279. Discuss the power of Ombudsman to conduct In our view, the present controversy simply boils down to
administrative investigations, and to impose preventive this pivotal question: Given the purpose of preventive
suspension. suspension and the circumstances of this case, did
respondent Deputy Ombudsman commit a grave abuse of
Held: Worth stressing, to resolve the present discretion when he set the period of preventive
controversy, we must recall that the authority of the suspension at six months?
Ombudsman to conduct administrative investigations is
mandated by no less than the Constitution. x x x Preventive suspension under Sec. 24, R.A. 6770 x x x may
be imposed when, among other factors, the evidence of
R.A. 6770, the Ombudsman Law, further grants the Office guilt is strong. The period for which an official may be
of the Ombudsman the statutory power to conduct preventively suspended must not exceed six months. In
administrative investigations. X x x this case, petitioner was preventively suspended and
ordered to cease and desist from holding office for the
Section 21 of R.A. 6770 names the officials subject to the entire period of six months, which is the maximum
Ombudsman’s disciplinary authority x x x. provided by law.

Petitioner is an elective local official accused of grave The determination of whether or not the evidence of guilt
misconduct and dishonesty. That the Office of the is strong as to warrant preventive suspension rests with
Ombudsman may conduct an administrative investigation the Ombudsman (Nera v. Garcia, 106 Phil. 1031 [1960];
into the acts complained of, appears clear from the others omitted.). The discretion as regards the period of
foregoing provisions of R.A. 6770. such suspension also necessarily belongs to the
Ombudsman, except that he cannot extend the period of
However, the question of whether or not the Ombudsman suspension beyond that provided by law (Castillo-Co v.
may conduct an investigation over a particular act or Barbers, supra.). But, in our view, both the strength of
omission, is different from the question of whether or not the evidence to warrant said suspension and the propriety
petitioner, after investigation, may be held of the length or period of suspension imposed on
administratively liable. This distinction ought here to be petitioner are properly raised in this petition for certiorari
kept in mind, even as we must also take note that the and prohibition. X x x
power to investigate is distinct from the power to suspend
preventively an erring public officer. Xxx

Likewise worthy of note, the power of the Office of the Given these findings, we cannot say now that there is no
Ombudsman to preventively suspend an official subject to evidence sufficiently strong to justify the imposition of
its administrative investigation is provided by specific preventive suspension against petitioner. But considering
provision of law. X x x its purpose and the circumstances in the case brought
before us, it does appear to us that the imposition of the
maximum period of six months is unwarranted.
X x x [G]ranting that now the evidence against petitioner “Indeed, there is nothing in the Local Government Code
is already strong, even without conceding that initially it to indicate that it has repealed, whether expressly or
was weak, it is clear to us that the maximum six-month impliedly, the pertinent provisions of the Ombudsman Act.
period is excessive and definitely longer than necessary The two statutes on the specific matter in question are
for the Ombudsman to make its legitimate case against not so inconsistent, let alone irreconcilable, as to compel
petitioner. We must conclude that the period during us to only uphold one and strike down the other.” (Hagad
which petitioner was already preventively suspended, has v. Gozo-Dadole, supra, 251-252)
been sufficient for the lawful purpose of preventing
petitioner from hiding and destroying needed documents, It was also argued in Hagad, that the six-month
or harassing and preventing witnesses who wish to preventive suspension under the Ombudsman Law is
appear against him. (Garcia v. Mojica, 314 SCRA 207, “much too repugnant” to the 60-day period that may be
Sept. 10, 1999, 2nd Div. [Quisumbing]) imposed under the Local Government Code. But per J.
Vitug, “the two provisions govern differently.” (Id., at
280. Distinguish preventive suspension under the 253-254)
Local Government Code from preventive suspension
under the Ombudsman Act. However, petitioner now contends that Hagad did
not settle the question of whether a local elective official
Held: We reach the foregoing conclusion, however, may be preventively suspended even before the issues
without necessarily subscribing to petitioner’s claim that could be joined. Indeed it did not, but we have held in
the Local Government Code, which he averred should other cases that there could be preventive suspension
apply to this case of an elective local official, has been even before the charges against the official are heard, or
violated. True, under said Code, preventive suspension before the official is given an opportunity to prove his
may only be imposed after the issues are joined, and only innocence (supra at note 14, excluding the case of
for a maximum period of sixty days. Here, petitioner was Buenaseda v. Flavier). Preventive suspension is
suspended without having had the chance to refute first merely a preliminary step in an administrative
the charges against him, and for the maximum period of investigation and is not in any way the final determination
six months provided by the Ombudsman Law. But as of the guilt of the official concerned.
respondents argue, administrative complaints
commenced under the Ombudsman Law are distinct from Petitioner also avers that the suspension order
those initiated under the Local Government Code. against him was issued in violation of Section 26[2] of the
Respondents point out that the shorter period of Ombudsman Law x x x.
suspension under the Local Government Code is intended
to limit the period of suspension that may be imposed by Petitioner argues that before an inquiry may be
a mayor, a governor, or the President, who may be converted into a full-blown administrative investigation,
motivated by partisan political considerations. In contrast the official concerned must be given 72 hours to answer
the Ombudsman, who can impose a longer period of the charges against him. In his case, petitioner says the
preventive suspension, is not likely to be similarly inquiry was converted into an administrative investigation
motivated because it is a constitutional body. The without him being given the required number of hours to
distinction is valid but not decisive, in our view, of whether answer.
there has been grave abuse of discretion in a specific case
of preventive suspension. Indeed, it does not appear that petitioner was
given the requisite 72 hours to submit a written answer
Xxx to the complaint against him. This, however, does not
make invalid the preventive suspension order issued
Respondents may be correct in pointing out the against him. As we have earlier stated, a preventive
reason for the shorter period of preventive suspension suspension order may be issued even before the charges
imposable under the Local Government Code. Political against the official concerned is heard.
color could taint the exercise of the power to suspend
local officials by the mayor, governor, or President’s Moreover, respondents state that petitioner was given 10
office. In contrast the Ombudsman, considering the days to submit his counter-affidavit to the complaint filed
constitutional origin of his Office, always ought to be by respondent Tagaan. We find this 10-day period is in
insulated from the vagaries of politics, as respondents keeping with Section 5[a] of the Rules of Procedure of the
would have us believe. Office of the Ombudsman x x x. (Garcia v. Mojica, 314
SCRA 207, Sept. 10, 1999, 2nd Div. [Quisumbing])
In Hagad v. Gozo-Dadole (251 SCRA 242 [1995]),
on the matter of whether or not the Ombudsman has 281. Does Section 13, Republic Act No. 3019 exclude
been stripped of his power to investigate local elective from its coverage the members of Congress and,
officials by virtue of the Local Government Code, we said: therefore, the Sandiganbayan erred in decreeing the
preventive suspension order against Senator Miriam upheld Sandiganbayan’s authority to decree the
Defensor-Santiago? Will the order of suspension suspension of public officials and employees indicted
prescribed by Republic Act No. 3019 not encroach on the before it.
power of Congress to discipline its own ranks under the
Constitution? Section 13 of Republic Act No. 3019 does not
state that the public officer concerned must be
Held: The petition assails the authority of the suspended only in the office where he is alleged to have
Sandiganbayan to decree a ninety-day preventive committed the acts with which he has been charged.
suspension of Mme. Miriam Defensor-Santiago, a Thus, it has been held that the use of the word “office”
Senator of the Republic of the Philippines, from any would indicate that it applies to any office which the
government position, and furnishing a copy thereof to officer charged may be holding, and not only the
the Senate of the Philippines for the implementation of particular office under which he stands accused. (Bayot
the suspension order. v. Sandiganbayan, supra; Segovia v. Sandiganbayan,
supra.)
The authority of the Sandiganbayan to order the
preventive suspension of an incumbent public official En passant, while the imposition of suspension is
charged with violation of the provisions of Republic Act not automatic or self-operative as the validity of the
No. 3019 has both legal and jurisprudential support. X x information must be determined in a pre-suspension
x hearing, there is no hard and fast rule as to the conduct
thereof. It has been said that –
In the relatively recent case of Segovia v.
Sandiganbayan (288 SCRA 328 [1998]), the Court “ ‘x x x No specific rules need be laid down for such pre-
reiterated: suspension hearing. Suffice it to state that the accused
should be given a fair and adequate opportunity to
“The validity of Section 13, R.A. 3019, as amended – challenge the VALIDITY OF THE CRIMINAL
treating of the suspension pendente lite of an accused PROCEEDINGS against him, e.g., that he has not been
public officer – may no longer be put at issue, having afforded the right of due preliminary investigation; that
been repeatedly upheld by this Court. the acts for which he stands charged do not constitute a
violation of the provisions of Republic Act 3019 or the
“X x x bribery provisions of the Revised Penal Code which
would warrant his mandatory suspension from office
“The provision of suspension pendente lite applies to all under Section 13 of the Act; or he may present a motion
persons indicted upon a valid information under the Act, to quash the information on any of the grounds provided
whether they be appointive or elective officials; or for in Rule 117 of the Rules of Court x x x.’
permanent or temporary employees, or pertaining to the
career or non-career service.” (At pp. 336-337) “x x x

It would appear, indeed, to be a ministerial duty “Likewise, he is accorded the right to challenge the
of the court to issue an order of suspension upon propriety of his prosecution on the ground that the acts
determination of the validity of the information filed for which he is charged do not constitute a violation of
before it. Once the information is found to be sufficient Rep. Act 3019, or of the provisions on bribery of the
in form and substance, the court is bound to issue an Revised Penal Code, and the right to present a motion to
order of suspension as a matter of course, and there quash the information on any other grounds provided in
seems to be “no ifs and buts about it.” (Libanan v. Rule 117 of the Rules of Court.
Sandiganbayan, 163 SCRA 163 [1988]) Explaining the
nature of the preventive suspension, the Court in the “However, a challenge to the validity of the criminal
case of Bayot v. Sandiganbayan (128 SCRA 383 [1984]) proceedings on the ground that the acts for which the
observed: accused is charged do not constitute a violation of the
provisions of Rep. Act No. 3019, or of the provisions on
“x x x It is not a penalty because it is not imposed as a bribery of the Revised Penal Code, should be treated
result of judicial proceedings. In fact, if acquitted, the only in the same manner as a challenge to the criminal
official concerned shall be entitled to reinstatement and proceeding by way of a motion to quash on the ground
to the salaries and benefits which he failed to receive provided in Paragraph (a), Section 2 of Rule 117 of the
during suspension.” (At p. 386) Rules of Court, i.e., that the facts charged do not
constitute an offense. In other words, a resolution of
In issuing the preventive suspension of the challenge to the validity of the criminal proceeding,
petitioner, the Sandiganbayan merely adhered to the on such ground, should be limited to an inquiry whether
clear and unequivocal mandate of the law, as well as the the facts alleged in the information, if hypothetically
jurisprudence in which the Court has, more than once, admitted, constitute the elements of an offense
punishable under Rep. Act 3019 or the provisions on
bribery of the Revised Penal Code.” (Luciano v. Mariano, “x x x. Petitioner’s invocation of Section 16 (3), Article
40 SCRA 187 [1971]; People v. Albano, 163 SCRA 511, VI of the Constitution – which deals with the power of
517-519 [1988]) each House of Congress inter alia to ‘punish its Members
for disorderly behavior,’ and ‘suspend or expel a
The law does not require that the guilt of the Member’ by a vote of two-thirds of all its Members
accused must be established in a pre-suspension subject to the qualification that the penalty of
proceeding before trial on the merits proceeds. Neither suspension, when imposed, should not exceed sixty days
does it contemplate a proceeding to determine (1) the – in unavailing, as it appears to be quite distinct from
strength of the evidence of culpability against him, (2) the suspension spoken of in Section 13 of RA 3019,
the gravity of the offense charged, or (3) whether or not which is not a penalty but a preliminary, preventive
his continuance in office could influence the witnesses or measure, prescinding from the fact that the latter is not
pose a threat to the safety and integrity of the records being imposed on petitioner for misbehavior as a
and other evidence before the court could have a valid Member of the House of Representatives.”
basis in decreeing preventive suspension pending the
trial of the case. All it secures to the accused is The doctrine of separation of powers by itself may not
adequate opportunity to challenge the validity or be deemed to have effectively excluded Members of
regularity of the proceedings against him, such as, that Congress from Republic Act No. 3019 nor from its
he has not been afforded the right to due preliminary sanctions. The maxim simply recognizes each of the
investigation, that the acts imputed to him do not three co-equal and independent, albeit coordinate,
constitute a specific crime warranting his mandatory branches of the government – the Legislative, the
suspension from office under Section 13 of Republic Act Executive and the Judiciary – has exclusive prerogatives
No. 3019, or that the information is subject to quashal and cognizance within its own sphere of influence and
on any of the grounds set out in Section 3, Rule 117, of effectively prevents one branch from unduly intruding
the Revised Rules on Criminal Procedure (Segovia v. into the internal affairs of either branch.
Sandiganbayan, supra; Resolution of the Supreme Court
in A.M. No. 00-05-03-SC, dated 03 October 2000, which Parenthetically, it might be well to elaborate a bit.
became effective on 01 December 2000) Section 1, Article VIII, of the 1987 Constitution,
empowers the Court to act not only in the settlement of
Xxx “actual controversies involving rights which are legally
demandable and enforceable,” but also in the
The pronouncement, upholding the validity of determination of “whether or not there has been a grave
the information filed against petitioner, behooved abuse of discretion amounting to lack or excess of
Sandiganbayan to discharge its mandated duty to jurisdiction on the part of any branch or instrumentality
forthwith issue the order of preventive suspension. of the government.” The provision allowing the Court to
look into any possible grave abuse of discretion
The order of suspension prescribed by Republic committed by any government instrumentality has
Act No. 3019 is distinct from the power of Congress to evidently been couched in general terms in order to
discipline its own ranks under the Constitution which make it malleable to judicial interpretation in the light of
provides that each – any emerging milieu. In its normal concept, the term
has been said to imply an arbitrary, despotic, capricious
“x x x house may determine the rules of its proceedings, or whimsical exercise of judgment amounting to lack or
punish its Members for disorderly behavior, and, with excess of jurisdiction. When the question, however,
the concurrence of two-thirds of all its Members, pertains to an affair internal to either of Congress or the
suspend or expel a Member. A penalty of suspension, Executive, the Court subscribes to the view that unless
when imposed, shall not exceed sixty days.” (Section an infringement of any specific Constitutional
16[3], Article VI, 1987 Constitution) proscription thereby inheres the Court should not deign
substitute its own judgment over that of any of the
The suspension contemplated in the above other two branches of government. It is an impairment
constitutional provision is a punitive measure that is or a clear disregard of a specific constitutional precept or
imposed upon determination by the Senate or the House provision that can unbolt the steel door for judicial
of Representatives, as the case may be, upon an erring intervention. If any part of the Constitution is not, or
member. Thus, in its resolution in the case of Ceferino ceases to be, responsive to contemporary needs, it is
Paredes, Jr. v. Sandiganbayan, et al. (G.R. No. 118364, the people, not the Court, who must promptly react in
08 August 1995), the Court affirmed the order of the manner prescribed by the Charter itself.
suspension of Congressman Paredes by the
Sandiganbayan, despite his protestations on the Republic Act No. 3019 does not exclude from its
encroachment by the court on the prerogatives of coverage the members of Congress and that, therefore,
Congress. The Court ruled:
the Sandiganbayan did not err in thus decreeing the Respondents, on the other hand, contend that while the
assailed preventive suspension order. contract in question was signed during the previous term
of petitioner, it was to commence or be effective only on
Attention might be called to the fact that Criminal Case September 1998 or during his current term. It is the
No. 16698 has been decided by the First Division of the respondents’ submission that petitioner “went beyond the
Sandiganbayan on 06 December 1999, acquitting herein protective confines” of jurisprudence when he “agreed to
petitioner. The Court, nevertheless, deems it extend his act to his current term of office.” Aguinaldo
appropriate to render this decision for future guidance cannot apply, according to respondents, because what is
on the significant issue raised by petitioner. (Santiago involved in this case is a misconduct committed during a
v. Sandiganbayan, 356 SCRA 636, April 18, 2001, previous term but to be effective during the current term.
En Banc [Vitug])
Respondents maintain that,

282. What is the doctrine of forgiveness or “x x x petitioner performed two acts with respect to the
condonation? Does it apply to pending criminal cases? contract: he provided for a suspensive period making the
supply contract commence or be effective during his
Held: 1. A public official cannot be removed for succeeding or current term and during his current term of
administrative misconduct committed during a prior term, office he acceded to the suspensive period making the
since his re-election to office operates as a condonation contract effective during his current term by causing the
of the officer’s previous misconduct to the extent of implementation of the contract.”
cutting off the right to remove him therefor. The
foregoing rule, however, finds no application to criminal Hence, petitioner cannot take refuge in the fact
cases pending against petitioner. (Aguinaldo v. of his reelection, according to respondents.
Santos, 212 SCRA 768, 773 [1992])
Further, respondents point out that the contract
2. A reelected local official may not be held in question was signed just four days before the date of
administratively accountable for misconduct committed the 1998 election and so it could not be presumed that
during his prior term of office. The rationale for this when the people of Cebu City voted petitioner to office,
holding is that when the electorate put him back into they did so with full knowledge of petitioner’s character.
office, it is presumed that it did so with full knowledge of
his life and character, including his past misconduct. If, On this point, petitioner responds that knowledge
armed with such knowledge, it still reelects him, then such of an official’s previous acts is presumed and the court
reelection is considered a condonation of his past need not inquire whether, in reelecting him, the electorate
misdeeds. (Mayor Alvin B. Garcia v. Hon. Arturo C. was actually aware of his prior misdeeds.
Mojica, et al., G.R. No. 139043, Sept. 10, 1999
[Quisumbing]) Petitioner cites our ruling in Salalima v. Guingona
(257 SCRA 55 [1996]), wherein we absolved Albay
283. What is the Doctrine of Condonation? Illustrative governor Ramon R. Salalima of his administrative liability
case. as regards a retainer agreement he signed in favor of a
law firm during his previous term, although
Held: Petitioner contends that, per our ruling in disbursements of public funds to cover payments under
Aguinaldo v. Santos (212 SCRA 768 [1992]), his reelection the agreement were still being done during his
has rendered the administrative case filed against him subsequent term. Petitioner argues that, following
moot and academic. This is because his reelection Salalima, the doctrine of Aguinaldo applies even where
operates as a condonation by the electorate of the the effects of the acts complained of are still evident
misconduct committed by an elective official during his during the subsequent term of the reelected official. The
previous term. Petitioner further cites the ruling of this implementation of the contract is a mere incident of its
Court in Pascual v. Hon. Provincial Board of Nueva Ecija execution. Besides, according to petitioner, the “sole act”
(106 Phil. 466, 472 [1959], citing Conant v. Brogan, 6 for which he has been administratively charged is the
N.Y.S.R. 332 [1887], cited in 17 A.L.R. 281, 63, So. 559, signing of the contract with F.E. Zuellig. The charge, in
50 LRA [NS] 553), that his view, excludes the contract’s execution or
implementation, or any act subsequent to the perfection
“x x x When the people have elected a man to office, it of the contract.
must be assumed that they did this with knowledge of his
life and character, and that they disregarded or forgave In Salalima, we recall that the Solicitor General
his faults or misconduct, if he had been guilty of any. It maintained that Aguinaldo did not apply to that case
is not for the court, by reason of such faults or misconduct because the administrative case against Governor Rodolfo
to practically overrule the will of the people.” Aguinaldo of Cagayan was already pending when he filed
his certificate of candidacy for his reelection bid.
Nevertheless, in Salalima, the Court applied the Aguinaldo enemies, who may not stop to hound the former during
doctrine, even if the administrative case against Governor his new term with administrative cases for acts alleged to
Salalima was filed after his reelection. have been committed during his previous term. His
second term may thus be devoted to defending himself in
Xxx the said cases to the detriment of public service x x x.”
(Emphasis added.) (Salalima v. Guingona, supra at 115)
We now come to the concluding inquiry. Granting
that the Office of the Ombudsman may investigate, for The above ruling in Salalima applies to this case.
purposes provided for by law, the acts of petitioner Petitioner cannot anymore be held administratively liable
committed prior to his present term of office; and that it for an act done during his previous term, that is, his
may preventively suspend him for a reasonable period, signing of the contract with F.E. Zuellig.
can that office hold him administratively liable for said
acts? The assailed retainer agreement in Salalima was
executed sometime in 1990. Governor Salalima was
In a number of cases, we have repeatedly held reelected in 1992 and payments for the retainer continued
that a reelected local official may not be held to be made during his succeeding term. This situation is
administratively accountable for misconduct committed no different from the one in the present case, wherein
during his prior term of office (Pascual v. Hon. Provincial deliveries of the asphalt under the contract with F.E.
Board of Nueva Ecija, 106 Phil. 466 [1959]; others Zuellig and the payments therefor were supposed to have
omitted). The rationale for this holding is that when the commenced on September 1998, during petitioner’s
electorate put him back into office, it is resumed that it second term.
did so with full knowledge of his life and character,
including his past misconduct. If, armed with such However, respondents argue that the contract,
knowledge, it still reelects him, then such reelection is although signed on May 7, 1998, during petitioner’s prior
considered a condonation of his past misdeeds. term, is to be made effective only during his present term.

However, in the present case, respondents point We fail to see any difference to justify a valid
out that the contract entered into by petitioner with F.E. distinction in the result. The agreement between
Zuellig was signed just four days before the date of the petitioner (representing Cebu City) and F.E. Zuellig was
elections. It was not made an issue during the election, perfected on the date the contract was signed, during
and so the electorate could not be said to have voted for petitioner’s prior term. At that moment, petitioner already
petitioner with knowledge of this particular aspect of his acceded to the terms of the contract, including
life and character. stipulations now alleged to be prejudicial to the city
government. Thus, any culpability petitioner may have in
For his part, petitioner contends that “the only signing the contract already became extant on the day
conclusive determining factor” as regards the people’s the contract was signed. It hardly matters that the
thinking on the matter is an election. On this point we deliveries under the contract are supposed to have been
agree with petitioner. That the people voted for an official made months later.
with knowledge of his character is presumed, precisely to
eliminate the need to determine, in factual terms, the While petitioner can no longer be held
extent of this knowledge. Such an undertaking will administratively liable for signing the contract with F.E.
obviously be impossible. Our rulings on the matter do not Zuellig, however, this should not prejudice the filing of
distinguish the precise timing or period when the any case other than administrative against petitioner. Our
misconduct was committed, reckoned from the date of ruling in this case, may not be taken to mean the total
the official’s reelection, except that it must be prior to said exoneration of petitioner for whatever wrongdoing, if any,
date. might have been committed in signing the subject
contract. The ruling now is limited to the question of
As held in Salalima, whether or not he may be held administratively liable
therefor, and it is our considered view that he may not.
“The rule adopted in Pascual, qualified in Aguinaldo (Garcia v. Mojica, 314 SCRA 207, Sept. 10, 1999,
insofar as criminal cases are concerned, is still a good law. 2nd Div. [Quisumbing])
Such a rule is not only founded on the theory that an
official’s reelection expresses the sovereign will of the 284. What are the situations covered by the law on
electorate to forgive or condone any act or omission nepotism?
constituting a ground for administrative discipline which
was committed during his previous term. We may add Held: Under the definition of nepotism, one is guilty of
that sound policy dictates it. To rule otherwise would nepotism if an appointment is issued in favor of a relative
open the floodgates to exacerbating endless partisan within the third civil degree of consanguinity or affinity of
contests between the reelected official and his political any of the following:
on a common date, and (2) that any vacancy due to
a) appointing authority; death, resignation or disability before the expiration of the
b) recommending authority; term should only be filled only for the unexpired balance
c) chief of the bureau or office; and of the term.”
d) person exercising immediate supervision over
the appointee. Consequently, the terms of the first Chairmen and
Commissioners of the Constitutional Commissions under
Clearly, there are four situations covered. In the last two the 1987 Constitution must start on a common date,
mentioned situations, it is immaterial who the appointing irrespective of the variations in the dates of appointments
or recommending authority is. To constitute a violation and qualifications of the appointees, in order that the
of the law, it suffices that an appointment is extended or expiration of the first terms of seven, five and three years
issued in favor of a relative within the third civil degree of should lead to the regular recurrence of the two-year
consanguinity or affinity of the chief of the bureau or interval between the expiration of the terms.
office, or the person exercising immediate supervision
over the appointee. (CSC v. Pedro O. Dacoycoy, G.R. Applying the foregoing conditions x x x, we rule that the
No. 135805, April 29, 1999, En Banc [Pardo]) appropriate starting point of the terms of office of the first
appointees to the Constitutional Commissions under the
285. What are the exemptions from the operation of 1987 Constitution must be on February 2, 1987, the date
the rules on nepotism? of the adoption of the 1987 Constitution. In case of a
belated appointment or qualification, the interval between
Ans.: The following are exempted from the operation of the start of the term and the actual qualification of the
the rules on nepotism: (a) persons employed in a appointee must be counted against the latter. (Thelma
confidential capacity, (b) teachers, (c) physicians, and (d) P. Gaminde v. COA, G.R. No. 140335, Dec. 13,
members of the Armed Forces of the Philippines. 2000, En Banc [Pardo])

The rules on nepotism shall likewise not be 288. What is the hold-over doctrine? What is its
applicable to the case of a member of any family who, purpose?
after his or her appointment to any position in an office
or bureau, contracts marriage with someone in the same Held: 1. The concept of holdover when applied to a
office or bureau, in which event the employment or public officer implies that the office has a fixed term and
retention therein of both husband and wife may be the incumbent is holding onto the succeeding term. It is
allowed. (Sec. 59, Chap. 7, Subtitle A, Title I, Bk. V, usually provided by law that officers elected or appointed
E.O. No. 292) for a fixed term shall remain in office not only for that
term but until their successors have been elected and
286. Distinguish “term” of office from “tenure” of the qualified. Where this provision is found, the office does
incumbent. not become vacant upon the expiration of the term if
there is no successor elected and qualified to assume it,
Held: In the law of public officers, there is a settled but the present incumbent will carry over until his
distinction between “term” and “tenure.” “[T]he term of successor is elected and qualified, even though it be
an office must be distinguished from the tenure of the beyond the term fixed by law.
incumbent. The term means the time during which the
officer may claim to hold office as of right, and fixes the Absent an express or implied constitutional or statutory
interval after which the several incumbents shall succeed provision to the contrary, an officer is entitled to stay in
one another. The tenure represents the term during office until his successor is appointed or chosen and has
which the incumbent actually holds the office. The term qualified. The legislative intent of not allowing holdover
of office is not affected by the hold-over. The tenure may must be clearly expressed or at least implied in the
be shorter than the term for reasons within or beyond the legislative enactment, otherwise it is reasonable to
power of the incumbent.” (Thelma P. Gaminde v. assume that the law-making body favors the same.
COA, G.R. No. 140335, Dec. 13, 2000, En Banc
[Pardo]) Indeed, the law abhors a vacuum in public offices, and
courts generally indulge in the strong presumption
287. Discuss the operation of the rotational plan against a legislative intent to create, by statute, a
insofar as the term of office of the Chairman and Members condition which may result in an executive or
of the Constitutional Commissions is concerned. administrative office becoming, for any period of time,
wholly vacant or unoccupied by one lawfully authorized to
Held: In Republic v. Imperial (96 Phil. 770 [1955]), we exercise its functions. This is founded on obvious
said that “the operation of the rotational plan requires two considerations of public policy, for the principle of
conditions, both indispensable to its workability: (1) that holdover is specifically intended to prevent public
the terms of the first three (3) Commissioners should start convenience from suffering because of a vacancy and to
avoid a hiatus in the performance of government Abandonment springs from and is accompanied by
functions. (Lecaroz v. Sandiganbayan, 305 SCRA deliberation and freedom of choice. Its concomitant
397, March 25, 1999, 2nd Div. [Bellosillo]) effect is that the former holder of an office can no longer
legally repossess it even by forcible reoccupancy.
2. The rule is settled that unless “holding over be
expressly or impliedly prohibited, the incumbent may Clear intention to abandon should be manifested by the
continue to hold over until someone else is elected and officer concerned. Such intention may be express or
qualified to assume the office.” This rule is demanded by inferred from his own conduct. Thus, the failure to
the “most obvious requirements of public policy, for perform the duties pertaining to the office must be with
without it there must frequently be cases where, from a the officer’s actual or imputed intention to abandon and
failure to elect or a refusal or neglect to qualify, the office relinquish the office. Abandonment of an office is not
would be vacant and the public service entirely wholly a matter of intention; it results from a complete
suspended.” Otherwise stated, the purpose is to prevent abandonment of duties of such continuance that the law
a hiatus in the government pending the time when the will infer a relinquishment. Therefore, there are two
successor may be chosen and inducted into office. essential elements of abandonment; first, an intention to
(Galarosa v. Valencia, 227 SCRA 728, Nov. 11, abandon and, second, an overt or “external” act by which
1993, En Banc [Davide, Jr.]) the intention is carried into effect. (Sangguniang
Bayan of San Andres, Catanduanes v. CA, 284
289. What is resignation? What are the requisites of a SCRA 276, Jan. 16, 1998)
valid resignation?
291. Petitioner claims that Benipayo has no authority
Held: 1. It is the act of giving up or the act of an officer to remove her as Director IV of the EID and reassign her
by which he declines his office and renounces the further to the Law Department. Petitioner further argues that
right to use it. It is an expression of the incumbent in only the COMELEC, acting as a collegial body, can
some form, express or implied, of the intention to authorize such reappointment. Moreover, petitioner
surrender, renounce, and relinquish the office and the maintains that a reassignment without her consent
acceptance by competent and lawful authority. To amounts to removal from office without due process and
constitute a complete and operative resignation from therefore illegal.
public office, there must be: (a) an intention to relinquish
a part of the term; (b) an act of relinquishment; and (c) Held: Petitioner’s posturing will hold water if
an acceptance by the proper authority. The last one is Benipayo does not possess any color of title to the office
required by reason of Article 238 of the Revised Penal of Chairman of the COMELEC. We have ruled, however,
Code. (Sangguniang Bayan of San Andres, that Benipayo is the de jure COMELEC Chairman, and
Catanduanes v. CA, 284 SCRA 276, Jan. 16, 1998) consequently he has full authority to exercise all the
powers of that office for so long as his ad interim
2. Resignation x x x is a factual question and its elements appointment remains effective. X x x. The Chairman,
are beyond quibble: there must be an intent to resign and as the Chief Executive of the COMELEC, is expressly
the intent must be coupled by acts of relinquishment empowered on his own authority to transfer or reassign
(Gonzales v. Hernandez, 2 SCRA 228 [1961]). The validity COMELEC personnel in accordance with the Civil Service
of a resignation is not governed by any formal Law. In the exercise of this power, the Chairman is not
requirement as to form. It can be oral. It can be written. required by law to secure the approval of the COMELEC
It can be express. It can be implied. As long as the en banc.
resignation is clear, it must be given legal effect.
(Estrada v. Desierto, G.R. Nos. 146710-15, March Petitioner’s appointment papers x x x
2, 2001, en Banc [Puno]) indisputably show that she held her Director IV position
in the EID only in an acting or temporary capacity.
Petitioner is not a Career Executive Service (CES), and
290. What is abandonment of an office? What are its neither does she hold Career Executive Service Eligibility,
requisites? How is it distinguished from resignation? which are necessary qualifications for holding the
position of Director IV as prescribed in the Qualifications
Held: Abandonment of an office has been defined as the Standards (Revised 1987) issued by the Civil Service
voluntary relinquishment of an office by the holder, with Commission. Obviously, petitioner does not enjoy
the intention of terminating his possession and control security of tenure as Director IV. X x x
thereof. Indeed, abandonment of office is a species of
resignation; while resignation in general is a formal Xxx
relinquishment, abandonment is a voluntary
relinquishment through nonuser. Having been appointed merely in a temporary or
acting capacity, and not possessed of the necessary
qualifications to hold the position of Director IV,
petitioner has no legal basis in claiming that her capacity, is Benipayo. The COMELEC en banc, in
reassignment was contrary to the Civil Service Law. X x COMELEC Resolution No. 3300, approved the transfer or
x reassignment of COMELEC personnel during the election
period. Thus, Benipayo’s order reassigning petitioner
Still, petitioner assails her reassignment, carried out from the EID to the Law Department does not violate
during the election period, as a prohibited act under Section 261 (h) of the Omnibus Election Code. For the
Section 261 (h) of the Omnibus Election Code x x x. same reason, Benipayo’s order designating Cinco
Officer-in-Charge of the EID is legally unassailable.
Xxx (Matibag v. Benipayo, 380 SCRA 49, April 2, 2002,
En Banc [Carpio])
Petitioner claims that Benipayo failed to secure the
approval of the COMELEC en banc to effect transfers or 292. Is a government employee who has been
reassignments of COMELEC personnel during the ordered arrested and detained for a non-bailable offense
election period. Moreover, petitioner insists that the and for which he was suspended for his inability to
COMELEC en banc must concur to every transfer or report for work until the termination of his case, still
reassignment of COMELEC personnel during the election required to file a formal application for leave of absence
period. to ensure his reinstatement upon his acquittal and thus
protect his security of tenure? Concomitantly, will his
Contrary to petitioner’s allegation, the COMELEC did in prolonged absence from office for more than one (1)
fact issue COMELEC Resolution No. 3300 dated year automatically justify his being dropped from the
November 6, 2000, exempting the COMELEC from rolls without prior notice despite his being allegedly
Section 261 (h) of the Omnibus Election Code. X x x placed under suspension by his employer until the
termination of his case, which finally resulted in his
Xxx acquittal for lack of evidence?

The proviso in COMELEC Resolution No. 3300, requiring EUSEBIA R. GALZOTE was employed as a lowly
due notice and hearing before any transfer or clerk in the service of the City Government of Makati
reassignment can be made within thirty days prior to City. With her meager income she was the lone provider
election day, refers only to COMELEC field personnel and for her children. But her simple life was disrupted
not to head office personnel like the petitioner. Under abruptly when she was arrested without warrant and
the Revised Administrative Code (see Section 7 [4], detained for more than three (3) years for a crime she
Chapter 2, Subtitle C, Book V of the Revised did not commit. Throughout her ordeal she trusted the
Administrative Code), the COMELEC Chairman is the sole city government that the suspension imposed on her
officer specifically vested with the power to transfer or was only until the final disposition of her case. As she
reassign COMELEC personnel. The COMELEC Chairman drew near her vindication she never did expect the worst
will logically exercise the authority to transfer or to come to her. On the third year of her detention the
reassign COMELEC personnel pursuant to COMELEC city government lifted her suspension, dropped her from
Resolution No. 3300. The COMELEC en banc cannot the rolls without prior notice and without her knowledge,
arrogate unto itself this power because that will mean much less gave her an opportunity to forthwith correct
amending the Revised Administrative Code, an act the the omission of an application for leave of absence
COMELEC en banc cannot legally do. belatedly laid on her.

COMELEC Resolution No. 3300 does not require that Upon her acquittal for lack of evidence and her
every transfer or reassignment of COMELEC personnel release from detention she was denied reinstatement to
should carry the concurrence of the COMELEC as a her position. She was forced to seek recourse in the
collegial body. Interpreting Resolution No. 3300 to Civil Service Commission which ordered her immediate
require such concurrence will render the resolution reinstatement with back wages from 19 October 1994,
meaningless since the COMELEC en banc will have to the date when she presented herself for reassumption of
approve every personnel transfer or reassignment, duties but was turned back by the city government, up
making the resolution utterly useless. Resolution No. to the time of her actual reinstatement.
3300 should be interpreted for what it is, an approval to
effect transfers and reassignments of personnel, without Xxx
need of securing a second approval from the COMELEC
en banc to actually implement such transfer or Plainly, the case of petitioner City Government
reassignment. of Makati City revolves around a rotunda of doubt, a
dilemma concerning the legal status and implications of
The COMELEC Chairman is the official expressly its suspension of private respondent Eusebia R. Galzote
authorized by law to transfer or reassign COMELEC and the automatic leave of absence espoused by the
personnel. The person holding that office, in a de jure Civil Service Commission. Against this concern is the
punctilious adherence to technicality, the requirement that petitioner City government of Makati City had
that private respondent should have filed an application placed her under suspension until the final disposition of
for leave of absence in proper form. The instant case is her criminal case. This act of petitioner indubitably
therefore a dispute between, at its worst, private recognized private respondent’s predicament and thus
respondent’s substantial compliance with the standing allowed her to forego reporting for work during the
rules, and the City Government’s insistence that the pendency of her criminal case without the needless
lowly clerk should have still gone through the formalities exercise of strict formalities. At the very least, this
of applying for leave despite her detention, of which official communication should be taken as an equivalent
petitioner had actual notice, and the suspension order of a prior approved leave of absence since it was her
couched in simple language that she was being employer itself which placed her under suspension and
suspended until the final disposition of her criminal case. thus excused her from further formalities in applying for
such leave. Moreover, the arrangement bound the City
The meaning of suspension until the final Government to allow private respondent to return to her
disposition of her case is that should her case be work after the termination of her case, i.e., if acquitted
dismissed she should be reinstated to her position with of the criminal charge. This pledge sufficiently served as
payment of back wages. She did not have to apply for legitimate reason for her to altogether dispense with the
leave of absence since she was already suspended by formal application for leave; there was no reason to, as
her employer until her case would be terminated. We in fact it was not required, since she was for all practical
have done justice to the workingman in the past; today purposes incapacitated or disabled to do so.
we will do no less by resolving all doubts in favor of the
humble employee in faithful obeisance to the Indeed, private respondent did not have the least
constitutional mandate to afford full protection to labor intention to go on AWOL from her post as Clerk III of
(Const., Art. XIII, Sec. 3, par. 1; Art. II, Sec. 18) petitioner, for AWOL means the employee leaving or
abandoning his post without justifiable reason and
Xxx without notifying his employer. In the instant case,
private respondent had a valid reason for failing to
As may be gleaned from the pleadings of the report for work as she was detained without bail.
parties, the issues are: (1) whether private respondent Hence, right after her release from detention, and when
Eusebia R. Galzote may be considered absent without finally able to do so, she presented herself to the
leave; (b) whether due process had been observed Municipal Personnel Officer of petitioner City
before she was dropped from the rolls; and, (3) whether Government to report for work. Certainly, had she been
she may be deemed to have abandoned her position, told that it was still necessary for her to file an
hence, not entitled to reinstatement with back salaries application for leave despite the 9 September 1991
for not having filed a formal application for leave. assurance from petitioner, private respondent would
Encapsulated, the issues may be reduced to whether have lost no time in filing such piece of document. But
private respondent may be considered absent without the situation momentarily suspending her from work
leave or whether she abandoned her job as to justify persisted: petitioner City Government did not alter the
being dropped from the service for not filing a formal modus vivendi with private respondent and lulled her
application for leave. into believing that its commitment that her suspension
was only until the termination of her case was true and
Petitioner would have private respondent reliable. Under the circumstances private respondent
declared on AWOL and faults her for failing to file an was in, prudence would have dictated petitioner, more
application for leave of absence under Secs. 20 (Now particularly the incumbent city executive, in patria
Sec. 52 of Rule XVI, Leave of Absence, of Res. No. 91- potestas, to advise her that it was still necessary –
1631 dated 27 December 1991, as amended by CSC MC although indeed unnecessary and a useless ceremony –
No. 41, s. 1998) and 35 (Now Sec. 63 of Rule XVI, Leave to file such application despite the suspension order,
of Absence, of Res. No. 91-1631 dated 27 December before depriving her of her legitimate right to return to
1991, as amended by CSC MC Nos. 41, s. 1998 and 14, her position. Patria potestas in piatate debet, non in
s. 1999) of the CSC Rules and rejects the CSC’s ruling of atrocitate, consistere. Paternal power should consist or
an “automatic leave of absence for the period of her be exercised in affection, not in atrocity.
detention” since the “current Civil Service Law and Rules
do not contain any specific provision on automatic leave It is clear from the records that private respondent
of absence.” Galzote was arrested and detained without a warrant on
6 September 1991 for which reason she and her co-
The Court believes that private respondent accused were subjected immediately to inquest
cannot be faulted for failing to file prior to her detention proceedings. This fact is evident from the instant
an application for leave and obtain approval thereof. petition itself and its attachments x x x. Hence, her
The records clearly show that she had been advised ordeal in jail began on 6 September 1991 and ended
three (3) days after her arrest, or on 9 September 1991,
only after her acquittal, thus leaving her no time to
attend to the formality of filing a leave of absence. At any rate, statements are, or should be,
construed against the one responsible for the confusion;
But petitioner City Government would otherwise stated, petitioner must assume full
unceremoniously set aside its 9 September 1991 responsibility for the consequences of its own act,
suspension order claiming that it was superseded three hence, he should be made to answer for the mix-up of
(3) years later by a memorandum dropping her from the private respondent as regards the leave application. At
rolls effective 21 January 1993 for absence “for more the very least, it should be considered estopped from
than one (1) year without official leave.” Hence, the claiming that its order of suspension is void or that it did
suspension order was void since there was no pending not excuse private respondent from filing an application
administrative charge against private respondent so that for leave on account of her incarceration. It is a fact
she was not excused from filing an application for leave. that she relied upon this order, issued barely three (3)
days from the date of her arrest, and assumed that
We do not agree. In placing private respondent when the criminal case would be settled she could
under suspension until the final disposition of her return to work without need of any prior act. X x x
criminal case, the Municipal Personnel Officer acted with
competence, so he presumably knew that his order of Xxx
suspension was not akin to either suspension as penalty
or preventive suspension since there was no The holding of the Civil Service Commission that
administrative case against private respondent. As private respondent was on automatic leave of absence
competence on the part of the MPO is presumed, any during the period of her detention must be sustained.
error on his part should not prejudice private The CSC is the constitutionally mandated central
respondent, and that what he had in mind was to personnel agency of the Government tasked to
consider her as being on leave of absence without pay “establish a career service and adopt measures to
and their employer-employee relationship being merely promote morale, efficiency, integrity, responsiveness,
suspended, not severed, in the meantime. This progressiveness and courtesy in the civil service”
construction of the order of suspension is actually more (Const., Art. IX-B, Sec. 3) and “strengthen the merit and
consistent with logic as well as fairness and kindness to rewards system, integrate all human resources
its author, the MPO. Significantly, the idea of a development programs for all levels and ranks, and
suspended employer-employee relationship is widely institutionalize a management climate conducive to
accepted in labor law to account for situations wherein public accountability.” (Ibid.) Besides, the
laborers would have no work to perform for causes not Administrative Code of 1987 further empowers the CSC
attributable to them (see e.g., Visayan Stevedore to “prescribe, amend, and enforce rules and regulations
Transportation Company v. Court of Industrial Relations, for carrying into effect the provisions of the Civil Service
No. L-21696, 25 February 1967, 19 SCRA 426; Tomas Law and other pertinent laws,” (Bk. V, I (A), Ch. 3, Sec.
Lao Construction v. NLRC, G.R. No. 116781, 5 12) and for matters concerning leaves of absence, the
September 1997, 278 SCRA 716). We find no basis for Code specifically vests the CSC to ordain –
denying the application of this principle to the instant
case which also involves a lowly worker in the public Sec. 60. Leave of absence. – Officers and employees in
service. the Civil Service shall be entitled to leave of absence,
with or without pay, as may be provided by law and the
Moreover, we certainly cannot nullify the City rules and regulations of the Civil Service Commission in
Government’s order of suspension, as we have no the interest of the service.
reason to do so, much less retroactively apply such
nullification to deprive private respondent of a Pursuant thereto the CSC promulgated
compelling and valid reason for not filing the leave Resolution No. 91-1631 dated 27 December 1991
application. For as we have held, a void act though in entitled Rules Implementing Book V of Executive Order
law a mere scrap of paper nonetheless confers No. 292 and Other Pertinent Civil Service Laws which it
legitimacy upon past acts or omissions done in reliance has several times amended through memorandum
thereof (De Agbayani v. Philippine National Bank, G.R. circulars. It devotes Rule XVI to leaves of absence.
No. 231127, 29 April 1971, 38 SCRA 429; Municipality of Petitioner City Government relies upon Secs. 20 and 35
Malabang v. Benito, G.R No. 28113, 28 March 1969, 27 to debunk the CSC ruling of an automatic leave of
SCRA 545). Consequently, the existence of a statute or absence. Significantly, these provisions have been
executive order prior to its being adjudged void is an amended so that Sec. 20 of the Civil Service Rules is
operative fact to which legal consequences are attached now Sec. 52 of Rule XVI, on Leave of Absence, of
(De Agbayani, supra, p. 435). It would indeed be Resolution No. 91-1631 dated 27 December 1991 as
ghastly unfair to prevent private respondent from relying amended by CSC MC No. 41, s. 1998, and Sec. 35 is
upon the order of suspension in lieu of a formal leave now Sec. 63 as amended by CSC MC Nos. 41, s. 1998
application. and 14, s. 1999.
293. What is abandonment of office? What are its
Xxx essential elements?

As a general rule, Secs. 20 and 52, as well as Held: Abandonment of an office is the voluntary
Secs. 35 and 63, require an approved leave of absence relinquishment of an office by the holder, with the
to avoid being an AWOL. However, these provisions intention of terminating his possession and control
cannot be interpreted as exclusive and referring only to thereof (Sangguniang Bayan of San Andres, Catanduanes
one mode of securing the approval of a leave of absence v. Court of Appeals, 284 SCRA 276 [1998]). In order to
which would require an employee to apply for it, constitute abandonment of an office, it must be total and
formalities and all, before exceeding thirty (30) days of under such circumstances as clearly to indicate an
absence in order to avoid from being dropped from the absolute relinquishment (Airoso v. De Guzman, 49 Phil.
rolls. There are, after all, other means of seeking and 371 [1926]). There must be a complete abandonment of
granting an approved leave of absence, one of which is duties of such continuance that the law will infer a
the CSC recognized rule of automatic leave of absence relinquishment (67 C.J.S. Officers Sec. 100, citing Cosby
under specified circumstances. X x x v. Moore, 65 So.2d 178, 259 Ala. 41). Abandonment of
duties is a voluntary act (Ibid., citing Steingruber v. San
Xxx Antonio, Comm. App., 220 S.W. 77, 78) ; it springs from
and is accompanied by deliberation and freedom of choice
As properly noted, the CSC was only interpreting (Jorge v. Mayor, 10 SCRA 331 [1964], citing Teves v.
its own rules on leave of absence and not a statutory Sindiong, 81 Ohil. 658 [1948]). There are, therefore, two
provision (As a matter of fact, Sec. 60 of the essential elements of abandonment: first, an intention to
Administrative Code does not provide for any rule on abandon and second, an overt or “external” act by which
leave of absence other than that civil servants are the intention is carried into effect (67 C.J.S. Officers Sec.
entitled to leave of absence) in coming up with this 100, citing Rainwater v. State ex rel. Strickland, 178 So.
uniform rule. Undoubtedly, the CSC like any other 484, 237 Ala. 482, 121 A.L.R. 981)
agency has the power to interpret its own rules and any
phrase contained in them (Norwegian Nitrogen Products Generally speaking, a person holding a public office may
Co. v. United States of America, 288 SCRA 294, 325, 77 abandon such office by nonuser or acquiescence (Ibid.,
Led. 796, 812 [1933]) with its interpretation significantly citing Herbert v. State Oil and Gas Bd., 250 So.2d 597,
becoming part of the rules themselves. X x x 287 Ala. 221). Non-user refers to a neglect to use a right
or privilege or to exercise an office (Sangguniang Bayan
Xxx of San Andres, Catanduanes v. Court of Appeals, supra).
However, nonperformance of the duties of an office does
Under RA 6656 (An Act to Protect the Security of not constitute abandonment where such nonperformance
Tenure of Civil Service Officers and Employees in the results from temporary disability or from involuntary
Implementation of Government Reorganization) and RA failure to perform (67 C.J.S. Sec. 100, citing Doris v.
7160 (The Local Government Code of 1991), civil Heroux, 47 A.2d 633, 71 R.I. 491). Abandonment may
servants who are found illegally dismissed or retrenched also result from an acquiescence by the officer in his
are entitled to full pay for the period of their separation. wrongful removal or discharge, for instance, after a
summary removal, an unreasonable delay by an officer
Our final point. An efficient and honest illegally removed in taking steps to vindicate his rights
bureaucracy is never inconsistent with the emphasis on may constitute an abandonment of the office (Ibid., citing
and the recognition of the basic rights and privileges of Nicholas v. U.S., Ct. Cl., 42 S.Ct. 7, 257 U.S. 71, 66 L. Ed.
our civil servants or, for that matter, the constitutional 133). Where, while desiring and intending to hold the
mandates of the Civil Service Commission. In fact only office, and with no willful desire or intention to abandon
from an enlightened corps of government workers and it, the public officer vacates it in deference to the
an effective CSC grows the professionalization of the requirements of a statute which is afterwards declared
bureaucracy. Indeed the government cannot be left in unconstitutional, such a surrender will not be deemed an
the lurch; but neither could we decree that government abandonment and the officer may recover the effect.
personnel be separated from their jobs indiscriminately (Mechem, A Treatise on the Law of Public Offices and
regardless of fault. The fine line between these Officers, 1890 edition, p. 279, citing Turnipseed v.
concerns may be difficult to clearly draw but if we only Hudson, 50 Miss. 429, 19 Am. Rep. 15) (Canonizado v.
exerted extra effort to rebel against the allure of legal Aguirre, 351 SCRA 659, 665-668, Feb. 15, 2001, En
over-simplification, justice would have been done where Banc [Gonzaga-Reyes])
it is truly due. (City Government of Makati City v.
Civil Service Commission, 376 SCRA 248, Feb. 6, 294. By accepting another position in the government
2002, En Banc [Bellosillo]) during the pendency of a case – brought precisely to
assail the constitutionality of his removal - may a person
be deemed to have abandoned his claim for moment, but that which proceeds from the nature and
reinstatement? relations of the two positions to each other as to give rise
to contrariety and antagonism should one person attempt
Held: Although petitioners do not deny the appointment to faithfully and impartially discharge the duties of one
of Canonizado as Inspector General, they maintain that toward the incumbent of the other. (Ibid.)
Canonizado’s initiation and tenacious pursuance of the
present case would belie any intention to abandon his There is no question that the positions of
former office. Petitioners assert that Canonizado should NAPOLCOM Commissioner and Inspector General of the
not be faulted for seeking gainful employment during the IAS are incompatible with each other. As pointed out by
pendency of this case. Furthermore, petitioners point out respondents, RA 8551 prohibits any personnel of the IAS
that from the time Canonizado assumed office as from sitting in a committee charged with the task of
Inspector General he never received the salary pertaining deliberating on the appointment, promotion, or
to such position x x x. assignment of any PNP personnel, whereas the
NAPOLCOM has the power of control and supervision
Xxx over the PNP. However, the rule on incompatibility of
duties will not apply to the case at bar because at no
By accepting the position of Inspector General point did Canonizado discharge the functions of the two
during the pendency of the present case – brought offices simultaneously. Canonizado was forced out of
precisely to assail the constitutionality of his removal from his first office by the enactment of Section 8 of RA 8551.
the NAPOLCOM – Canonizado cannot be deemed to have Thus, when Canonizado was appointed as Inspector
abandoned his claim for reinstatement to the latter General x x x he had ceased to discharge his official
position. First of all, Canonizado did not voluntarily leave functions as NAPOLCOM Commissioner. X x x. Thus, to
his post as Commissioner, but was compelled to do so on reiterate, the incompatibility of duties rule never had a
the strength of Section 8 of RA 8551 x x x chance to come into play for petitioner never occupied
the two positions, of Commissioner and Inspector
In our decision of 25 January 2000, we struck General, nor discharged their respective functions,
down the abovequoted provision for being violative of concurrently.
petitioner’s constitutionally guaranteed right to security
of tenure. Thus, Canonizado harbored no willful desire Xxx
or intention to abandon his official duties. In fact,
Canonizado, together with petitioners x x x lost no time As in the Tan (Tan v. Gimenez, 107 Phil. 17 [1960]) and
disputing what they perceived to be an illegal removal; a Gonzales (Gonzales v. Hernandez, 2 SCRA 228 [1961])
few weeks after RA 8551 took effect x x x petitioners cases, Canonizado was compelled to leave his position
instituted the current action x x x assailing the as Commissioner, not by an erroneous decision, but by
constitutionality of certain provisions of said law. The an unconstitutional provision of law. Canonizado, like
removal of petitioners from their positions by virtue of a the petitioners in the above mentioned cases, held a
constitutionally infirm act necessarily negates a finding second office during the period that his appeal was
of voluntary relinquishment. (Canonizado v. Aguirre, pending. As stated in the Comment filed by petitioners,
351 SCRA 659, 665-668, Feb. 15, 2001, En Banc Canonizado was impelled to accept this subsequent
[Gonzaga-Reyes]) position by a desire to continue serving the country, in
whatever capacity. Surely, this selfless and noble
295. What is the effect of acceptance of an aspiration deserves to be placed on at least equal
incompatible office to a claim for reinstatement? footing with the worthy goal of providing for oneself and
one’s family, either of which are sufficient to justify
Held: The next issue is whether Canonizado’s Canonizado’s acceptance of the position of Inspector
appointment to and acceptance of the position of General. A Contrary ruling would deprive petitioner of
Inspector General should result in an abandonment of his his right to live, which contemplates not only a right to
claim for reinstatement to the NAPOLCOM. It is a well- earn a living, as held in previous cases, but also a right
settled rule that he who, while occupying one office, to lead a useful and productive life. Furthermore,
accepts another incompatible with the first, ipso facto prohibiting Canonizado from accepting a second position
vacates the first office and his title is thereby terminated during the pendency of his petition would be to unjustly
without any other act or proceeding (Mechem, A Treatise compel him to bear the consequences of an
on the Law of Public Offices and Officers, 1890 edition, p. unconstitutional act which under no circumstance can be
267). Public policy considerations dictate against allowing attributed to him. However, before Canonizado can re-
the same individual to perform inconsistent and assume his post as Commissioner, he should first resign
incompatible duties (Ibid.). The incompatibility as Inspector General of the IAS-PNP. (Canonizado v.
contemplated is not the mere physical impossibility of one Aguirre, 351 SCRA 659, Feb. 15, 2001, En Banc
person’s performing the duties of the two offices due to a [Gonzaga-Reyes])
lack of time or the inability to be in two places at the same
296. May the appointment of a person assuming a Section 20 of Rule VI also provides:
position in the civil service under a completed
appointment be validly recalled or revoked? Sec. 20. Notwithstanding the initial approval of an
appointment, the same may be recalled on any of the
Held: It has been held that upon the issuance of an following grounds:
appointment and the appointee’s assumption of the
position in the civil service, “he acquires a legal right (e) Non-compliance with the
which cannot be taken away either by revocation of the procedures/criteria provided in the agency’s Merit
appointment or by removal except for cause and with Promotion Plan;
previous notice and hearing.” (Mauna v. Civil Service (f) Failure to pass through the agency’s
Commission, 232 SCRA 388, 398 [1994]) Moreover, Selection/Promotion Board;
it is well-settled that the person assuming a position in (g) Violation of the existing collective
the civil service under a completed appointment acquires agreement between management and employees relative
a legal, not just an equitable, right to the position. This to promotion; or
right is protected not only by statute, but by the (h) Violation of other existing civil
Constitution as well, which right cannot be taken away by service law, rules and regulations.
either revocation of the appointment, or by removal,
unless there is valid cause to do so, provided that there is Accordingly, the appointments of the private respondents
previous notice and hearing. (Aquino v. Civil Service may only be recalled on the above-cited grounds. And
Commission, 208 SCRA 240, 248 [1992]) yet, the only reason advanced by the petitioner to justify
the recall was that these were “midnight appointments.”
Petitioner admits that his very first official act upon The CSC correctly ruled, however, that the constitutional
assuming the position of town mayor was to issue Office prohibition on so-called “midnight appointments,”
Order No. 95-01 which recalled the appointments of the specifically those made within two (2) months
private respondents. There was no previous notice, much immediately prior to the next presidential elections,
less a hearing accorded to the latter. Clearly, it was applies only to the President or Acting President. (De
petitioner who acted in undue haste to remove the private Rama v. Court of Appeals, 353 SCRA 94, Feb. 28,
respondents without regard for the simple requirements 2001, En Banc [Ynares-Santiago])
of due process of law. While he argues that the
appointing power has the sole authority to revoke said 297. When may unconsented transfers be considered
appointments, there is no debate that he does not have anathema to security of tenure?
blanket authority to do so. Neither can he question the
CSC’s jurisdiction to affirm or revoke the recall. Held: As held in Sta. Maria v. Lopez (31 SCRA 637, 653
citing Ibanez v. Commission on Elections, L-26558, April
Rule V, Section 9 of the Omnibus Implementing 27, 1967, 19 SCRA 1002, 1012 and Section 12 of the Tax
Regulations of the Revised Administrative Code Code).
specifically provides that “an appointment accepted by
the appointee cannot be withdrawn or revoked by the "x x x the rule that outlaws unconsented transfers as
appointing authority and shall remain in force and in anathema to security of tenure applies only to an officer
effect until disapproved by the Commission.” Thus, it is who is appointed - not merely assigned - to a particular
the CSC that is authorized to recall an appointment station. Such a rule does not pr[o]scribe a transfer
initially approved, but only when such appointment and carried out under a specific statute that empowers the
approval are proven to be in disregard of applicable head of an agency to periodically reassign the employees
provisions of the civil service law and regulations and officers in order to improve the service of the agency.
(Debulgado v. Civil Service Commission, 237 SCRA x x x"
184, 200 [1994]).
The guarantee of security of tenure under the
Moreover, Section 10 of the same rule provides: Constitution is not a guarantee of perpetual employment.
It only means that an employee cannot be dismissed (or
Sec. 10. An appointment issued in accordance with transferred) from the service for causes other than those
pertinent laws and rules shall take effect immediately provided by law and after due process is accorded the
upon its issuance by the appointing authority, and if the employee. What it seeks to prevent is capricious exercise
appointee has assumed the duties of the position, he shall of the power to dismiss. But where it is the law-making
be entitled to receive his salary at once without awaiting authority itself which furnishes the ground for the transfer
the approval of his appointment by the Commission. The of a class of employees, no such capriciousness can be
appointment shall remain effective until disapproved by raised for so long as the remedy proposed to cure a
the Commission. In no case shall an appointment take perceived evil is germane to the purposes of the law.
effect earlier than the date of its issuance. (Agripino A. De Guzman, Jr., et al. v. COMELEC,
G.R. No. 129118, July 19, 2000, En Banc or units therein, including the lines of control, authority
[Purisima]) and responsibility between them. It involves a reduction
of personnel, consolidation of offices, or abolition thereof
298. Discuss Abolition of Office? by reason of economy or redundancy of functions.
Naturally, it may result in the loss of one's position
Held: The creation and abolition of public offices is through removal or abolition of an office. However, for a
primarily a legislative function. It is acknowledged that reorganization to be valid, it must also pass the test of
Congress may abolish any office it creates without good faith, laid down in Dario v. Mison (176 SCRA 84
impairing the officer's right to continue in the position held [1989]):
and that such power may be exercised for various
reasons, such as the lack of funds or in the interest of x x x As a general rule, a reorganization is carried out in
economy. However, in order for the abolition to be valid, "good faith" if it is for the purpose of economy or to make
it must be made in good faith, not for political or personal bureaucracy more efficient. In that event, no dismissal
reasons, or in order to circumvent the constitutional (in case of dismissal) or separation actually occurs
security of tenure of civil service employees. because the position itself ceases to exist. And in that
case, security of tenure would not be a Chinese wall. Be
An abolition of office connotes an intention to do away that as it may, if the "abolition" which is nothing else but
with such office wholly and permanently, as the word a separation or removal, is done for political reasons or
"abolished" denotes. Where one office is abolished and purposely to defeat security of tenure, or otherwise not in
replaced with another office vested with similar functions, good faith, no valid "abolition" takes place and whatever
the abolition is a legal nullity. Thus, in U.P. Board of "abolition" is done, is void ab initio. There is an invalid
Regents v. Rasul (200 SCRA 685 [1991]) we said: "abolition" as where there is merely a change of
nomenclature of positions, or where claims of economy
It is true that a valid and bona fide abolition of an office are belied by the existence of ample funds.
denies to the incumbent the right to security of tenure (Alexis C. Canonizado, et al. v. Hon. Alexander P.
(De la Llana v. Alba, 112 SCRA 294 [1982]). However, in Aguirre, et al., G.R. No. 133132, Jan. 25, 2000, En
this case, the renaming and restructuring of the PGH and Banc [Gonzaga-Reyes])
its component units cannot give rise to a valid and bona
fide abolition of the position of PGH Director. This is 2. While the President’s power to reorganize can
because where the abolished office and the offices not be denied, this does not mean however that the
created in its place have similar functions, the abolition reorganization itself is properly made in accordance with
lacks good faith (Jose L. Guerrero v. Hon. Antonio V. law. Well-settled is the rule that reorganization is
Arizabal, G.R. No. 81928, June 4, 1990, 186 SCRA 108 regarded as valid provided it is pursued in good faith.
[1990]). We hereby apply the principle enunciated in Thus, in Dario v. Mison, this Court has had the occasion
Cezar Z. Dario v. Hon. Salvador M. Mison (176 SCRA 84 to clarify that:
[1989]) that abolition which merely changes the
nomenclature of positions is invalid and does not result in “As a general rule, a reorganization is carried out in ‘good
the removal of the incumbent. faith’ if it is for the purpose of economy or to make the
bureaucracy more efficient. In that event no dismissal or
The above notwithstanding, and assuming that the separation actually occurs because the position itself
abolition of the position of the PGH Director and the ceases to exist. And in that case the security of tenure
creation of a UP-PGH Medical Center Director are valid, would not be a Chinese wall. Be that as it may, if the
the removal of the incumbent is still not justified for the abolition which is nothing else but a separation or
reason that the duties and functions of the two positions removal, is done for political reasons or purposely to
are basically the same. defeat security of tenure, or otherwise not in good faith,
no valid abolition takes place and whatever abolition done
This was also our ruling in Guerrero v. Arizabal (186 SCRA is void ab initio. There is an invalid abolition as where
108 [1990]), wherein we declared that the substantial there is merely a change of nomenclature of positions or
identity in the functions between the two offices was where claims of economy are belied by the existence of
indicia of bad faith in the removal of petitioner pursuant ample funds.” (176 SCRA 84)
to a reorganization. (Alexis C. Canonizado, et al. v. (Larin v. Executive Secretary, 280 SCRA 713, Oct.
Hon. Alexander P. Aguirre, et al., G.R. No. 133132, 16, 1997)
Jan. 25, 2000, En Banc [Gonzaga-Reyes])
300. What are the circumstances evidencing bad faith
299. What is reorganization? When is it valid? When in the removal of employees as a result of reorganization
is it invalid? and which may give rise to a claim for reinstatement or
reappointment)?
Held: 1. Reorganization takes place when there is an
alteration of the existing structure of government offices Held:
others, the process of registration. Specifically, a citizen
1) Where there is a significant increase in the in order to be qualified to exercise his right to vote, in
number of positions in the new staffing pattern of the addition to the minimum requirements set by the
department or agency concerned; fundamental charter, is obliged by law to register, at
2) Where an office is abolished and another present, under the provisions of Republic Act No. 8189,
performing substantially the same functions is created; otherwise known as the “Voter’s Registration Act of
3) Where incumbents are replaced by those less 1996.” (Akbayan-Youth v. COMELEC, 355 SCRA
qualified in terms of status of appointment, performance 318, Mar. 26, 2001, En Banc [Buena])
and merit;
4) Where there is a reclassification of offices in 302. Discuss the reason behind the principle of ballot
the department or agency concerned and the reclassified secrecy. May the conduct of exit polls transgress the
offices perform substantially the same functions as the sanctity and the secrecy of the ballot to justify its
original offices; prohibition?
5) Where the removal violates the order of
separation provided in Section 3 hereof. Held: The reason behind the principle of ballot secrecy
(Sec. 2, R.A. No. 6656; Larin v. Executive is to avoid vote buying through voter identification. Thus,
Secretary, 280 SCRA 713, Oct. 16, 1997) voters are prohibited from exhibiting the contents of their
official ballots to other persons, from making copies
thereof, or from putting distinguishing marks thereon so
ELECTION LAWS as to be identified. Also proscribed is finding out the
contents of the ballots cast by particular voters or
301. Discuss the Right of Suffrage, and its substantive disclosing those of disabled or illiterate voters who have
and procedural requirements. been assisted. Clearly, what is forbidden is the
association of voters with their respective votes, for the
Held: In a representative democracy such as purpose of assuring that the votes have been cast in
ours, the right of suffrage, although accorded a prime accordance with the instructions of a third party. This
niche in the hierarchy of rights embodied in the result cannot, however, be achieved merely through the
fundamental law, ought to be exercised within the voters’ verbal and confidential disclosure to a pollster of
proper bounds and framework of the Constitution and whom they have voted for.
must properly yield to pertinent laws skillfully enacted by
the Legislature, which statutes for all intents and In exit polls, the contents of the official ballot are not
purposes, are crafted to effectively insulate such so actually exposed. Furthermore, the revelation of whom
cherished right from ravishment and preserve the an elector has voted for is not compulsory, but voluntary.
democratic institutions our people have, for so long, Voters may also choose not to reveal their identities.
guarded against the spoils of opportunism, debauchery Indeed, narrowly tailored countermeasures may be
and abuse. prescribed by the Comelec, so as to minimize or suppress
incidental problems in the conduct of exit polls, without
To be sure, the right of suffrage x x x is not at all transgressing the fundamental rights of our people.
absolute. Needless to say, the exercise of the right of (ABS-CBN Broadcasting Corporation v. COMELEC,
suffrage, as in the enjoyment of all other rights, is subject G.R. No. 133486, Jan. 28, 2000, En Banc
to existing substantive and procedural requirements [Panganiban])
embodied in our Constitution, statute books and other
repositories of law. Thus, as to the substantive aspect, 303. Discuss the meaning and purpose of residency
Section 1, Article V of the Constitution provides: requirement in Election Law.

“SECTION 1. SUFFRAGE MAY BE EXERCISED BY Held: 1. The meaning and purpose of the residency
ALL CITIZENS OF THE PHILIPPINES NOT OTHERWISE requirement were explained recently in our decision in
DISQUALIFIED BY LAW, WHO ARE AT LEAST EIGHTEEN Aquino v. Comelec (248 SCRA 400, 420-421 [1995]), as
YEARS OF AGE, AND WHO SHALL HAVE RESIDED IN THE follows:
PHILIPPINES FOR AT LEAST ONE YEAR AND IN THE
PLACE WHEREIN THEY PROPOSE TO VOTE FOR AT LAST X x x [T]he place “where a party actually or constructively
SIX MONTHS IMMEDIATELY PRECEDING THE ELECTION. has his permanent home,” where he, no matter where he
NO LITERACY, PROPERTY, OR OTHER SUBSTANTIVE may be found at any given time, eventually intends to
REQUIREMENT SHALL BE IMPOSED ON THE EXERCISE return and remain, i.e., his domicile, is that to which the
OF SUFFRAGE.” Constitution refers when it speaks of residence for the
purposes of election law. The manifest purpose of this
As to the procedural limitation, the right of a deviation from the usual conceptions of residency in law
citizen to vote is necessarily conditioned upon certain as explained in Gallego v. Vera is “to exclude strangers or
procedural requirements he must undergo: among newcomers unfamiliar with the conditions and needs of
the community” from taking advantage of favorable 304. Does the fact that a person is registered as a
circumstances existing in that community for electoral voter in one district proof that he is not domiciled in
gain. While there is nothing wrong with the practice of another district?
establishing residence in a given area for meeting election
law requirements, this nonetheless defeats the essence of Held: The fact that a person is registered as a voter in
representation, which is to place through the assent of one district is not proof that he is not domiciled in another
voters those most cognizant and sensitive to the needs of district. Thus, in Faypon v. Quirino (96 Phil. 294 [1954]),
a particular district, if a candidate falls short of the period this Court held that the registration of a voter in a place
of residency mandated by law for him to qualify. That other than his residence of origin is not sufficient to
purpose could be obviously best met by individuals who consider him to have abandoned or lost his residence.
have either had actual residence in the area for a given (Marcita Mamba Perez v. COMELEC, G.R. No.
period or who have been domiciled in the same area 133944, Oct. 28, 1999, En Banc [Mendoza])
either by origin or by choice.
(Marcita Mamba Perez v. COMELEC, G.R. No.
133944, Oct. 28, 1999, En Banc [Mendoza]) 305. Discuss the nature of Voter’s Registration.

2. The Constitution and the law requires Held: Stated differently, the act of registration
residence as a qualification for seeking and holding is an indispensable precondition to the right of suffrage.
elective public office, in order to give candidates the For registration is part and parcel of the right to vote and
opportunity to be familiar with the needs, difficulties, an indispensable element in the election process. Thus,
aspirations, potentials for growth and all matters vital to x x x registration cannot and should not be denigrated to
the welfare of their constituencies; likewise, it enables the the lowly stature of a mere statutory requirement.
electorate to evaluate the office seekers’ qualifications Proceeding from the significance of registration as a
and fitness for the job they aspire for. Inasmuch as necessary requisite to the right to vote, the State
Vicente Y. Emano has proven that he, together with his undoubtedly, in the exercise of its inherent police power,
family, (1) had actually resided in a house he bought in may then enact laws to safeguard and regulate the act of
1973 in Cagayan de Oro City; (2) had actually held office voter’s registration for the ultimate purpose of conducting
there during his three terms as provincial governor of honest, orderly and peaceful election, to the incidental yet
Misamis Oriental, the provincial capitol being located generally important end, that even pre-election activities
therein; and (3) has registered as voter in the city during could be performed by the duly constituted authorities in
the period required by law, he could not be deemed “a a realistic and orderly manner – one which is not
stranger or newcomer” when he ran for and was indifferent and so far removed from the pressing order of
overwhelmingly voted as city mayor. Election laws must the day and the prevalent circumstances of the times.
be liberally construed to give effect to the popular (Akbayan-Youth v. COMELEC, 355 SCRA 318, Mar.
mandate. (Torayno, Sr. v. COMELEC, 337 SCRA 574, 26, 2001, En Banc [Buena])
Aug. 9, 2000, En Banc [Panganiban])
306. What is the Lone Candidate Law? What are its
3. Generally, in requiring candidates to have a minimum salient provisions?
period of residence in the area in which they seek to be
elected, the Constitution or the law intends to prevent the Answer: The Lone Candidate Law is Republic Act No.
possibility of a “stranger or newcomer unacquainted with 8295, enacted on June 6, 1997. Section 2 thereof
the conditions and needs of a community and not provides that “Upon the expiration of the deadline for the
identified with the latter from [seeking] an elective office filing of the certificate of candidacy in a special election
to serve that community.” Such provision is aimed at called to fill a vacancy in an elective position other than
excluding outsiders “from taking advantage of favorable for President and Vice-President, when there is only one
circumstances existing in that community for electoral (1) qualified candidate for such position, the lone
gain.” Establishing residence in a community merely to candidate shall be proclaimed elected to the position by
meet an election law requirement defeats the purpose of proper proclaiming body of the Commission on Elections
representation: to elect through the assent of voters without holding the special election upon certification by
those most cognizant and sensitive to the needs of the the Commission on Elections that he is the only candidate
community. This purpose is “best met by individuals who for the office and is thereby deemed elected.”
have either had actual residence in the area for a given
period or who have been domiciled in the same area Section 3 thereof provides that “the lone candidate so
either by origin or by choice.” (Torayno, Sr. v. proclaimed shall assume office not earlier than the
COMELEC, 337 SCRA 574, Aug. 9, 2000, En Banc scheduled election day, in the absence of any lawful
[Panganiban]) ground to deny due course or cancel the certificate of
candidacy in order to prevent such proclamation, as
provided for under Sections 69 and 78 of Batas Pambansa
Bilang 881 also known as the Omnibus Election Code.”
All told, a disqualified candidate may only be substituted
307. Who are disqualified to run in a special election if he had a valid certificate of candidacy in the first place
under the Lone Candidate Law? because, if the disqualified candidate did not have a valid
and seasonably filed certificate of candidacy, he is and
Answer: Section 4 of the Lone Candidate Law provides was not a candidate at all. If a person was not a
that “In addition to the disqualifications mentioned in candidate, he cannot be substituted under Section 77 of
Sections 12 and 68 of the Omnibus Election Code and the Code. (Miranda v. Abaya, G.R. No. 136351, July
Section 40 of Republic Act No. 7160, otherwise known as 28, 1999, en Banc [Melo])
the Local Government Code, whenever the evidence of
guilt is strong, the following persons are disqualified to 310. Should the votes cast for the substituted
run in a special election called to fill the vacancy in an candidate be considered votes for the substitute
elective office, to wit: candidate?

a) Any elective official who has resigned from Answer: Republic Act No. 9006, otherwise known as
his office by accepting an appointive office or for the Fair Election Act, provides in Section 12 thereof: “In
whatever reason which he previously occupied but has case of valid substitutions after the official ballots have
caused to become vacant due to his resignation; and been printed, the votes cast for the substituted
b) Any person who, directly or indirectly, candidates shall be considered as stray votes but shall not
coerces, bribes, threatens, harasses, intimidates or invalidate the whole ballot. For this purpose, the official
actually causes, inflicts or produces any violence, injury, ballots shall provide spaces where the voters may write
punishment, torture, damage, loss or disadvantage to any the name of the substitute candidates if they are voting
person or persons aspiring to become a candidate or that for the latter: Provided, however, That if the substitute
of the immediate member of his family, his honor or candidate is of the same family name, this provision shall
property that is meant to eliminate all other potential not apply.”
candidate.”
311. What is the effect of the filing of certificate of
308. What is the purpose of the law in requiring the candidacy by elective officials?
filing of certificate of candidacy and in fixing the time limit
therefor? Answer: COMELEC Resolution No. 3636,
promulgated March 1, 2001, implementing the Fair
Held: The evident purpose of the law in requiring the Election Act (R.A. No. 9006) provides in Section 26
filing of certificate of candidacy and in fixing the time limit thereof: “any elective official, whether national or local,
therefor are: (a) to enable the voters to know, at least who has filed a certificate of candidacy for the same or
sixty days before the regular election, the candidates any other office shall not be considered resigned from his
among whom they are to make the choice, and (b) to office.”
avoid confusion and inconvenience in the tabulation of the
votes cast. For if the law did not confine the choice or Note that Section 67 of the Omnibus Election Code and
election by the voters to the duly registered candidates, the first proviso in the third paragraph of Section 11 of
there might be as many persons voted for as there are Republic Act No. 8436 which modified said Section 67,
voters, and votes might be cast even for unknown or were expressly repealed and rendered ineffective,
fictitious persons as a mark to identify the votes in favor respectively, by Section 14 (Repealing Clause) of The Fair
of a candidate for another office in the same election. Election Act (R.A. No. 9006).
(Miranda v. Abaya, G.R. No. 136351, July 28, 1999)
312. What kind of “material misrepresentation” is
309. May a disqualified candidate and whose contemplated by Section 78 of the Omnibus Election Code
certificate of candidacy was denied due course and/or as a ground for disqualification of a candidate? Does it
canceled by the Comelec be validly substituted? include the use of surname?

Held: Even on the most basic and fundamental Held: Therefore, it may be concluded that the material
principles, it is readily understood that the concept of a misrepresentation contemplated by Section 78 of the
substitute presupposes the existence of the person to be (Omnibus Election) Code refers to qualifications for
substituted, for how can a person take the place of elective office. This conclusion is strengthened by the fact
somebody who does not exist or who never was. The that the consequences imposed upon a candidate guilty
Court has no other choice but to rule that in all instances of having made a false representation in his certificate of
enumerated in Section 77 of the Omnibus Election Code, candidacy are grave – to prevent the candidate from
the existence of a valid certificate of candidacy seasonably running or, if elected, from serving, or to prosecute him
filed is a requisite sine qua non. for violation of the election laws. It could not have been
the intention of the law to deprive a person of such a basic
and substantial political right to be voted for a public of the election. Note that the cause of such failure of
office upon just any innocuous mistake. election could only be any of the following: force majeure,
violence, terrorism, fraud or other analogous causes.
[A]side from the requirement of materiality, a false
representation under Section 78 must consist of a Thus, in Banaga, Jr. v. COMELEC (336 SCRA 701, July
“deliberate attempt to mislead, misinform, or hide a fact 31, 2000, En Banc [Quisumbing]), the SC held:
which would otherwise render a candidate ineligible.” In
other words, it must be made with an intention to deceive “We have painstakingly examined the petition filed by
the electorate as to one’s qualifications for public office. petitioner Banaga before the Comelec. But we found that
The use of a surname, when not intended to mislead or petitioner did not allege at all that elections were either
deceive the public as to one’s identity, is not within the not held or suspended. Neither did he aver that although
scope of the provision. (Victorino Salcedo II v. there was voting, nobody was elected. On the contrary,
COMELEC, G.R. No. 135886, Aug. 16, 1999, En he conceded that an election took place for the office of
Banc [Gonzaga-Reyes]) vice-mayor of Paranaque City, and that private
respondent was, in fact, proclaimed elected to that post.
313. Who has authority to declare failure of elections While petitioner contends that the election was tainted
and the calling of special election? What are the three with widespread anomalies, it must be noted that to
instances where a failure of election may be declared? warrant a declaration of failure of election the commission
of fraud must be such that it prevented or suspended the
Held: The COMELEC’s authority to declare failure of holding of an election, or marred fatally the preparation
elections is provided in our election laws. Section 4 of RA and transmission, custody and canvass of the election
7166 provides that the Comelec sitting en banc by a returns. These essential facts ought to have been alleged
majority vote of its members may decide, among others, clearly by the petitioner below, but he did not.”
the declaration of failure of election and the calling of
special election as provided in Section 6 of the Omnibus 315. Cite instances when Comelec may or may not
Election Code. X x x validly declare failure of elections.

There are three instances where a failure of election may Held: In Mitmug v. COMELEC (230 SCRA 54 [1994]),
be declared, namely, (a) the election in any polling place petitioner instituted with the COMELEC an action to
has not been held on the date fixed on account of force declare failure of election in forty-nine precincts where
majeure, violence, terrorism, fraud or other analogous less than a quarter of the electorate were able to cast
causes; (b) the election in any polling place has been their votes. He also lodged an election protest with the
suspended before the hour fixed by law for the closing of Regional Trial Court disputing the result of the election in
the voting on account of force majeure, violence, all precincts in his municipality. The Comelec denied motu
terrorism, fraud or other analogous causes; or (c) after proprio and without due notice and hearing the petition
the voting and during the preparation and transmission of to declare failure of election despite petitioner’s argument
the election returns or in the custody or canvass thereof, that he has meritorious grounds in support thereto, that
such election results in a failure to elect on account of is, massive disenfranchisement of voters due to terrorism.
force majeure, violence, terrorism, fraud or other On review, we ruled that the Comelec did not gravely
analogous causes. In these instances, there is a resulting abuse its discretion in denying the petition. It was not
failure to elect. This is obvious in the first two scenarios, proven that no actual voting took place. Neither was it
where the election was not held and where the election shown that even if there was voting, the results thereon
was suspended. As to the third scenario, where the would be tantamount to failure to elect. Considering that
preparation and the transmission of the election returns there is no concurrence of the conditions seeking to
give rise to the consequence of failure to elect, it must x declare failure of election, there is no longer need to
x x, be interpreted to mean that nobody emerged as a receive evidence on alleged election irregularities.
winner. (Banaga, Jr. v. COMELEC, 336 SCRA 701,
July 31, 2000, En Banc [Quisumbing]) In Sardea v. COMELEC (225 SCRA 374 [1993]), all
election materials and paraphernalia with the municipal
314. What are the two conditions that must concur board of canvassers were destroyed by the sympathizers
before the COMELEC can act on a verified petition seeking of the losing mayoralty candidate. The board then
to declare a afailure of election? decided to use the copies of election returns furnished to
the municipal trial court. Petitioner therein filed a petition
Held: Before the COMELEC can act on a verified petition to stop the proceedings of the board of canvassers on the
seeking to declare a failure of election two conditions ground that it had no authority to use said election returns
must concur, namely: (1) no voting took place in the obtained from the municipal trial court. The petition was
precinct or precincts on the date fixed by law, or even if denied. Next, he filed a petition assailing the composition
there was voting, the election resulted in a failure to elect; of the board of canvassers. Despite that petition, the
and (2) the votes not cast would have affected the result board of canvassers proclaimed the winning candidates.
Later on, petitioner filed a petition to declare a failure of resolution, order or ruling of a Division, the Clerk of Court
election alleging that the attendant facts would justify concerned shall, within twenty-four (24) hours from the
declaration of such failure. On review, we ruled that filing thereof, notify the presiding Commissioner. The
petitioner’s first two actions involved pre-proclamation latter shall within two (2) days thereafter certify the case
controversies which can no longer be entertained after to the Commission en banc.”
the winning candidates have been proclaimed. Regarding
the petition to declare a failure of election, we held that Under the above-quoted rule, the acts of a
the destruction and loss of copies of election returns Division that are subject of a motion for reconsideration
intended for the municipal board of canvassers on must have a character of finality before the same can be
account of violence is not one of the causes that would elevated to the COMELEC en banc. The elementary rule
warrant the declaration of failure of election. The reason is that an order is final in nature if it completely disposes
is that voting actually took place as scheduled and other of the entire case. But if there is something more to be
valid election returns still existed. Moreover, the done in the case after its issuance, that order is
destruction or loss did not affect the result of the election. interlocutory.
We also declared that there is failure of elections only
when the will of the electorate has been muted and As correctly pointed out by public respondent in
cannot be ascertained. If the will of the people is its assailed order of November 29, 1999, the October 11,
determinable, the same must as far as possible be 1999 did not dispose of the case completely as there is
respected. something more to be done which is to decide the election
protest. As such, it is the herein public respondent
Xxx (Second Division of the COMELEC) which issued the
interlocutory order of October 11, 1999 that should
In Loong v. COMELEC (257 SCRA 1 [1996]), the petition resolve petitioner’s motion for reconsideration, not the
for annulment of election results or to declare failure of COMELEC en banc (Bulaong v. COMELEC, First Division,
elections in Parang, Sulu, on the ground of statistical 220 SCRA 745, 749 [1993]). Accordingly, the applicable
improbability and massive fraud was granted by the rule on the subject is Section 5(c), Rule 3 of the COMELEC
COMELEC. Even before the technical examination of Rules of Procedure, which states:
election documents was conducted, the Comelec already
observed badges of fraud just by looking at the election “Rule 3, Section 5(c). Any motion to reconsider a
results in Parang. Nevertheless, the Comelec dismissed decision, resolution, order or ruling of a Division shall be
the petition for annulment of election results or to declare resolved by the Commission en banc except motions on
failure of elections in the municipalities of Tapul, Panglima interlocutory orders of the division, which shall be
Estino, Pata, Siasi and Kalinggalang Calauag. The resolved by the divisions which issued the order.”
COMELEC dismissed the latter action on ground of
untimeliness of the petition, despite a finding that the That only final orders of a Division may be raised
same badges of fraud evident from the results of the before the COMELEC en banc is in accordance with Article
election based on the certificates of canvass of votes in IX-C, Section 3 of the Constitution which mandates that
Parang, are also evident in the election results of the five only motions for reconsideration of final decisions shall be
mentioned municipalities. We ruled that Comelec decided by the Commission on Elections en banc, thus:
committed grave abuse of discretion in dismissing the
petition as there is no law which provides for a “Sec. 3. The Commission on Elections may sit en banc or in
reglementary period to file annulment of elections when two divisions, and shall promulgate its rules of procedure in
there is yet no proclamation. The election resulted in a order to expedite disposition of election cases, including pre-
failure to elect on account of fraud. Accordingly, we proclamation controversies. All such election cases shall be
ordered the Comelec to reinstate the aforesaid petition. heard and decided in division, provided that motions for
Those circumstances, however, are not present in this reconsideration of decisions shall be decided by the
case, so that reliance on Loong by petitioner Banaga is Commission en banc.”
misplaced. (Banaga, Jr. v. COMELEC, 336 SCRA 701,
July 31, 2000, En Banc [Quisumbing]) It bears stressing that under this constitutional
provision, the COMELEC en banc shall decide motions for
316. What acts of a Division of the COMELEC may be reconsideration only of “decisions” of a Division, meaning
subject of a motion for reconsideration of the COMELEC those acts of final character. Clearly, the assailed order
en banc? denying petitioner's demurrer to evidence, being
interlocutory, may not, be resolved by the COMELEC en
Held: Section 5, Rule 19 of the COMELEC Rules of banc (Ambil, Jr. v. Commission on elections, G.R. No.
Procedure, provides: 143398, Oct. 25, 2000, 344 SCRA 358). (Gementiza v.
Commission on Elections, 353 SCRA 724, March 6,
“SEC. 5. How Motion for Reconsideration Disposed of. - 2001, En Banc [Sandoval-Gutierrez])
Upon the filing of a motion to reconsider a decision,
317. Is a petition to declare failure of election different An election protest is governed by Rule 20 on ordinary
from a petition to annul the election results? actions, while a petition to declare failure of elections is
covered by Rule 26 under special actions.
Held: A prayer to declare failure of elections and a prayer
to annul the election results x x x are actually of the same In this case, petitioner filed his petition as a special action
nature. Whether an action is for declaration of failure of and paid the corresponding fee therefor. Thus, the
elections or for annulment of election results, based on petition was docketed as SPA-98-383. This conforms to
allegations of fraud, terrorism, violence or analogous, the petitioner’s categorization of his petition as one to declare
Omnibus Election Code denominates them similarly. a failure of elections or annul election results. In contrast,
(Banaga, Jr. v. COMELEC, 336 SCRA 701, July 31, an election protest is assigned a docket number starting
2000, En Banc [Quisumbing]) with “EPC,” meaning election protest case.

318. What conditions must concur before the Comelec Third, petitioner did not comply with the requirements for
can act on a verified petition seeking to declare a failure filing an election protest. He failed to pay the required
of election? Is low turn-out of voters enough basis to filing fee and cash deposits for an election protest. Failure
grant the petition? to pay filing fees will not vest the election tribunal
jurisdiction over the case. Such procedural lapse on the
Held: Before COMELEC can act on a verified petition part of a petitioner would clearly warrant the outright
seeking to declare a failure of election, two (2) conditions dismissal of his action.
must concur: first, no voting has taken place in the
precinct or precincts on the date fixed by law or, even if Fourth, an en banc decision of Comelec in an ordinary
there was voting, the election nevertheless results in action becomes final and executory after thirty (30) days
failure to elect; and, second, the votes not cast would from its promulgation, while an en banc decision in a
affect the result of the election. special action becomes final and executory after five (5)
days from promulgation, unless restrained by the
There can be failure of election in a political unit only if Supreme Court (Comelec Rules of Procedure, Rule 18,
the will of the majority has been defiled and cannot be Section 13 [a], [b]). For that reason, a petition cannot be
ascertained. But, if it can be determined, it must be treated as both an election protest and a petition to
accorded respect. After all, there is no provision in our declare failure of elections.
election laws which requires that a majority of registered
voters must cast their votes. All the law requires is that Fifth, the allegations in the petition decisively determine
a winning candidate must be elected by a plurality of valid its nature. Petitioner alleged that the local elections for
votes, regardless of the actual number of ballots cast. the office of vice-mayor in Paranaque City held on May
Thus, even if less than 25% of the electorate in the 11, 1998, denigrates the true will of the people as it was
questioned precincts cast their votes, the same must still marred with widespread anomalies on account of vote
be respected. (Mitmug v. COMELEC, 230 SCRA 54, buying, flying voters and glaring discrepancies in the
Feb. 10, 1994, En Banc [Bellosillo]) election returns. He averred that those incidents warrant
the declaration of a failure of elections.
319. Distinguish a petition to declare failure of
elections from an election protest. Given these circumstances, public respondent cannot be
said to have gravely erred in treating petitioner’s action
Held: While petitioner may have intended to institute an as a petition to declare failure of elections or to annul
election protest by praying that said action may also be election results. (Banaga, Jr. v. COMELEC, 336 SCRA
considered an election protest, in our view, petitioner’s 701, July 31, 2000, En Banc [Quisumbing])
action is a petition to declare a failure of elections or annul
election results. It is not an election protest. 320. What are pre-proclamation cases, and exceptions
thereto? What Court has jurisdiction over pre-
First, his petition before the Comelec was instituted proclamation cases?
pursuant to Section 4 of Republic Act No. 7166 in relation
to Section 6 of the Omnibus Election Code. Section 4 of Held: As a general rule, candidates and registered
RA 7166 refers to “postponement, failure of election and political parties involved in an election are allowed to file
special elections” while Section 6 of the Omnibus Election pre-proclamation cases before the Comelec. Pre-
Code relates to “failure of election.” It is simply captioned proclamation cases refer to any question pertaining to or
as “Petition to Declare Failure of Elections and/or For affecting the proceedings of the board of canvassers
Annulment of Elections.” which may be raised by any candidate or by any
registered political party or coalition of political parties
Second, an election protest is an ordinary action while a before the board or directly with the Commission, or any
petition to declare a failure of elections is a special action matter raised under Sections 233, 234, 235 and 236 in
under the 1993 Comelec Rules of Procedure as amended. relation to the preparation, transmission, receipt, custody
and appreciation of election returns (Section 241, Section 5, however, of the same rule states that a petition
Omnibus Election Code). The Comelec has exclusive for correction of manifest error may be filed directly with
jurisdiction over all pre-proclamation controversies the Commission en banc provided that such errors could
(Section 242, supra). As an exception, however, to the not have been discovered during the canvassing despite
general rule, Section 15 of Republic Act 7166 prohibits the exercise of due diligence and proclamation of the
candidates in the presidential, vice-presidential, senatorial winning candidate had already been made. (Federico S.
and congressional elections from filing pre-proclamation Sandoval v. COMELEC, G.R. No. 133842, Jan. 26,
cases. It states: 2000 [Puno])

“Sec. 15. Pre-Proclamation Cases Not Allowed in 322. Distinguish Election Protest from Petition for Quo
Elections for President, Vice-President, Senator, and Warranto.
Members of the House of Representatives. - For
purposes of the elections for President, Vice-President, Held: In Samad v. COMELEC, we explained that a
Senator and Member of the House of Representatives, no petition for quo warranto under the Omnibus Election
pre-proclamation cases shall be allowed on matters Code raises in issue the disloyalty or ineligibility of the
relating to the preparation, transmission, receipt, custody winning candidate. It is a proceeding to unseat the
and appreciation of election returns or the certificates of respondent from office but not necessarily to install the
canvass, as the case may be. However, this does not petitioner in his place. An election protest is a contest
preclude the authority of the appropriate canvassing body between the defeated and winning candidates on the
motu proprio or upon written complaint of an interested ground of frauds or irregularities in the casting and
person to correct manifest errors in the certificate of counting of the ballots, or in the preparation of the
canvass or election returns before it.” returns. It raises the question of who actually obtained
the plurality of the legal votes and therefore is entitled to
The prohibition aims to avoid delay in the proclamation of hold the office. (Dumayas, Jr. v. COMELEC, G.R. Nos.
the winner in the election, which delay might result in a 141952-53, April 20, 2001, En Banc [Quisumbing])
vacuum in these sensitive posts. The law, nonetheless,
provides an exception to the exception. The second 323. What is a counter-protest? When should it be
sentence of Section 15 allows the filing of petitions for filed?
correction of manifest errors in the certificate of canvass
or election returns even in elections for president, vice- Held: Under the Comelec Rules of Procedure, the
president and members of the House of Representatives protestee may incorporate in his answer a counter-
for the simple reason that the correction of manifest error protest. It has been said that a counter-protest is
will not prolong the process of canvassing nor delay the tantamount to a counterclaim in a civil action and may
proclamation of the winner in the election. The rule is be presented as a part of the answer within the time he
consistent with and complements the authority of the is required to answer the protest, i.e., within five (5) days
Comelec under the Constitution to "enforce and upon receipt of the protest, unless a motion for extension
administer all laws and regulations relative to the conduct is granted, in which case it must be filed before the
of an election, plebiscite, initiative, referendum and recall" expiration of the extended time.
(Section 2[1], Article IX-C, 1987 Constitution) and its
power to "decide, except those involving the right to vote, As early as in the case of Arrieta v. Rodriguez (57 Phil.
all questions affecting elections." (Section 2[3], Article IX- 717), the SC had firmly settled the rule that the counter-
C, supra) (Federico S. Sandoval v. COMELEC, G.R. protest must be filed within the period provided by law,
No. 133842, Jan. 26, 2000 [Puno]) otherwise, the forum loses its jurisdiction to entertain the
belatedly filed counter-protest. (Kho v. COMELEC, 279
321. Who has authority to rule on petitions for SCRA 463, Sept. 25, 1997, En Banc [Torres])
correction of manifest error in the certificate of canvass
or election returns? 324. What is the effect of death of a party in an
election protest? Should it warrant the dismissal of the
Held: The authority to rule on petitions for correction of protest?
manifest error is vested in the Comelec en banc. Section
7 of Rule 27 of the 1993 COMELEC Rules of Procedure Held: An election protest involves both the private
(took effect on February 15, 1993) provides that if the interests of the rival candidates and the public interest in
error is discovered before proclamation, the board of the final determination of the real choice of the electorate,
canvassers may motu proprio, or upon verified petition by and for this reason, an election contest necessarily
any candidate, political party, organization or coalition of survives the death of the protestant or the protestee. It
political parties, after due notice and hearing, correct the is true that a public office is personal to the public officer
errors committed. The aggrieved party may appeal the and is not a property transmissible to his heirs upon
decision of the board to the Commission and said appeal death, thus, upon the death of the incumbent, no heir of
shall be heard and decided by the Commission en banc. his may be allowed to continue holding his office in his
place. But while the right to a public office is personal proclamation which marks off the jurisdiction of the courts
and exclusive to the public officer, an election protest is from the jurisdiction of election officials.
not purely personal and exclusive to the protestant or to
the protestee such that after the death of either would The case of Jose M. Mercado v. Board of Election
oust the court of all authority to continue the protest Supervisors (243 SCRA 423, G.R. No. 109713, April 6,
proceedings. An election contest, after all, involves not 1995), in which this Court ruled that election protests
merely conflicting private aspirations but is imbued with involving SK elections are to be determined by the Board
paramount public interests. The death of the protestant of Election Supervisors was decided under the aegis of
neither constitutes a ground for the dismissal of the Comelec Resolution No. 2499, which took effect on
contest nor ousts the trial court of its jurisdiction to decide August 27, 1992. However, Comelec Resolution No.
the election contest. (De Castro v. COMELEC, 267 2824, which took effect on February 6, 1996 and was
SCRA 806, Feb. 7, 1997) passed pursuant to R.A. 7808, in relation to Arts. 252-253
of the Omnibus Election Code, has since transferred the
325. Does the fact that one or a few candidates in an cognizance of such cases from the Board of Election
election got zero votes in one or a few precincts Supervisors to the MTCs, MCTCs and MeTCs. Thus, the
adequately support a finding that the election returns are doctrine of Mercado is no longer controlling. (Francis
statistically improbable? King L. Marquez v. COMELEC, G.R. No. 127318,
Aug. 25, 1999, En Banc [Purisima])
Held: From experiences in past elections, it is possible
for one candidate or even a few candidates to get zero 327. What acts of a Division of the COMELEC may be
votes in one or a few precincts. subject of a motion for reconsideration of the COMELEC
en banc?
Standing alone and without more, the bare fact that a
candidate for public office received zero votes in one or Held: Section 5, Rule 19 of the COMELEC Rules of
two precincts can not adequately support a finding that Procedure, provides:
the subject election returns are statistically improbable. A
no-vote for a particular candidate in election returns is but “SEC. 5. How Motion for Reconsideration Disposed of. -
one strand in the web of circumstantial evidence that Upon the filing of a motion to reconsider a decision,
those election returns were prepared under “duress, force resolution, order or ruling of a Division, the Clerk of Court
and intimidation.” In the case of Una Kibad v. Comelec concerned shall, within twenty-four (24) hours from the
(23 SCRA 588 [1968]), the SC warned that the doctrine filing thereof, notify the presiding Commissioner. The
of statistical improbability must be viewed restrictively, latter shall within two (2) days thereafter certify the case
the utmost care being taken lest in penalizing the to the Commission en banc.”
fraudulent and corrupt practices, innocent voters become
disenfranchised, a result which hardly commends itself. Under the above-quoted rule, the acts of a
Moreover, the doctrine of statistical improbability involves Division that are subject of a motion for reconsideration
a question of fact and a more prudential approach must have a character of finality before the same can be
prohibits its determination ex parte. (Arthur V. Velayo elevated to the COMELEC en banc. The elementary rule
v. COMELEC, G.R. No. 135613, March 9, 2000, En is that an order is final in nature if it completely disposes
Banc [Puno]) of the entire case. But if there is something more to be
done in the case after its issuance, that order is
326. What Court has jurisdiction over election protests interlocutory.
and quo warranto proceedings involving Sangguniang
Kabataan (SK) elections? As correctly pointed out by public respondent in
its assailed order of November 29, 1999, the October 11,
Held: Any contest relating to the election of members of 1999 did not dispose of the case completely as there is
the Sangguniang Kabataan (including the chairman) – something more to be done which is to decide the election
whether pertaining to their eligibility or the manner of protest. As such, it is the herein public respondent
their election – is cognizable by MTCs, MCTCs, and (Second Division of the COMELEC) which issued the
MeTCs. Section 6 of Comelec Resolution No. 2824 which interlocutory order of October 11, 1999 that should
provides that cases involving the eligibility or qualification resolve petitioner’s motion for reconsideration, not the
of SK candidates shall be decided by the City/Municipal COMELEC en banc (Bulaong v. COMELEC, First Division,
Election Officer whose decision shall be final, applies only 220 SCRA 745, 749 [1993]). Accordingly, the applicable
to proceedings before the election. Before proclamation, rule on the subject is Section 5(c), Rule 3 of the COMELEC
cases concerning eligibility of SK officers and members Rules of Procedure, which states:
are cognizable by the Election Officer. But after the
election and proclamation, the same cases become quo “Rule 3, Section 5(c). Any motion to reconsider a
warranto cases cognizable by MTCs, MCTCs, and MeTCs. decision, resolution, order or ruling of a Division shall be
The distinction is based on the principle that it is the resolved by the Commission en banc except motions on
interlocutory orders of the division, which shall be 329. Is there a conflict between the power of the
resolved by the divisions which issued the order.” President to merge administrative regions with the
constitutional provision requiring a plebiscite in the
That only final orders of a Division may be raised merger of local government units?
before the COMELEC en banc is in accordance with Article
IX-C, Section 3 of the Constitution which mandates that Held: There is no conflict between the power of the
only motions for reconsideration of final decisions shall be President to merge administrative regions with the
decided by the Commission on Elections en banc, thus: constitutional provision requiring a plebiscite in the
merger of local government units because the
“Sec. 3. The Commission on Elections may sit en banc or in requirement of a plebiscite in a merger expressly applies
two divisions, and shall promulgate its rules of procedure in only to provinces, cities, municipalities or barangays, not
order to expedite disposition of election cases, including pre- to administrative regions. (Abbas v. COMELEC, 179
proclamation controversies. All such election cases shall be SCRA 287, Nov. 10, 1989, En Banc [Cortes])
heard and decided in division, provided that motions for
reconsideration of decisions shall be decided by the 330. What is the Metropolitan Manila Development
Commission en banc.” Authority (MMDA)? Is it a local government unit or public
corporation endowed with legislative power? May it
It bears stressing that under this constitutional validly exercise police power? How is it distinguished
provision, the COMELEC en banc shall decide motions for from the former Metro Manila Council (MMC) created
reconsideration only of “decisions” of a Division, meaning under PD No. 824?
those acts of final character. Clearly, the assailed order
denying petitioner's demurrer to evidence, being Held: Metropolitan or Metro Manila is a body composed
interlocutory, may not, be resolved by the COMELEC en of several local government units – i.e., twelve (12) cities
banc (Ambil, Jr. v. Commission on elections, G.R. No. and five (5) municipalities x x x. With the passage of
143398, Oct. 25, 2000, 344 SCRA 358). (Gementiza v. Republic Act No. 7924 in 1995, Metropolitan Manila was
Commission on Elections, 353 SCRA 724, March 6, declared as a “special development and administrative
2001, En Banc [Sandoval-Gutierrez]) region” and the Administration of “metrowide” basic
services affecting the region placed under “a development
authority” referred to as the MMDA.

The governing board of the MMDA is the Metro Manila


THE LAW OF PUBLIC CORPORATIONS Council. The Council is composed of the mayors of the
component 12 cities and 5 municipalities, the president of
328. What is an autonomous region? the Metro Manila Vice-Mayors’ League and the president
of the Metro Manila Councilors’ League. The Council is
Answer: An autonomous region consists of provinces, headed by a Chairman who is appointed by the President
cities, municipalities, and geographical areas sharing and vested with the rank of cabinet member. As the
common and distinctive historical and cultural heritage, policy-making body of the MMDA, the Metro Manila
economic and social structures, and other relevant Council approves metro-wide plans, programs and
characteristics within the framework of the Constitution projects, and issues the necessary rules and regulations
and the national sovereignty as well as the territorial for the implementation of said plans; it approves the
integrity of the Republic of the Philippines. (Sec. 15, Art. annual budget of the MMDA and promulgates the rules
X, 1987 Constitution) and regulations for the delivery of basic services,
collection of service and regulatory fees, fines and
329. What are administrative regions? Are they penalties. X x x
considered territorial and political subdivisions of the
State? Who has the power to create administrative Clearly, the scope of the MMDA’s function is limited to the
regions? delivery of the seven (7) basic services. One of these is
transport and traffic management x x x.
Held: Administrative regions are mere groupings of
contiguous provinces for administrative purposes. They It will be noted that the powers of the MMDA are limited
are not territorial and political subdivisions like provinces, to the following acts: formulation, coordination,
cities, municipalities and barangays. While the power to regulation, implementation, preparation, management,
merge administrative regions is not expressly provided for monitoring, setting of policies, installation of a system and
in the Constitution, it is a power which has traditionally administration. There is no syllable in R.A. No. 7924 that
been lodged with the President to facilitate the exercise grants the MMDA police power, let alone legislative
of the power of general supervision over local power. Even the Metro Manila Council has not been
governments. (Abbas v. COMELEC, 179 SCRA 287, delegated any legislative power. Unlike the legislative
Nov. 10, 1989, En Banc [Cortes]) bodies of the local government units, there is no provision
in R.A. No. 7924 that empowers the MMDA or its Council Xxx
“to enact ordinances, approve resolutions and appropriate
funds for the general welfare” of the inhabitants of Metro The creation of the MMC also carried with it the creation
Manila. The MMDA is x x x a “development authority.” It of the Sangguniang Bayan. This was composed of the
is an agency created for the purpose of laying down members of the component city and municipal councils,
policies and coordinating with the various national barangay captains chosen by the MMC and sectoral
government agencies, people’s organizations, non- representatives appointed by the President. The
governmental organizations and the private sector for the Sangguniang Bayan had the power to recommend to the
efficient and expeditious delivery of basic services in the MMC the adoption of ordinances, resolutions or measures.
vast metropolitan area. All its functions are administrative It was the MMC itself, however, that possessed legislative
in nature and these are actually summed up in the charter powers. All ordinances, resolutions and measures
itself x x x. recommended by the Sangguniang Bayan were subject to
the MMC’s approval. Moreover, the power to impose
Secondly, the MMDA is not the same entity as the MMC in taxes and other levies, the power to appropriate money,
Sangalang. Although the MMC is the forerunner of the and the power to pass ordinances or resolutions with
present MMDA, an examination of Presidential Decree No. penal sanctions were vested exclusively in the MMC.
824, the charter of the MMC, shows that the latter
possessed greater powers which were not bestowed on Thus, Metropolitan Manila had a “central government,”
the present MMDA. i.e., the MMC which fully possessed legislative and police
powers. Whatever legislative powers the component
Metropolitan Manila was first created in 1975 by cities and municipalities had were all subject to review
Presidential Decree No. 824. It comprised the Greater and approval by the MMC.
Manila Area composed of the contiguous four (4) cities of
Manila, Quezon, Pasay and Caloocan, and the thirteen After President Corazon Aquino assumed power, there
(13) municipalities x x x. Metropolitan Manila was was a clamor to restore the autonomy of the local
created as a response to the finding that the rapid growth government units in Metro Manila. Hence, Sections 1 and
of population and the increase of social and economic 2 of Article X of the 1987 Constitution x x x. The
requirements in these areas demand a call for Constitution, however, recognized the necessity of
simultaneous and unified development; that the public creating metropolitan regions not only in the existing
services rendered by the respective local governments National Capital Region but also in potential equivalents
could be administered more efficiently and economically in the Visayas and Mindanao. X x x
if integrated under a system of central planning; and this
coordination, “especially in the maintenance of peace and The Constitution itself expressly provides that Congress
order and the eradication of social and economic ills that may, by law, create “special metropolitan political
fanned the flames of rebellion and discontent [were] part subdivisions” which shall be subject to approval by a
of the reform measures under Martial Law essential to the majority of the votes cast in a plebiscite in the political
safety and security of the State.” units directly affected; the jurisdiction of this subdivision
shall be limited to basic services requiring coordination;
Metropolitan Manila was established as a “public and the cities and municipalities comprising this
corporation” x x x. subdivision shall retain their basic autonomy and their
own local executive and legislative assemblies (Section
The administration of Metropolitan Manila was placed 11, Article X, 1987 Constitution). Pending enactment of
under the Metro Manila Commission (MMC) x x x. this law, the Transitory Provisions of the Constitution gave
the President of the Philippines the power to constitute
The MMC was the “central government” of Metro Manila the Metropolitan Authority x x x.
for the purpose of establishing and administering
programs providing services common to the area. As a In 1990, President Aquino issued Executive Order No. 392
“central government” it had the power to levy and collect and constituted the Metropolitan Manila Authority (MMA).
taxes and special assessments, the power to charge and The powers and functions of the MMC were devolved to
collect fees; the power to appropriate money for its the MMA. It ought to be stressed, however, that not all
operation, and at the same time, review appropriations powers and functions of the MMC were passed to the
for the city and municipal units within its jurisdiction. It MMA. The MMA’s power was limited to the “delivery of
was bestowed the power to enact or approve ordinances, basic urban services requiring coordination in
resolutions and fix penalties for violation of such Metropolitan Manila.” The MMA’s governing body, the
ordinances and resolutions. It also had the power to Metropolitan Manila Council, although composed of the
review, amend, revise or repeal all ordinances, resolutions mayors of the component cities and municipalities, was
and acts of any of the x x x cities and x x x municipalities merely given the power of: (1) formulation of policies on
comprising Metro Manila. the delivery of basic services requiring coordination and
consolidation; and (2) promulgation of resolutions and
other issuances, approval of a code of basic services and the welfare of the community. It is the local government
the exercise of its rule-making power.” units, acting through their respective legislative councils,
that possess legislative power and police power. In the
Under the 1987 Constitution, the local government units case at bar, the Sangguniang Panlungsod of Makati City
became primarily responsible for the governance of their did not pass any ordinance or resolution ordering the
respective political subdivisions. The MMA’s jurisdiction opening of Neptune Street, hence, its proposed opening
was limited to addressing common problems involving by petitioner MMDA is illegal x x x. (MMDA v. Bel-Air
basic services that transcended local boundaries. It did Village Association, Inc., 328 SCRA 836, March 27,
not have legislative power. Its power was merely to 2000, 1st Div. [Puno])
provide the local government units technical assistance in
the preparation of local development plans. Any 331. Discuss the concept of local autonomy.
semblance of legislative power it had was confined to a
“review [of] legislation proposed by the local legislative Held: Autonomy is either decentralization of
assemblies to ensure consistency among local administration or decentralization of power. There is
governments and with the comprehensive development decentralization of administration when the central
plan of Metro Manila,” and to “advise the local government delegates administrative powers to political
governments accordingly.” subdivisions in order to broaden the base of government
and in the process to make local governments more
When R.A. No. 7924 took effect, Metropolitan Manila responsive and accountable, and ensure their fullest
became a “special development and administrative development as self-reliant communities and make them
region” and the MMDA a “special development authority” more effective partners in the pursuit of national
whose functions were “without prejudice to the autonomy development and social progress. At the same time, it
of the affected local government units.” The character of relieves the central government of the burden of
the MMDA was clearly defined in the legislative debates managing local affairs and enables it to concentrate on
enacting its charter. national concerns. The President exercises general
supervision over them, but only to ensure that local affairs
Xxx are administered according to law. He has no control over
their acts in the sense that he can substitute their
Clearly, the MMDA is not a political unit of government. judgments with his own.
The power delegated to the MMDA is that given to the
Metro Manila Council to promulgate administrative rules Decentralization of power, on the other hand, involves an
and regulations in the implementation of the MMDA’s abdication of political power in favor of local government
functions. There is no grant of authority to enact units declared autonomous. In that case, the
ordinances and regulations for the general welfare of the autonomous government is free to chart its own destiny
inhabitants of the metropolis. This was explicitly stated and shape its own future with minimum intervention from
in the last Committee deliberations prior to the bill’s central authorities. According to a constitutional author,
presentation to Congress. X x x decentralization of power amounts to “self-immolation,”
since in that event, the autonomous government
It is thus beyond doubt that the MMDA is not a local becomes accountable not to the central authorities but to
government unit or a public corporation endowed with its constituency. (Limbona v. Mangelin, 170 SCRA
legislative power. It is not even a “special metropolitan 786, Feb. 28, 1989, En Banc [Sarmiento])
political subdivision” as contemplated in Section 11,
Article X of the Constitution. The creation of a “special 332. What kind of local autonomy is contemplated by
metropolitan political subdivision” requires the approval the Constitution? What about the kind of autonomy
by a majority of the votes cast in a plebiscite in the contemplated insofar as the autonomous regions are
political units directly affected. R.A. No. 7924 was not concerned?
submitted to the inhabitants of Metro Manila in a
plebiscite. The Chairman of the MMDA is not an official Held: 1. The principle of local autonomy under the 1987
elected by the people, but appointed by the President Constitution simply means “decentralization.” It does not
with the rank and privileges of a cabinet member. In fact, make local governments sovereign within the state or an
part of his function is to perform such other duties as may “imperium in imperio.” Remaining to be an intra
be assigned to him by the President, whereas in local sovereign subdivision of one sovereign nation, but not
government units, the President merely exercises intended, however, to be an imperium in imperio,” the
supervisory authority. This emphasizes the administrative local government unit is autonomous in the sense that it
character of the MMDA. is given more powers, authority, responsibilities and
resources. Power which used to be highly centralized in
Clearly then, the MMC under P.D. No. 824 is not the same Manila, is thereby deconcentrated, enabling especially the
entity as the MMDA under R.A. No. 7924. Unlike the peripheral local government units to develop not only at
MMC, the MMDA has no power to enact ordinances for their own pace and discretion but also with their own
resources and assets. (Alvarez v. Guingona, Jr., 252 courts, unless such construction is clearly shown to be in
SCRA 695, Jan. 31, 1996, En Banc [Hermosisima]) sharp conflict with the Constitution, the governing
statute, or other laws. (Alvarez v. Guingona, Jr., 252
2. The constitutional guarantee of local autonomy in the SCRA 695, Jan. 31, 1996, En Banc [Hermosisima,
Constitution refers to the administrative autonomy of local Jr., J.])
government units or, cast in more technical language, the
decentralization of government authority. 334. State the importance of drawing with precise
strokes the territorial boundaries of a local government
On the other hand, the creation of autonomous regions in unit.
Muslim Mindanao and the Cordilleras, which is peculiar to
the 1987 Constitution, contemplates the grant of political Held: The importance of drawing with precise strokes
autonomy and not just administrative autonomy to these the territorial boundaries of a local unit of government
regions. Thus, the provision in the Constitution for an cannot be overemphasized. The boundaries must be
autonomous regional government with a basic structure clear for they define the limits of the territorial jurisdiction
consisting of an executive department and a legislative of a local government unit. It can legitimately exercise
assembly and special courts with personal, family and powers of government only within the limits of its
property law jurisdiction in each of the autonomous territorial jurisdiction. Beyond these limits, its acts are
regions. (Cordillera Broad Coalition v. COA, 181 ultra vires. Needless to state, any uncertainty in the
SCRA 495, Jan. 29, 1990, En Banc [Cortes]) boundaries of local government units will sow costly
conflicts in the exercise of governmental powers which
333. Whether or not the Internal Revenue allotments ultimately will prejudice the people’s welfare. This is the
(IRAs) are to be included in the computation of the evil sought to be avoided by the Local Government Code
average annual income of a municipality for purposes of in requiring that the land area of a local government unit
its conversion into an independent component city? must be spelled out in metes and bounds, with technical
descriptions. (Mariano, Jr. v. COMELEC, 242 SCRA
Held: Yes. The IRAs are items of income because they 211, 217-219, Mar. 7, 1995, En Banc [Puno])
form part of the gross accretion of the funds of the local
government unit. The IRAs regularly and automatically 335. R.A. 7854 was enacted converting the
accrue to the local treasury without need of any further Municipality of Makati into a highly urbanized city. Section
action on the part of the local government unit. They 2 thereof did not provide for a cadastral type of
thus constitute income which the local government can description of its boundary but merely provided that the
invariably rely upon as the source of much needed funds. boundary of the new city of Makati shall be the boundary
of the present municipality of Makati. Petitioners
Xxx contended in a petition brought the SC that R.A. 7854 was
defective because it did not comply with the requirement
[T]o reiterate, IRAs are a regular, recurring item of in the Local Government Code that “the territorial
income; nil is there a basis, too, to classify the same as a jurisdiction of newly created or converted cities should be
special fund or transfer, since IRAs have a technical described by metes and bounds, with technical
definition and meaning all its own as used in the Local descriptions.” Note that at the time the law was enacted,
Government Code that unequivocally makes it distinct there was a pending boundary dispute between Makati
from special funds or transfers referred to when the Code and one of its neighbors, Taguig, before the regular court.
speaks of “funding support from the national government, Should the contention be upheld?
its instrumentalities and government-owned or controlled
corporations.” Held: Given the facts of the cases at bench, we cannot
perceive how this evil (uncertainty in the boundaries of
Thus, Department of Finance Order No. 35-93 correctly local government units will sow costly conflicts in the
encapsulizes the full import of the above disquisition when exercise of government powers which ultimately will
it defined ANNUAL INCOME to be “revenues and receipts prejudice the people’s welfare) can be brought about by
realized by provinces, cities and municipalities from the description made in Section 2 of R.A. No. 7854.
regular sources of the Local General Fund including the Petitioners have not demonstrated that the delineation of
internal revenue allotment and other shares provided for the land area of the proposed City of Makati will cause
in Sections 284, 290 and 291 of the Code, but exclusive confusion as to its boundaries. We note that said
of non-recurring receipts, such as other national aids, delineation did not change even by an inch the land area
grants, financial assistance, loan proceeds, sales of fixed previously covered by Makati as a municipality. Section 2
assets, and similar others”. Such order, constituting did not add, subtract, divide, or multiply the established
executive or contemporaneous construction of a statute land area of Makati. In language that cannot be any
by an administrative agency charged with the task of clearer, Section 2 stated that the city’s land area “shall
interpreting and applying the same, is entitled to full comprise the present territory of the municipality.”
respect and should be accorded great weight by the
The deliberations of Congress will reveal that there is a serve the letter but defeat the spirit of the Code. It then
legitimate reason why the land area of the proposed City becomes a case of the master serving the slave, instead
of Makati was not defined by metes and bounds, with of the other way around. This could not be the
technical descriptions. At the time of the consideration of intendment of the law.” X x x
R.A. No. 7854, the territorial dispute between the (Mariano, Jr. v. COMELEC, 242 SCRA 211, 217-219,
municipalities of Makati and Taguig over Fort Bonifacio Mar. 7, 1995, En Banc [Puno])
was under court litigation. Out of a becoming sense of
respect to a co-equal department of government, the 336. What is the meaning of "devolution"?
legislators felt that the dispute should be left to the courts
to decide. They did not want to foreclose the dispute by Answer: The term "devolution" refers to the act by
making a legislative finding of fact which could decide the which the National government confers power and
issue. This would have ensued if they defined the land authority upon the various local government units to
area of the proposed city by its exact metes and bounds, perform specific functions and responsibilities. (Sec.
with technical descriptions. We take judicial notice of the 17[e], 2nd par., Local Government Code)
fact that Congress has also refrained from using the
metes and bounds description of the land area of other 337. Have the powers of the Land Transportation
local government units with unsettled boundary disputes. Office (LTO) to register, tricycles in particular, as well as
to issue licenses for the driving thereof, been devolved
We hold that the existence of a boundary dispute does likewise to local government units?
not per se present an insurmountable difficulty which will
prevent Congress from defining with reasonable certitude Held: Only the powers of the Land Transportation
the territorial jurisdiction of a local government unit. In Franchising Regulatory Board (LTFRB) to regulate the
the cases at bench, Congress maintained the existing operation of tricycles-for-hire and to grant franchises for
boundaries of the proposed City of Makati but as an act the operation thereof have been devolved to local
of fairness, made them subject to the ultimate resolution governments under the Local Government Code. Clearly
by the courts. Considering these peculiar circumstances, unaffected by the Local Government Code are the powers
we are not prepared to hold that Section 2 of R.A. No. of the LTO under R.A. No. 4136 requiring the registration
7854 is unconstitutional. We sustain the submission of of all kinds of motor vehicles “used or operated on or upon
the Solicitor General in this regard, viz: any public highway” in the country. This can be gleaned
from the explicit language of the statute itself, as well as
“Going now to Sections 7 and 450 of the Local the corresponding guidelines issued by the DOTC. In fact,
Government Code, it is beyond cavil that the requirement even the power of LGUs to regulate the operation of
started therein, viz: ‘the territorial jurisdiction of newly tricycles and to grant franchises for the operation thereof
created or converted cities should be described by metes is still subject to the guidelines prescribed by the DOTC.
and bounds, with technical descriptions” – was made in (LTO v. City of Butuan, G.R. No. 131512, Jan. 20,
order to provide a means by which the area of said cities 2000, 3rd Div. [Vitug])
may be reasonably ascertained. In other words, the
requirement on metes and bounds was meant merely as 338. Distinguish the power to grant a license or permit
a tool in the establishment of local government units. It to do business and the power to issue a license to engage
is not an end in itself. Ergo, so long as the territorial in the practice of a particular profession.
jurisdiction of a city may be reasonably ascertained, i.e.,
by referring to common boundaries with neighboring Held: Distinction must be made between the grant of a
municipalities, as in this case, then, it may be concluded license or permit to do business and the issuance of a
that the legislative intent behind the law has been license to engage in the practice of a particular
sufficiently served. profession. The first is usually granted by the local
authorities and the second is issued by the Board or
Certainly, Congress did not intend that laws creating new Commission tasked to regulate the particular profession.
cities must contain therein detailed technical descriptions A business permit authorizes the person, natural or
similar to those appearing in Torrens titles, as petitioners otherwise, to engage in business or some form of
seem to imply. To require such description in the law as commercial activity. A professional license, on the other
a condition sine qua non for its validity would be to defeat hand, is the grant of authority to a natural person to
the very purpose which the Local Government Code seeks engage in the practice or exercise of his or her profession.
to serve. The manifest intent of the Code is to empower
local government units and to give them their rightful due. In the case at bar, what is sought by petitioner (Acebedo
It seeks to make local governments more responsive to Optical Company, Inc.) from respondent City Mayor is a
the needs of their constituents while at the same time permit to engage in the business of running an optical
serving as a vital cog in national development. To shop. It does not purport to seek a license to engage in
invalidate R.A. No. 7854 on the mere ground that no the practice of optometry as a corporate body or entity,
cadastral type of description was used in the law would although it does have in its employ, persons who are duly
licensed to practice optometry by the Board of Examiners 4) A valid and definite offer has been previously
in Optometry. made to the owner of the property sought to be
expropriated, but said offer was not accepted.
Xxx (Municipality of Paranaque v. V.M. Realty Corp.,
292 SCRA 678, July 20, 1998 [Panganiban])
In the present case, the objective of the imposition of
subject conditions on petitioner’s business permit could 341. May the Sangguniang Panlalawigan validly
be attained by requiring the optometrists in petitioner’s disapprove a resolution or ordinance of a municipality
employ to produce a valid certificate of registration as calling for the expropriation of private property to be
optometrists, from the Board of Examiners in Optometry. made site of a Farmers Center and Other Government
A business permit is issued primarily to regulate the Sports Facilities on the ground that said “expropriation is
conduct of business and the City Mayor cannot, through unnecessary considering that there are still available lots
the issuance of such permit, regulate the practice of a of the municipality for the establishment of a government
profession, like that of optometry. Such a function is center”?
within the exclusive domain of the administrative agency
specifically empowered by law to supervise the Held: Under the Local Government Code, the
profession, in this case the Professional Regulations Sangguniang Panlalawigan is granted the power to
Commission and the Board of Examiners in Optometry. declare a municipal resolution invalid on the sole ground
(Acebedo Optical Company, Inc. v. CA, 329 SCRA that it is beyond the power of the Sangguniang Bayan or
314, March 31, 2000, En Banc [Purisima]) Mayor to issue. As held in Velazco v. Blas (G.R. No. L-
30456, July 30, 1982, 115 SCRA 540, 544-545), “The only
339. May a local government unit validly authorize an ground upon which a provincial board may declare any
expropriation of private property through a mere municipal resolution, ordinance or order invalid is when
resolution of its lawmaking body? such resolution, ordinance, or order is ‘beyond the powers
conferred upon the council or president making the
Held: The Local Government Code expressly and clearly same.’ A strictly legal question is before the provincial
requires an ordinance or a local law for that purpose. A board in its consideration of a municipal resolution,
resolution that merely expresses the sentiment or opinion ordinance, or order. The provincial board’s disapproval of
of the Municipal Council will not suffice. The case of any resolution, ordinance, or order must be premised
Province of Camarines Sur v. Court of Appeals which held specifically upon the fact that such resolution, ordinance,
that a mere resolution may suffice to support the exercise or order is outside the scope of the legal powers conferred
of eminent domain by a local government unit is not in by law. If a provincial board passes these limits, it usurps
point because the applicable law at that time was B.P. the legislative functions of the municipal council or
337, the previous Local Government Code, which had president. Such has been the consistent course of
provided that a mere resolution would enable an LGU to executive authority.” (Moday v. CA, 268 SCRA 586,
exercise eminent domain. In contrast, R.A. 7160, the Feb. 20, 1997)
present Local Government Code, explicitly required an
ordinance for this purpose. (Municipality of 342. Under Section 8, Article X of the Constitution,
Paranaque v. V.M. Realty Corp., 292 SCRA 678, "[T]he term of office of elective local officials x x x shall
July 20, 1998 [Panganiban]) be three years and no such official shall serve for more
than three consecutive terms." How is this term limit for
340. What are the requisites before a Local elective local officials to be interpreted?
Government Unit can validly exercise the power of
eminent domain? Held: The term limit for elective local officials must be
taken to refer to the right to be elected as well as the
Held: right to serve in the same elective position.
Consequently, it is not enough that an individual has
1) An ordinance is enacted by the local served three consecutive terms in an elective local office,
legislative council authorizing the local chief executive, in he must also have been elected to the same position for
behalf of the LGU, to exercise the power of eminent the same number of times before the disqualification can
domain or pursue expropriation proceedings over a apply. (Borja, Jr. v. COMELEC and Capco, Jr., G.R.
particular private property; No. 133495, Sept. 3, 1998, 295 SCRA 157, En Banc
2) The power of eminent domain is exercised for [Mendoza])
public use, purpose or welfare, or for the benefit of the
poor and the landless; Case No. 1. Suppose A is a vice-mayor who becomes
3) There is payment of just compensation, as mayor by reason of the death of the incumbent. Six
required under Section 9, Article III of the Constitution, months before the next election, he resigns and is twice
and other pertinent laws; elected thereafter. Can he run again for mayor in the
next election?
enhancing the freedom of choice of the people. To
Answer: Yes, because although he has already first consider, therefore, only stay in office regardless of how
served as mayor by succession and subsequently resigned the official concerned came to that office – whether by
from office before the full term expired, he has not election or by succession by operation of law – would be
actually served three full terms in all for the purpose of to disregard one of the purposes of the constitutional
applying the term limit. Under Art. X, Sec. 8, voluntary provision in question. (Borja, Jr. v. COMELEC and
renunciation of the office is not considered as an Capco, Jr., G.R. No. 133495, Sept. 3, 1998, 295
interruption in the continuity of his service for the full term SCRA 157, En Banc [Mendoza])
only if the term is one “for which he was elected.” Since
A is only completing the service of the term for which the 344. Lonzanida was previously elected and served
deceased and not he was elected, A cannot be considered two consecutive terms as mayor of San Antonio,
to have completed one term. His resignation constitutes Zambales prior to the May 1995 mayoral elections. In the
an interruption of the full term. May 1995 elections he again ran for mayor of San
Antonio, Zambales and was proclaimed winner. He
Case No. 2. Suppose B is elected Mayor and, during his assumed office and discharged the rights and duties of
first term, he is twice suspended for misconduct for a total mayor until March 1998 when he was ordered to vacate
of 1 year. If he is twice reelected after that, can he run the post by reason of the COMELEC decision on the
for one more term in the next election? election protest against him which declared his opponent
Juan Alvez the duly elected mayor. Alvez served the
Answer: Yes, because he has served only two full terms remaining portion of the 1995-1998 mayoral term. Is
successively. Lonzanida still qualified to run for mayor of San Antonio,
Zambales in the May 1998 local elections?
In both cases, the mayor is entitled to run for reelection
because the two conditions for the application of the Held: The two requisites for the application of the three
disqualification provisions have not concurred, namely, term rule was absent. First, Lonzanida cannot be
that the local official concerned has been elected three considered as having been duly elected to the post in the
consecutive times and that he has fully served three May 1995 elections, and second, he did not fully serve the
consecutive terms. In the first case, even if the local 1995-1998 mayoral term by reason of involuntary
official is considered to have served three full terms relinquishment of office. After a re-appreciation and
notwithstanding his resignation before the end of the first revision of the contested ballots the COMELEC itself
term, the fact remains that he has not been elected three declared by final judgment that Lonzanida lost in the May
times. In the second case, the local official has been 1995 mayoral elections and his previous proclamation as
elected three consecutive times, but he has not fully winner was declared null and void. His assumption of
served three consecutive terms. office as mayor cannot be deemed to have been by
reason of a valid election but by reason of a void
Case No. 3. The case of vice-mayor C who becomes proclamation. It has been repeatedly held by the SC that
mayor by succession involves a total failure of the two a proclamation subsequently declared void is no
conditions to concur for the purpose of applying Art. X, proclamation at all and while a proclaimed candidate may
Sec. 8. Suppose he is twice elected after that term, is he assume office on the strength of the proclamation of the
qualified to run again in the next election? Board of Canvassers he is only a presumptive winner who
assumes office subject to the final outcome of the election
Answer: Yes, because he was not elected to the office protest. Lonzanida did not serve a term as mayor of San
of mayor in the first term but simply found himself thrust Antonio, Zambales from May 1995 to March 1998 because
into it by operation of law. Neither had he served the full he was not duly elected to the post; he merely assumed
term because he only continued the service, interrupted office as presumptive winner, which presumption was
by the death, of the deceased mayor. (Borja, Jr. v. later overturned by the COMELEC when it decided with
COMELEC and Capco, Jr., G.R. No. 133495, Sept. 3, finality that Lonzanida lost in the May 1995 mayoral
1998, 295 SCRA 157, En Banc [Mendoza]) elections.

343. What are the policies embodied in the Second, Lonzanida cannot be deemed to have served the
constitutional provision barring elective local officials, with May 1995 to 1998 term because he was ordered to vacate
the exception of barangay officials, from serving more his post before the expiration of the term. His opponents'
than three consecutive terms? contention that Lonzanida should be deemed to have
served one full term from May 1995-1998 because he
Held: To prevent the establishment of political dynasties served the greater portion of that term has no legal basis
is not the only policy embodied in the constitutional to support it; it disregards the second requisite for the
provision in question (barring elective local officials, with application of the disqualification, i.e., that he has fully
the exception of barangay officials, from serving more served three consecutive terms. The second sentence of
than three consecutive terms). The other policy is that of the constitutional provision under scrutiny states,
"Voluntary renunciation of office for any length of time
shall not be considered as an interruption in the continuity Section 4 of AO 372, however, orders the withholding,
of service for the full term for which he was elected." The effective January 1, 1998, of 10 percent of the LGUs’ IRA
clear intent of the framers of the Constitution to bar any “pending the assessment and evaluation by the
attempt to circumvent the three-term limit by a voluntary Development Budget Coordinating Committee of the
renunciation of office and at the same time respect the emerging fiscal situation” in the country. Such
people's choice and grant their elected official full service withholding clearly contravenes the Constitution and the
of a term is evident in this provision. Voluntary law. Although, temporary, it is equivalent to a holdback,
renunciation of a term does not cancel the renounced which means “something held back or withheld. Often
term in the computation of the three term limit; temporarily.” Hence, the “temporary” nature of the
conversely, involuntary severance from office for any retention by the national government does not matter.
length of time short of the full term provided by law Any retention is prohibited.
amounts to an interruption of continuity of service.
Lonzanida vacated his post a few months before the next In sum, while Section 1 of AO 372 may be upheld as an
mayoral elections, not by voluntary renunciation but in advisory effected in times of national crisis, Section 4
compliance with the legal process of writ of execution thereof has no color of validity at all. The latter provision
issued by the COMELEC to that effect. Such involuntary effectively encroaches on the fiscal autonomy of local
severance from office is an interruption of continuity of governments. Concededly, the President was well-
service and thus, Lonzanida did not fully serve the 1995- intentioned in issuing his Order to withhold the LGUs’ IRA,
1998 mayoral term. but the rule of law requires that even the best intentions
must be carried out within the parameters of the
In sum, Lonzanida was not the duly elected mayor and Constitution and the law. Verily, laudable purposes must
that he did not hold office for the full term; hence, his be carried out by legal methods. (Pimentel, Jr. v.
assumption of office from May 1995 to March 1998 cannot Aguirre, G.R. No. 132988, 336 SCRA 201, July 19,
be counted as a term for purposes of computing the three 2000, En Banc [Panganiban])
term limit. (Lonzanida v. COMELEC, 311 SCRA 602,
July 28, 1999, En Banc [Gonzaga-Reyes]) 346. What is meant by fiscal autonomy of Local
Governments? Does it rule out in any manner national
345. May the President validly withhold a portion of the government intervention by way of supervision in order
internal revenue allotments of Local Government Units to ensure that local programs are consistent with national
legally due them by administrative fiat? goals?

Held: The Constitution vests the President with the Held: Under existing law, local government units, in
power of supervision, not control, over local government addition to having administrative autonomy in the
units (LGUs). Such power enables him to see to it that exercise of their functions, enjoy fiscal autonomy as well.
LGUs and their officials execute their tasks in accordance Fiscal autonomy means that local governments have the
with law. While he may issue advisories and seek their power to create their own sources of revenue in addition
cooperation in solving economic difficulties, he cannot to their equitable share in the national taxes released by
prevent them from performing their tasks and using the national government, as well as the power to allocate
available resources to achieve their goals. He may not their resources in accordance with their own priorities. It
withhold or alter any authority or power given them by extends to the preparation of their budgets, and local
the law. Thus, the withholding of a portion of internal officials in turn have to work within the constraints
revenue allotments legally due them cannot be directed thereof. They are not formulated at the national level and
by administrative fiat. imposed on local governments, whether they are relevant
to local needs and resources or not. Hence, the necessity
Xxx of a balancing of viewpoints and the harmonization of
proposals from both local and national officials, who in
Section 4 of AO 372 cannot x x x be upheld. A basic any case are partners in the attainment of national goals.
feature of local fiscal autonomy is the automatic release
of the shares of LGUs in the National internal revenue. Local fiscal autonomy does not, however, rule out any
This is mandated by no less than the Constitution . The manner of national government intervention by way of
Local Government Code (Sec. 286[a]) specifies further supervision, in order to ensure that local programs, fiscal
that the release shall be made directly to the LGU and otherwise, are consistent with national goals.
concerned within five (5) days after every quarter of the Significantly, the President, by constitutional fiat, is the
year and “shall not be subject to any lien or holdback that head of the economic and planning agency of the
may be imposed by the national government for whatever government (Section 9, Article XII of the Constitution),
purpose.” As a rule, the term “shall” is a word of primarily responsible for formulating and implementing
command that must be given a compulsory meaning.” continuing, coordinated and integrated social and
The provision is, therefore, imperative. economic policies, plans and programs (Section 3,
Chapter 1, Subtitle C, Title II, Book V, EO 292 Corporation, 292 SCRA 678, July 20, 1998
[Administrative Code of 1987]) for the entire country. [Panganiban])
However, under the Constitution, the formulation and the
implementation of such policies and programs are subject 349. On its first regular session, may the Sanggunian
to “consultations with the appropriate public agencies, transact business other than the matter of adopting or
various private sectors, and local government units.” The updating its existing rules or procedure?
President cannot do so unilaterally. (Pimentel, Jr. v.
Aguirre, 336 SCRA 201, July 19, 2000, En Banc Held: We cannot infer the mandate of the (Local
[Panganiban]) Government) Code that no other business may be
transacted on the first regular session except to take up
347. What are the requisites before the President may the matter of adopting or updating rules. All that the law
interfere in local fiscal matters? requires is that “on the first regular session x x x the
sanggunian concerned shall adopt or update its existing
Held: x x x [T]he Local Government Code provides (Sec. rules or procedures.” There is nothing in the language
284. See also Art. 379 of the Rules and Regulations thereof that restricts the matters to be taken up during
Implementing the Local Government Code of 1991) : the first regular session merely to the adoption or
updating of the house rules. If it were the intent of
“x x x [I]n the event the national government incurs an Congress to limit the business of the local council to such
unmanaged public sector deficit, the President of the matters, then it would have done so in clear and
Philippines is hereby authorized, upon the unequivocal terms. But as it is, there is no such intent.
recommendation of [the] Secretary of Finance, Secretary
of the Interior and Local Government and Secretary of Moreover, adopting or updating of house rules would
Budget and Management, and subject to consultation necessarily entail work beyond the day of the first regular
with the presiding officers of both Houses of Congress and session. Does this mean that prior thereto, the local
the presidents of the liga, to make the necessary council's hands were tied and could not act on any other
adjustments in the internal revenue allotment of local matter? That would certainly be absurd for it would result
government units but in no case shall the allotment be in a hiatus and a paralysis in the local legislature's work
less than thirty percent (30%) of the collection of national which could not have been intended by the law.
internal revenue taxes of the third fiscal year preceding (Malonzo v. Zamora, 311 SCRA 224, July 27, 1999,
the current fiscal year x x x” En Banc [Romero])

There are therefore several requisites before the 350. May an incumbent Vice-Governor, while
President may interfere in local fiscal matters: (1) an concurrently the Acting Governor, continue to preside
unmanaged public sector deficit of the national over the sessions of the Sangguniang Panlalawigan (SP)?
government; (2) consultations with the presiding officers If no, who may preside in the meantime?
of the Senate and the House of Representatives and the
presidents of the various local leagues; and (3) the Held: Being the acting governor, the Vice-governor
corresponding recommendation of the secretaries of the cannot continue to simultaneously exercise the duties of
Department of Finance, Interior and Local Government, the latter office, since the nature of the duties of the
and Budget and Management. Furthermore, any Provincial Governor calls for a full-time occupant to
adjustment in the allotment shall in no case be less than discharge them. Such is not only consistent with but also
thirty percent (30%) of the collection of national internal appears to be the clear rationale of the new (Local
revenue taxes of the third fiscal year preceding the Government) Code wherein the policy of performing dual
current one. (Pimentel, Jr. v. Aguirre, 336 SCRA 201, functions in both offices has already been abandoned. To
July 19, 2000, En Banc [Panganiban]) repeat, the creation of a temporary vacancy in the office
of the Governor creates a corresponding vacancy in the
348. Distinguish an ordinance from a mere resolution. office of the Vice-Governor whenever the latter acts as
Governor by virtue of such temporary vacancy. This
Held: A municipal ordinance is different from a event constitutes an “inability” on the part of the regular
resolution. An ordinance is a law, but a resolution is presiding officer (Vice-Governor) to preside during the SP
merely a declaration of the sentiment or opinion of a sessions, which thus calls for the operation of the remedy
lawmaking body on a specific matter. An ordinance set in Article 49(b) of the Local Government Code –
possesses a general and permanent character, but a concerning the election of a temporary presiding officer.
resolution is temporary in nature. Additionally, the two The continuity of the Acting Governor’s (Vice-Governor)
are enacted differently – a third reading is necessary for powers as presiding officer of the SP is suspended so long
an ordinance, but not for a resolution, unless decided as he is in such capacity. Under Section 49(b), “in the
otherwise by a majority of all the Sanggunian members . event of the inability of the regular presiding officer to
(Municipality of Paranaque v. V.M. Realty preside at the sanggunian session, the members present
and constituting a quorum shall elect from among
themselves a temporary presiding officer.” (Gamboa, Held: We reject this submission of the respondents. The
Jr. v. Aguirre, Jr., G.R. No. 134213, July 20, 1999, due process clause of the Constitution requiring notice as
En Banc [Ynares-Santiago]) an element of fairness is inviolable and should always be
considered part and parcel of every law in case of its
351. What is recall? silence. The need for notice to all the members of the
assembly is also imperative for these members represent
Held: Recall is a mode of removal of a public officer by the different sectors of the electorate of Bataan. To the
the people before the end of his term of office. The extent that they are not notified of the meeting of the
people's prerogative to remove a public officer is an assembly, to that extent is the sovereign voice of the
incident of their sovereign power and in the absence of people they represent nullified. The resolution to recall
constitutional restraint, the power is implied in all should articulate the majority will of the members of the
governmental operations. Such power has been held to assembly but the majority will can be genuinely
be indispensable for the proper administration of public determined only after all the members of the assembly
affairs. Not undeservedly, it is frequently described as a have been given a fair opportunity to express the will of
fundamental right of the people in a representative their constituents. Needless to stress, the requirement of
democracy. (Garcia v. COMELEC, 227 SCRA 108, notice is mandatory for it is indispensable in determining
Oct. 5, 1993, En Banc [Puno]) the collective wisdom of the members of the Preparatory
Recall Assembly. Its non-observance is fatal to the
352. What is the ground for recall? Is this subject to validity of the resolution to recall petitioner Garcia as
judicial inquiry? Governor of the province of Bataan. (Garcia v.
COMELEC, G.R. No. 111511, Sept. 21, 1993; 227
Held: Former Senator Aquilino Pimentel, Jr., a major SCRA 100, Oct. 5, 1993, En Banc [Puno]
author of the subject law in his book The Local
Government Code of 1991: The Key to National 354. The members of the Preparatory Recall
Development, stressed the same reason why the Assembly (PRA) of Puerto Princesa City met and adopted
substantive content of a vote of lack of confidence is a resolution calling for the recall of incumbent Mayor
beyond any inquiry, thus: Dennis Victorino M. Socrates on the ground of loss of
confidence on July 2, 2002. Mayor Socrates argued that
“There is only one ground for recall of local government they have no authority to adopt said Recall Resolution
officials: loss of confidence. This means that the people because a majority of PRA members were seeking a new
may petition or the Preparatory Recall Assembly may electoral mandate in the barangay elections scheduled
resolve to recall any local elective official without on July 15, 2002. Should his contention be sustained?
specifying any particular ground except loss of
confidence. There is no need for them to bring up any Held: This argument deserves scant consideration
charge of abuse or corruption against the local elective considering that when the PRA members adopted the
officials who are subject of any recall petition. Recall Resolution their terms of office had not yet expired.
They were all de jure sangguniang barangay members
In the case of Evardone v. Commission on Elections, et with no legal disqualification to participate in the recall
al., 204 SCRA 464, 472 (1991), the Court ruled that ‘loss assembly under Section 70 of the Local Government
of confidence’ as a ground for recall is a political question. Code. (Victorino Dennis M. Socrates v. The
In the words of the Court, 'whether or not the electorate Commission on Elections, G.R. No. 154512, Nov.
of the municipality of Sulat has lost confidence in the 12, 2002, En Banc [Carpio])
incumbent mayor is a political question.’”
(Garcia v. COMELEC, 227 SCRA 108, Oct. 5, 1993, 355. Whether or not an elective official who became
En Banc [Puno]) City Mayor by legal succession can be the subject of a
recall election by virtue of a Preparatory Recall Assembly
353. The members of the Preparatory Recall Assembly Resolution which was passed or adopted when the said
(PRA) of the province of Bataan adopted a resolution elective official was still the Vice-Mayor.
calling for the recall of Governor Garcia. It was admitted,
however, by the proponents of the recall resolution that Held: The specific purpose of the Preparatory
only those members of the assembly inclined to agree Recall Assembly was to remove Amelita S. Navarro as
were notified of the meeting where said resolution was the elected Vice-Mayor of Santiago City since PRA
adopted “as a matter of strategy and security.” They Resolution No. 1 dated July 12, 1999 expressly states
justified these selective notices on the ground that the that “ x x x it is hereby resolved to invoke the rescission
law (Local Government Code) does not specifically of the electoral mandate of the incumbent City Vice-
mandate the giving of notice. Should this submission be Mayor Amelita S. Navarro for loss of confidence through
sustained? a recall election to be set by the Commission on Election
as provided for under Section 71 of the Local
Government Code of 1991.” However, the said PRA
Resolution No. 1 is no longer applicable to her inasmuch After three consecutive terms, an elective local
as she had already vacated the office of Vice-Mayor on official cannot seek immediate reelection for a fourth
October 11, 1999 when she assumed the position of City term. The prohibited election refers to the next regular
Mayor of Santiago City. election for the same office following the end of the third
consecutive term. Any subsequent election, like a recall
Even if the Preparatory Recall Assembly were to election, is no longer covered by the prohibition for two
reconvene to adopt another resolution for the recall of reasons. First, a subsequent election like a recall election
Amelita Navarro, this time as Mayor of Santiago City, the is no longer an immediate reelection after three
same would still not prosper in view of Section 74 (b) of consecutive terms. Second, the intervening period
the Local Government Code of 1991 which provides that constitutes an involuntary interruption in the continuity of
“No recall shall take place within one (1) year from the service.
date of the official’s assumption of office or one (1) year
immediately preceding a regular election.” There is no Xxx
more allowable time in the light of that law within which
to hold recall elections for that purpose. The then Vice- Clearly, what the Constitution prohibits is an
Mayor Amelita S. Navarro assumed office as Mayor of immediate reelection for a fourth term following three
Santiago City on October 11, 1999. One year after her consecutive terms. The Constitution, however, does not
assumption of office as Mayor will be October 11, 2000 prohibit a subsequent reelection for a fourth term as long
which is already within the one (1) year prohibited period as the reelection is not immediately after the end of the
immediately preceding the next regular election in May third consecutive term. A recall election mid-way in the
2001. (Afiado v. Commission on Elections, 340 term following the third consecutive term is a subsequent
SCRA 600, Sept. 18, 2000, En Banc [De Leon] election but not an immediate reelection after the third
term.
356. Mayor Edward S. Hagedorn of Puerto Princesa
City was elected for three consecutive times in the 1992, Neither does the Constitution prohibit one barred
1995 and 1998 elections and served in full his three from seeking immediate reelection to run in any other
consecutive terms as Mayor. In the 2001 elections, he subsequent election involving the same term of office.
ran for Governor of the Province of Palawan and lost. What the Constitution prohibits is a consecutive fourth
Socrates ran and won as Mayor of Puerto Princesa in term. The debates in the Constitutional Commission
that election. On July 2, 2002, the Preparatory Recall evidently show that the prohibited election referred to by
Assembly (PRA) of Puerto Princesa City adopted a the framers of the Constitution is the immediate
Resolution calling for the recall of incumbent Mayor reelection after the third term, not any other subsequent
Socrates. The COMELEC scheduled a Special Recall election.
Election for Mayor of that City on September 24, 2002.
Is Mayor Hagedorn qualified to run again for Mayor in Xxx
that Special Recall Election considering the
circumstances? In the case of Hagedorn, his candidacy in the
recall election on September 24, 2002 is not an immediate
Held: The three-term limit rule for reelection after his third consecutive term which ended on
elective local officials is found in Section 8, Article June 30, 2001. The immediate reelection that the
X of the Constitution x x x. Constitution barred Hagedorn from seeking referred to
the regular elections in 2001. Hagedorn did not seek
This three-term limit rule is reiterated in Section reelection in the 2001 elections.
43 (b) of RA No. 7160, otherwise known as the Local
Government Code x x x. Xxx

These constitutional and statutory provisions From June 30, 2001 until the recall election on
have two parts. The first part provides that an elective September 24, 2002, the mayor of Puerto Princesa was
local official cannot serve for more than three consecutive Socrates. This period is clearly an interruption in the
terms. The clear intent is that only consecutive terms continuity of Hagedorn’s service as mayor, not because
count in determining the three-term limit rule. The of his voluntary renunciation, but because of a legal
second part states that voluntary renunciation of office for prohibition. Hagedorn’s three consecutive terms ended
any length of time does not interrupt the continuity of on June 30, 2001. Hagedorn’s new recall term from
service. The clear intent is that involuntary severance September 24, 2002 to June 30, 2004 is not a seamless
from office for any length of time interrupts continuity of continuation of his previous three consecutive terms as
service and prevents the service before and after the mayor. One cannot stitch together Hagedorn’s previous
interruption from being joined together to form a three-terms with his new recall term to make the recall
continuous service or consecutive terms. term a fourth consecutive term because factually it is
not. An involuntary interruption occurred from June 30,
2001 to September 24, 2002 which broke the continuity interlocutory orders of the division, which shall be
or consecutive character of Hagedorn’s service as resolved by the divisions which issued the order.”
mayor.
That only final orders of a Division may be raised
X x x In Hagedorn’s case, the nearly 15-month period he before the COMELEC en banc is in accordance with Article
was out of office, although short of a full term of three IX-C, Section 3 of the Constitution which mandates that
years, constituted an interruption in the continuity of his only motions for reconsideration of final decisions shall be
service as mayor. The Constitution does not require the decided by the Commission on Elections en banc, thus:
interruption or hiatus to be a full term of three years. The
clear intent is that interruption “for any length of time,” “Sec. 3. The Commission on Elections may sit en banc or in
as long as the cause is involuntary, is sufficient to break two divisions, and shall promulgate its rules of procedure in
an elective local official’s continuity of service. order to expedite disposition of election cases, including pre-
(Victorino Dennis M. Socrates v. The Commission proclamation controversies. All such election cases shall be
on Elections, G.R. No. 154512, Nov. 12, 2002, En heard and decided in division, provided that motions for
Banc [Carpio]) reconsideration of decisions shall be decided by the
Commission en banc.”

It bears stressing that under this constitutional


357. What acts of a Division of the COMELEC may be provision, the COMELEC en banc shall decide motions for
subject of a motion for reconsideration of the COMELEC reconsideration only of “decisions” of a Division, meaning
en banc? those acts of final character. Clearly, the assailed order
denying petitioner's demurrer to evidence, being
Held: Section 5, Rule 19 of the COMELEC Rules of interlocutory, may not, be resolved by the COMELEC en
Procedure, provides: banc (Ambil, Jr. v. Commission on elections, G.R. No.
143398, Oct. 25, 2000, 344 SCRA 358). (Gementiza v.
“SEC. 5. How Motion for Reconsideration Disposed of. - Commission on Elections, 353 SCRA 724, March 6,
Upon the filing of a motion to reconsider a decision, 2001, En Banc [Sandoval-Gutierrez])
resolution, order or ruling of a Division, the Clerk of Court
concerned shall, within twenty-four (24) hours from the 358. Will it be proper for the Commission on Elections
filing thereof, notify the presiding Commissioner. The to act on a petition for recall signed by just one person?
latter shall within two (2) days thereafter certify the case
to the Commission en banc.” Held: A petition for recall signed by just one person is in
violation of the statutory 25% minimum requirement as
Under the above-quoted rule, the acts of a to the number of signatures supporting any petition for
Division that are subject of a motion for reconsideration recall. Sec. 69(d) of the Local Government Code of 1991
must have a character of finality before the same can be expressly provides that 'recall of any elective x x x
elevated to the COMELEC en banc. The elementary rule municipal x x x official may also be validly initiated upon
is that an order is final in nature if it completely disposes petition of at least twenty-five percent (25%) of the total
of the entire case. But if there is something more to be number of registered voters in the local government unit
done in the case after its issuance, that order is concerned during the election in which the local official
interlocutory. sought to be recalled was elected.' The law is plain and
unequivocal as to what constitutes recall proceedings:
As correctly pointed out by public respondent in only a petition of at least 25% of the total number of
its assailed order of November 29, 1999, the October 11, registered voters may validly initiate recall proceedings.
1999 did not dispose of the case completely as there is (Angobung v. COMELEC, G.R. No. 126576, March 5,
something more to be done which is to decide the election 1997)
protest. As such, it is the herein public respondent
(Second Division of the COMELEC) which issued the 359. Section 74 of the Local Government Code
interlocutory order of October 11, 1999 that should provides that “no recall shall take place within one year x
resolve petitioner’s motion for reconsideration, not the x x immediately preceding a regular local election.” What
COMELEC en banc (Bulaong v. COMELEC, First Division, does the term “regular local election,” as used in this
220 SCRA 745, 749 [1993]). Accordingly, the applicable section, mean?
rule on the subject is Section 5(c), Rule 3 of the COMELEC
Rules of Procedure, which states: Held: The term “regular local election” under Sec. 74 of
the Local Government Code of 1991 which provides that
“Rule 3, Section 5(c). Any motion to reconsider a “no recall shall take place within one (1) year x x x
decision, resolution, order or ruling of a Division shall be immediately preceding a regular local election” refers to
resolved by the Commission en banc except motions on one where the position of the official sought to be recalled
is to be actually contested and filled by the electorate
(Paras v. Comelec, G.R. No. 123169, Nov. 4, 1996). The 1) Because Sec. 74 speaks of limitations on
one-year time bar will not apply where the local official “recall” which, according to Sec. 69, is a power which shall
sought to be recalled is a Mayor and the approaching be exercised by the registered voters of a local
election is a barangay election. (Angobung v. government unit. Since the voters do not exercise such
COMELEC, G.R. No. 126576, March 5, 1997) right except in an election, it is clear that the initiation of
recall proceedings is not prohibited within the one-year
360. Does the word “Recall” in paragraph (b) of period provided in paragraph (b);
Section 74 of the Local Government Code include the 2) Because the purpose of the first limitation in
convening of the Preparatory Recall Assembly and the paragraph (b) is to provide voters a sufficient basis for
filing by it of a recall resolution? Discuss. judging an elective local official, and final judging is not
done until the day of the election; and
Held: Petitioner contends that the term “recall” in Sec. 3) Because to construe the limitation in
74 (b) refers to a process, in contrast to the term “recall paragraph (b) as including the initiation of recall
election” found in Sec. 74 (a), which obviously refers to proceedings would unduly curtail freedom of speech and
an election. He claims that “when several barangay of assembly guaranteed in the Constitution.
chairmen met and convened on May 19, 1999 and (Jovito O. Claudio v. COMELEC, G.R. No. 140560,
unanimously resolved to initiate the recall, followed by the May 4, 2000, En Banc [Mendoza])
taking of votes by the PRA on May 29, 1999 for the
purpose of adopting a resolution ‘to initiate the recall of 361. Who has the legal authority to represent a
Jovito Claudio as Mayor of Pasay City for loss of municipality in lawsuits?
confidence,’ the process of recall began” and, since May
29, 1999 was less than a year after he had assumed Held: Only the provincial fiscal, provincial attorney, and
office, the PRA was illegally convened and all proceedings municipal attorney should represent a municipality in its
held thereafter, including the filing of the recall petition lawsuits. Only in exceptional instances may a private
on July 2, 1999, were null and void. attorney be hired by a municipality to represent it in
lawsuits. (Ramos v. CA, 269 SCRA 34, March 3,
The COMELEC, on the other hand, maintains that the 1997)
process of recall starts with the filing of the petition for
recall and ends with the conduct of the recall election, and 362. What are the exceptional instances when a
that, since the petition for recall in this case was filed on private attorney may be validly hired by a municipality in
July 2, 1999, exactly one year and a day after petitioner’s its lawsuits?
assumption of office, the recall was validly initiated
outside the one-year prohibited period. Held: In Alinsug v. RTC Br. 58, San Carlos City, Negros
Occidental (225 SCRA 553, Aug. 23, 1993), it was held
Both petitioner Claudio and the COMELEC thus agree that that “the law allows a private counsel to be hired by a
the term “recall” as used in Sec. 74 refers to a process. municipality only when the municipality is an adverse
They disagree only as to when the process starts for party in a case involving the provincial government or
purpose of the one-year limitation in paragraph (b) of another municipality or city within the province. This
Sec. 74. provision has its apparent origin in De Guia v. The Auditor
General (44 SCRA 169, March 29, 1979) where the Court
We can agree that recall is a process which begins with held that the municipality’s authority to employ a private
the convening of the preparatory recall assembly or the attorney is expressly limited only to situations where the
gathering of the signatures at least 25% of the registered provincial fiscal would be disqualified to serve and
voters of a local government unit, and then proceeds to represent it.” (Ramos v. CA, 269 SCRA 34, March 3,
the filing of a recall resolution or petition with the 1997)
COMELEC, the verification of such resolution or petition,
the fixing of the date of the recall election, and the holding 363. Cite instances when the provincial fiscal may be
of the election on the scheduled date. However, as used disqualified to represent in court a particular municipality.
in paragraph (b) of Sec. 74, “recall” refers to the election
itself by means of which voters decide whether they Held: As held in Enriquez, Sr. v. Gimenez (107 Phil. 932
should retain their local official or elect his replacement. [1960]), the provincial fiscal may be disqualified to
represent in court a particular municipality in the following
Xxx instances:

To sum up, the term “recall” in paragraph (b) refers to 1) If and when original jurisdiction of case
the recall election and not to the preliminary proceedings involving the municipality is vested in the Supreme Court;
to initiate recall – 2) When the municipality is a party adverse to
the provincial government or to some other municipality
in the same province; and
3) When, in a case involving the municipality, Applying the rule that the power to appoint includes the
he, or his wife, or child, is pecuniarily involved, as heir, power to remove, the questioned dismissal from office of
legatee, creditor or otherwise. the barangay officials by the punong barangay without
(Ramos v. CA, 269 SCRA 34, March 3, 1997) the concurrence of the majority of all the members of the
Sangguniang Barangay cannot be legally justified. To rule
364. May a municipality be represented by a private otherwise could also create an absurd situation of the
law firm which had volunteered its services gratis, in Sangguniang Barangay members refusing to give their
collaboration with the municipal attorney and the fiscal? approval to the replacements selected by the punong
barangay who has unilaterally terminated the services of
Held: No. Such representation will be violative of Section the incumbents. It is likely that the legislature did not
1983 of the old Administrative Code. This strict coherence intend this absurdity to follow from its enactment of the
to the letter of the law appears to have been dictated by law. (Ramon Alquizola, Sr. v. Gallardo Ocol, G.R.
the fact that “the municipality should not be burdened No. 132413, Aug. 27, 1999, 3rd Div. [Vitug])
with expenses of hiring a private lawyer” and that “the
interests of the municipality would be best protected if a 367. What acts of a Division of the COMELEC may be
government lawyer handles its litigations.” subject of a motion for reconsideration of the COMELEC
en banc?
Private lawyers may not represent municipalities on their
own. Neither may they do so even in collaboration with Held: Section 5, Rule 19 of the COMELEC Rules of
authorized government lawyers. This is anchored on the Procedure, provides:
principle that only accountable public officers may act for
and in behalf of public entities and that public funds “SEC. 5. How Motion for Reconsideration Disposed of. -
should not be expended to hire private lawyers. (Ramos Upon the filing of a motion to reconsider a decision,
v. CA, 269 SCRA 34, March 3, 1997) resolution, order or ruling of a Division, the Clerk of Court
concerned shall, within twenty-four (24) hours from the
365. May a municipality adopt the work already filing thereof, notify the presiding Commissioner. The
performed in good faith by a private lawyer, which work latter shall within two (2) days thereafter certify the case
proved beneficial to it? to the Commission en banc.”

Held: Although a municipality may not hire a private Under the above-quoted rule, the acts of a
lawyer to represent it in litigations, in the interest of Division that are subject of a motion for reconsideration
substantial justice, however, it was held that a must have a character of finality before the same can be
municipality may adopt the work already performed in elevated to the COMELEC en banc. The elementary rule
good faith by such private lawyer, which work is beneficial is that an order is final in nature if it completely disposes
to it (1) provided that no injustice is thereby heaped on of the entire case. But if there is something more to be
the adverse party and (2) provided further that no done in the case after its issuance, that order is
compensation in any guise is paid therefor by said interlocutory.
municipality to the private lawyer. Unless so expressly
adopted, the private lawyer’s work cannot bind the As correctly pointed out by public respondent in
municipality. (Ramos v. CA, 269 SCRA 34, March 3, its assailed order of November 29, 1999, the October 11,
1997) 1999 did not dispose of the case completely as there is
something more to be done which is to decide the election
366. May the Punong Barangay validly appoint or protest. As such, it is the herein public respondent
remove the barangay treasurer, the barangay secretary, (Second Division of the COMELEC) which issued the
and other appointive barangay officials without the interlocutory order of October 11, 1999 that should
concurrence of the majority of all the members of the resolve petitioner’s motion for reconsideration, not the
Sangguniang Barangay? COMELEC en banc (Bulaong v. COMELEC, First Division,
220 SCRA 745, 749 [1993]). Accordingly, the applicable
Held: The Local Government Code explicitly vests on the rule on the subject is Section 5(c), Rule 3 of the COMELEC
punong barangay, upon approval by a majority of all the Rules of Procedure, which states:
members of the sangguniang barangay, the power to
appoint or replace the barangay treasurer, the barangay “Rule 3, Section 5(c). Any motion to reconsider a
secretary, and other appointive barangay officials. Verily, decision, resolution, order or ruling of a Division shall be
the power of appointment is to be exercised conjointly by resolved by the Commission en banc except motions on
the punong barangay and a majority of all the members interlocutory orders of the division, which shall be
of the sangguniang barangay. Without such conjoint resolved by the divisions which issued the order.”
action, neither an appointment nor a replacement can be
effectual. That only final orders of a Division may be raised
before the COMELEC en banc is in accordance with Article
IX-C, Section 3 of the Constitution which mandates that disbursement of funds from any public treasury or
only motions for reconsideration of final decisions shall be depository therefor. It can thus be plainly seen that the
decided by the Commission on Elections en banc, thus: law invoked by petitioner Quezon City itself provides that
an appropriation law is not the only authority upon
“Sec. 3. The Commission on Elections may sit en banc or in which public funds shall be disbursed.
two divisions, and shall promulgate its rules of procedure in
order to expedite disposition of election cases, including pre- Furthermore, then Mayor Brigido Simon, Jr. did
proclamation controversies. All such election cases shall be not enter into the subject contract without legal
heard and decided in division, provided that motions for authority. The Local Government Code of 1983, or B.P.
reconsideration of decisions shall be decided by the Blg. 337, which was then in force, specifically and
Commission en banc.” exclusively empowered the city mayor to “represent the
city in its business transactions, and sign all warrants
It bears stressing that under this constitutional drawn on the city treasury and all bonds, contracts and
provision, the COMELEC en banc shall decide motions for obligations of the city.” Such power granted to the city
reconsideration only of “decisions” of a Division, meaning mayor by B.P. Blg. 337 was not qualified nor restricted
those acts of final character. Clearly, the assailed order by any prior action or authority of the city council. We
denying petitioner's demurrer to evidence, being note that while the subsequent Local Government Code
interlocutory, may not, be resolved by the COMELEC en of 1991, which took effect after the execution of the
banc (Ambil, Jr. v. Commission on elections, G.R. No. subject contracts, provides that the mayor’s
143398, Oct. 25, 2000, 344 SCRA 358). (Gementiza v. representation must be “upon authority of the
Commission on Elections, 353 SCRA 724, March 6, sangguniang panlungsod or pursuant to law or
2001, En Banc [Sandoval-Gutierrez]) ordinance,” there was no such qualification under the
old code. (Citations omitted)

368. Is a contract entered into by the city mayor We must differentiate the provisions of the old
involving the expenditure of public funds by the local Local Government Code of 1983, B.P. Blg. 337, which
government without prior appropriation by the city was then in force, from that of the Local Government
council valid and binding? Code of 1991, R.A. No. 7160, which now requires that
the mayor’s representation of the city in its business
Held: If we are to limit our disquisition to the cited transactions must be “upon authority of the
provisions of Presidential Decree No. 1445, or the sangguniang panlungsod or pursuant to law or
Auditing Code of the Philippines, in conjunction with ordinance” (Section 455 [vi]. No such prior authority
Section 177 (b) of Batas Pambansa Blg. 337, or the was required under B.P. Blg. 337. This restriction,
Local Government Code of 1983, which empowered the therefore, cannot be imposed on the city mayor then
Sangguniang Panlungsod to “appropriate funds for since the two contracts were entered into before R.A.
expenses of the city government, and fix the salaries of No. 7160 was even enacted.
its officers and employees according to law,” there
would be no debate that prior appropriation by the city Under B.P. Blg. 337, while the city mayor has no
council and a certification that funds are available power to appropriate funds to support the contracts,
therefore is indeed mandatorily required. neither does said law prohibit him from entering into
contracts unless and until funds are appropriated
Xxx therefor. In fact, it is his bounden duty to so represent
the city in all its business transactions. On the other
However, the very same Presidential Decree No. hand, the city council must provide for the “depositing,
1445, which is the cornerstone of petitioner’s leaving or throwing of garbage” and to appropriate
arguments, does not provide that the absence of an funds for such expenses. (Section 177 [b]). It cannot
appropriation law ipso facto makes a contract entered refuse to so provide and appropriate public funds for
into by a local government unit null and void. Section such services which are very vital to the maintenance of
84 of the statute specifically provides: cleanliness of the city and the good health of its
inhabitants.
Revenue funds shall not be paid out of any public
treasury or depository except in pursuance of an By entering into the two contracts, Mayor Simon
appropriation law or other specific statutory authority. did not usurp the city council’s power to provide for the
proper disposal of garbage and to appropriate funds
Consequently, public funds may be disbursed therefor. The execution of contracts to address such a
not only pursuant to an appropriation law, but also in need is his statutory duty, just as it is the city council’s
pursuance of other specific statutory authority, i.e., duty to provide for said services. There is no provision
Section 84 of PD 1445. Thus, when a contract is in B.P. Blg. 337, however, that prohibits the city mayor
entered into by a city mayor pursuant to specific from entering into contracts for the public welfare,
statutory authority, the law, i.e., PD 1445 allows the
unless and until there is prior authority from the city constitution and by-laws shall govern other matters
council. This requirement was imposed much later by affecting internal organization of the liga not otherwise
R.A. No. 7160, long after the contracts had already been provided for in the Local Government Code provided that
executed and implemented. the constitution and by-laws shall be suppletory to the
provisions of Book III, Title VI of the Local Government
Even the very Charter of Quezon City, more Code and shall always conform to the provisions of the
particularly Section 9(f), Section 12(a)and Section 12(m) Constitution and existing laws (Sec. 507, Local
thereof, simply provide that the mayor shall exercise Government Code).
general powers and duties, such as signing “all warrants
drawn on the city treasurer and all bonds, contracts, and Having in mind the foregoing principles, we rule
obligations of the city,” even as it grants the City Council that Memorandum Circular No. 97-193 of the DILG
the power, by ordinance or resolution, “to make all insofar as it authorizes the filing a Petition for Review of
appropriations for the expenses of the government of the BES with the regular courts in a post proclamation
the city,” as well as “to prohibit the throwing or electoral protest is of doubtful constitutionality. We
depositing of offal, garbage, refuse, or other offensive agree with both the petitioner and the Solicitor General
matter in the same, and to provide for its collection and that in authorizing the filing of the petition for review of
disposition x x x.” (Citations omitted) the decision of the BES with the regular courts, the DILG
Secretary in effect amended and modified the
While the powers and duties of the Mayor and GUIDELINES promulgated by the National Liga Board
the City Council are clearly delineated, there is nothing and adopted by the LIGA which provides that the
in the cited provisions, nor even in the statute itself, that decision of the BES shall be subject to review by the
requires “prior authorization by the city council by National Liga Board. The amendment of the
proper enactment of an ordinance” before the City GUIDELINES is more than an exercise of the power of
Mayor can enter into contracts. supervision but is an exercise of the power of control,
which the President does not have over the LIGA.
Private respondent Lexber asserts that the Although the DILG is given the power to prescribe rules,
subject contract was entered into by Mayor Simon in regulations and other issuances, the Administrative Code
behalf of the Quezon City government pursuant to specific limits its authority to merely “monitoring compliance” by
statutory authority, more particularly the provisions of local government units of such issuances (Taule v.
Executive Order No. 392 (Constituting the Metro Manila Santos, 200 SCRA 512, 523 [1991]). To monitor means
Authority [MMA]). x x x (City of Quezon v. Lexber “to watch, observe or check” and is compatible with the
Incorporated, 354 SCRA 493, Mar. 15, 2001, 1st power of supervision of the DILG Secretary over local
Div. [Ynares-Santiago]) governments, which is limited to checking whether the
local government unit concerned or the officers thereof
perform their duties as per statutory enactments (Ibid.).
369. Does the President’s power of general Besides, any doubt as to the power of the DILG
supervision extend to the liga ng mga barangay, which Secretary to interfere with local affairs should be
is not a local government unit? resolved in favor of the greater autonomy of the local
government (Ibid.)
Held: We rule in the affirmative. In Opinion
No. 41, Series of 1995, the Department of Justice ruled The public respondent judge therefore committed
that the liga ng mga barangay is a government grave abuse of discretion amounting to lack or excess of
organization, being an association, federation, league or jurisdiction in not dismissing the respondent’s Petition for
union created by law or by authority of law, whose Review for failure to exhaust all administrative remedies
members are either appointed or elected government and for lack of jurisdiction. (Bito-Onon v. Fernandez,
officials. The Local Government Code defines the liga ng 350 SCRA 732, Jan. 31, 2001, 3rd Div. [Gonzaga-
mga barangay as an organization of all barangays for Reyes])
the primary purpose of determining the representation
of the liga in the sanggunians, and for ventilating, 370. Petitioners would seek the disqualification of
articulating and crystallizing issues affecting barangay respondent Leonardo B. Roman on the ground of his
government administration and securing, through proper having transgressed the three-term limit under Section
and legal means, solutions thereto (Sec. 491, Local 8, Article X, of the 1987 Constitution and Section 43 of
Government Code). X x x Republic Act No. 7160 (Local Government Code). The
focal issue presented before the Court x x x would
Xxx revolve on the question of whether or not private
respondent Roman exceeded the three-term limit for
The ligas are primarily governed by the elective local officials, expressed in the Constitution and
provisions of the Local Government Code (Book III, Title the Local Government Code, when he again ran for the
VI, Local Government Code). However, their respective position of Governor in the 14th of May 2001 elections,
having occupied and served in that position following the by leaders freely chosen by them in credible elections. He
1993 recall elections, as well as the 1995 and 1998 concluded that, in election cases, when two conflicting
regular elections, immediately prior to the 2001 legal positions are of almost equal weight, the scales of
elections. In fine, should respondent’s incumbency to justice should be tilted in favor of the people’s
the post of Governor following the recall elections be overwhelming choice.
included in determining the three-consecutive term limit
fixed by law? AZCUNA, J., joined by BELLOSILLO, J., also voted to
dismiss, arguing that it is clear from the constitutional
Held: After due deliberation, the Court voted 8 to 7 to provision that the disqualification applies only if the terms
DISMISS the petition. are consecutive and the service is full and continuous.
Hence, service for less than a term, except only in case of
VITUG, J., joined by YNARES-SANTIAGO, J., voted to voluntary renunciation, should not count to disqualify an
dismiss the petition. He contended that as revealed by elective local official from running for the same position.
the records of the Constitutional Commission, the This case is different from Socrates, where the full three
Constitution envisions a continuous and an uninterrupted consecutive terms had been continuously served so that
service for three full terms before the proscription applies. disqualification had clearly attached.
Therefore, not being a full term, a recall term should not
be counted or used as a basis for the disqualification On the other hand, SANDOVAL-GUTIERREZ, J., with
whether served prior (as in this case) or subsequent (as whom DAVIDE, C.J., and AUSTRIA-MARTINEZ, CORONA,
in the Socrates case) to the nine-year, full three-term and CALLEJO, SR., JJ., concurred, holds the view that the
limit. recall term served by respondent Roman, comprising the
period June 28, 1994 to June 30, 1995, should be
MENDOZA, J., in whose opinion QUISUMBING, J., joined, considered as one term. Since he thereafter served for
voted to dismiss the petition on the ground that, in two consecutive terms from 1995 to 1998 and from 1998
accordance with the ruling in Borja, Jr. v. COMELEC, 295 to 2001, his election on May 14, 2001 was actually his
SCRA 157 [1998]; Arcos v. COMELEC, G.R. No. 133639, fourth term and contravenes Art. X, Sec. 8 of the
Oct. 6, 1998 (res.); Lonzanida v. COMELEC, 311 SCRA 602 Constitution. For this reason, she voted to grant the
[1999]; and Adormeo v. COMELEC, G.R. No. 147927, Feb. petition and to declare respondent’s election on May 14,
4, 2002, a term during which succession to a local elective 2002 as null and void.
office takes place or a recall election is held should not be
counted in determining whether an elective local official CARPIO, J., joined by CARPI0-MORALES, J., also
has served more than three consecutive terms. He dissented and voted to grant the petition. He held that a
argued that the Constitution does not prohibit elective recall term constitutes one term and that to totally ignore
local officials from serving for more than three a recall term in determining the three-term limit would
consecutive terms because, in fact, it excludes from the allow local officials to serve for more than nine
three-term limit interruptions in the continuity of service, consecutive years contrary to the manifest intent of the
so long as such interruptions are not due to the voluntary framers of the Constitution. He contended that
renunciation of the office by the incumbent. Hence, the respondent Roman’s election in 2001 cannot exempt him
period from June 28, 1994 to June 30, 1995, during which from the three-term limit imposed by the Constitution.
respondent Leonardo B. Roman served as governor of
Bataan by virtue of a recall election held in 1993, should In his Separate Opinion, Justice Vitug voted to dismiss
not be counted. Since on May 14, 2001 respondent had the petition on the following considerations:
previously served as governor of Bataan for only two
consecutive terms (1995-1998 and 1998-2001), his In order that the three-consecutive term limit
election on that day was actually only his third term for can apply, two conditions must concur, i.e., (1) that the
the same position. elective local official concerned has been elected for
three consecutive terms to the same local government
PANGANIBAN, J., joined by PUNO, J., also voted to position, and (2) that he has served three consecutive
dismiss the petition. He argued that a recall term should full terms, albeit a voluntary renunciation of the
not be considered as one full term, because a contrary office for any length of time shall not be deemed to be
interpretation would in effect cut short the elected an interruption in the continuity of the service for the full
official’s service to less than nine years and shortchange term for which he is elected. The constitutional
his constituents. The desire to prevent monopoly of provision does not appear to be all that imprecise for
political power should be balanced against the need to and in its application. Section 8, Article X, of the
uphold the voters’ obvious preference who, in the present Constitution is explicit that the “term of office of elective
case, is Roman who received 97 percent of the votes cast. local officials x x x shall be three years” which phrase is
He explained that, in Socrates, he also voted to affirm the forthwith followed by its mandate that “no such official
clear choice of the electorate, because in a democracy the shall serve for more than three consecutive terms,” and
people should, as much as legally possible, be governed that “[v]oluntary renunciation of the office for any length
of time shall not be considered as an interruption in the its familiarity with political realities, and its peculiar
continuity of his service for the full term for which he expertise in dealing with election controversies, should
[is] elected.” The law evidently contemplates a be in a good vantage point to resolve issues of this
continuous full three-year term before the proscription nature. Concededly, no ready made formulae are
can apply. always extant to address occasional complex issues,
allowing time and experience to merely evolve and
The Constitutional Commission, in its ultimately provide acceptable solutions. In the
deliberations, referred to a full nine (9) years of service administration of election laws, it would be unsound by
for each elective local government official in the an excessive zeal to remove from the Commission on
application of the prohibition, envisioning at the same Elections the initiative it takes on such questions which,
time a continuous and uninterrupted period of nine in fact, by legal mandate properly belong to it (See
years by providing for only one exception, i.e., when an Loong v. COMELEC, 305 SCRA 832, Pangandaman v.
incumbent voluntarily gives up the office. COMELEC, 319 SCRA 283).

Xxx Nor should it be ignored that the law here


involved is a limitation on the right of suffrage not only
A winner who dislodges in a recall election an on the candidate for office but also, and most
incumbent elective local official merely serves the importantly, on the electorate. Respondent Roman has
balance of the latter’s term of office; it is not a full won the election to the post of Governor of Bataan with
three-year term. It also goes without saying that an a comfortable margin against his closest opponent.
incumbent elective local official against whom a recall Where a candidate appears to be the clear choice of the
election is initiated and who nevertheless wins in a recall people, doubts on the candidate’s eligibility, even only
election must be viewed as being a continuing term of as a practical matter, must be so resolved as to respect
office and not as a break in reckoning his three and carry out, not defeat, the paramount will of the
consecutive terms. X x x electorate. While the Constitution would attempt to
prevent the monopolization of political power, indeed a
If involuntary severance from the service wise rule, the precept of preserving the freedom of
which results in the incumbent’s being unable to finish choice of the people on who shall rightfully hold the
his term of office because of his ouster through valid reins of government for them is no less than
recall proceedings negates “one term” for purposes of fundamental in looking at its overriding intent. (Melanio
applying the three-term limit, as so intimated in L. Mendoza and Mario E. Ibarra v. Commission on
Lonzanida, it stands to reason that the balance of the Elections and Leonardo B. Roman, G.R. No.
term assumed by the newly elected local official in a 149736, Dec. 17, 2002, En Banc)
recall election should not also be held to be one term in
reckoning the three-term limit. In both situations, 371. On May 3, 2001, petitioner filed with the
neither the elective local official who is unable to finish Provincial Election Supervisor in Pagadian City a petition
his term nor the elected local official who only assumes for the disqualification of respondent Sulong, pursuant
the balance of the term of the ousted local official to Sec. 40[b] of Republic Act No. 7160 (Local
following the recall election could be considered to have Government Code), which disqualifies from running for
served a full three-year term set by the Constitution. any elective local position “those removed from office as
a result of an administrative case.” It appears that
This view is not inconsistent, but indeed in line, respondent Sulong had previously won as mayor of
with the conclusion ultimately reached in Socrates v. Lapuyan on January 18, 1988. In the May 11, 1992,
Commission on Elections, where the Court has and again in the May 8, 1995 elections, he was
considered Hagedorn, following his three full terms of reelected. In a petition for disqualification, petitioner
nine years, still qualified to run in a recall election alleged that in 1991, during his first term as mayor of
conducted about a year and a half after the most recent Lapuyan, respondent Sulong, along with a municipal
regular local elections. A recall term then, not being a councilor of Lapuyan and several other individuals, was
full three-year term, is not to be counted or used as a administratively charged (AC No. 12-91) with various
basis for disqualification whether it is held prior or offenses, and that, on February 4, 1992, the
subsequent to the nine year full three-term limit. Sangguniang Panlalawigan of Zamboanga del Sur found
him guilty of the charges and ordered his removal from
This same issue has been passed and ruled office. Petitioner claimed that this decision had become
upon by the Commission on Elections no less than five final and executory, and consequently the then vice-
times. Consistently, it has held that the term of a mayor of Lapuyan, Vicente Imbing, took his oath as
newcomer in recall elections cannot be counted as a full mayor vice respondent Sulong on March 3, 1992.
term and may not thus be included in counting the
three-term limit prescribed under the law. The Respondent Sulong denied that the decision in AC No.
Commission on Elections, with its fact-finding facilities, 12-91 had become final and executory. He averred that
after receiving a copy of the decision on February 17, Indeed, considering the failure of the
1992, he filed a motion for reconsideration and/or notice Sangguniang Panlalawigan to resolve respondent’s
of appeal thereof on February 18, 1992; that on motion, it is unfair to the electorate to be told after they
February 27, 1992, the Sangguniang Panlalawigan have voted for respondent Sulong that after all he is
required Jim Lingating, the complainant in AC No. 12-91, disqualified, especially since at the time of the elections
to comment on respondent Sulong’s motion for on May 14, 2001, the decision of the Sangguniang
reconsideration and/or notice of appeal; that the said Panlalawigan had been rendered nearly ten years ago.
complainant had not yet complied therewith and his (Atty. Miguel M. Lingating v. Commission on
(respondent Sulong’s) motion had consequently Elections and Cesar B. Sulong, G.R. No. 153475,
remained pending. Respondent Sulong denied he had Nov. 13, 2002, En Banc [Mendoza])
been removed from office by virtue of the decision in AC
No. 12-91. 372. When may a permanent vacancy arise under
Section 44 of the Local Government Code?
Held: Petitioner contends that the COMELEC en
banc erred in applying the ruling in Aguinaldo v. Held: Under Section 44, a permanent vacancy
Commission on Elections (212 SCRA 768 [1992]) in arises when an elective official fills a higher vacant office,
holding that the reelection of respondent Sulong in 1992 refuses to assume office, fails to qualify, dies, is removed
and 1995 as mayor of Lapuyan had the effect of from office, voluntarily resigns, or is otherwise
condoning the misconduct for which he was ordered permanently incapacitated to discharge the functions of
dismissed by the Sangguniang Panlalawigan of his office. (Navarro v. Court of Appeals, 355 SCRA
Zamboanga del Sur. Petitioner cites Reyes v. 672, Mar. 28, 2001, 1st Div. [Kapunan])
Commission on Elections (254 SCRA 514, 525-526
[1996]) in which we held that an elective local executive 373. How is Section 45(b) of the Local Government
officer, who is removed before the expiration of the term Code to be interpreted? What is the reason behind the
for which he was elected, is disqualified from being a right given to a political party to nominate a replacement
candidate for a local elective position under Section where a permanent vacancy occurs in the Sanggunian?
40[b] of the Local Government Code.
Held: What is crucial is the interpretation of
Xxx Section 45(b) providing that “x x x only the nominee of
the political party under which the Sanggunian member
However, Reyes cannot be applied to this case because concerned has been elected and whose elevation to the
it appears that the 1992 decision of the Sangguniang position next higher in rank created the last vacancy in
Panlalawigan, finding respondent Sulong guilty of the Sanggunian shall be appointed in the manner
dishonesty, falsification and malversation of public hereinabove provided. The appointee shall come from
funds, has not until now become final. X x x The filing the political party as that of the Sanggunian member who
of his motion for reconsideration prevented the decision caused the vacancy x x x.”
of Sangguniang Panlalawigan from becoming final.
The reason behind the right given to a political
While R.A. No. 7160 on disciplinary actions is party to nominate a replacement where a permanent
silent on the filing of a motion for reconsideration, the vacancy occurs in the Sanggunian is to maintain the party
same cannot be interpreted as a prohibition against the representation as willed by the people in the election
filing of a motion for reconsideration. x x x. (Aquilino Pimentel, the Local Government Code of 1991,
The Key to National Development, p. 150).
There is thus no decision finding respondent
guilty to speak of. As Provincial Secretary of With the elevation of petitioner Tamayo, who
Zamboanga del Sur Wilfredo Cimafranca attested, the belonged to REFORMA-LM, to the position of Vice-Mayor,
Sangguniang Panlalawigan simply considered the matter a vacancy occurred in the Sanggunian that should be filled
as having become moot and academic because it was up with someone who should belong to the political party
“overtaken by the local elections of May [11], 1992.” of petitioner Tamayo. Otherwise, REFORMA-LM’s
representation in the Sanggunian would be diminished.
Neither can the succession of the then vice- To argue that the vacancy created was that formerly held
mayor of Lapuyan x x x and the highest ranking by Rolando Lalas, a LAKAS-NUCD-Kampi member, would
municipal councilor of Lapuyan x x x to the offices of result in the increase of that party’s representation in the
mayor and vice-mayor, respectively, be considered proof Sanggunian at the expense of the REFORMA-LM. This
that the decision in AC No. 12-91 had become final interpretation is contrary to the letter and spirit of the law
because it appears to have been made pursuant to Sec. and thus violative of a fundamental rule in statutory
68 of the Local Government Code, which makes construction which is to ascertain and give effect to the
decisions in administrative cases immediately executory. intent and purpose of the law (Manila Lodge No. 761 v.
Court of Appeals, 73 SCRA 12 [1976]). As earlier pointed
out, the reason behind par. (b), section 44 of the Local the City of Pasig presents a prejudicial question which
Government Code is the maintenance of party must first be decided before the plebiscites for the
representation in the Sanggunian in accordance with the creation of the proposed barangays may be held.
will of the electorate.
Xxx
The “last vacancy” in the Sanggunian refers to
that created by the elevation of the member formerly In the case at bar, while the City of Pasig vigorously
occupying the next higher in rank which in turn also had claims that the areas covered by the proposed Barangays
become vacant by any of the causes already enumerated. Karangalan and Napico are within its territory, it can not
The term “last vacancy” is thus used in Sec. 45 (b) to deny that portions of the same area are included in the
differentiate it from the other vacancy previously created. boundary dispute case pending before the Regional Trial
The term by no means refers to the vacancy in the No. 8 Court of Antipolo. Surely, whether the areas in
position which occurred with the elevation of Rolando controversy shall be decided as within the territorial
Lalas to the seventh position in the Sanggunian. Such jurisdiction of the Municipality of Cainta or the City of
construction will result in absurdity. (Navarro v. Court Pasig has material bearing to the creation of the proposed
of Appeals, 355 SCRA 672, Mar. 28, 2001, 1st Div. Barangays Karangalan and Napico. Indeed, a requisite
[Kapunan]) for the creation of a barangay is for its territorial
jurisdiction to be properly identified by metes and bounds
374. May local elective officials practice their or by more or less permanent natural boundaries (Sec.
profession or engage in any occupation? 386[b], R.A. No. 7160). Precisely because territorial
jurisdiction is an issue raised in the pending civil case,
Ans.: All governors, city and municipal mayors until and unless such issue is resolved with finality, to
are prohibited from practicing their profession or define the territorial jurisdiction of the proposed
engaging in any occupation other than the exercise of barangays would only be an exercise in futility. Not only
their functions as local chief executives. (Sec. 90[a], that, we would be paving the way for potentially ultra
LGC) vires acts of such barangays. X x x

Sanggunian members may practice their Moreover, considering the expenses entailed in the
profession, engage in any occupation, or teach in schools holding of plebiscites, it is far more prudent to hold in
except during session hours. (Sec. 90[b], LGC) abeyance the conduct of the same, pending final
determination of whether or not the entire area of the
Sanggunian members who are also members of proposed barangays are truly within the territorial
the Bar shall not: jurisdiction of the City of Pasig.

(a) Appear as counsel before any court in any Neither do we agree that merely because a plebiscite had
civil case wherein a local government unit or any office, already been held in the case of the proposed Barangay
agency, or instrumentality of the government is the Napico, the petition of the Municipality of Cainta has
adverse party; already been rendered moot and academic. The issue
(b) Appear as counsel in any criminal case raised by the Municipality of Cainta in its petition before
wherein an officer or employee of the national or local the COMELEC against the holding of the plebiscite for the
government is accused of an offense committed in creation of Barangay Napico are still pending
relation to his office; determination before the Antipolo Regional Trial Court.
(c) Collect any fee for their appearance in
administrative proceedings involving the local Xxx
government unit of which he is an official; and
(d) Use property and personnel of the Therefore, the plebiscite on the creation of Barangay
government except when the sanggunian member Karangalan should be held in abeyance pending final
concerned is defending the interest of the Government. resolution of the boundary dispute between the City of
(Sec. 90[b], LGC) Pasig and the Municipality of Cainta by the Regional Trial
Court of Antipolo City. In the same vein, the plebiscite
held on March 15, 1997 to ratify the creation of Barangay
375. Whether or not the plebiscites scheduled for the Napico, Pasig City, should be annulled and set aside.
creation of Barangays Karangalan and Napico should be (City of Pasig v. COMELEC, 314 SCRA 179, Sept. 10,
suspended or cancelled in view of the pending boundary 1999, En Banc [Ynares-Santiago])
dispute between the two local governments.

Held: To begin with, we agree with the position of the


COMELEC that Civil Case No. 94-3006 involving the
boundary dispute between the Municipality of Cainta and
PUBLIC INTERNATIONAL LAW Jeffrey Liang [Huefeng] v. People, G.R. No.
125865, Mar. 26, 2001, 1st Div. [Motion for
376. Discuss the contemporary view on the rightful Reconsideration])
place of an Individual in International Law? Does he
remain a mere “object” of International Law, or is he now
a proper “subject” of International Law?

Held: Then came the long and still ongoing 377. What is the doctrine of incorporation? How is it
debate on what should be the subject of international law. applied by local courts?
The 20th century saw the dramatic rise and fall of different
types and hues of authoritarianism – the fascism of Italy’s Held: Under the doctrine of incorporation, rules of
Mussolini and Germany’s Hitler, the militarism of Japan’s international law form part of the law of the land and no
Hirohito and the communism of Russia’s Stalin, etc. The further legislative action is needed to make such rules
sinking of these isms led to the elevation of the rights of applicable in the domestic sphere.
the individual against the state. Indeed, some species of
human rights have already been accorded universal The doctrine of incorporation is applied whenever
recognition (See Universal Declaration of Human Rights municipal tribunals (or local courts) are confronted with
[1948], The International Covenant on Economic, Social situations in which there appears to be a conflict between
and Cultural Rights [1966] and The International a rule of international law and the provisions of the
Covenant on Civil and Political Rights [1966]). Today, the Constitution or statute of the local state. Efforts should
drive to internationalize rights of women and children is first be exerted to harmonize them, so as to give effect to
also on high gear (The Convention on the Elimination of both since it is to be presumed that municipal law was
All Forms of Discrimination Against Women [CEDAW] enacted with proper regard for the generally accepted
otherwise known as “The Bill of Rights for principles of international law in observance of the
Women” was adopted by the UN General Assembly in Incorporation Clause in Section 2, Article II of the
December 1979. As of November 1999, one hundred Constitution. In a situation however, where the conflict
sixty seven [167] states including the Philippines have is irreconcilable and a choice has to be made between a
ratified or acceded to it. See Statement of Angela King, rule of international law and municipal law, jurisprudence
Special Adviser to the Secretary General of the UN on dictates that municipal law should be upheld by the
Gender Issues and Advancement of Women, Judicial municipal courts for the reason that such courts are
Colloquium on the Application of International Human organs of municipal law and are accordingly bound by it
Rights Law at the Domestic Level, Vienna, Austria, in all circumstances. The fact that international law has
October 27, 1999). The higher rating given to human been made part of the law of the land does not pertain to
rights on the hierarchy of values necessarily led to the re- or imply the primacy of international law over national or
examination of the rightful place of the individual in municipal law in the municipal sphere. The doctrine of
international law. Given the harshest eye is the moss- incorporation, as applied in most countries, decrees that
covered doctrine that international law deals only with rules of international law are given equal standing with,
States and that individuals are not its subject. For its but are not superior to, national legislative enactments.
undesirable corollary is that sub-doctrine that an Accordingly, the principle of lex posterior derogat priori
individual’s right in international law is a near cipher. takes effect – a treaty may repeal a statute and a statute
Translated in extradition law, the view that once may repeal a treaty. In states where the Constitution is
commanded a consensus is that since a fugitive is a mere the highest law of the land, such as the Republic of the
object and not a subject of international law, he is bereft Philippines, both statutes and treaties may be invalidated
of rights. An extraditee, so it was held, is a mere “object if they are in conflict with the Constitution. (Secretary
transported from one state to the other as an exercise of of Justice v. Hon. Ralph C. Lantion, G.R. No.
the sovereign will of the two states involved.” (Blakesley 139465, Jan. 18, 2000, En Banc [Melo])
and Lagodny, Finding Harmony Amidst Disagreement
Over Extradition, Jurisdiction, The Role of Human Rights 378. Is sovereignty really absolute and all-
and Issues of Extraterritoriality Under International encompassing? If not, what are its restrictions and
Criminal Law, Vanderbilt Journal of Transnational Law, limitations?
Vol. 24, No. 1, p. 44 [1991]) The re-examination
consigned this pernicious doctrine to the museum of ideas Held: While sovereignty has traditionally been deemed
(See generally Kelsen, Principles of International Law, 2 nd absolute and all-encompassing on the domestic level, it is
ed. [1966]; Korowicz, The Problem of the International however subject to restrictions and limitations voluntarily
Personality of Individuals, 50 Am. J., Int’l. Law 553 agreed to by the Philippines, expressly or impliedly, as a
[1966]). The new thinkers of international law then gave member of the family of nations. By the doctrine of
a significant shape to the role and rights of the individual incorporation, the country is bound by generally accepted
in state-concluded treaties and other international principles of international law, which are considered to be
agreements. X x x (Concurring Opinion, Puno J., in automatically part of our own laws. One of the oldest and
most fundamental rules in international law is pacta sunt “By taking up the case of one of its subjects and by
servanda – international agreements must be performed resorting to diplomatic action or international judicial
in good faith. A state which has contracted valid proceedings on his behalf, a State is in reality asserting
international obligations is bound to make in its its own rights - its right to ensure, in the person of its
legislations such modifications as may be necessary to subjects, respect for the rules of international law” (The
ensure the fulfillment of the obligations. Mavrommatis Palestine Concessions, 1 Hudson, World
Court Reports 293, 302 [1924]).
By their inherent nature, treaties really limit or restrict the (Holy See, The v. Rosario, Jr., 238 SCRA 524, 538-
absoluteness of sovereignty. By their voluntary act, 539, Dec. 1, 1994, En Banc [Quiason])
nations may surrender some aspects of their state power
in exchange for greater benefits granted by or derived
from a convention or pact. After all, states, like 380. Discuss the Status of the Vatican and the Holy
individuals, live with coequals, and in pursuit of mutually See in International Law.
covenanted objectives and benefits, they also commonly
agree to limit the exercise of their otherwise absolute Held: Before the annexation of the Papal States by Italy
rights. Thus, treaties have been used to record in 1870, the Pope was the monarch and he, as the Holy
agreements between States concerning such widely See, was considered a subject of International Law. With
diverse matters as, for example, the lease of naval bases, the loss of the Papal States and the limitation of the
the sale or cession of territory, the termination of war, the territory under the Holy See to an area of 108.7 acres, the
regulation of conduct of hostilities, the formation of position of the Holy See in International Law became
alliances, the regulation of commercial relations, the controversial.
settling of claims, the laying down of rules governing
conduct in peace and the establishment of international In 1929, Italy and the Holy See entered into the Lateran
organizations. The sovereignty of a state therefore Treaty, where Italy recognized the exclusive dominion
cannot in fact and in reality be considered absolute. and sovereign jurisdiction of the Holy See over the Vatican
Certain restrictions enter into the picture: (1) limitations City. It also recognized the right of the Holy See to
imposed by the very nature of membership in the family receive foreign diplomats, to send its own diplomats to
of nations and (2) limitations imposed by treaty foreign countries, and to enter into treaties according to
stipulations. (Tanada v. Angara, 272 SCRA 18, May International Law.
2, 1997 [Panganiban])
The Lateran Treaty established the statehood of the
379. What must a person who feels aggrieved by the Vatican City “for the purpose of assuring to the Holy See
acts of a foreign sovereign do to espouse his cause? absolute and visible independence and of guaranteeing to
it indisputable sovereignty also in the field of international
Held: Private respondent is not left without any relations.”
legal remedy for the redress of its grievances. Under both
Public International Law and Transnational Law, a person In view of the wordings of the Lateran Treaty, it is difficult
who feels aggrieved by the acts of a foreign sovereign can to determine whether the statehood is vested in the Holy
ask his own government to espouse his cause through See or in the Vatican City. Some writers even suggested
diplomatic channels. that the treaty created two international persons - the
Holy See and Vatican City.
Private respondent can ask the Philippine
government, through the Foreign Office, to espouse its The Vatican City fits into none of the established
claims against the Holy See. Its first task is to persuade categories of states, and the attribution to it of
the Philippine government to take up with the Holy See “sovereignty” must be made in a sense different from that
the validity of its claim. Of course, the Foreign Office shall in which it is applied to other states. In a community of
first make a determination of the impact of its espousal national states, the Vatican City represents an entity
on the relations between the Philippine government and organized not for political but for ecclesiastical purposes
the Holy See (Young, Remedies of Private Claimants and international objects. Despite its size and object, the
Against Foreign States, Selected Readings on Protection Vatican City has an independent government of its own,
by Law of Private Foreign Investments 905, 919 [1964]). with the Pope, who is also head of the Roman Catholic
Once the Philippine government decides to espouse the Church, as the Holy See or Head of State, in conformity
claim, the latter ceases to be a private cause. with its traditions, and the demands of its mission in the
world. Indeed, the world-wide interests and activities of
According to the Permanent Court of the Vatican City are such as to make it in a sense an
International Justice, the forerunner of the International “international state.”
Court of Justice:
One authority wrote that the recognition of the Vatican
City as a state has significant implication - that it is
possible for any entity pursuing objects essentially 383. Discuss the two conflicting concepts of sovereign
different from those pursued by states to be invested with immunity from suit.
international personality.
Held: There are two conflicting concepts of sovereign
Inasmuch as the Pope prefers to conduct foreign relations immunity, each widely held and firmly established.
and enter into transactions as the Holy See and not in the According to the classical or absolute theory, a sovereign
name of the Vatican City, one can conclude that in the cannot, without its consent, be made a respondent in the
Pope's own view, it is the Holy See that is the international courts of another sovereign. According to the newer or
person. restrictive theory, the immunity of the sovereign is
recognized only with regard to public acts or acts jure
The Republic of the Philippines has accorded the Holy See imperii of a state, but not with regard to private acts or
the status of a foreign sovereign. The Holy See, through acts jure gestionis.
its Ambassador, the Papal Nuncio, has had diplomatic
representations with the Philippine government since Some states passed legislation to serve as guidelines for
1957. This appears to be the universal practice in the executive or judicial determination when an act may
international relations. (Holy See, The v. Rosario, Jr., be considered as jure gestionis. The United States passed
238 SCRA 524, 533-534, Dec. 1, 1994, En Banc the Foreign Sovereign Immunities Act of 1976, which
[Quiason]) defines a commercial activity as “either a regular course
of commercial conduct or a particular commercial
381. What are international organizations? Discuss transaction or act.” Furthermore, the law declared that
their nature. the “commercial character of the activity shall be
determined by reference to the nature of the course of
Held: International organizations are institutions conduct or particular transaction or act, rather than by
constituted by international agreement between two or reference to its purpose.” The Canadian Parliament
more States to accomplish common goals. The legal enacted in 1982 an Act to Provide For State Immunity in
personality of these international organizations has been Canadian Courts. The Act defines a “commercial activity”
recognized not only in municipal law, but in international as any particular transaction, act or conduct or any
law as well. regular course of conduct that by reason of its nature, is
of a “commercial character.”
Permanent international commissions and administrative
bodies have been created by the agreement of a The restrictive theory, which is intended to be a solution
considerable number of States for a variety of to the host of problems involving the issue of sovereign
international purposes, economic or social and mainly immunity, has created problems of its own. Legal
non-political. In so far as they are autonomous and treatises and the decisions in countries which follow the
beyond the control of any one State, they have distinct restrictive theory have difficulty in characterizing whether
juridical personality independent of the municipal law of a contract of a sovereign state with a private party is an
the State where they are situated. As such, they are act jure gestionis or an act jure imperii.
deemed to possess a species of international personality
of their own. (SEAFDEC-AQD v. NLRC, 206 SCRA The restrictive theory came about because of the entry of
283, Feb. 14, 1992) sovereign states into purely commercial activities
remotely connected with the discharge of governmental
382. Discuss the basic immunities of international functions. This is particularly true with respect to the
organizations and the reason for affording them such Communist states which took control of nationalized
immunities. business activities and international trading. (Holy See,
The v. Rosario, Jr., 238 SCRA 524, Dec. 1, 1994, En
Held: One of the basic immunities of an international Banc [Quiason])
organization is immunity from local jurisdiction, i.e., that
it is immune from legal writs and processes issued by the 384. Cite some transactions by a foreign state with
tribunals of the country where it is found. The obvious private parties that were considered by the Supreme
reason for this is that the subjection of such an Court as acts “jure imperii” and acts “jure gestionis.”
organization to the authority of the local courts would
afford a convenient medium through which the host Held: This Court has considered the following
government may interfere in their operations or even transactions by a foreign state with private parties as acts
influence or control its policies and decisions; besides, jure imperii: (1) the lease by a foreign government of
such subjection to local jurisdiction would impair the apartment buildings for use of its military officers (Syquia
capacity of such body to discharge its responsibilities v. Lopez, 84 Phil. 312 [1949]); (2) the conduct of public
impartially on behalf of its member-states. (SEAFDEC- bidding for the repair of a wharf at a United States Naval
AQD v. NLRC, 206 SCRA 283, Feb. 4, 1992) Station (United States of America v. Ruiz, supra.); and (3)
the change of employment status of base employees be made site of its mission or the Apostolic Nunciature in
(Sanders v. Veridiano, 162 SCRA 88 [1988]). the Philippines but which purpose cannot be
accomplished as the land was occupied by squatters who
On the other hand, this Court has considered the refused to vacate the area?
following transactions by a foreign state with private
parties as acts jure gestionis: (1) the hiring of a cook in Held: In the case at bench, if petitioner (Holy See) has
the recreation center, consisting of three restaurants, a bought and sold lands in the ordinary course of a real
cafeteria, a bakery, a store, and a coffee and pastry shop estate business, surely the said transaction can be
at the John Hay Air Station in Baguio City, to cater to categorized as an act jure gestionis. However, petitioner
American servicemen and the general public (United has denied that the acquisition and subsequent disposal
States of America v. Rodrigo, 182 SCRA 644 [1990]; and of Lot 5-A were made for profit but claimed that it
(2) the bidding for the operation of barber shops in Clark acquired said property for the site of its mission or the
Air Base in Angeles City (United States of America v. Apostolic Nunciature in the Philippines. X x x
Guinto, 182 SCRA 644 [1990]). The operation of the
restaurants and other facilities open to the general public Lot 5-A was acquired by petitioner as a donation from the
is undoubtedly for profit as a commercial and not a Archdiocese of Manila. The donation was made not for
governmental activity. By entering into the employment commercial purpose, but for the use of petitioner to
contract with the cook in the discharge of its proprietary construct thereon the official place of residence of the
function, the United States government impliedly divested Papal Nuncio. The right of a foreign sovereign to acquire
itself of it sovereign immunity from suit. (Holy See, The property, real or personal, in a receiving state, necessary
v. Rosario, Jr., 238 SCRA 524, Dec. 1, 1994, En for the creation and maintenance of its diplomatic
Banc [Quiason]) mission, is recognized in the 1961 Vienna Convention on
Diplomatic Relations. This treaty was concurred in by the
385. What should be the guidelines to determine what Philippine Senate and entered into force in the Philippines
activities and transactions shall be considered on November 15, 1965.
“commercial” and as constituting acts “jure gestionis” by
a foreign state? In Article 31(a) of the Convention, a diplomatic envoy is
granted immunity from the civil and administrative
Held: In the absence of legislation defining what jurisdiction of the receiving state over any real action
activities and transactions shall be considered relating to private immovable property situated in the
“commercial” and as constituting acts jure gestionis, we territory of the receiving state which the envoy holds on
have to come out with our own guidelines, tentative they behalf of the sending state for the purposes of the
may be. mission. If this immunity is provided for a diplomatic
envoy, with all the more reason should immunity be
Certainly, the mere entering into a contract by a foreign recognized as regards the sovereign itself, which in this
state with a private party cannot be the ultimate test. case is the Holy See.
Such an act can only be the start of the inquiry. The
logical question is whether the foreign state is engaged in The decision to transfer the property and the subsequent
the activity in the regular course of business. If the disposal thereof are likewise clothed with a governmental
foreign state is not engaged regularly in a business or character. Petitioner did not sell Lot 5-A for profit or gain.
trade, the particular act or transaction must then be It merely wanted to dispose off the same because the
tested by its nature. If the act is in pursuit of a sovereign squatters living thereon made it almost impossible for
activity, or an incident thereof, then it is an act jure petitioner to use it for the purpose of the donation. (Holy
imperii, especially when it is not undertaken for gain or See, The v. Rosario, Jr., 238 SCRA 524, Dec. 1,
profit. 1994, En Banc [Quiason])

As held in United States of America v. Guinto (supra.) : 387. How is sovereign or diplomatic immunity pleaded
in a foreign court?
“There is no question that the United States of America,
like any other state, will be deemed to have impliedly Held: In Public International Law, when a state or
waived its non-suability if it has entered into a contract in international agency wishes to plead sovereign or
its proprietary or private capacity. It is only when the diplomatic immunity in a foreign court, it requests the
contract involves its sovereign or governmental capacity Foreign Office of the state where it is sued to convey to
that no such waiver may be implied.” the court that said defendant is entitled to immunity.
(Holy See, The v. Rosario, Jr., 238 SCRA 524, Dec.
1, 1994, En Banc [Quiason]) In the United States, the procedure followed is the
process of “suggestion,” where the foreign state or the
386. May the Holy See be sued for selling the land it international organization sued in an American court
acquired by donation from the Archdiocese of Manila to requests the Secretary of State to make a determination
as to whether it is entitled to immunity. If the Secretary formally intervened in this case and officially certified that
of State finds that the defendant is immune from suit, he, the Embassy of the Holy See is a duly accredited
in turn, asks the Attorney General to submit to the court diplomatic mission to the Republic of the Philippines
a “suggestion” that the defendant is entitled to immunity. exempt from local jurisdiction and entitled to all the rights,
In England, a similar procedure is followed, only the privileges and immunities of a diplomatic mission or
Foreign Office issues a certification to that effect instead embassy in this country. The determination of the
of submitting a “suggestion”. executive arm of government that a state or
instrumentality is entitled to sovereign or diplomatic
In the Philippines, the practice is for the foreign immunity is a political question that is conclusive upon the
government or the international organization to first courts. Where the plea of immunity is recognized and
secure an executive endorsement of its claim of sovereign affirmed by the executive branch, it is the duty of the
or diplomatic immunity. But how the Philippine Foreign courts to accept this claim so as not to embarrass the
Office conveys its endorsement to the courts varies. In executive arm of the government in conducting the
International Catholic Migration Commission v. Calleja, country’s foreign relations. As in International Catholic
190 SCRA 130 (1990), the Secretary of Foreign Affairs just Migration Commission and in World Health Organization,
sent a letter directly to the Secretary of Labor and we abide by the certification of the Department of Foreign
Employment, informing the latter that the respondent- Affairs. (Holy See, The v. Rosario, Jr., 238 SCRA 524,
employer could not be sued because it enjoyed diplomatic Dec. 1, 1994, En Banc [Quiason])
immunity. In World Health Organization v. Aquino, 48
SCRA 242 (1972), the Secretary of Foreign Affairs sent 389. Discuss the Status of the Vatican and the Holy
the trial court a telegram to that effect. In Baer v. Tizon, See in International Law.
57 SCRA 1 (1974), the U.S. Embassy asked the Secretary
of Foreign Affairs to request the Solicitor General to make, Held: Before the annexation of the Papal States
in behalf of the commander of the United States Naval by Italy in 1870, the Pope was the monarch and he, as
Base at Olongapo City, Zambales, a “suggestion” to the Holy See, was considered a subject of International
respondent Judge. The Solicitor General embodied the Law. With the loss of the Papal States and the limitation
“suggestion” in a Manifestation and Memorandum as of the territory under the Holy See to an area of 108.7
amicus curiae. acres, the position of the Holy See in International Law
became controversial (Salonga and Yap, Public
In the case at bench, the Department of Foreign Affairs, International Law 36-37 [1992]).
through the Office of Legal Affairs moved with this Court
to be allowed to intervene on the side of petitioner. The In 1929, Italy and the Holy See entered into the
Court allowed the said Department to file its Lateran Treaty, where Italy recognized the exclusive
memorandum in support of petitioner’s claim of sovereign dominion and sovereign jurisdiction of the Holy See over
immunity. the Vatican City. It also recognized the right of the Holy
See to receive foreign diplomats, to send its own
In some cases, the defense of sovereign immunity was diplomats to foreign countries, and to enter into treaties
submitted directly to the local courts by the respondents according to International Law (Garcia, Questions and
through their private counsels. In cases where the Problems in International Law, Public and Private 81
foreign states bypass the Foreign Office, the courts can [1948]).
inquire into the facts and make their own determination
as to the nature of the acts and transactions involved. The Lateran Treaty established the statehood of
(Holy See, The v. Rosario, Jr., 238 SCRA 524, Dec. the Vatican City "for the purpose of assuring to the Holy
1, 1994, En Banc [Quiason]) See absolute and visible independence and of
guaranteeing to it indisputable sovereignty also in the
388. Is the determination of the executive branch of field of international relations" (O'Connell, I International
the government that a state or instrumentality is entitled Law 311 [1965]).
to sovereign or diplomatic immunity subject to judicial
review, or is it a political question and therefore, In view of the wordings of the Lateran Treaty, it
conclusive upon the courts? is difficult to determine whether the statehood is vested
in the Holy See or in the Vatican City. Some writers even
Held: The issue of petitioner’s (The Holy See) non- suggested that the treaty created two international
suability can be determined by the trial court without persons - the Holy See and Vatican City (Salonga and Yap,
going to trial in light of the pleadings x x x. Besides, the supra, 37).
privilege of sovereign immunity in this case was
sufficiently established by the Memorandum and The Vatican City fits into none of the established
Certification of the Department of Foreign Affairs. As the categories of states, and the attribution to it of
department tasked with the conduct of the Philippines’ “sovereignty” must be made in a sense different from that
foreign relations, the Department of Foreign Affairs has in which it is applied to other states (Fenwick,
International Law 124-125 [1948]; Cruz, International Held: Extradition was first practiced by the Egyptians,
Law 37 [1991]). In a community of national states, the Chinese, Chaldeans and Assyro-Babylonians but their
Vatican City represents an entity organized not for basis for allowing extradition was unclear. Sometimes, it
political but for ecclesiastical purposes and international was granted due to pacts; at other times, due to plain
objects. Despite its size and object, the Vatican City has good will. The classical commentators on international
an independent government of its own, with the Pope, law thus focused their early views on the nature of the
who is also head of the Roman Catholic Church, as the duty to surrender an extraditee --- whether the duty is
Holy See or Head of State, in conformity with its legal or moral in character. Grotius and Vattel led the
traditions, and the demands of its mission in the world. school of thought that international law imposed a legal
Indeed, the world-wide interests and activities of the duty called civitas maxima to extradite criminals. In sharp
Vatican City are such as to make it in a sense an contrast, Puffendorf and Billot led the school of thought
“international state” (Fenwick, supra, 125; Kelsen, that the so-called duty was but an "imperfect obligation
Principles of International Law 160 [1956]). which could become enforceable only by a contract or
agreement between states.
One authority wrote that the recognition of the
Vatican City as a state has significant implication - that it Modern nations tilted towards the view of Puffendorf and
is possible for any entity pursuing objects essentially Billot that under international law there is no duty to
different from those pursued by states to be invested with extradite in the absence of treaty, whether bilateral or
international personality (Kunz, The Status of the Holy multilateral. Thus, the US Supreme Court in US v.
See in International Law, 46 The American Journal of Rauscher (119 US 407, 411, 7 S Ct. 234, 236, 30 L. ed.
International Law 308 [1952]). 425 [1886]), held: “x x x it is only in modern times that
the nations of the earth have imposed upon themselves
Inasmuch as the Pope prefers to conduct foreign the obligation of delivering up these fugitives from justice
relations and enter into transactions as the Holy See and to the states where their crimes were committed, for trial
not in the name of the Vatican City, one can conclude that and punishment. This has been done generally by
in the Pope's own view, it is the Holy See that is the treaties x x x Prior to these treaties, and apart from them
international person. there was no well-defined obligation on one country to
deliver up such fugitives to another; and though such
The Republic of the Philippines has accorded the delivery was often made it was upon the principle of
Holy See the status of a foreign sovereign. The Holy See, comity x x x.” (Dissenting Opinion, Puno, J., in
through its Ambassador, the Papal Nuncio, has had Secretary of Justice v. Hon. Ralph C. Lantion, G.R.
diplomatic representations with the Philippine No. 139465, Jan. 18, 2000, En Banc)
government since 1957. This appears to be the universal
practice in international relations. (Holy See, The v. 392. What is the nature of an extradition proceeding?
Rosario, Jr., 238 SCRA 524, 533-534, Dec. 1, 1994, Is it akin to a criminal proceeding?
En Banc [Quiason])
Held: [A]n extradition proceeding is sui generis. It is not
a criminal proceeding which will call into operation all the
390. What is extradition? To whom does it apply? rights of an accused as guaranteed by the Bill of Rights.
To begin with, the process of extradition does not involve
Held: It is the “process by which persons charged with the determination of the guilt or innocence of an accused.
or convicted of crime against the law of a State and found His guilt or innocence will be adjudged in the court of the
in a foreign State are returned by the latter to the former state where he will be extradited. Hence, as a rule,
for trial or punishment. It applies to those who are merely constitutional rights that are only relevant to determine
charged with an offense but have not been brought to the guilt or innocence of an accused cannot be invoked
trial; to those who have been tried and convicted and by an extraditee especially by one whose extradition
have subsequently escaped from custody; and those who papers are still undergoing evaluation. As held by the US
have been convicted in absentia. It does not apply to Supreme Court in United States v. Galanis:
persons merely suspected of having committed an
offense but against whom no charge has been laid or to “An extradition proceeding is not a criminal prosecution,
a person whose presence is desired as a witness or for and the constitutional safeguards that accompany a
obtaining or enforcing a civil judgment.” (Weston, Falk, criminal trial in this country do not shield an accused from
D' Amato, International Law and Order, 2nd ed., p. extradition pursuant to a valid treaty.” (Wiehl, Extradition
630 [1990], cited in Dissenting Opinion, Puno, J., Law at the Crossroads: The Trend Toward Extending
in Secretary of Justice v. Hon. Ralph C. Lantion, Greater Constitutional Procedural Protections To Fugitives
G.R. No. 139465, Jan. 18, 2000, En Banc) Fighting Extradition from the United States, 19 Michigan
Journal of International Law 729, 741 [1998], citing
391. Discuss the basis for allowing extradition. United States v. Galanis, 429 F. Supp. 1215 [D. Conn.
1977])
accordance with the ordinary meaning to be given to the
There are other differences between an extradition terms of the treaty in their context and in light of its object
proceeding and a criminal proceeding. An extradition and purpose.” X x x. It cannot be gainsaid that today,
proceeding is summary in natural while criminal countries like the Philippines forge extradition treaties to
proceedings involve a full-blown trial. In contradistinction arrest the dramatic rise of international and transnational
to a criminal proceeding, the rules of evidence in an crimes like terrorism and drug trafficking. Extradition
extradition proceeding allow admission of evidence under treaties provide the assurance that the punishment of
less stringent standards. In terms of the quantum of these crimes will not be frustrated by the frontiers of
evidence to be satisfied, a criminal case requires proof territorial sovereignty. Implicit in the treaties should be
beyond reasonable doubt for conviction while a fugitive the unbending commitment that the perpetrators of these
may be ordered extradited “upon showing of the crimes will not be coddled by any signatory state.
existence of a prima facie case.” Finally, unlike in a
criminal case where judgment becomes executory upon It ought to follow that the RP-US Extradition Treaty calls
being rendered final, in an extradition proceeding, our for an interpretation that will minimize if not prevent the
courts may adjudge an individual extraditable but the escape of extraditees from the long arm of the law and
President has the final discretion to extradite him . The expedite their trial. X x x
United States adheres to a similar practice whereby the
Secretary of State exercises wide discretion in balancing [A]n equally compelling factor to consider is the
the equities of the case and the demands of the nation's understanding of the parties themselves to the RP-US
foreign relations before making the ultimate decision to Extradition Treaty as well as the general interpretation of
extradite. the issue in question by other countries with similar
treaties with the Philippines . The rule is recognized that
As an extradition proceeding is not criminal in character while courts have the power to interpret treaties, the
and the evaluation stage in an extradition proceeding is meaning given them by the departments of government
not akin to a preliminary investigation, the due process particularly charged with their negotiation and
safeguards in the latter do not necessarily apply to the enforcement is accorded great weight. The reason for
former. This we hold for the procedural due process the rule is laid down in Santos III v. Northwest Orient
required by a given set of circumstances “must begin with Airlines, et al. (210 SCRA 256, 261 [1992]), where we
a determination of the precise nature of the government stressed that a treaty is a joint executive-legislative act
function involved as well as the private interest that has which enjoys the presumption that “it was first carefully
been affected by governmental action.” The concept of studied and determined to be constitutional before it was
due process is flexible for “not all situations calling for adopted and given the force of law in the country.”
procedural safeguards call for the same kind of (Secretary of Justice v. Hon. Ralph C. Lantion, G.R.
procedure.” (Secretary of Justice v. Hon. Ralph C. No. 139465, Oct. 17, 2000, En Banc [Puno])
Lantion, G.R. No. 139465, Oct. 17, 2000, En Banc
[Puno]) 395. Is respondent Mark Jimenez entitled to bail
during the pendency of the Extradition Proceeding?
393. Will the retroactive application of an extradition
treaty violate the constitutional prohibition against "ex Held: We agree with petitioner. As suggested by the
post facto" laws? use of the word “conviction,” the constitutional provision
on bail x x x, as well as Section 4 of Rule 114 of the Rules
Held: The prohibition against ex post facto law applies of Court, applies only when a person has been arrested
only to criminal legislation which affects the substantial and detained for violation of Philippine criminal laws. It
rights of the accused. This being so, there is no merit in does not apply to extradition proceedings, because
the contention that the ruling sustaining an extradition extradition courts do not render judgments of conviction
treaty’s retroactive application violates the constitutional or acquittal.
prohibition against ex post facto laws. The treaty is
neither a piece of criminal legislation nor a criminal Moreover, the constitutional right to bail “flows
procedural statute. (Wright v. CA, 235 SCRA 341, from the presumption of innocence in favor of every
Aug. 15, 1994 [Kapunan]) accused who should not be subjected to the loss of
freedom as thereafter he would be entitled to acquittal,
394. Discuss the rules in the interpretation of unless his guilt be proved beyond reasonable doubt.” (De
extradition treaties. la Camara v. Enage, 41 SCRA 1, 6, September 17, 1971,
per Fernando, Jr. [later CJ]) It follows that the
Held: [A]ll treaties, including the RP-US Extradition constitutional provision on bail will not apply to a case like
Treaty, should be interpreted in light of their intent. extradition, where the presumption of innocence is not an
Nothing less than the Vienna Convention on the Law of issue.
Treaties to which the Philippines is a signatory provides
that “a treaty shall be interpreted in good faith in
The provision in the Constitution stating that the right to normally a judicial prerogative. Hence, any intrusion by the
bail shall not be impaired even when the privilege of the courts into the exercise of this power should be characterized
writ of habeas corpus is suspended” does not detract from by caution, so that the vital international and bilateral interests
the rule that the constitutional right to bail is available of our country will not be unreasonably impeded or
only in criminal proceedings. It must be noted that the compromised. In short, while this Court is ever protective of
suspension of the privilege of the writ of habeas corpus “the sporting idea of fair play,” it also recognizes the limits of
finds application “only to persons judicially charged for its own prerogatives and the need to fulfill international
rebellion or offenses inherent in or directly connected with obligations. (Government of the United States of America v.
Hon. Guillermo Purganan, G.R. No. 148571, Sept. 24, 2002,
invasion.” (Sec. 18, Article VII, Constitution) Hence, the
En Banc [Panganiban])
second sentence in the constitutional provision on bail
merely emphasizes the right to bail in criminal
397. Are there special circumstances compelling
proceedings for the aforementioned offenses. It cannot
enough for the Court to grant Mark Jimenez’s request
be taken to mean that the right is available even in
for provisional release on bail?
extradition proceedings that are not criminal in nature.
Held: Along this line, Jimenez contends that there are
That the offenses for which Jimenez is sought to be extradited
special circumstances that are compelling enough for the
are bailable in the United States is not an argument to grant him
one in the present case. To stress, extradition proceedings are Court to grant his request for provisional release on bail.
separate and distinct from the trial for the offenses for which he We have carefully examined these circumstances and
is charged. He should apply for bail before the courts trying the shall now discuss them.
criminal cases against him, not before the extradition court.
(Government of the United States of America v. Hon. 1. Alleged Disenfranchisement
Guillermo Purganan, G.R. No. 148571, Sept. 24, 2002, En
Banc [Panganiban]) While his extradition was pending, Respondent
Jimenez was elected as a member of the House of
396. What is the exception to the “No Bail” Rule in Representatives. On that basis, he claims that his
Extradition Proceedings? detention will disenfranchise his Manila district of
600,000 residents. We are not persuaded. In People v.
Held: The rule x x x is that bail is not a matter of right Jalosjos (324 SCRA 689, February 3, 2000, per Ynares-
in extradition cases. However, the judiciary has the Santiago, J.), the Court has already debunked the
constitutional duty to curb grave abuse of discretion and disenfranchisement argument x x x.
tyranny, as well as the power to promulgate rules to
protect and enforce constitutional rights. Furthermore, It must be noted that even before private
we believe that the right to due process is broad enough respondent ran for and won a congressional seat in
to include the grant of basic fairness to extraditees. Manila, it was already of public knowledge that the
Indeed, the right to due process extends to the “life, United States was requesting his extradition. Hence, his
liberty or property” of every person. It is “dynamic and constituents were or should have been prepared for the
resilient, adaptable to every situation calling for its consequences of the extradition case against their
application.” (I.A. Cruz, Constitutional Law, 1998 ed., p. representative, including his detention pending the final
98) resolution of the case. Premises considered and in line
with Jalosjos, we are constrained to rule against his
Accordingly and to best serve the ends of justice, we claim that his election to public office is by itself a
believe and so hold that, after a potential extraditee has been compelling reason to grant him bail.
arrested or placed under the custody of the law, bail may be
applied for and granted as an exception, only upon a clear and 2. Anticipated Delay
convincing showing (1) that, once granted bail, the applicant
will not be a flight risk or a danger to the community; and (2) Respondent Jimenez further contends that
that there exist special, humanitarian and compelling because the extradition proceedings are lengthy, it
circumstances including, as a matter of reciprocity, those cited would be unfair to confine him during the pendency of
by the highest court in the requesting state when it grants the case. Again we are not convinced. We must
provisional liberty in extradition cases therein. emphasize that extradition cases are summary in nature.
They are resorted to merely to determine whether the
Since this exception has no express or specific
extradition petition and its annexes conform to the
statutory basis, and since it is derived essentially from general
principles of justice and fairness, the applicant bears the Extradition Treaty, not to determine his guilt or
burden of proving the above two-tiered requirement with innocence. Neither is it, as a rule, intended to address
clarity, precision and emphatic forcefulness. The Court issues relevant to the constitutional rights available to
realizes that extradition is basically an executive, not a the accused in a criminal action.
judicial, responsibility arising from the presidential power to
conduct foreign relations. In its barest concept, it partakes of We are not overruling the possibility that
the nature of police assistance amongst states, which is not petitioner may, in bad faith, unduly delay the
proceedings. This is quite another matter that is not at methods of dealing with criminals and crimes that
issue here. Thus, any further discussion of this point transcend international boundaries.
would be merely anticipatory and academic.
Today, “a majority of nations in the world
However, if the delay is due to maneuverings of community have come to look upon extradition as the
respondent, with all the more reason would the grant of major effective instrument of international co-operation in
bail not be justified. Giving premium to delay by the suppression of crime.” (Bassiouni, supra, p. 21) It is
considering it as a special circumstance for the grant of the only regular system that has been devised to return
bail would be tantamount to giving him the power to fugitives to the jurisdiction of a court competent to try
grant bail to himself. It would also encourage him to them in accordance with municipal and international law
stretch out and unreasonably delay the extradition (Id., p. 67).
proceedings even more. This we cannot allow.
Xxx
3. Not a Flight Risk?
Indeed, in this era of globalization, easier and
Jimenez further claims that he is not a flight faster international travel, and an expanding ring of
risk. To support this claim, he stresses that he learned international crimes and criminals, we cannot afford to be
of the extradition request in June 1999; yet, he has not an isolationist state. We need to cooperate with other
fled the country. True, he has not actually fled during states in order to improve our chances of suppressing
the preliminary stages of the request for his extradition. crime in our country.
Yet, this fact cannot be taken to mean that he will not
flee as the process moves forward to its conclusion, as 2. The Requesting State Will Accord Due Process
he hears the footsteps of the requesting government to the Accused.
inching closer and closer. That he has not yet fled from
the Philippines cannot be taken to mean that he will Second, an extradition treaty presupposes that
stand his ground and still be within reach of our both parties thereto have examined, and that both accept
government if and when it matters; that is, upon the and trust, each other’s legal system and judicial process
resolution of the Petition for Extradition. (Coquia, “On Implementation of the RP-US Extradition
Treaty,” The Lawyers Review, August 31, 2000, p. 4).
In any event, it is settled that bail may be More pointedly, our duly authorized representative’s
applied for and granted by the trial court at anytime signature on an extradition treaty signifies our confidence
after the applicant has been taken into custody and prior in the capacity and willingness of the other state to
to judgment, even after bail has been previously denied. protect the basic rights of the person sought to be
In the present case, the extradition court may continue extradited (See Bassiouni, p. 546; citing 221 US 508, 512
hearing evidence on the application for bail, which may [1910]). That signature signifies our full faith that the
be granted in accordance with the guidelines in this accused will be given, upon extradition to the requesting
Decision. (Government of the United States of state, all relevant and basic rights in the criminal
America v. Hon. Guillermo Purganan, G.R. No. proceedings that will take place therein; otherwise, the
148571, Sept. 24, 2002, En Banc [Panganiban]) treaty would not have been signed, or would have been
directly attacked for its unconstitutionality.

3. The Proceedings Are Sui Generis.


398. Discuss the Five Postulates of Extradition.
Third, as pointed out in Secretary of Justice v.
Held: Lantion (Supra), extradition proceedings are not criminal
in nature. In criminal proceedings, the constitutional
1. Extradition Is a Major Instrument for the Suppression rights of the accused are at fore; in extradition which is
of Crime. sui generis – in a class by itself – they are not.

First, extradition treaties are entered into for the Xxx


purpose of suppressing crime by facilitating the arrest and
custodial transfer (Bassiouni, International Extradition, Given the foregoing, it is evident that the
1987 ed., p. 68) of a fugitive from one state to the other. extradition court is not called upon to ascertain the guilt
or the innocence of the person sought to be extradited
With the advent of easier and faster means of (Secretary of Justice v. Lantion, supra.). Such
international travel, the flight of affluent criminals from determination during the extradition proceedings will only
one country to another for the purpose of committing result in needless duplication and delay. Extradition is
crime and evading prosecution has become more merely a measure of international judicial assistance
frequent. Accordingly, governments are adjusting their through which a person charged with or convicted of a
crime is restored to a jurisdiction with the best claim to requested state despite learning that the requesting state
try that person. It is not part of the function of the is seeking his return and that the crimes he is charged
assisting authorities to enter into questions that are the with are bailable – eloquently speak of his aversion to the
prerogative of that jurisdiction (Shearer, Extradition in processes in the requesting state, as well as his
International Law, 1971 ed., p. 157). The ultimate predisposition to avoid them at all cost. These
purpose of extradition proceedings in court is only to circumstances point to an ever-present, underlying high
determine whether the extradition request complies with risk of flight. He has demonstrated that he has the
the Extradition Treaty, and whether the person sought is capacity and the will to flee. Having fled once, what is
extraditable (Id., p. 545). there to stop him, given sufficient opportunity, from
fleeing a second time? (Government of the United
4. Compliance Shall Be in Good Faith. States of America v. Hon. Guillermo Purganan, G.R.
No. 148571, Sept. 24, 2002, En Banc
Fourth, our executive branch of government voluntarily [Panganiban])
entered into the Extradition Treaty, and our legislative
branch ratified it. Hence, the Treaty carries the 399. Discuss the Ten Points to consider in Extradition
presumption that its implementation will serve the Proceedings?
national interest.
Held: 1. The ultimate purpose of extradition
Fulfilling our obligations under the Extradition Treaty promotes proceedings is to determine whether the request
comity (In line with the Philippine policy of cooperation and expressed in the petition, supported by its annexes and
amity with all nations set forth in Article II, Section 2, the evidence that may be adduced during the hearing of
Constitution). On the other hand, failure to fulfill our the petition, complies with the Extradition Treaty and
obligations thereunder paints at bad image of our country before Law; and whether the person sought is extraditable. The
the world community. Such failure would discourage other proceedings are intended merely to assist the requesting
states from entering into treaties with us, particularly an state in bringing the accused – or the fugitive who has
extradition treaty that hinges on reciprocity.
illegally escaped – back to its territory, so that the criminal
process may proceed therein.
Verily, we are bound by pacta sunt servanda to comply in
good faith with our obligations under the Treaty 2. By entering into an extradition treaty, the Philippines
(Secretary of Justice v. Lantion, supra.). This principle is deemed to have reposed its trust in the reliability or
requires that we deliver the accused to the requesting soundness of the legal and judicial system of its treaty
country if the conditions precedent to extradition, as set partner; as well as in the ability and the willingness of
forth in the Treaty, are satisfied. In other words, “[t]he the latter to grant basic rights to the accused in the
demanding government, when it has done all that the pending criminal case therein.
treaty and the law require it to do, is entitled to the
delivery of the accused on the issue of the proper warrant, 3. By nature then, extradition proceedings are not
and the other government is under obligation to make the equivalent to a criminal case in which guilt or innocence
surrender.” (Wright v. Henkel, 190 U.S. 40, 62, March 23, is determined. Consequently, an extradition case is not
1903) Accordingly, the Philippines must be ready and in one in which the constitutional rights of the accused are
a position to deliver the accused, should it be found necessarily available. It is more akin, if at all, to a
proper. court’s request to police authorities for the arrest of the
accused who is at large or has escaped detention or
5. There Is an Underlying Risk of Flight. jumped bail. Having once escaped the jurisdiction of the
requesting state, the reasonable prima facie
Fifth, persons to be extradited are presumed to be flight presumption is that the person would escape again if
risks. This prima facie presumption finds reinforcement given the opportunity.
in the experience of the executive branch: nothing short
of confinement can ensure that the accused will not flee 4. Immediately upon receipt of the petition for
the jurisdiction of the requested state in order to thwart extradition and its supporting documents, the judge
their extradition to the requesting state. shall make a prima facie finding whether the petition is
sufficient in form and substance, whether it complies
The present extradition case further validates the premise with the Extradition Treaty and Law, and whether the
that persons sought to be extradited have a propensity to person sought is extraditable. The magistrate has
flee. Indeed, extradition hearings would not even begin, discretion to require the petitioner to submit further
if only the accused were willing to submit to trial in the documentation, or to personally examine the affiants or
requesting country (Persily, “International Extradition and witnesses. If convinced that a prima facie case exists,
the Right to Bail,” 34 Stan. J. Int’l L. 407 [Summer 1988]). the judge immediately issues a warrant for the arrest of
Prior acts of herein respondent – (1) leaving the the potential extraditee and summons him or her to
requesting state right before the conclusion of his
indictment proceedings there; and (2) remaining in the
answer and to appear at scheduled hearings on the G.R. No. 148571, Sept. 24, 2002, En Banc
petition. [Panganiban])

5. After being taken into custody, potential extraditees


may apply for bail. Since the applicants have a history
of absconding, they have the burden of showing that (a) 400. What is a Treaty? Discuss.
there is no flight risk and no danger to the community;
and (b) there exist special, humanitarian or compelling Held: A treaty, as defined by the Vienna Convention on
circumstances. The grounds used by the highest court the Law of Treaties, is “an international instrument
in the requesting state for the grant of bail therein may concluded between States in written form and governed
be considered, under the principle of reciprocity as a by international law, whether embodied in a single
special circumstance. In extradition cases, bail is not a instrument or in two or more related instruments, and
matter of right; it is subject to judicial discretion in the whatever its particular designation.” There are many
context of the peculiar facts of each case. other terms used for a treaty or international agreement,
some of which are: act, protocol, agreement, compromis
6. Potential extraditees are entitled to the rights to due d' arbitrage, concordat, convention, declaration,
process and to fundamental fairness. Due process does exchange of notes, pact, statute, charter and modus
not always call for a prior opportunity to be heard. A vivendi. All writers, from Hugo Grotius onward, have
subsequent opportunity is sufficient due to the flight risk pointed out that the names or titles of international
involved. Indeed, available during the hearings on the agreements included under the general term treaty have
petition and the answer is the full chance to be heard little or no significance. Certain terms are useful, but they
and to enjoy fundamental fairness that is compatible furnish little more than mere description
with the summary nature of extradition.
Article 2(2) of the Vienna Convention provides that “the
7. This Court will always remain a protector of human provisions of paragraph 1 regarding the use of terms in
rights, a bastion of liberty, a bulwark of democracy and the present Convention are without prejudice to the use
the conscience of society. But it is also well aware of of those terms, or to the meanings which may be given
the limitations of its authority and of the need for to them in the internal law of the State.” (BAYAN
respect for the prerogatives of the other co-equal and [Bagong Alyansang Makabayan] v. Executive
co-independent organs of government. Secretary Ronaldo Zamora, G.R. No. 138570, Oct.
10, 2000, En Banc [Buena])
8. We realize that extradition is essentially an executive,
not a judicial, responsibility arising out of the 401. Discuss the binding effect of treaties and
presidential power to conduct foreign relations and to executive agreements in international law.
implement treaties. Thus, the Executive Department of
government has broad discretion in its duty and power Held: [I]n international law, there is no difference
of implementation. between treaties and executive agreements in their
binding effect upon states concerned, as long as the
9. On the other hand, courts merely perform oversight functionaries have remained within their powers .
functions and exercise review authority to prevent or International law continues to make no distinction
excise grave abuse and tyranny. They should not allow between treaties and executive agreements: they are
contortions, delays and “over-due process” every little equally binding obligations upon nations. (BAYAN
step of the way, lest these summary extradition [Bagong Alyansang Makabayan] v. Executive
proceedings become not only inutile but also sources of Secretary Ronaldo Zamora, G.R. No. 138570, Oct.
international embarrassment due to our inability to 10, 2000, En Banc [Buena])
comply in good faith with a treaty partner’s simple
request to return a fugitive. Worse, our country should 402. Does the Philippines recognize the binding effect
not be converted into a dubious haven where fugitives of executive agreements even without the concurrence of
and escapees can unreasonably delay, mummify, mock, the Senate or Congress?
frustrate, checkmate and defeat the quest for bilateral
justice and international cooperation. Held: In our jurisdiction, we have recognized the binding
effect of executive agreements even without the
10. At bottom, extradition proceedings should be concurrence of the Senate or Congress. In Commissioner
conducted with all deliberate speed to determine of Customs v. Eastern Sea Trading (3 SCRA 351, 356-357
compliance with the Extradition Treaty and Law; and, [1961]), we had occasion to pronounce:
while safeguarding basic individual rights, to avoid the
legalistic contortions, delays and technicalities that may “x x x the right of the Executive to enter into binding
negate that purpose. (Government of the United agreements without the necessity of subsequent
States of America v. Hon. Guillermo Purganan, Congressional approval has been confirmed by long
usage. From the earliest days of our history we have v. S.C. Johnson and Son, Inc., 309 SCRA 87, 107-
entered into executive agreements covering such subjects 108, June 25, 1999, 3rd Div. [Gonzaga-Reyes])
as commercial and consular relations, most-favored-
nation rights, patent rights, trademark and copyright 405. What is the essence of the principle behind the
protection, postal and navigation arrangements and the "most-favored-nation" clause as applied to tax treaties?
settlement of claims. The validity of these has never
been seriously questioned by our courts. " Held: The essence of the principle is to allow the
(BAYAN [Bagong Alyansang Makabayan] v. taxpayer in one state to avail of more liberal provisions
Executive Secretary Ronaldo Zamora, G.R. No. granted in another tax treaty to which the country of
138570, Oct. 10, 2000, En Banc [Buena]) residence of such taxpayer is also a party provided that
the subject matter of taxation x x x is the same as that in
403. What is a "protocol de cloture"? Will it require the tax treaty under which the taxpayer is liable.
concurrence by the Senate?
In Commissioner of Internal Revenue v. S.C.
Held: A final act, sometimes called protocol de cloture, Johnson and Son, Inc., 309 SCRA 87, June 25,
is an instrument which records the winding up of the 1999, the SC did not grant the claim filed by S.C. Johnson
proceedings of a diplomatic conference and usually and Son, Inc., a non-resident foreign corporation based
includes a reproduction of the texts of treaties, in the USA, with the BIR for refund of overpaid
conventions, recommendations and other acts agreed withholding tax on royalties pursuant to the most-
upon and signed by the plenipotentiaries attending the favored-nation clause of the RP-US Tax Treaty in relation
conference. It is not the treaty itself. It is rather a to the RP-West Germany Tax Treaty. It held:
summary of the proceedings of a protracted conference
which may have taken place over several years. It will Given the purpose underlying tax treaties and the
not require the concurrence of the Senate. The rationale for the most favored nation clause, the
documents contained therein are deemed adopted concessional tax rate of 10 percent provided for in the RP-
without need for ratification. (Tanada v. Angara, 272 Germany Tax Treaty should apply only if the taxes
SCRA 18, May 2, 1997 [Panganiban]) imposed upon royalties in the RP-US Tax Treaty and in
the RP-Germany Tax Treaty are paid under similar
404. What is the “most-favored-nation” clause? What circumstances. This would mean that private respondent
is its purpose? (S.C. Johnson and Son, Inc.) must prove that the RP-US
Tax Treaty grants similar tax reliefs to residents of the
Answer: 1. The most-favored-nation clause may be United States in respect of the taxes imposable upon
defined, in general, as a pledge by a contracting party to royalties earned from sources within the Philippines as
a treaty to grant to the other party treatment not less those allowed to their German counterparts under the RP-
favorable than that which has been or may be granted to Germany Tax Treaty.
the “most favored” among other countries. The clause
has been commonly included in treaties of commercial The RP-US and the RP-West Germany Tax Treaties do not
nature. contain similar provisions on tax crediting. Article 24 of
the RP-Germany Tax Treaty x x x expressly allows
There are generally two types of most-favored-nation crediting against German income and corporation tax of
clause, namely, conditional and unconditional. According 20% of the gross amount of royalties paid under the law
to the clause in its unconditional form, any advantage of of the Philippines. On the other hand, Article 23 of the
whatever kind which has been or may in future be granted RP-US Tax Treaty, which is the counterpart provision with
by either of the contracting parties to a third State shall respect to relief for double taxation, does not provide for
simultaneously and unconditionally be extended to the similar crediting of 20% of the gross amount of royalties
other under the same or equivalent conditions as those paid. X x x
under which it has been granted to the third State.
(Salonga & Yap, Public International Law, 5th X x x The entitlement of the 10% rate by U.S. firms
Edition, 1992, pp. 141-142) despite the absence of matching credit (20% for
royalties) would derogate from the design behind the
2. The purpose of a most favored nation clause is to grant most favored nation clause to grant equality of
to the contracting party treatment not less favorable than international treatment since the tax burden laid upon the
that which has been or may be granted to the "most income of the investor is not the same in the two
favored" among other countries. The most favored nation countries. The similarity in the circumstances of payment
clause is intended to establish the principle of equality of of taxes is a condition for the enjoyment of most favored
international treatment by providing that the citizens or nation treatment precisely to underscore the need for
subjects of the contracting nations may enjoy the equality of treatment.
privileges accorded by either party to those of the most
favored nation (Commissioner of Internal Revenue
406. What is ratification? Discuss its function in the
treaty-making process. Held: This principle is expressed in Article 5 of the Hague
Convention of 1930 on the Conflict of Nationality Laws as
Held: Ratification is generally held to be an executive follows:
act, undertaken by the head of state or of the
government, as the case may be, through which the Art. 5. Within a third State a person having more than
formal acceptance of the treaty is proclaimed. A State one nationality shall be treated as if he had only one.
may provide in its domestic legislation the process of Without prejudice to the application of its law in matters
ratification of a treaty. The consent of the State to be of personal status and of any convention in force, a third
bound by a treaty is expressed by ratification when: (a) State shall, of the nationalities which any such person
the treaty provides for such ratification, (b) it is otherwise possesses, recognize exclusively in its territory either the
established that the negotiating States agreed that nationality of the country in which he is habitually and
ratification should be required, (c) the representative of principally resident or the nationality of the country with
the State has signed the treaty subject to ratification, or which in the circumstances he appears to be in fact most
(d) the intention of the State to sign the treaty subject to closely connected. (Frivaldo v. COMELEC, 174 SCRA
ratification appears from the full powers of its 245, June 23, 1989)
representative, or was expressed during the negotiation.
(BAYAN [Bagong Alyansang Makabayan] v. 410. What are the conditions before foreign military
Executive Secretary Ronaldo Zamora, G.R. No. bases, troops, or facilities may be allowed in the
138570, Oct. 10, 2000, En Banc [Buena]) Philippines?

407. Explain the “pacta sunt servanda” rule. Ans.: After the expiration in 1991 of the
Agreement between the Republic of the Philippines and
Held: One of the oldest and most fundamental rules in the United States of America concerning Military Bases,
international law is pacta sunt servanda – international foreign military bases, troops, or facilities shall not be
agreements must be performed in good faith. “A treaty allowed in the Philippines except under a treaty duly
engagement is not a mere moral obligation but creates a concurred in by the Senate and, when the Congress so
legally binding obligation on the parties x x x. A state requires, ratified by a majority of the votes cast by the
which has contracted valid international obligations is people in a national referendum held for that purpose,
bound to make in its legislations such modifications as and recognized as a treaty by the other contracting
may be necessary to ensure the fulfillment of the State. (Sec. 25, Art. XVIII, 1987 Constitution)
obligations undertaken.” (Tanada v. Angara, 272
SCRA 18, May 2, 1997 [Panganiban]) 411. Which provision of the Constitution applies with
regard to the exercise by the Senate of its constitutional
408. Explain the "rebus sic stantibus" rule (i.e., things power to concur with the Visiting Forces Agreement
remaining as they are). Does it operate automatically to (VFA)?
render a treaty inoperative?
Held: One focal point of inquiry in this controversy is the
Held: According to Jessup, the doctrine constitutes an determination of which provision of the Constitution
attempt to formulate a legal principle which would justify applies, with regard to the exercise by the Senate of its
non-performance of a treaty obligation if the conditions constitutional power to concur with the VFA. Petitioners
with relation to which the parties contracted have argue that Section 25, Article XVIII is applicable
changed so materially and so unexpectedly as to create a considering that the VFA has for its subject the presence
situation in which the exaction of performance would be of foreign military troops in the Philippines. Respondents,
unreasonable. The key element of this doctrine is the vital on the contrary, maintain that Section 21, Article VII
change in the condition of the contracting parties that should apply inasmuch as the VFA is not a basing
they could not have foreseen at the time the treaty was arrangement but an agreement which involves merely the
concluded. temporary visits of United States personnel engaged in
joint military exercises.
The doctrine of rebus sic stantibus does not operate
automatically to render the treaty inoperative. There is a The 1987 Philippine contains two provisions
necessity for a formal act of rejection, usually made by requiring the concurrence of the Senate on treaties or
the head of state, with a statement of the reasons why international agreements. Section 21, Article VII x x x
compliance with the treaty is no longer required. reads:
(Santos III v. Northwest Orient Airlines, 210 SCRA
256, June 23, 1992) "No treaty or international agreement shall be valid and
effective unless concurred in by at least two-thirds of all
409. What is the “doctrine of effective nationality” the Members of the Senate."
(genuine link doctrine)?
Section 25, Article XVIII, provides: Senate is mandatory to comply with the strict
constitutional requirements.
"After the expiration in 1991 of the Agreement between
the Republic of the Philippines and the United States of On the whole, the VFA is an agreement which
America concerning Military Bases, foreign military bases, defines the treatment of United States troops and
troops, or facilities shall not be allowed in the Philippines personnel visiting the Philippines. It provides for the
except under a treaty duly concurred in by the Senate guidelines to govern such visits of military personnel, and
and, when the Congress so requires, ratified by a majority further defines the rights of the United States and the
of the votes cast by the people in a national referendum Philippine government in the matter of criminal
held for that purpose, and recognized as a treaty by the jurisdiction, movement of vessels and aircraft,
other contracting State." importation and exportation of equipment, materials and
supplies.
Section 21, Article VII deals with treaties or
international agreements in general, in which case, the Undoubtedly, Section 25, Article XVIII, which
concurrence of at least two-thirds (2/3) of all the specifically deals with treaties involving foreign military
Members of the Senate is required to make the subject bases, troops, or facilities, should apply in the instant
treaty, or international agreement, valid and binding on case. To a certain extent and in a limited sense, however,
the part of the Philippines. This provision lays down the the provisions of Section 21, Article VII will find
general rule on treaties or international agreements and applicability with regard to the issue and for the sole
applies to any form of treaty with a wide variety of subject purpose of determining the number of votes required to
matter, such as, but not limited to, extradition or tax obtain the valid concurrence of the Senate x x x.
treaties or those economic in nature. All treaties or
international agreements entered into by the Philippines, It is a finely-imbedded principle in statutory
regardless of subject matter, coverage, or particular construction that a special provision or law prevails over
designation or appellation, requires the concurrence of a general one. Lex specialis derogat generali. Thus,
the Senate to be valid and effective. where there is in the same statute a particular enactment
and also a general one which, in its most comprehensive
In contrast, Section 25, Article XVIII is a special sense, would include what is embraced in the former, the
provision that applies to treaties which involve the particular enactment must be operative, and the general
presence of foreign military bases, troops or facilities in enactment must be taken to affect only such cases within
the Philippines. Under this provision, the concurrence of its general language which are not within the provision of
the Senate is only one of the requisites to render the particular enactment (Manila Railroad Co. v. Collector
compliance with the constitutional requirements and to of Customs, 52 Phil. 950). (BAYAN [Bagong
consider the agreement binding on the Philippines. Alyansang Makabayan] v. Executive Secretary
Section 25, Article XVIII further requires that "foreign Ronaldo Zamora, G.R. No. 138570 and Companion
military bases, troops, or facilities" may be allowed in the Cases, Oct. 10, 2000, 342 SCRA 449, 481-492, En
Philippines only by virtue of a treaty duly concurred in by Banc [Buena])
the Senate, ratified by a majority of the votes cast in a
national referendum held for that purpose if so required 412. Should the contention that since the VFA merely
by Congress, and recognized as such by the other involved the temporary visits of United States personnel
contracting State. engaged in joint military exercises and not a basing
agreement, therefore, Sec. 21, Art. VII of the 1987
It is our considered view that both constitutional Constitution, and not Sec. 25, Art. XVIII, should apply to
provisions, far from contradicting each other, actually the VFA, be upheld?
share some common ground. These constitutional
provisions both embody phrases in the negative and thus, Held: It is specious to argue that Section 25,
are deemed prohibitory in mandate and character. In Article XVIII is inapplicable to mere transient agreements
particular, Section 21 opens with the clause "No treaty x for the reason that there is no permanent placing of
x x," and Section 25 contains the phrase "shall not be structure for the establishment of a military base. On this
allowed." Additionally, in both instances, the concurrence score, the Constitution makes no distinction between
of the Senate is indispensable to render the treaty or "transient" and "permanent". Certainly, we find nothing
international agreement valid and effective. in Section 25, Article XVIII that requires foreign troops or
facilities to be stationed or placed permanently in the
To our mind, the fact that the President referred Philippines.
the VFA to the Senate under Section 21, Article VII, and
that the Senate extended its concurrence under the same It is a rudiment in legal hermeneutics that when
provision, is immaterial. For in either case, whether under no distinction is made by law, the Court should not
Section 21, Article VII or Section 25, Article XVIII, the distinguish - Ubi lex non distinguit nec nos distinguire
fundamental law is crystalline that the concurrence of the debemos.
accordance with the provisions of the Constitution,
In like manner, we do not subscribe to the whether under the general requirement in Section 21,
argument that Section 25, Article XVIII is not controlling Article VII, or the specific mandate mentioned in Section
since no foreign military bases, but merely foreign troops 25, Article XVIII, the provision in the latter article
and facilities, are involved in the VFA. Notably, a perusal requiring ratification by a majority of the votes cast in a
of said constitutional provision revels that the proscription national referendum being unnecessary since Congress
covers "foreign military bases, troops, or facilities." has not required it.
Stated differently, this prohibition is not limited to the
entry of troops and facilities without any foreign bases As to the matter of voting, Section 21, Article VII
being established. The clause does not refer to foreign particularly requires that a treaty or international
military bases, troops, or facilities" collectively but treats agreement, to be valid and effective, must be concurred
them as separate and independent subjects. The use of in by at least two-thirds of all the members of the Senate.
comma and disjunctive word "or" clearly signifies On the other hand, Section 25, Article XVIII simply
disassociation and independence of one thing from the provides that the treaty be "duly concurred in by the
others included in the enumeration (Castillo-Co v. Senate."
Barbers, 290 SCRA 717, 723 [1998]), such that, the
provision contemplates three different situations - a Applying the foregoing constitutional provisions, a two-
military treaty the subject of which could be either (a) thirds vote of all the members of the Senate is clearly
foreign bases (b) foreign troops, or (c) foreign facilities - required so that the concurrence contemplated by law
any of the three standing alone places it under the may be validly obtained and deemed present. While it is
coverage of Section 25, Article XVIII. true that Section 25, Article XVIII requires, among other
things, that the treaty - the VFA, in the instant case - be
To this end, the intention of the framers of the "duly concurred in by the Senate," it is very true however
Charter, as manifested during the deliberations of the that said provision must be related and viewed in light of
1986 Constitutional Commission, is consistent with this the clear mandate embodied in Section 21, Article VII,
interpretation x x x. which in more specific terms, requires that the
concurrence of a treaty, or international agreement, be
Moreover, military bases established within the made by a two-thirds vote of all the members of the
territory of another state is no longer viable because of Senate. Indeed, Section 25, Article XVIII must not be
the alternatives offered by new means and weapons of treated in isolation to Section 21, Article VII.
warfare such as nuclear weapons, guided missiles as well
as huge sea vessels that can stay afloat in the sea even As noted, the "concurrence requirement" under Section
for months and years without returning to their home 25, Article XVIII must be construed in relation to the
country. These military warships are actually used as provisions of Section 21, Article VII. In a more particular
substitutes for a land-home base not only of military language, the concurrence of the Senate contemplated
aircraft but also of military personnel and facilities. under Section 25, Article XVIII means that at least two-
Besides, vessels are mobile as compared to a land-based thirds of all the members of the Senate favorably vote to
military headquarters. (BAYAN [Bagong Alyansang concur with the treaty - the VFA in the instant case.
Makabayan] v. Executive Secretary Ronaldo
Zamora, G.R. No. 138570 and Companion Cases, xxx
Oct. 10, 2000, 342 SCRA 449, 481-492, En Banc
[Buena]) Having resolved that the first two requisites prescribed in
Section 25, Article XVIII are present, we shall now pass upon
413. Were the requirements of Sec. 25, Art. XVIII of and delve on the requirement that the VFA should be
the 1987 Constitution complied with when the Senate recognized as a treaty by the United States of America.
gave its concurrence to the VFA?
xxx
Held: Section 25, Article XVIII disallows foreign military
bases, troops, or facilities in the country, unless the This Court is of the firm view that the phrase "recognized
following conditions are sufficiently met, viz: (a) it must as a treaty" means that the other contracting party
be under a treaty; (b) the treaty must be duly concurred accepts or acknowledges the agreement as a treaty
in by the Senate and, when so required by Congress, (Ballantine's Legal Dictionary, 1995). To require the other
ratified by a majority of the votes cast by the people in a contracting state, The United States of America in this
national referendum; and (c) recognized as a treaty by case, to submit the VFA to the United States Senate for
the other contracting state. concurrence pursuant to its Constitution, is to accord
strict meaning to the phrase.
There is no dispute as to the presence of the first two
requisites in the case of the VFA. The concurrence Well-entrenched is the principle that the words used in
handed by the Senate through Resolution No. 18 is in the Constitution are to be given their ordinary meaning
except where technical terms are employed, in which case has been described as the “core” of the defense
the significance thus attached to them prevails. Its relationship between the Philippines and its traditional
language should be understood in the sense they have in ally, the United States. Its aim is to enhance the
common use (J.M. Tuason & Co., Inc. v. Land Tenure strategic and technological capabilities of our armed
Association, 31 SCRA 413 [1970]) forces through joint training with its American
counterparts; the “Balikatan” is the largest such training
Moreover, it is inconsequential whether the United States exercise directly supporting the MDT’s objectives. It is
treats the VFA only as an executive agreement because, this treaty to which the VFA adverts and the obligations
under international law, an executive agreement is as thereunder which it seeks to reaffirm.
binding as a treaty (Altman Co. v. United States, 224 US
263 [1942], cited in Coquia and Defensor-Santiago, The lapse of the US-Philippine Bases Agreement in 1992
International Law, 1998 Ed. P. 497). To be sure, as long and the decision not to renew it created a vacuum in
as the VFA possesses the elements of an agreement US-Philippine defense relations, that is, until it was
under international law, the said agreement is to be taken replaced by the Visiting Forces Agreement. It should be
equally as a treaty. recalled that on October 10, 2000, by a vote of eleven to
three, this Court upheld the validity of the VFA (BAYAN,
xxx et. Al. v. Zamora, et. al., 342 SCRA 449 [2000]). The
VFA provides the “regulatory mechanism” by which
The records reveal that the United States Government, “United States military and civilian personnel [may visit]
through Ambassador Thomas C. Hubbard, has stated that temporarily in the Philippines in connection with
the United States government has fully committed to activities approved by the Philippine Government.” It
living up to the terms of the VFA. For as long as the contains provisions relative to entry and departure of
United States of America accepts or acknowledges the American personnel, driving and vehicle registration,
VFA as a treaty, and binds itself further to comply with its criminal jurisdiction, claims, importation and exportation,
obligations under the treaty, there is indeed marked movement of vessels and aircraft, as well as the
compliance with the mandate of the Constitution. duration of the agreement and its termination. It is the
VFA which gives continued relevance to the MDT despite
Worth stressing too, is that the ratification, by the the passage of years. Its primary goal is to facilitate the
President, of the VFA and the concurrence of the Senate promotion of optimal cooperation between American and
should be taken as a clear and unequivocal expression of Philippine military forces in the event of an attack by a
our nation's consent to be bound by said treaty, with the common foe.
concomitant duty to uphold the obligations and
responsibilities embodied thereunder. The first question that should be addressed is whether
“Balikatan 02-1” is covered by the Visiting Forces
xxx Agreement. To resolve this, it is necessary to refer to
the VFA itself. Not much help can be had therefrom,
With the ratification of the VFA, which is equivalent to unfortunately, since the terminology employed is itself
final acceptance, and with the exchange of notes between the source of the problem. The VFA permits United
the Philippines and the United States of America, it now States personnel to engage, on an impermanent basis,
becomes obligatory and incumbent on our part, under the in “activities,” the exact meaning of which was left
principles of international law, to be bound by the terms undefined. The expression is ambiguous, permitting a
of the agreement. Thus, no less than Section 2, Article II wide scope of undertakings subject only to the approval
of the Constitution, declares that the Philippines adopts of the Philippine government. The sole encumbrance
the generally accepted principles of international law as placed on its definition is couched in the negative, in
part of the law of the land and adheres to the policy of that United States personnel must “abstain from any
peace, equality, justice, freedom, cooperation and amity activity inconsistent with the spirit of this agreement,
with all nations. (BAYAN [Bagong Alyansang and in particular, from any political activity.” All other
Makabayan] v. Executive Secretary Ronaldo activities, in other words, are fair game.
Zamora, G.R. No. 138570 and Companion Cases,
Oct. 10, 2000, 342 SCRA 449, 481-492, En Banc We are not completely unaided, however. The Vienna
[Buena]) Convention on the Law of Treaties, which contains
provisos governing interpretations of international
414. Are the “Balikatan” exercises covered by the agreements, state x x x.
Visiting Forces Agreement?
It is clear from the foregoing that the cardinal rule of
Held: The holding of “Balikatan 02-1” must be interpretation must involve an examination of the text,
studied in the framework of the treaty antecedents to which is presumed to verbalize the parties’ intentions.
which the Philippines bound itself. The first of these is The Convention likewise dictates what may be used as
the Mutual Defense Treaty (MDT, for brevity). The MDT aids to deduce the meaning of terms, which it refers to
as the context of the treaty, as well as other elements responsible for violations committed by any branch or
may be taken into account alongside the aforesaid subdivision of its government or any official thereof. As
context. X x x an integral part of the community of nations, we are
responsible to assure that our government, Constitution
The Terms of Reference rightly fall within the context of and laws will carry out our international obligation (Louis
the VFA. Henkin, Richard C. Pugh, Oscar Schachter, Hans Smit,
International Law, Cases and Materials, 2nd Ed., American
After studied reflection, it appeared farfetched that the Casebook Series, p. 136). Hence, we cannot readily plead
ambiguity surrounding the meaning of the word the Constitution as a convenient excuse for non-
“activities” arose from accident. In our view, it was compliance with our obligations, duties and
deliberately made that way to give both parties a certain responsibilities under international law.
leeway in negotiation. In this manner, visiting US forces
may sojourn in Philippine territory for purposes other Beyond this, Article 13 of the Declaration of Rights and
than military. As conceived, the joint exercises may Duties of States adopted by the International Law
include training on new techniques of patrol and Commission in 1949 provides: Every State has the duty to
surveillance to protect the nation’s marine resources, carry out in good faith its obligations arising from treaties
sea search-and-destroy operations to assist vessels in and other sources of international law, and it may not
distress, disaster relief operations, civic action projects invoke provisions in its constitution or its laws as an
such as the building of school houses, medical and excuse for failure to perform this duty. (Gerhard von
humanitarian missions, and the like. Glahn, supra, p. 487)

Under these auspices, the VFA gives legitimacy to the Equally important is Article 26 of the Convention which
current Balikatan exercises. It is only logical to assume provides that “Every treaty in force is binding upon the
that “Balikatan 02-1,” a “mutual anti-terrorism advising, parties to it and must be performed by them in good
assisting and training exercise,” falls under the umbrella faith.” This is known as the principle of pacta sunt
of sanctioned or allowable activities in the context of the servanda which preserves the sanctity of treaties and
agreement. Both the history and intent of the Mutual have been one of the most fundamental principles of
Defense Treaty and the VFA support the conclusion that positive international law, supported by the jurisprudence
combat-related activities – as opposed to combat itself – of international tribunals (Harris, p. 634 cited in Coquia,
such as the one subject of the instant petition, are International Law, supra, p. 512). (BAYAN [Bagong
indeed authorized. (Arthur D. Lim and Paulino R. Alyansang Makabayan] v. Executive Secretary
Ersando v. Honorable Executive Secretary, G.R. Ronaldo Zamora, G.R. No. 138570, Oct. 10, 2000,
No. 151445, April 11, 2002, En Banc [De Leon]) 342 SCRA 449, 492-493, En Banc [Buena])

415. With the ratification of the Visiting Forces 416. What must a person who feels aggrieved by the
Agreement (VFA), has it now become obligatory and acts of a foreign sovereign do to espouse his cause?
incumbent on our part to be bound by its terms even if it
is asserted that said agreement contravenes the Held: Private respondent is not left without any
Constitution? legal remedy for the redress of its grievances. Under both
Public International Law and Transnational Law, a person
Held: With the ratification of the VFA, which is equivalent who feels aggrieved by the acts of a foreign sovereign can
to final acceptance, and with the exchange of notes ask his own government to espouse his cause through
between the Philippines and the United States of America, diplomatic channels.
it now becomes obligatory and incumbent on our part,
under the principles of international law, to be bound by Private respondent can ask the Philippine
the terms of the agreement. Thus, no less than Section government, through the Foreign Office, to espouse its
2, Article II of the Constitution, declares that the claims against the Holy See. Its first task is to persuade
Philippines adopts the generally accepted principles of the Philippine government to take up with the Holy See
international law as part of the law of the land and the validity of its claim. Of course, the Foreign Office shall
adheres to the policy of peace, equality, justice, freedom, first make a determination of the impact of its espousal
cooperation and amity with all nations. on the relations between the Philippine government and
the Holy See (Young, Remedies of Private Claimants
As a member of the family of nations, the Philippines Against Foreign States, Selected Readings on Protection
agrees to be bound by generally accepted rules for the by Law of Private Foreign Investments 905, 919 [1964]).
conduct of its international relations. While the Once the Philippine government decides to espouse the
international obligation devolves upon the state and not claim, the latter ceases to be a private cause.
upon any particular branch, institution, or individual
member of its government, the Philippines is nonetheless
According to the Permanent Court of Held: Courts cannot blindly adhere and take on
International Justice, the forerunner of the International its face the communication from the DFA that petitioner
Court of Justice: is covered by any immunity. The DFA’s determination
that a certain person is covered by immunity is only
“By taking up the case of one of its subjects and by preliminary which has no binding effect in courts. In
resorting to diplomatic action or international judicial receiving ex parte the DFA’s advice and in motu proprio
proceedings on his behalf, a State is in reality asserting dismissing the two criminal cases without notice to the
its own rights - its right to ensure, in the person of its prosecution, the latter’s right to due process was violated.
subjects, respect for the rules of international law” (The It should be noted that due process is a right of the
Mavrommatis Palestine Concessions, 1 Hudson, World accused as much as it is of the prosecution. The needed
Court Reports 293, 302 [1924]). inquiry in what capacity petitioner was acting at the time
(Holy See, The v. Rosario, Jr., 238 SCRA 524, 538- of the alleged utterances requires for its resolution
539, Dec. 1, 1994, En Banc [Quiason]) evidentiary basis that has yet to be presented at the
proper time (See United States v. Guinto, 182 SCRA 644
417. What are the conditions before the rights of [1990]). At any rate, it has been ruled that the mere
belligerency may be accorded the rebels? invocation of the immunity clause does not ipso facto
result in the dropping of the charges (Chavez v.
Ans.: As a matter of legal theory, the rebels have Sandiganbayan, 193 SCRA 282 [1991]). (Liang v.
to fulfill certain conditions before the rights of belligerency People, 323 SCRA 692, Jan. 28, 2000, 1st Div.
are accorded them, namely: [Ynares-Santiago])

1) An organized civil government that has 420. Discuss the basis of the argument that a
control and direction over the armed struggle launched by determination by the DFA that a person is entitled to
the rebels; diplomatic immunity is a political question binding on the
2) Occupation of a substantial portion of the courts.
national territory;
3) Seriousness of the struggle, which must be Held: Petitioner’s argument that a determination
so widespread thereby leaving no doubt as to the by the Department of Foreign Affairs that he is entitled to
outcome; diplomatic immunity is a political question binding on the
4) Willingness on the part of the rebels to courts, is anchored on the ruling enunciated in the case
observe the rules and customs of war. of WHO, et al. v. Aquino, et al. (48 SCRA 242 [1972]), viz:

418. Discuss the legal consequences that follow “It is a recognized principle of international law and under
recognition of belligerency. our system of separation of powers that diplomatic
immunity is essentially a political question and courts
Ans.: should refuse to look beyond a determination by the
executive branch of the government, and where the plea
1) Before recognition as such, it is the legitimate of diplomatic immunity is recognized and affirmed by the
government that is responsible for the acts of the rebels executive branch of the government as in the case at bar,
affecting foreign nationals and their properties. Once it is then the duty of the courts to accept the claim of
recognition is given, the legitimate government may no longer immunity upon appropriate suggestion by the principal
be held responsible for their acts; responsibility is shifted to the law officer of the government, the Solicitor General in this
rebel government; case, or other officer acting under his direction. Hence,
2) The legitimate government, once it recognizes the in adherence to the settled principle that courts may not
rebels as belligerents, is bound to observe the laws and customs
so exercise their jurisdiction by seizure and detention of
of war in conducting the hostilities;
property, as to embarrass the executive arm of the
3) From the point of view of third States, the effect
government in conducting foreign relations, it is accepted
of recognition of belligerency is to put them under obligation to
observe strict neutrality and abide by the consequences arising doctrine that in such cases the judicial department of the
from that position; government follows the action of the political branch and
4) On the side of the rebels, recognition of will not embarrass the latter by assuming an antagonistic
belligerency puts them under responsibility to third States and jurisdiction.”
to the legitimate government for all their acts which do not
conform to the laws and customs of war. (Salonga & Yap, This ruling was reiterated in the subsequent cases
Public International Law, 5th Ed. [1992], p. 33) of International Catholic Migration Commission v. Calleja
(190 SCRA 130 [1990]); The Holy See v. Rosario, Jr. (238
419. Should Courts blindly adhere and take on its face SCRA 524 [1994]); Lasco v. United Nations (241 SCRA
the communication from the Department of Foreign 681 [1995]); and DFA v. NLRC (262 SCRA 38 [1996]).
Affairs (DFA) that a person is covered by any immunity?
The case of WHO v. Aquino involved the search mainly as a means for conducting general international
and seizure of personal effects of petitioner Leonce business in which the member states have an interest.”
Verstuyft, an official of the WHO. Verstuyft was certified (ICMC v. Calleja, supra note 2)
to be entitled to diplomatic immunity pursuant to the Host
Agreement executed between the Philippines and the International public officials have been defined
WHO. as:

ICMC v. Calleja concerned a petition for “x x x persons who, on the basis of an international treaty
certification election filed against ICMC and IRRI. As constituting a particular international community, are
international organizations, ICMC and IRRI were declared appointed by this international community, or by an organ
to possess diplomatic immunity. It was held that they are of it, and are under its control to exercise, in a continuous
not subject to local jurisdictions. It was ruled that the way, functions in the interest of this particular
exercise of jurisdiction by the Department of Labor over international community, and who are subject to a
the case would defeat the very purpose of immunity, particular personal status.” (John Kerry King, The
which is to shield the affairs of international organizations Privileges and Immunities of the Personnel of
from political pressure or control by the host country and International Organizations xiii [1949], citing: Suzanne
to ensure the unhampered performance of their Basdevant, Les Functionnaires Internationaux [Paris:
functions. 1931], Chapter I)

Holy See v. Rosario, Jr. involved an action for “Specialized agencies” are international
annulment of sale of land against the Holy See, as organizations having functions in particular fields, such as
represented by the Papal Nuncio. The Court upheld the posts, telecommunications, railways, canals, rivers, sea
petitioner’s defense of sovereign immunity. It ruled that transport, civil aviation, meteorology, atomic energy,
where a diplomatic envoy is granted immunity from the finance, trade, education and culture, health and refugees
civil and administrative jurisdiction of the receiving state (ICMC v. Calleja, et al., citing Articles 57 and 63 of the
over any real action relating to private immovable United Nations Charter). (Concurring Opinion, Puno
property situated in the territory of the receiving state, J., in Jeffrey Liang [Huefeng] v. People, G.R. No.
which the envoy holds on behalf of the sending state for 125865, Mar. 26, 2001, 1st Div. [Motion for
the purposes of the mission, with all the more reason Reconsideration])
should immunity be recognized as regards the sovereign
itself, which in that case is the Holy See. 421. What are the differences between Diplomatic and
International Immunities? Discuss.
In Lasco v. United Nations, the United Nations
Revolving Fund for Natural Resources Exploration was Held: There are three major differences
sued before the NLRC for illegal dismissal. The Court between diplomatic and international immunities. Firstly,
again upheld the doctrine of diplomatic immunity invoked one of the recognized limitations of diplomatic immunity
by the Fund. is that members of the diplomatic staff of a mission may
be appointed from among the nationals of the receiving
Finally, DFA v. NLRC involved an illegal dismissal State only with the express consent of that State; apart
case filed against the Asian Development Bank. Pursuant from inviolability and immunity from jurisdiction in respect
to its Charter and the Headquarters Agreement, the of official acts performed in the exercise of their functions,
diplomatic immunity of the Asian Development Bank was nationals enjoy only such privileges and immunities as
recognized by the Court. may be granted by the receiving State. International
immunities may be specially important in relation to the
It bears to stress that all of these cases pertain State of which the official is a national. Secondly, the
to the diplomatic immunity enjoyed by international immunity of a diplomatic agent from the jurisdiction of the
organizations. Petitioner asserts that he is entitled to the receiving State does not exempt him from the jurisdiction
same diplomatic immunity and he cannot be prosecuted of the sending State; in the case of international
for acts allegedly done in the exercise of his official immunities there is no sending State and an equivalent
functions. for the jurisdiction of the Sending State therefore has to
be found either in waiver of immunity or in some
The term “international organizations” – international disciplinary or judicial procedure. Thirdly,
the effective sanctions which secure respect for
“is generally used to describe an organization set up by diplomatic immunity are the principle of reciprocity and
agreement between two or more states. Under the danger of retaliation by the aggrieved State;
contemporary international law, such organizations are international immunities enjoy no similar protection (C.
endowed with some degree of international legal Wilfred Jenks, Contemporary Development in
personality such that they are capable of exercising International Immunities xxxvii [1961]) (Concurring
specific rights, duties and powers. They are organized Opinion, Puno J., in Jeffrey Liang [Huefeng] v.
People, G.R. No. 125865, Mar. 26, 2001, 1st Div. located. This is the case with the League of Nations, the
[Motion for Reconsideration]) Permanent Court of Justice, and the United Nations (J.K.
King, supra note 12, at 81).
422. Discuss the immunity of International Officials.
The Asian Development Bank and its Personnel
Held: The generally accepted principles which are now fall under this third category.
regarded as the foundation of international immunities
are contained in the ILO Memorandum, which reduced There is a connection between diplomatic
them in three basic propositions, namely: (1) that privileges and immunities and those extended to
international institutions should have a status which international officials. The connection consists in the
protects them against control or interference by any one granting, by contractual provisions, of the relatively well-
government in the performance of functions for the established body of diplomatic privileges and immunities
effective discharge of which they are responsible to to international functionaries. This connection is purely
democratically constituted international bodies in which historical. Both types of officials find the basis of their
all the nations concerned are represented; (2) that no special status in the necessity of retaining functional
country should derive any financial advantage by levying independence and freedom from interference by the state
fiscal charges on common international funds; and (3) of residence. However, the legal relationship between an
that the international organization should, as a collectivity ambassador and the state to which he is accredited is
of States Members, be accorded the facilities for the entirely different from the relationship between the
conduct of its official business customarily extended to international official and those states upon whose
each other by its individual member States. The thinking territory he might carry out its functions (See Id. at 255).
underlying these propositions is essentially institutional in
character. It is not concerned with the status, dignity or The privileges and immunities of diplomats and those of
privileges of individuals, but with the elements of international officials rest upon different legal
functional independence necessary to free international foundations. Whereas those immunities awarded to
institutions from national control and to enable them to diplomatic agents are a right of the sending state based
discharge their responsibilities impartially on behalf of all on customary international law, those granted to
their members (Id. at 17). (Concurring Opinion, Puno international officials are based on treaty or conventional
J., in Jeffrey Liang [Huefeng] v. People, G.R. No. law. Customary international law places no obligation on
125865, Mar. 26, 2001, 1st Div. [Motion for a state to recognize a special status of an international
Reconsideration]) official or to grant him jurisdictional immunities. Such an
obligation can only result from specific treaty provisions
423. What are the three methods of granting privileges (Id. at 25-26).
and immunities to the personnel of international
organizations? Under what category does the Asian The special status of the diplomatic envoy is regulated by
Development Bank and its Personnel fall? the principle of reciprocity by which a state is free to treat
the envoy of another state as its envoys are treated by
Held: Positive international law has devised three that state. The juridical basis of the diplomat’s position is
methods of granting privileges and immunities to the firmly established in customary international law. The
personnel of international organizations. The first is by diplomatic envoy is appointed by the sending State but it
simple conventional stipulation, as was the case in the has to make certain that the agreement of the receiving
Hague Conventions of 1899 and 1907. The second is by State has been given for the person it proposes to accredit
internal legislation whereby the government of a state, as head of the mission to that State (Article 4, Vienna
upon whose territory the international organization is to Convention on Diplomatic Relations).
carry out its functions, recognizes the international
character of the organization and grants, by unilateral The staff personnel of an international organization – the
measures, certain privileges and immunities to better international officials – assume a different position as
assure the successful functioning of the organization and regards their special status. They are appointed or
its personnel. In this situation, treaty obligation for the elected to their position by the organization itself, or by a
state in question to grant concessions is lacking. Such competent organ of it; they are responsible to the
was the case with the Central Commission of the Rhine at organization and their official acts are imputed to it. The
Strasbourg and the International Institute of Agriculture juridical basis of their special position is found in
at Rome. The third is a combination of the first two. In conventional law (J.K. King, supra note 12, at xiii), since
this third method, one finds a conventional obligation to there is no established basis of usage or custom in the
recognize a certain status of an international organization case of the international official. Moreover, the
and its personnel, but the status is described in broad and relationship between an international organization and a
general terms. The specific definition and application of member-state does not admit of the principle of
those general terms are determined by an accord reciprocity (Id. at 27), for it is contradictory to the basic
between the organization itself and the state wherein it is principle of equality of states. An international
organization carries out functions in the interest of every Rhine Treaty of 1804 between the German Empire and
member state equally. The international official does not France which provided “all the rights of neutrality” to
carry out his functions in the interest of any state, but in persons employed in regulating navigation in the
serving the organization he serves, indirectly, each state international interest; The Treaty of Berlin of 1878 which
equally. He cannot be, legally, the object of the operation granted the European Commission of the Danube
of the principle of reciprocity between states under such “complete independence of territorial authorities” in the
circumstances. It is contrary to the principle of equality exercise of its functions; The Convention of the League
of states for one state member of an international which granted “diplomatic immunities and privileges.”
organization to assert a capacity to extract special Today, the age of the United Nations finds the scope of
privileges for its nationals from other member states on protection narrowed. The current tendency is to reduce
the basis of a status awarded by it to an international privileges and immunities of personnel of international
organization. It is upon this principle of sovereign organizations to a minimum. The tendency cannot be
equality that international organizations are built. considered as a lowering of the standard but rather as a
recognition that the problem on the privileges and
It follows from this same legal circumstance that a state immunities of international officials is new. The solution
called upon to admit an official of an international to the problem presented by the extension of diplomatic
organization does not have a capacity to declare him prerogatives to international functionaries lies in the
persona non grata. general reduction of the special position of both types of
agents in that the special status of each agent is granted
The functions of the diplomat and those of the in the interest of function. The wide grant of diplomatic
international official are quite different. Those of the prerogatives was curtailed because of practical necessity
diplomat are functions in the national interest. The task and because the proper functioning of the organization
of the ambassador is to represent his state, and its did not require such extensive immunity for its officials.
specific interest, at the capital of another state. The While the current direction of the law seems to be to
functions of the international official are carried out in the narrow the prerogatives of the personnel of international
international interest. He does not represent a state or organizations, the reverse is true with respect to the
the interest of any specific state. He does not usually prerogatives of the organizations themselves, considered
“represent” the organization in the true sense of that as legal entities. Historically, states have been more
term. His functions normally are administrative, although generous in granting privileges and immunities to
they may be judicial or executive, but they are rarely organizations than they have to the personnel of these
political or functions of representation, such as those of organizations (J.K. King, supra note 12, at 253-268).
the diplomat.
Thus, Section 2 of the General Convention on the
There is a difference of degree as well as of kind. The Privileges and Immunities of the United Nations states
interruption of the activities of a diplomatic agent is likely that the UN shall enjoy immunity from every form of legal
to produce serious harm to the purposes for which his process except insofar as in any particular case it has
immunities were granted. But the interruption of the expressly waived its immunity. Section 4 of the
activities of the international official does not, usually, Convention on the Privileges and Immunities of the
cause serious dislocation of the functions of an Specialized Agencies likewise provides that the specialized
international secretariat (id. at 254-257). agencies shall enjoy immunity from every form of legal
process subject to the same exception. Finally, Article
On the other hand, they are similar in the sense that acts 50[1] of the ADB Charter and Section 5 of the
performed in an official capacity by either a diplomatic Headquarters Agreement similarly provide that the bank
envoy or an international official are not attributable to shall enjoy immunity from every form of legal process,
him as an individual but are imputed to the entity he except in cases arising out of or in connection with the
represents, the state in the case of the diplomat, and the exercise of its powers to borrow money, to guarantee
organization in the case of the international official (Id. at obligations, or to buy and sell or underwrite the sale of
103). ). (Concurring Opinion, Puno J., in Jeffrey securities.
Liang [Huefeng] v. People, G.R. No. 125865, Mar.
26, 2001, 1st Div. [Motion for Reconsideration]) The phrase “immunity from every form of legal process”
as used in the UN General Convention has been
424. What is the reason behind the current tendency interpreted to mean absolute immunity from a state’s
of reducing privileges and immunities of personnel of jurisdiction to adjudicate or enforce its law by legal
international organizations to a minimum? process, and it is said that states have not sought to
restrict that immunity of the United Nations by
Held: Looking back over 150 years of privileges and interpretation or amendment. Similar provisions are
immunities granted to the personnel of international contained in the Special Agencies Convention as well as
organizations, it is clear that they were accorded a wide in the ADB Charter and Headquarters Agreement. These
scope of protection in the exercise of their functions – The organizations were accorded privileges and immunities in
their charters by language similar to that applicable to the Held: Section 18 [a] of the General Convention
United Nations. It is clear therefore that these has been interpreted to mean that officials of the specified
organizations were intended to have similar privileges and categories are denied immunity from local jurisdiction for
immunities (1 Restatement of the Law Third 498-501). acts of their private life and empowers local courts to
From this, it can be easily deduced that international assume jurisdiction in such cases without the necessity of
organizations enjoy absolute immunity similar to the waiver (Id. at 186). It has earlier been mentioned that
diplomatic prerogatives granted to diplomatic envoys. historically, international officials were granted diplomatic
privileges and immunities and were thus considered
Even in the United States this seems to be the prevailing immune for both private and official acts. In practice, this
rule x x x. wide grant of diplomatic prerogatives was curtailed
because of practical necessity and because the proper
On the other hand, international officials are governed by functioning of the organization did not require such
a different rule. Section 18[a] of the General Convention exclusive immunity for its officials. Thus, the current
on Privileges and Immunities of the United Nations states status of the law does not maintain that states grant
that officials of the United Nations shall be immune from jurisdictional immunity to international officials for acts of
legal process in respect of words spoken or written and their private lives (But see id. at 259. It is important to
all acts performed by them in their official capacity. The note that the submission of international officials to local
Convention on Specialized Agencies carries exactly the jurisdiction for private acts is not completely accepted in
same provision. The Charter of the ADB provides under doctrine and theory. Jenks, in particular, has argued for
Article 55[i] that officers and employees of the bank shall complete jurisdictional immunity, as has Hammarskjold.).
be immune from legal process with respect to acts This much is explicit from the charter and Headquarters
performed by them in their official capacity except when Agreement of the ADB which contain substantially similar
the Bank waives immunity. Section 45 [a] of the ADB provisions to that of the General convention.
Headquarters Agreement accords the same immunity to (Concurring Opinion, Puno J., in Jeffrey Liang
the officers and staff of the bank. There can be no [Huefeng] v. People, G.R. No. 125865, Mar. 26,
dispute that international officials are entitled to immunity 2001, 1st Div. [Motion for Reconsideration])
only with respect to acts performed in their official
capacity, unlike international organizations which enjoy 426. Who is competent to determine whether a given
absolute immunity. act of international officials and representatives is private
or official?
Clearly, the most important immunity to an international
official, in the discharge of his international functions, is Held: In connection with this question, the
immunity from local jurisdiction. There is no argument in current tendency to narrow the scope of privileges ad
doctrine or practice with the principle that an international immunities of international officials and representatives is
official is independent of the jurisdiction of the local most apparent. Prior to the regime of the United Nations,
authorities for his official acts. Those acts are not his, but the determination of this question rested with the
are imputed to the organization, and without waiver the organization and its decision was final. By the new
local courts cannot hold him liable for them. In strict law, formula, the state itself tends to assume this competence.
it would seem that even the organization itself could have If the organization is dissatisfied with the decision, under
no right to waive an official’s immunity for his official acts. the provisions of the General Convention of the United
This permits local authorities to assume jurisdiction over Nations, or the Special Convention for Specialized
an individual for an act which is not, in the wider sense of Agencies, the Swiss Arrangement, and other current
the term, his act al all. It is the organization itself, as a dominant instruments, it may appeal to an international
juristic person, which should waive its own immunity and tribunal by procedures outlined in these instruments.
appear in court, not the individual, except insofar as he Thus, the state assumes this competence in the first
appears in the name of the organization. Provisions for instance. It means that, if a local court assumes
immunity from jurisdiction for official acts appear, aside jurisdiction over an act without the necessity of waiver
from the aforementioned treatises, in the constitution of from the organization, the determination of the nature of
most modern international organizations. The the act is made at the national level (Id. at 260-261).
acceptance of the principle is sufficiently widespread to
be regarded as declaratory of international law (J.K. King, It appears that the inclination is to place the
supra note 12, at 258-259) (Concurring Opinion, Puno competence to determine the nature of an act as private
J., in Jeffrey Liang [Huefeng] v. People, G.R. No. or official in the courts of the state concerned. That the
125865, Mar. 26, 2001, 1st Div. [Motion for practical notion seems to be to leave to the local courts
Reconsideration]) determination of whether or not a given act is official or
private does not necessarily mean that such
425. What is the status of the international official with determination is final. If the United Nations questions the
respect to his private acts? decision of the Court, it may invoke proceedings for
settlement of disputes between the organization and the
member states as provided in Section 30 of the General
Convention. Thus, the decision as to whether a given act 427. Discuss the extent of the international official’s
is official or private is made by the national courts in the immunity for official acts.
first instance, but it may be subjected to review in the
international level if questioned by the United Nations (Id. Held: One final point. The international official’s
at 189). immunity for official acts may be likened to a consular
official’s immunity from arrest, detention, and criminal or
xxx civil process which is not absolute but applies only to acts
or omissions in the performance of his official functions,
Under the Third Restatement of the Law, it is suggested in the absence of special agreement. Since a consular
that since an international official does not enjoy personal officer is not immune from all legal processes, he must
inviolability from arrest or detention and has immunity respond to any process and plead and prove immunity on
only with respect to official acts, he is subject to judicial the ground that the act or omission underlying the
or administrative process and must claim his immunity in process was in the performance of his official functions.
the proceedings by showing that the act in question was The issue has not been authoritatively determined, but
an official act. Whether an act was performed in the apparently the burden is on the consular official to prove
individual’s official capacity is a question for the court in his status as well as his exemption in the circumstances.
which a proceeding is brought, but if the international In the United States, the US Department of State
organization disputes the court’s finding, the dispute generally has left it to the courts to determine whether a
between that organization and the state of the forum is particular act was within a consular officer’s official duties
to be resolved by negotiation, by an agreed mode of (1 Restatement of the Law Third 475-477). (Concurring
settlement or by advisory opinion of the International Opinion, Puno J., in Jeffrey Liang [Huefeng] v.
Court of Justice (Restatement of the Law Third 512). People, G.R. No. 125865, Mar. 26, 2001, 1st Div.
[Motion for Reconsideration])
Recognizing the difficulty that by reason of the right of a
national court to assume jurisdiction over private acts 428. State the occasions when the use of force may be
without a waiver of immunity, the determination of the allowed under the UN Charter.
official or private character of a particular act may pass
from international to national, Jenks proposes three ways Ans.: There are only two occasions when the use of force
of avoiding difficulty in the matter. The first would be for is allowed under the UN Charter. The first is when it is
a municipal court before which a question of the official authorized in pursuance of the enforcement action that
or private character of a particular act arose to accept as may be decreed by the Security Council under Art. 42.
conclusive in the matter any claim by the international The second is when it is employed in the exercise of the
organization that the act was official in character, such a inherent right of self-defense under conditions prescribed
claim being regarded as equivalent to a governmental in Art. 51. (Justice Isagani A. Cruz, in an article
claim that a particular act is an act of State. Such a claim entitled “A New World Order” written in his
would be in effect a claim by the organization that the column “Separate Opinion” published in the March
proceedings against the official were a violation of the 30, 2003 issue of the Philippines Daily Inquirer)
jurisdictional immunity of the organization itself which is
unqualified and therefore not subject to delimitation in 429. Is the United States justified in invading Iraq
the discretion of the municipal court. The second would invoking its right to defend itself against an expected
be for a court to accept as conclusive in the matter a attack by Iraq with the use of its biological and chemical
statement by the executive government of the country weapons of mass destruction?
where the matter arises certifying the official character of
the act. The third would be to have recourse to the Ans.: The United States is invoking its right to defend
procedure of international arbitration. Jenks opines that itself against an expected attack by Iraq with the use of
it is possible that none of these three solutions would be its biological and chemical weapons of mass destruction.
applicable in all cases; the first might be readily There is no evidence of such a threat, but Bush is
acceptable only in the clearest cases and the second is probably invoking the modern view that a state does not
available only if the executive government of the country have to wait until the potential enemy fires first. The
where the matter arises concurs in the view of the cowboy from Texas says that outdrawing the foe who is
international organization concerning the official about to shoot is an act of self-defense.
character of the act. However, he surmises that taken in
combination, these various possibilities may afford the Art. 51 says, however, that there must first be an “armed
elements of a solution to the problem (Jenks, supra note attack” before a state can exercise its inherent right of
14, at 117-118). (Concurring Opinion, Puno J., in self-defense, and only until the Security Council, to which
Jeffrey Liang [Huefeng] v. People, G.R. No. the aggression should be reported, shall have taken the
125865, Mar. 26, 2001, 1st Div. [Motion for necessary measures to maintain international peace and
Reconsideration]) security. It was the United States that made the “armed
attack” first, thus becoming the aggressor, not Iraq. Iraq
is now not only exercising its inherent right of self-defense
as recognized by the UN Charter. (Justice Isagani A.
Cruz, in an article entitled “A New World Order”
written in his column “Separate Opinion”
published in the March 30, 2003 issue of the
Philippines Daily Inquirer)

430. Will the subsequent discovery of weapons of


mass destruction in Iraq after its invasion by the US justify
the attack initiated by the latter?

Ans.: Even if Iraq’s hidden arsenal is discovered – or


actually used – and the United States is justified in its
suspicions, that circumstance will not validate the
procedure taken against Iraq. It is like searching a person
without warrant and curing the irregularity with the
discovery of prohibited drugs in his possession. The
process cannot be reversed. The warrant must first be
issued before the search and seizure can be made.

The American invasion was made without permission


from the Security Council as required by the UN Charter.
Any subsequent discovery of the prohibited biological and
chemical weapons will not retroactively legalize that
invasion, which was, legally speaking, null and void ab
initio. (Justice Isagani A. Cruz, in an article entitled
“A New World Order” written in his column
“Separate Opinion” published in the March 30,
2003 issue of the Philippines Daily Inquirer)

You might also like